ЭлСктромагнитная индукция Ρ„ΠΎΡ€ΠΌΡƒΠ»Π°: ЭлСктромагнитная индукция – Ρ„ΠΎΡ€ΠΌΡƒΠ»Π°, Ρ‚Π°Π±Π»ΠΈΡ†Π°, ΠΏΡ€ΠΈΠΌΠ΅Ρ€Ρ‹ ΠΊΡ€Π°Ρ‚ΠΊΠΎ

Π‘ΠΎΠ΄Π΅Ρ€ΠΆΠ°Π½ΠΈΠ΅

ΠžΡΠ½ΠΎΠ²Π½Ρ‹Π΅ Ρ„ΠΎΡ€ΠΌΡƒΠ»Ρ‹ ΠΈ мСтодичСскиС Ρ€Π΅ΠΊΠΎΠΌΠ΅Π½Π΄Π°Ρ†ΠΈΠΈ ΠΏΠΎ Ρ€Π΅ΡˆΠ΅Π½ΠΈΡŽ Π·Π°Π΄Π°Ρ‡ Π½Π° ΡΠ»Π΅ΠΊΡ‚Ρ€ΠΎΠΌΠ°Π³Π½ΠΈΡ‚Π½ΡƒΡŽ ΠΈΠ½Π΄ΡƒΠΊΡ†ΠΈΡŽ

Β«ΠŸΡ€Π΅Π²Ρ€Π°Ρ‚ΠΈΡ‚ΡŒ ΠΌΠ°Π³Π½Π΅Ρ‚ΠΈΠ·ΠΌ Π² элСктричСство…»

Майкл Π€Π°Ρ€Π°Π΄Π΅ΠΉ

Данная Ρ‚Π΅ΠΌΠ° Π±ΡƒΠ΄Π΅Ρ‚ посвящСна Ρ€Π°ΡΡΠΌΠΎΡ‚Ρ€Π΅Π½ΠΈΡŽ основных Ρ„ΠΎΡ€ΠΌΡƒΠ» ΠΈ мСтодичСских Ρ€Π΅ΠΊΠΎΠΌΠ΅Π½Π΄Π°Ρ†ΠΈΠΉ ΠΏΠΎ Ρ€Π΅ΡˆΠ΅Π½ΠΈΡŽ Π·Π°Π΄Π°Ρ‡ Π½Π° ΡΠ»Π΅ΠΊΡ‚Ρ€ΠΎΠΌΠ°Π³Π½ΠΈΡ‚Π½ΡƒΡŽ ΠΈΠ½Π΄ΡƒΠΊΡ†ΠΈΡŽ

Рассмотрим основныС понятия элСктромагнитной ΠΈΠ½Π΄ΡƒΠΊΡ†ΠΈΠΈ. ΠœΠ°Π³Π½ΠΈΡ‚Π½Ρ‹ΠΉ ΠΏΠΎΡ‚ΠΎΠΊ – это скалярная физичСская Π²Π΅Π»ΠΈΡ‡ΠΈΠ½Π°, числСнно равная ΠΏΡ€ΠΎΠΈΠ·Π²Π΅Π΄Π΅Π½ΠΈΡŽ модуля Π²Π΅ΠΊΡ‚ΠΎΡ€Π° ΠΌΠ°Π³Π½ΠΈΡ‚Π½ΠΎΠΉ ΠΈΠ½Π΄ΡƒΠΊΡ†ΠΈΠΈ Π½Π° ΠΏΠ»ΠΎΡ‰Π°Π΄ΡŒ повСрхности, ΠΎΠ³Ρ€Π°Π½ΠΈΡ‡Π΅Π½Π½ΠΎΠΉ ΠΊΠΎΠ½Ρ‚ΡƒΡ€ΠΎΠΌ, ΠΈ Π½Π° косинус ΡƒΠ³Π»Π° ΠΌΠ΅ΠΆΠ΄Ρƒ Π½ΠΎΡ€ΠΌΠ°Π»ΡŒΡŽ ΠΊ повСрхности ΠΈ Π½Π°ΠΏΡ€Π°Π²Π»Π΅Π½ΠΈΠ΅ΠΌ Π»ΠΈΠ½ΠΈΠΉ ΠΌΠ°Π³Π½ΠΈΡ‚Π½ΠΎΠΉ ΠΈΠ½Π΄ΡƒΠΊΡ†ΠΈΠ΅ΠΉ.

ИзмСнСниС ΠΌΠ°Π³Π½ΠΈΡ‚Π½ΠΎΠ³ΠΎ ΠΏΠΎΡ‚ΠΎΠΊΠ° Π²Π»Π΅Ρ‡Π΅Ρ‚ Π·Π° собой Ρ‚Π°ΠΊΠΎΠ΅ явлСниС, ΠΊΠ°ΠΊ элСктромагнитная индукция. Π§Π΅ΠΌ быстрСС измСняСтся ΠΌΠ°Π³Π½ΠΈΡ‚Π½Ρ‹ΠΉ ΠΏΠΎΡ‚ΠΎΠΊ, Ρ‚Π΅ΠΌ большая сила Ρ‚ΠΎΠΊΠ° Π²ΠΎΠ·Π½ΠΈΠΊΠ°Π΅Ρ‚ Π² Π·Π°ΠΌΠΊΠ½ΡƒΡ‚ΠΎΠΌ ΠΊΠΎΠ½Ρ‚ΡƒΡ€Π΅.

Π’ Ρ€Π΅Π·ΡƒΠ»ΡŒΡ‚Π°Ρ‚Π΅ явлСния элСктромагнитной ΠΈΠ½Π΄ΡƒΠΊΡ†ΠΈΠΈ, Π² ΠΊΠΎΠ½Ρ‚ΡƒΡ€Π΅ Π²ΠΎΠ·Π½ΠΈΠΊΠ°Π΅Ρ‚ элСктродвиТущая сила – ΠΎΠ½Π° Ρ‚Π°ΠΊ ΠΈ называСтся

Π­Π”Π‘ ΠΈΠ½Π΄ΡƒΠΊΡ†ΠΈΠΈ.

ΠŸΠΎΡΠΊΠΎΠ»ΡŒΠΊΡƒ сила Ρ‚ΠΎΠΊΠ° связана с ΠΈΠ½Π΄ΡƒΠΊΡ†ΠΈΠ΅ΠΉ ΠΏΠΎΡ€ΠΎΠΆΠ΄Π°Π΅ΠΌΠΎΠ³ΠΎ ΠΈΠΌ ΠΌΠ°Π³Π½ΠΈΡ‚Π½ΠΎΠ³ΠΎ поля, Π° магнитная индукция, Π² свою ΠΎΡ‡Π΅Ρ€Π΅Π΄ΡŒ, связана с ΠΌΠ°Π³Π½ΠΈΡ‚Π½Ρ‹ΠΌ ΠΏΠΎΡ‚ΠΎΠΊΠΎΠΌ, Π²ΠΎΠ·Π½ΠΈΠΊΠ°Π΅Ρ‚ явлСниС самоиндукции. Бамоиндукция – это явлСниС возникновСния Π­Π”Π‘ ΠΈΠ½Π΄ΡƒΠΊΡ†ΠΈΠΈ Π² проводящСм ΠΊΠΎΠ½Ρ‚ΡƒΡ€Π΅ ΠΏΡ€ΠΈ ΠΈΠ·ΠΌΠ΅Π½Π΅Π½ΠΈΠΈ ΠΏΡ€ΠΎΡ‚Π΅ΠΊΠ°ΡŽΡ‰Π΅Π³ΠΎ Ρ‡Π΅Ρ€Π΅Π· ΠΊΠΎΠ½Ρ‚ΡƒΡ€ Ρ‚ΠΎΠΊΠ°. Π’ΠΎ Π΅ΡΡ‚ΡŒ, ΠΏΡ€ΠΈ ΠΈΠ·ΠΌΠ΅Π½Π΅Π½ΠΈΠΈ силы Ρ‚ΠΎΠΊΠ°, Π² Ρ†Π΅ΠΏΠΈ Π²ΠΎΠ·Π½ΠΈΠΊΠ°Π΅Ρ‚ ΠΈΠ½Π΄ΡƒΠΊΡ†ΠΈΠΎΠ½Π½Ρ‹ΠΉ Ρ‚ΠΎΠΊ, ΠΊΠΎΡ‚ΠΎΡ€Ρ‹ΠΉ стрСмится ΠΏΡ€Π΅ΠΏΡΡ‚ΡΡ‚Π²ΠΎΠ²Π°Ρ‚ΡŒ этому измСнСнию. Π’ связи с этим, вводится такая Π²Π΅Π»ΠΈΡ‡ΠΈΠ½Π°, ΠΊΠ°ΠΊ ΠΈΠ½Π΄ΡƒΠΊΡ‚ΠΈΠ²Π½ΠΎΡΡ‚ΡŒ – коэффициСнт ΠΏΡ€ΠΎΠΏΠΎΡ€Ρ†ΠΈΠΎΠ½Π°Π»ΡŒΠ½ΠΎΡΡ‚ΠΈ ΠΌΠ΅ΠΆΠ΄Ρƒ элСктричСским Ρ‚ΠΎΠΊΠΎΠΌ, Ρ‚Π΅ΠΊΡƒΡ‰ΠΈΠΌ Π² ΠΊΠ°ΠΊΠΎΠΌ-Π»ΠΈΠ±ΠΎ Π·Π°ΠΌΠΊΠ½ΡƒΡ‚ΠΎΠΌ ΠΊΠΎΠ½Ρ‚ΡƒΡ€Π΅, ΠΈ ΠΌΠ°Π³Π½ΠΈΡ‚Π½Ρ‹ΠΌ ΠΏΠΎΡ‚ΠΎΠΊΠΎΠΌ, создаваСмым этим Ρ‚ΠΎΠΊΠΎΠΌ Ρ‡Π΅Ρ€Π΅Π· ΠΏΠΎΠ²Π΅Ρ€Ρ…Π½ΠΎΡΡ‚ΡŒ, ΠΊΡ€Π°Π΅ΠΌ ΠΊΠΎΡ‚ΠΎΡ€ΠΎΠΉ являСтся этот ΠΊΠΎΠ½Ρ‚ΡƒΡ€. Π˜Π½Ρ‹ΠΌΠΈ словами, ΠΈΠ½Π΄ΡƒΠΊΡ‚ΠΈΠ²Π½ΠΎΡΡ‚ΡŒ Ρ…Π°Ρ€Π°ΠΊΡ‚Π΅Ρ€ΠΈΠ·ΡƒΠ΅Ρ‚ ΡΠΏΠΎΡΠΎΠ±Π½ΠΎΡΡ‚ΡŒ ΠΏΡ€ΠΎΠ²ΠΎΠ΄Π½ΠΈΠΊΠ° Π²Π»ΠΈΡΡ‚ΡŒ Π½Π° быстроту установлСния Ρ‚ΠΎΠΊΠ° Π² Ρ†Π΅ΠΏΠΈ. Она, ΠΊΠΎΠ½Π΅Ρ‡Π½ΠΎ, ΠΎΠ±Π½Π°Ρ€ΡƒΠΆΠΈΠ²Π°Π΅Ρ‚ сСбя Ρ‚ΠΎΠ»ΡŒΠΊΠΎ ΠΏΡ€ΠΈ ΠΈΠ·ΠΌΠ΅Π½Π΅Π½ΠΈΠΈ силы Ρ‚ΠΎΠΊΠ° Π² Ρ†Π΅ΠΏΠΈ.

Π‘Π²Π΅Π΄Ρ‘ΠΌ Π² Ρ‚Π°Π±Π»ΠΈΡ†Ρƒ основныС Ρ„ΠΎΡ€ΠΌΡƒΠ»Ρ‹ ΠΏΠΎ рассматриваСмой Ρ‚Π΅ΠΌΠ΅.

Π€ΠΎΡ€ΠΌΡƒΠ»Π°

ОписаниС Ρ„ΠΎΡ€ΠΌΡƒΠ»Ρ‹

ΠœΠ°Π³Π½ΠΈΡ‚Π½Ρ‹ΠΉ ΠΏΠΎΡ‚ΠΎΠΊ Ρ‡Π΅Ρ€Π΅Π· ΠΊΠΎΠ½Ρ‚ΡƒΡ€ ΠΏΠ»ΠΎΡ‰Π°Π΄ΡŒΡŽ

S, Π³Π΄Π΅ B – ΠΌΠΎΠ΄ΡƒΠ»ΡŒ Π²Π΅ΠΊΡ‚ΠΎΡ€Π° ΠΌΠ°Π³Π½ΠΈΡ‚Π½ΠΎΠΉ ΠΈΠ½Π΄ΡƒΠΊΡ†ΠΈΠΈ, a – ΡƒΠ³ΠΎΠ» ΠΌΠ΅ΠΆΠ΄Ρƒ Π½Π°ΠΏΡ€Π°Π²Π»Π΅Π½ΠΈΠ΅ΠΌ Π²Π΅ΠΊΡ‚ΠΎΡ€Π° ΠΌΠ°Π³Π½ΠΈΡ‚Π½ΠΎΠΉ ΠΈΠ½Π΄ΡƒΠΊΡ†ΠΈΠΈ ΠΈ Π½ΠΎΡ€ΠΌΠ°Π»ΡŒΡŽ ΠΊ плоскости ΠΊΠΎΠ½Ρ‚ΡƒΡ€Π°.

Π­Π”Π‘ ΠΈΠ½Π΄ΡƒΠΊΡ†ΠΈΠΈ, Π²ΠΎΠ·Π½ΠΈΠΊΠ°ΡŽΡ‰Π°Ρ Π² ΠΊΠΎΠ½Ρ‚ΡƒΡ€Π΅ ΠΏΡ€ΠΈ ΠΈΠ·ΠΌΠ΅Π½Π΅Π½ΠΈΠΈ ΠΌΠ°Π³Π½ΠΈΡ‚Π½ΠΎΠ³ΠΎ ΠΏΠΎΡ‚ΠΎΠΊΠ° Π½Π° Π²Π΅Π»ΠΈΡ‡ΠΈΠ½Ρƒ DF Π·Π° ΠΏΡ€ΠΎΠΌΠ΅ΠΆΡƒΡ‚ΠΎΠΊ Π²Ρ€Π΅ΠΌΠ΅Π½ΠΈ Dt.

Π­Π”Π‘ ΠΈΠ½Π΄ΡƒΠΊΡ†ΠΈΠΈ, Π²ΠΎΠ·Π½ΠΈΠΊΠ°ΡŽΡ‰Π°Ρ Π² двиТущСмся со ΡΠΊΠΎΡ€ΠΎΡΡ‚ΡŒΡŽ Β ΠΏΡ€ΠΎΠ²ΠΎΠ΄Π½ΠΈΠΊΠ΅ Π΄Π»ΠΈΠ½ΠΎΠΉ , Π³Π΄Π΅ a – ΡƒΠ³ΠΎΠ» ΠΌΠ΅ΠΆΠ΄Ρƒ Π½Π°ΠΏΡ€Π°Π²Π»Π΅Π½ΠΈΠ΅ΠΌ Π²Π΅ΠΊΡ‚ΠΎΡ€Π° ΠΌΠ°Π³Π½ΠΈΡ‚Π½ΠΎΠΉ ΠΈΠ½Π΄ΡƒΠΊΡ†ΠΈΠΈ ΠΈ Π½Π°ΠΏΡ€Π°Π²Π»Π΅Π½ΠΈΠ΅ΠΌ Π²Π΅ΠΊΡ‚ΠΎΡ€Π° скорости.

ΠšΠΎΡΡ„Ρ„ΠΈΡ†ΠΈΠ΅Π½Ρ‚ самоиндукции (ΠΈΠ½Π΄ΡƒΠΊΡ‚ΠΈΠ²Π½ΠΎΡΡ‚ΡŒ) ΠΊΠΎΠ½Ρ‚ΡƒΡ€Π°.

Π­Π”Π‘ самоиндукции, Π²ΠΎΠ·Π½ΠΈΠΊΠ°ΡŽΡ‰Π°Ρ Π² ΠΊΠΎΠ½Ρ‚ΡƒΡ€Π΅ ΠΏΡ€ΠΈ ΠΈΠ·ΠΌΠ΅Π½Π΅Π½ΠΈΠΈ силы Ρ‚ΠΎΠΊΠ° Π½Π° Π²Π΅Π»ΠΈΡ‡ΠΈΠ½Ρƒ DI Π·Π° ΠΏΡ€ΠΎΠΌΠ΅ΠΆΡƒΡ‚ΠΎΠΊ Π²Ρ€Π΅ΠΌΠ΅Π½ΠΈ Dt.

Π˜Π½Π΄ΡƒΠΊΡ‚ΠΈΠ²Π½ΠΎΡΡ‚ΡŒ солСноида ΠΎΠ±ΡŠΡ‘ΠΌΠΎΠΌ V, Π³Π΄Π΅ m – магнитная ΠΏΡ€ΠΎΠ½ΠΈΡ†Π°Π΅ΠΌΠΎΡΡ‚ΡŒ срСды, m0 – магнитная постоянная Β Π“Π½/ΠΌ,

n – число Π²ΠΈΡ‚ΠΊΠΎΠ² Π½Π° Π΅Π΄ΠΈΠ½ΠΈΡ†Ρƒ Π΄Π»ΠΈΠ½Ρ‹.

ЭнСргия ΠΌΠ°Π³Π½ΠΈΡ‚Π½ΠΎΠ³ΠΎ поля ΠΊΠ°Ρ‚ΡƒΡˆΠΊΠΈ с ΠΈΠ½Π΄ΡƒΠΊΡ‚ΠΈΠ²Π½ΠΎΡΡ‚ΡŒΡŽ L, Π³Π΄Π΅ I – сила Ρ‚ΠΎΠΊΠ°, F – ΠΌΠ°Π³Π½ΠΈΡ‚Π½Ρ‹ΠΉ ΠΏΠΎΡ‚ΠΎΠΊ.

ЭнСргия ΠΌΠ°Π³Π½ΠΈΡ‚Π½ΠΎΠ³ΠΎ поля солСноида ΠΎΠ±ΡŠΡ‘ΠΌΠΎΠΌ V, Π³Π΄Π΅ B β€” ΠΌΠΎΠ΄ΡƒΠ»ΡŒ Π²Π΅ΠΊΡ‚ΠΎΡ€Π° ΠΌΠ°Π³Π½ΠΈΡ‚Π½ΠΎΠΉ ΠΈΠ½Π΄ΡƒΠΊΡ†ΠΈΠΈ.

ΠœΠ΅Ρ‚ΠΎΠ΄ΠΈΡ‡Π΅ΡΠΊΠΈΠ΅ Ρ€Π΅ΠΊΠΎΠΌΠ΅Π½Π΄Π°Ρ†ΠΈΠΈ ΠΏΠΎ Ρ€Π΅ΡˆΠ΅Π½ΠΈΡŽ Π·Π°Π΄Π°Ρ‡ Π½Π° ΡΠ»Π΅ΠΊΡ‚Ρ€ΠΎΠΌΠ°Π³Π½ΠΈΡ‚Π½ΡƒΡŽ ΠΈΠ½Π΄ΡƒΠΊΡ†ΠΈΡŽ

1. Π£ΡΡ‚Π°Π½ΠΎΠ²ΠΈΡ‚ΡŒ ΠΏΡ€ΠΈΡ‡ΠΈΠ½Ρƒ измСнСния ΠΌΠ°Π³Π½ΠΈΡ‚Π½ΠΎΠ³ΠΎ ΠΏΠΎΡ‚ΠΎΠΊΠ° Ρ‡Π΅Ρ€Π΅Π· ΠΊΠΎΠ½Ρ‚ΡƒΡ€. Π˜ΡΡ…ΠΎΠ΄Ρ ΠΈΠ· Ρ„ΠΎΡ€ΠΌΡƒΠ»Ρ‹, ΠΏΡ€ΠΈΡ‡ΠΈΠ½ΠΎΠΉ ΠΌΠΎΠΆΠ΅Ρ‚ ΡΡ‚Π°Ρ‚ΡŒ Π»ΠΈΠ±ΠΎ ΠΈΠ·ΠΌΠ΅Π½Π΅Π½ΠΈΠ΅ ΠΌΠ°Π³Π½ΠΈΡ‚Π½ΠΎΠΉ ΠΈΠ½Π΄ΡƒΠΊΡ†ΠΈΠΈ поля, Π»ΠΈΠ±ΠΎ ΠΈΠ·ΠΌΠ΅Π½Π΅Π½ΠΈΠ΅ ΠΏΠ»ΠΎΡ‰Π°Π΄ΠΈ ΠΊΠΎΠ½Ρ‚ΡƒΡ€Π°, Π° Ρ‚Π°ΠΊΠΆΠ΅ ΡƒΠ³Π»Π° ΠΌΠ΅ΠΆΠ΄Ρƒ Π½Π°ΠΏΡ€Π°Π²Π»Π΅Π½ΠΈΠ΅ΠΌ Π»ΠΈΠ½ΠΈΠΉ ΠΌΠ°Π³Π½ΠΈΡ‚Π½ΠΎΠΉ ΠΈΠ½Π΄ΡƒΠΊΡ†ΠΈΠΈ ΠΈ Π½ΠΎΡ€ΠΌΠ°Π»ΡŒΡŽ ΠΊ плоскости ΠΊΠΎΠ½Ρ‚ΡƒΡ€Π° (Ρ‡Π°Ρ‰Π΅ всСго, это ΠΏΠΎΠ²ΠΎΡ€ΠΎΡ‚ Ρ€Π°ΠΌΠΊΠΈ с Ρ‚ΠΎΠΊΠΎΠΌ).

2. Π—Π°ΠΏΠΈΡΠ°Ρ‚ΡŒ Π·Π°ΠΊΠΎΠ½ элСктромагнитной ΠΈΠ½Π΄ΡƒΠΊΡ†ΠΈΠΈ (Π·Π°ΠΊΠΎΠ½ ЀарадСя).

3. Если Ρ€Π΅Ρ‡ΡŒ ΠΈΠ΄Π΅Ρ‚ ΠΎ ΠΏΠΎΡΡ‚ΡƒΠΏΠ°Ρ‚Π΅Π»ΡŒΠ½ΠΎΠΌ Π΄Π²ΠΈΠΆΠ΅Π½ΠΈΠΈ ΠΏΡ€ΠΎΠ²ΠΎΠ΄Π½ΠΈΠΊΠ°, ΠΏΡ€ΠΈΠΌΠ΅Π½ΠΈΡ‚ΡŒ Ρ„ΠΎΡ€ΠΌΡƒΠ»Ρƒ, ΠΏΠΎ ΠΊΠΎΡ‚ΠΎΡ€ΠΎΠΉ вычисляСтся Π­Π”Π‘ ΠΈΠ½Π΄ΡƒΠΊΡ†ΠΈΠΈ Π² двиТущСмся ΠΏΡ€ΠΎΠ²ΠΎΠ΄Π½ΠΈΠΊΠ΅.

4. ΠžΠΏΡ€Π΅Π΄Π΅Π»ΠΈΡ‚ΡŒ ΠΈΠ·ΠΌΠ΅Π½Π΅Π½ΠΈΠ΅ ΠΌΠ°Π³Π½ΠΈΡ‚Π½ΠΎΠ³ΠΎ ΠΏΠΎΡ‚ΠΎΠΊΠ°, рассматривая Π΅Π³ΠΎ Π² Π²Ρ‹Π±Ρ€Π°Π½Π½Ρ‹Π΅ ΠΌΠΎΠΌΠ΅Π½Ρ‚Ρ‹ Π²Ρ€Π΅ΠΌΠ΅Π½ΠΈ t1 ΠΈ t2 (ΠΊΠ°ΠΊ ΠΏΡ€Π°Π²ΠΈΠ»ΠΎ, это Π΄ΠΎΠ»ΠΆΠ½Ρ‹ Π±Ρ‹Ρ‚ΡŒ Ρ‚Π΅ ΠΌΠΎΠΌΠ΅Π½Ρ‚Ρ‹ Π²Ρ€Π΅ΠΌΠ΅Π½ΠΈ, ΠΊΠΎΡ‚ΠΎΡ€Ρ‹Π΅ ΠΎΠΏΠΈΡΡ‹Π²Π°ΡŽΡ‚ΡΡ Π² Π·Π°Π΄Π°Ρ‡Π΅).

5. ΠŸΠΎΠ΄ΡΡ‚Π°Π²ΠΈΡ‚ΡŒ Π½Π°ΠΉΠ΄Π΅Π½Π½ΠΎΠ΅ Π²Ρ‹Ρ€Π°ΠΆΠ΅Π½ΠΈΠ΅ для измСнСния ΠΌΠ°Π³Π½ΠΈΡ‚Π½ΠΎΠ³ΠΎ ΠΏΠΎΡ‚ΠΎΠΊΠ° Π² Π·Π°ΠΊΠΎΠ½ ЀарадСя. ΠŸΡ€ΠΈ нСобходимости, ΠΈΡΠΏΠΎΠ»ΡŒΠ·ΡƒΡ Π΄ΠΎΠΏΠΎΠ»Π½ΠΈΡ‚Π΅Π»ΡŒΠ½Ρ‹Π΅ уравнСния, ΡΠΎΡΡ‚Π°Π²ΠΈΡ‚ΡŒ систСму ΠΈ Ρ€Π΅ΡˆΠΈΡ‚ΡŒ Π΅Ρ‘ ΠΎΡ‚Π½ΠΎΡΠΈΡ‚Π΅Π»ΡŒΠ½ΠΎ искомых Π²Π΅Π»ΠΈΡ‡ΠΈΠ½.

Π·Π°ΠΊΠΎΠ½ элСктромагнитной ΠΈΠ½Π΄ΡƒΠΊΡ†ΠΈΠΈ β€” Ρ„ΠΎΡ€ΠΌΡƒΠ»Π° явлСния. ΠžΡ‚ Ρ‡Π΅Π³ΠΎ зависят сила ΠΈ Π½Π°ΠΏΡ€Π°Π²Π»Π΅Π½ΠΈΠ΅ ΠΈΠ½Π΄ΡƒΠΊΡ†ΠΈΠΎΠ½Π½ΠΎΠ³ΠΎ Ρ‚ΠΎΠΊΠ°

ΠœΠΠ“ΠΠ˜Π’ΠΠžΠ• ΠŸΠžΠ›Π•

ΠœΠ°Π³Π½ΠΈΡ‚Π½ΠΎΠ΅ взаимодСйствиС двиТущихся элСктричСских зарядов согласно прСдставлСниям Ρ‚Π΅ΠΎΡ€ΠΈΠΈ поля ΠΎΠ±ΡŠΡΡΠ½ΡΠ΅Ρ‚ΡΡ ΡΠ»Π΅Π΄ΡƒΡŽΡ‰ΠΈΠΌ ΠΎΠ±Ρ€Π°Π·ΠΎΠΌ: всякий двиТущийся элСктричСский заряд создаСт Π² ΠΎΠΊΡ€ΡƒΠΆΠ°ΡŽΡ‰Π΅ΠΌ пространствС ΠΌΠ°Π³Π½ΠΈΡ‚Π½ΠΎΠ΅ ΠΏΠΎΠ»Π΅, способноС Π΄Π΅ΠΉΡΡ‚Π²ΠΎΠ²Π°Ρ‚ΡŒ Π½Π° Π΄Ρ€ΡƒΠ³ΠΈΠ΅ двиТущиСся элСктричСскиС заряды.

Π’ – физичСская Π²Π΅Π»ΠΈΡ‡ΠΈΠ½Π°, ΡΠ²Π»ΡΡŽΡ‰Π°ΡΡΡ силовой характСристикой ΠΌΠ°Π³Π½ΠΈΡ‚Π½ΠΎΠ³ΠΎ поля. Она называСтся ΠΌΠ°Π³Π½ΠΈΡ‚Π½ΠΎΠΉ ΠΈΠ½Π΄ΡƒΠΊΡ†ΠΈΠ΅ΠΉ (ΠΈΠ»ΠΈ ΠΈΠ½Π΄ΡƒΠΊΡ†ΠΈΠ΅ΠΉ ΠΌΠ°Π³Π½ΠΈΡ‚Π½ΠΎΠ³ΠΎ поля).

ΠœΠ°Π³Π½ΠΈΡ‚Π½Π°Ρ индукция – вСкторная Π²Π΅Π»ΠΈΡ‡ΠΈΠ½Π°. ΠœΠΎΠ΄ΡƒΠ»ΡŒ Π²Π΅ΠΊΡ‚ΠΎΡ€Π° ΠΌΠ°Π³Π½ΠΈΡ‚Π½ΠΎΠΉ ΠΈΠ½Π΄ΡƒΠΊΡ†ΠΈΠΈ Ρ€Π°Π²Π΅Π½ ΠΎΡ‚Π½ΠΎΡˆΠ΅Π½ΠΈΡŽ максимального значСния силы АмпСра, Π΄Π΅ΠΉΡΡ‚Π²ΡƒΡŽΡ‰Π΅ΠΉ Π½Π° прямой ΠΏΡ€ΠΎΠ²ΠΎΠ΄Π½ΠΈΠΊ с Ρ‚ΠΎΠΊΠΎΠΌ, ΠΊ силС Ρ‚ΠΎΠΊΠ° Π² ΠΏΡ€ΠΎΠ²ΠΎΠ΄Π½ΠΈΠΊΠ΅ ΠΈ Π΅Π³ΠΎ Π΄Π»ΠΈΠ½Π΅:

Π•Π΄ΠΈΠ½ΠΈΡ†Π° ΠΌΠ°Π³Π½ΠΈΡ‚Π½ΠΎΠΉ ΠΈΠ½Π΄ΡƒΠΊΡ†ΠΈΠΈ . Π’ ΠœΠ΅ΠΆΠ΄ΡƒΠ½Π°Ρ€ΠΎΠ΄Π½ΠΎΠΉ систСмС Π΅Π΄ΠΈΠ½ΠΈΡ† Π·Π° Π΅Π΄ΠΈΠ½ΠΈΡ†Ρƒ ΠΌΠ°Π³Π½ΠΈΡ‚Π½ΠΎΠΉ ΠΈΠ½Π΄ΡƒΠΊΡ†ΠΈΠΈ принята индукция Ρ‚Π°ΠΊΠΎΠ³ΠΎ ΠΌΠ°Π³Π½ΠΈΡ‚Π½ΠΎΠ³ΠΎ поля, Π² ΠΊΠΎΡ‚ΠΎΡ€ΠΎΠΌ Π½Π° ΠΊΠ°ΠΆΠ΄Ρ‹ΠΉ ΠΌΠ΅Ρ‚Ρ€ Π΄Π»ΠΈΠ½Ρ‹ ΠΏΡ€ΠΎΠ²ΠΎΠ΄Π½ΠΈΠΊΠ° ΠΏΡ€ΠΈ силС Ρ‚ΠΎΠΊΠ° 1 А дСйствуСт максимальная сила АмпСра 1 Н.

Π­Ρ‚Π° Π΅Π΄ΠΈΠ½ΠΈΡ†Π° называСтся тСсла (сокращСнно: Π’Π»), Π² Ρ‡Π΅ΡΡ‚ΡŒ Π²Ρ‹Π΄Π°ΡŽΡ‰Π΅Π³ΠΎΡΡ югославского Ρ„ΠΈΠ·ΠΈΠΊΠ° Н. ВСсла:

Π‘Π˜Π›Π Π›ΠžΠ Π•ΠΠ¦Π

Π”Π²ΠΈΠΆΠ΅Π½ΠΈΠ΅ ΠΏΡ€ΠΎΠ²ΠΎΠ΄Π½ΠΈΠΊΠ° с Ρ‚ΠΎΠΊΠΎΠΌ Π² ΠΌΠ°Π³Π½ΠΈΡ‚Π½ΠΎΠΌ ΠΏΠΎΠ»Π΅ ΠΏΠΎΠΊΠ°Π·Ρ‹Π²Π°Π΅Ρ‚, Ρ‡Ρ‚ΠΎ ΠΌΠ°Π³Π½ΠΈΡ‚Π½ΠΎΠ΅ ΠΏΠΎΠ»Π΅ дСйствуСт Π½Π° двиТущиСся элСктричСскиС заряды. На ΠΏΡ€ΠΎΠ²ΠΎΠ΄Π½ΠΈΠΊ дСйствуСт сила АмпСра F А = IBlsin a , Π° сила Π›ΠΎΡ€Π΅Π½Ρ†Π° дСйствуСт Π½Π° двиТущийся заряд:

Π³Π΄Π΅ a – ΡƒΠ³ΠΎΠ» ΠΌΠ΅ΠΆΠ΄Ρƒ Π²Π΅ΠΊΡ‚ΠΎΡ€Π°ΠΌΠΈ B ΠΈ v .

Π”Π²ΠΈΠΆΠ΅Π½ΠΈΠ΅ заряТСнных частиц Π² ΠΌΠ°Π³Π½ΠΈΡ‚Π½ΠΎΠΌ ΠΏΠΎΠ»Π΅. Π’ ΠΎΠ΄Π½ΠΎΡ€ΠΎΠ΄Π½ΠΎΠΌ ΠΌΠ°Π³Π½ΠΈΡ‚Π½ΠΎΠΌ ΠΏΠΎΠ»Π΅ Π½Π° Π·Π°Ρ€ΡΠΆΠ΅Π½Π½ΡƒΡŽ частицу, Π΄Π²ΠΈΠΆΡƒΡ‰ΡƒΡŽΡΡ со ΡΠΊΠΎΡ€ΠΎΡΡ‚ΡŒΡŽ пСрпСндикулярно линиям ΠΈΠ½Π΄ΡƒΠΊΡ†ΠΈΠΈ ΠΌΠ°Π³Π½ΠΈΡ‚Π½ΠΎΠ³ΠΎ поля, дСйствуСт сила ΠΌ, постоянная ΠΏΠΎ ΠΌΠΎΠ΄ΡƒΠ»ΡŽ ΠΈ направлСнная пСрпСндикулярно Π²Π΅ΠΊΡ‚ΠΎΡ€Ρƒ скорости.Под дСйствиСм ΠΌΠ°Π³Π½ΠΈΡ‚Π½ΠΎΠΉ силы частица ΠΏΡ€ΠΈΠΎΠ±Ρ€Π΅Ρ‚Π°Π΅Ρ‚ ускорСниС, ΠΌΠΎΠ΄ΡƒΠ»ΡŒ ΠΊΠΎΡ‚ΠΎΡ€ΠΎΠ³ΠΎ Ρ€Π°Π²Π΅Π½:

Π’ ΠΎΠ΄Π½ΠΎΡ€ΠΎΠ΄Π½ΠΎΠΌ ΠΌΠ°Π³Π½ΠΈΡ‚Π½ΠΎΠΌ ΠΏΠΎΠ»Π΅ эта частица двиТСтся ΠΏΠΎ окруТности. Радиус ΠΊΡ€ΠΈΠ²ΠΈΠ·Π½Ρ‹ Ρ‚Ρ€Π°Π΅ΠΊΡ‚ΠΎΡ€ΠΈΠΈ, ΠΏΠΎ ΠΊΠΎΡ‚ΠΎΡ€ΠΎΠΉ двиТСтся частица, опрСдСляСтся ΠΈΠ· условияоткуда слСдуСт,

Радиус ΠΊΡ€ΠΈΠ²ΠΈΠ·Π½Ρ‹ Ρ‚Ρ€Π°Π΅ΠΊΡ‚ΠΎΡ€ΠΈΠΈ являСтся Π²Π΅Π»ΠΈΡ‡ΠΈΠ½ΠΎΠΉ постоянной, ΠΏΠΎΡΠΊΠΎΠ»ΡŒΠΊΡƒ сила, пСрпСндикулярная Π²Π΅ΠΊΡ‚ΠΎΡ€Ρƒ скорости, мСняСтся Ρ‚ΠΎΠ»ΡŒΠΊΠΎ Π΅Π΅ Π½Π°ΠΏΡ€Π°Π²Π»Π΅Π½ΠΈΠ΅, Π½ΠΎ Π½Π΅ ΠΌΠΎΠ΄ΡƒΠ»ΡŒ. А это ΠΈ ΠΎΠ·Π½Π°Ρ‡Π°Π΅Ρ‚, Ρ‡Ρ‚ΠΎ данная траСктория являСтся ΠΎΠΊΡ€ΡƒΠΆΠ½ΠΎΡΡ‚ΡŒΡŽ.

ΠŸΠ΅Ρ€ΠΈΠΎΠ΄ обращСния частицы Π² ΠΎΠ΄Π½ΠΎΡ€ΠΎΠ΄Π½ΠΎΠΌ ΠΌΠ°Π³Π½ΠΈΡ‚Π½ΠΎΠΌ ΠΏΠΎΠ»Π΅ Ρ€Π°Π²Π΅Π½:

ПослСднСС Π²Ρ‹Ρ€Π°ΠΆΠ΅Π½ΠΈΠ΅ ΠΏΠΎΠΊΠ°Π·Ρ‹Π²Π°Π΅Ρ‚, Ρ‡Ρ‚ΠΎ ΠΏΠ΅Ρ€ΠΈΠΎΠ΄ обращСния частицы Π² ΠΎΠ΄Π½ΠΎΡ€ΠΎΠ΄Π½ΠΎΠΌ ΠΌΠ°Π³Π½ΠΈΡ‚Π½ΠΎΠΌ ΠΏΠΎΠ»Π΅ Π½Π΅ зависит ΠΎΡ‚ скорости ΠΈ радиуса Ρ‚Ρ€Π°Π΅ΠΊΡ‚ΠΎΡ€ΠΈΠΈ Π΅Π΅ двиТСния.

Если Π½Π°ΠΏΡ€ΡΠΆΠ΅Π½Π½ΠΎΡΡ‚ΡŒ элСктричСского поля Ρ€Π°Π²Π½Π° Π½ΡƒΠ»ΡŽ, Ρ‚ΠΎ сила Π›ΠΎΡ€Π΅Π½Ρ†Π° Π» Ρ€Π°Π²Π½Π° ΠΌΠ°Π³Π½ΠΈΡ‚Π½ΠΎΠΉ силС ΠΌ:

Π­Π›Π•ΠšΠ’Π ΠžΠœΠΠ“ΠΠ˜Π’ΠΠΠ― Π˜ΠΠ”Π£ΠšΠ¦Π˜Π―

Π―Π²Π»Π΅Π½ΠΈΠ΅ элСктромагнитной ΠΈΠ½Π΄ΡƒΠΊΡ†ΠΈΠΈ ΠΎΡ‚ΠΊΡ€Ρ‹Π» Π€Π°Ρ€Π°Π΄Π΅ΠΉ, ΠΊΠΎΡ‚ΠΎΡ€Ρ‹ΠΉ установил, Ρ‡Ρ‚ΠΎ Π² Π·Π°ΠΌΠΊΠ½ΡƒΡ‚ΠΎΠΌ проводящСм ΠΊΠΎΠ½Ρ‚ΡƒΡ€Π΅ Π²ΠΎΠ·Π½ΠΈΠΊΠ°Π΅Ρ‚ элСктричСский Ρ‚ΠΎΠΊ ΠΏΡ€ΠΈ любом ΠΈΠ·ΠΌΠ΅Π½Π΅Π½ΠΈΠΈ ΠΌΠ°Π³Π½ΠΈΡ‚Π½ΠΎΠ³ΠΎ поля, ΠΏΡ€ΠΎΠ½ΠΈΠ·Ρ‹Π²Π°ΡŽΡ‰Π΅Π³ΠΎ ΠΊΠΎΠ½Ρ‚ΡƒΡ€.

ΠœΠΠ“ΠΠ˜Π’ΠΠ«Π™ ПОВОК

ΠœΠ°Π³Π½ΠΈΡ‚Π½Ρ‹ΠΉ ΠΏΠΎΡ‚ΠΎΠΊ Π€ (ΠΏΠΎΡ‚ΠΎΠΊ ΠΌΠ°Π³Π½ΠΈΡ‚Π½ΠΎΠΉ ΠΈΠ½Π΄ΡƒΠΊΡ†ΠΈΠΈ) Ρ‡Π΅Ρ€Π΅Π· ΠΏΠΎΠ²Π΅Ρ€Ρ…Π½ΠΎΡΡ‚ΡŒ ΠΏΠ»ΠΎΡ‰Π°Π΄ΡŒΡŽ S – Π²Π΅Π»ΠΈΡ‡ΠΈΠ½Π°, равная ΠΏΡ€ΠΎΠΈΠ·Π²Π΅Π΄Π΅Π½ΠΈΡŽ модуля Π²Π΅ΠΊΡ‚ΠΎΡ€Π° ΠΌΠ°Π³Π½ΠΈΡ‚Π½ΠΎΠΉ ΠΈΠ½Π΄ΡƒΠΊΡ†ΠΈΠΈ Π½Π° ΠΏΠ»ΠΎΡ‰Π°Π΄ΡŒ S ΠΈ косинус ΡƒΠ³Π»Π° Π° ΠΌΠ΅ΠΆΠ΄Ρƒ Π²Π΅ΠΊΡ‚ΠΎΡ€ΠΎΠΌ ΠΈ Π½ΠΎΡ€ΠΌΠ°Π»ΡŒΡŽ ΠΊ повСрхности:

Π€=BScos

Π’ БИ Π΅Π΄ΠΈΠ½ΠΈΡ†Π° ΠΌΠ°Π³Π½ΠΈΡ‚Π½ΠΎΠ³ΠΎ ΠΏΠΎΡ‚ΠΎΠΊΠ° 1 Π’Π΅Π±Π΅Ρ€ (Π’Π±) – ΠΌΠ°Π³Π½ΠΈΡ‚Π½Ρ‹ΠΉ ΠΏΠΎΡ‚ΠΎΠΊ Ρ‡Π΅Ρ€Π΅Π· ΠΏΠΎΠ²Π΅Ρ€Ρ…Π½ΠΎΡΡ‚ΡŒ ΠΏΠ»ΠΎΡ‰Π°Π΄ΡŒΡŽ 1 ΠΌ 2 , Ρ€Π°ΡΠΏΠΎΠ»ΠΎΠΆΠ΅Π½Π½ΡƒΡŽ пСрпСндикулярно Π½Π°ΠΏΡ€Π°Π²Π»Π΅Π½ΠΈΡŽ ΠΎΠ΄Π½ΠΎΡ€ΠΎΠ΄Π½ΠΎΠ³ΠΎ ΠΌΠ°Π³Π½ΠΈΡ‚Π½ΠΎΠ³ΠΎ поля, индукция ΠΊΠΎΡ‚ΠΎΡ€ΠΎΠ³ΠΎ Ρ€Π°Π²Π½Π° 1 Π’Π»:

ЭлСктромагнитная индукция -явлСниС возникновСния элСктричСского Ρ‚ΠΎΠΊΠ° Π² Π·Π°ΠΌΠΊΠ½ΡƒΡ‚ΠΎΠΌ проводящСм ΠΊΠΎΠ½Ρ‚ΡƒΡ€Π΅ ΠΏΡ€ΠΈ любом ΠΈΠ·ΠΌΠ΅Π½Π΅Π½ΠΈΠΈ ΠΌΠ°Π³Π½ΠΈΡ‚Π½ΠΎΠ³ΠΎ ΠΏΠΎΡ‚ΠΎΠΊΠ°, ΠΏΡ€ΠΎΠ½ΠΈΠ·Ρ‹Π²Π°ΡŽΡ‰Π΅Π³ΠΎ ΠΊΠΎΠ½Ρ‚ΡƒΡ€.

Π’ΠΎΠ·Π½ΠΈΠΊΠ°ΡŽΡ‰ΠΈΠΉ Π² Π·Π°ΠΌΠΊΠ½ΡƒΡ‚ΠΎΠΌ ΠΊΠΎΠ½Ρ‚ΡƒΡ€Π΅, ΠΈΠ½Π΄ΡƒΠΊΡ†ΠΈΠΎΠ½Π½Ρ‹ΠΉ Ρ‚ΠΎΠΊ ΠΈΠΌΠ΅Π΅Ρ‚ Ρ‚Π°ΠΊΠΎΠ΅ Π½Π°ΠΏΡ€Π°Π²Π»Π΅Π½ΠΈΠ΅, Ρ‡Ρ‚ΠΎ своим ΠΌΠ°Π³Π½ΠΈΡ‚Π½Ρ‹ΠΌ ΠΏΠΎΠ»Π΅ΠΌ противодСйствуСт Ρ‚ΠΎΠΌΡƒ измСнСнию ΠΌΠ°Π³Π½ΠΈΡ‚Π½ΠΎΠ³ΠΎ ΠΏΠΎΡ‚ΠΎΠΊΠ°, ΠΊΠΎΡ‚ΠΎΡ€Ρ‹ΠΌ ΠΎΠ½ Π²Ρ‹Π·Π²Π°Π½ (ΠΏΡ€Π°Π²ΠΈΠ»ΠΎ Π›Π΅Π½Ρ†Π°).

Π—ΠΠšΠžΠ Π­Π›Π•ΠšΠ’Π ΠžΠœΠΠ“ΠΠ˜Π’ΠΠžΠ™ Π˜ΠΠ”Π£ΠšΠ¦Π˜Π˜

ΠžΠΏΡ‹Ρ‚Ρ‹ ЀарадСя ΠΏΠΎΠΊΠ°Π·Π°Π»ΠΈ, Ρ‡Ρ‚ΠΎ сила ΠΈΠ½Π΄ΡƒΠΊΡ†ΠΈΠΎΠ½Π½ΠΎΠ³ΠΎ Ρ‚ΠΎΠΊΠ° I i Π² проводящСм ΠΊΠΎΠ½Ρ‚ΡƒΡ€Π΅ прямо ΠΏΡ€ΠΎΠΏΠΎΡ€Ρ†ΠΈΠΎΠ½Π°Π»ΡŒΠ½Π° скорости измСнСния числа Π»ΠΈΠ½ΠΈΠΉ ΠΌΠ°Π³Π½ΠΈΡ‚Π½ΠΎΠΉ ΠΈΠ½Π΄ΡƒΠΊΡ†ΠΈΠΈ, ΠΏΡ€ΠΎΠ½ΠΈΠ·Ρ‹Π²Π°ΡŽΡ‰ΠΈΡ… ΠΏΠΎΠ²Π΅Ρ€Ρ…Π½ΠΎΡΡ‚ΡŒ, ΠΎΠ³Ρ€Π°Π½ΠΈΡ‡Π΅Π½Π½ΡƒΡŽ этим ΠΊΠΎΠ½Ρ‚ΡƒΡ€ΠΎΠΌ.

ΠŸΠΎΡΡ‚ΠΎΠΌΡƒ сила ΠΈΠ½Π΄ΡƒΠΊΡ†ΠΈΠΎΠ½Π½ΠΎΠ³ΠΎ Ρ‚ΠΎΠΊΠ° ΠΏΡ€ΠΎΠΏΠΎΡ€Ρ†ΠΈΠΎΠ½Π°Π»ΡŒΠ½Π° скорости измСнСния ΠΌΠ°Π³Π½ΠΈΡ‚Π½ΠΎΠ³ΠΎ ΠΏΠΎΡ‚ΠΎΠΊΠ° Ρ‡Π΅Ρ€Π΅Π· ΠΏΠΎΠ²Π΅Ρ€Ρ…Π½ΠΎΡΡ‚ΡŒ, ΠΎΠ³Ρ€Π°Π½ΠΈΡ‡Π΅Π½Π½ΡƒΡŽ ΠΊΠΎΠ½Ρ‚ΡƒΡ€ΠΎΠΌ:

Π˜Π·Π²Π΅ΡΡ‚Π½ΠΎ, Ρ‡Ρ‚ΠΎ Ссли Π² Ρ†Π΅ΠΏΠΈ появился Ρ‚ΠΎΠΊ, это Π·Π½Π°Ρ‡ΠΈΡ‚, Ρ‡Ρ‚ΠΎ Π½Π° свободныС заряды ΠΏΡ€ΠΎΠ²ΠΎΠ΄Π½ΠΈΠΊΠ° Π΄Π΅ΠΉΡΡ‚Π²ΡƒΡŽΡ‚ сторонниС силы. Π Π°Π±ΠΎΡ‚Π° этих сил ΠΏΠΎ ΠΏΠ΅Ρ€Π΅ΠΌΠ΅Ρ‰Π΅Π½ΠΈΡŽ Π΅Π΄ΠΈΠ½ΠΈΡ‡Π½ΠΎΠ³ΠΎ заряда вдоль Π·Π°ΠΌΠΊΠ½ΡƒΡ‚ΠΎΠ³ΠΎ ΠΊΠΎΠ½Ρ‚ΡƒΡ€Π° называСтся элСктродвиТущСй силой (Π­Π”Π‘). НайдСм Π­Π”Π‘ ΠΈΠ½Π΄ΡƒΠΊΡ†ΠΈΠΈ Ξ΅ i .

По Π·Π°ΠΊΠΎΠ½Ρƒ Ома для Π·Π°ΠΌΠΊΠ½ΡƒΡ‚ΠΎΠΉ Ρ†Π΅ΠΏΠΈ

Π’Π°ΠΊ ΠΊΠ°ΠΊ R Π½Π΅ зависит ΠΎΡ‚ , Ρ‚ΠΎ

Π­Π”Π‘ ΠΈΠ½Π΄ΡƒΠΊΡ†ΠΈΠΈ совпадаСт ΠΏΠΎ Π½Π°ΠΏΡ€Π°Π²Π»Π΅Π½ΠΈΡŽ с ΠΈΠ½Π΄ΡƒΠΊΡ†ΠΈΠΎΠ½Π½Ρ‹ΠΌ Ρ‚ΠΎΠΊΠΎΠΌ, Π° этот Ρ‚ΠΎΠΊ Π² соотвСтствии с ΠΏΡ€Π°Π²ΠΈΠ»ΠΎΠΌ Π›Π΅Π½Ρ†Π° Π½Π°ΠΏΡ€Π°Π²Π»Π΅Π½ Ρ‚Π°ΠΊ, Ρ‡Ρ‚ΠΎ созданный ΠΈΠΌ ΠΌΠ°Π³Π½ΠΈΡ‚Π½Ρ‹ΠΉ ΠΏΠΎΡ‚ΠΎΠΊ противодСйствуСт измСнСнию внСшнСго ΠΌΠ°Π³Π½ΠΈΡ‚Π½ΠΎΠ³ΠΎ ΠΏΠΎΡ‚ΠΎΠΊΠ°.

Π—Π°ΠΊΠΎΠ½ элСктромагнитной ΠΈΠ½Π΄ΡƒΠΊΡ†ΠΈΠΈ

Π­Π”Π‘ ΠΈΠ½Π΄ΡƒΠΊΡ†ΠΈΠΈ Π² Π·Π°ΠΌΠΊΠ½ΡƒΡ‚ΠΎΠΌ ΠΊΠΎΠ½Ρ‚ΡƒΡ€Π΅ Ρ€Π°Π²Π½Π° взятой с ΠΏΡ€ΠΎΡ‚ΠΈΠ²ΠΎΠΏΠΎΠ»ΠΎΠΆΠ½Ρ‹ΠΌ Π·Π½Π°ΠΊΠΎΠΌ скорости измСнСния ΠΌΠ°Π³Π½ΠΈΡ‚Π½ΠΎΠ³ΠΎ ΠΏΠΎΡ‚ΠΎΠΊΠ°, ΠΏΡ€ΠΎΠ½ΠΈΠ·Ρ‹Π²Π°ΡŽΡ‰Π΅Π³ΠΎ ΠΊΠΎΠ½Ρ‚ΡƒΡ€:

Π‘ΠΠœΠžΠ˜ΠΠ”Π£ΠšΠ¦Π˜Π―. Π˜ΠΠ”Π£ΠšΠ’Π˜Π’ΠΠžΠ‘Π’Π¬

ΠžΠΏΡ‹Ρ‚ ΠΏΠΎΠΊΠ°Π·Ρ‹Π²Π°Π΅Ρ‚, Ρ‡Ρ‚ΠΎ ΠΌΠ°Π³Π½ΠΈΡ‚Π½Ρ‹ΠΉ ΠΏΠΎΡ‚ΠΎΠΊ Π€ , связанный с ΠΊΠΎΠ½Ρ‚ΡƒΡ€ΠΎΠΌ, прямо ΠΏΡ€ΠΎΠΏΠΎΡ€Ρ†ΠΈΠΎΠ½Π°Π»Π΅Π½ силС Ρ‚ΠΎΠΊΠ° Π² этом ΠΊΠΎΠ½Ρ‚ΡƒΡ€Π΅:

Π€ = L*I .

Π˜Π½Π΄ΡƒΠΊΡ‚ΠΈΠ²Π½ΠΎΡΡ‚ΡŒ ΠΊΠΎΠ½Ρ‚ΡƒΡ€Π° L – коэффициСнт ΠΏΡ€ΠΎΠΏΠΎΡ€Ρ†ΠΈΠΎΠ½Π°Π»ΡŒΠ½ΠΎΡΡ‚ΠΈ ΠΌΠ΅ΠΆΠ΄Ρƒ проходящим ΠΏΠΎ ΠΊΠΎΠ½Ρ‚ΡƒΡ€Ρƒ Ρ‚ΠΎΠΊΠΎΠΌ ΠΈ созданным ΠΈΠΌ ΠΌΠ°Π³Π½ΠΈΡ‚Π½Ρ‹ΠΌ ΠΏΠΎΡ‚ΠΎΠΊΠΎΠΌ.

Π˜Π½Π΄ΡƒΠΊΡ‚ΠΈΠ²Π½ΠΎΡΡ‚ΡŒ ΠΏΡ€ΠΎΠ²ΠΎΠ΄Π½ΠΈΠΊΠ° зависит ΠΎΡ‚ Π΅Π³ΠΎ Ρ„ΠΎΡ€ΠΌΡ‹, Ρ€Π°Π·ΠΌΠ΅Ρ€ΠΎΠ² ΠΈ свойств ΠΎΠΊΡ€ΡƒΠΆΠ°ΡŽΡ‰Π΅ΠΉ срСды.

Бамоиндукция – явлСниС возникновСния Π­Π”Π‘ ΠΈΠ½Π΄ΡƒΠΊΡ†ΠΈΠΈ Π² ΠΊΠΎΠ½Ρ‚ΡƒΡ€Π΅ ΠΏΡ€ΠΈ ΠΈΠ·ΠΌΠ΅Π½Π΅Π½ΠΈΠΈ ΠΌΠ°Π³Π½ΠΈΡ‚Π½ΠΎΠ³ΠΎ ΠΏΠΎΡ‚ΠΎΠΊΠ°, Π²Ρ‹Π·Π²Π°Π½Π½ΠΎΠΌ ΠΈΠ·ΠΌΠ΅Π½Π΅Π½ΠΈΠ΅ΠΌ Ρ‚ΠΎΠΊΠ°, проходящСго Ρ‡Π΅Ρ€Π΅Π· сам ΠΊΠΎΠ½Ρ‚ΡƒΡ€.

Бамоиндукция – частный случай элСктромагнитной ΠΈΠ½Π΄ΡƒΠΊΡ†ΠΈΠΈ.

Π˜Π½Π΄ΡƒΠΊΡ‚ΠΈΠ²Π½ΠΎΡΡ‚ΡŒ – Π²Π΅Π»ΠΈΡ‡ΠΈΠ½Π°, числСнно равная Π­Π”Π‘ самоиндукции, Π²ΠΎΠ·Π½ΠΈΠΊΠ°ΡŽΡ‰Π΅ΠΉ Π² ΠΊΠΎΠ½Ρ‚ΡƒΡ€Π΅ ΠΏΡ€ΠΈ ΠΈΠ·ΠΌΠ΅Π½Π΅Π½ΠΈΠΈ силы Ρ‚ΠΎΠΊΠ° Π² Π½Π΅ΠΌ Π½Π° Π΅Π΄ΠΈΠ½ΠΈΡ†Ρƒ Π·Π° Π΅Π΄ΠΈΠ½ΠΈΡ†Ρƒ Π²Ρ€Π΅ΠΌΠ΅Π½ΠΈ. Π’ БИ Π·Π° Π΅Π΄ΠΈΠ½ΠΈΡ†Ρƒ индуктивности ΠΏΡ€ΠΈΠ½ΠΈΠΌΠ°ΡŽΡ‚ ΠΈΠ½Π΄ΡƒΠΊΡ‚ΠΈΠ²Π½ΠΎΡΡ‚ΡŒ Ρ‚Π°ΠΊΠΎΠ³ΠΎ ΠΏΡ€ΠΎΠ²ΠΎΠ΄Π½ΠΈΠΊΠ°, Π² ΠΊΠΎΡ‚ΠΎΡ€ΠΎΠΌ ΠΏΡ€ΠΈ ΠΈΠ·ΠΌΠ΅Π½Π΅Π½ΠΈΠΈ силы Ρ‚ΠΎΠΊΠ° Π½Π° 1 А Π·Π° 1 с Π²ΠΎΠ·Π½ΠΈΠΊΠ°Π΅Ρ‚ Π­Π”Π‘ самоиндукции 1 Π’. Π­Ρ‚Π° Π΅Π΄ΠΈΠ½ΠΈΡ†Π° называСтся Π³Π΅Π½Ρ€ΠΈ (Π“Π½):

Π­ΠΠ•Π Π“Π˜Π― ΠœΠΠ“ΠΠ˜Π’ΠΠžΠ“Πž ΠŸΠžΠ›Π―

Π―Π²Π»Π΅Π½ΠΈΠ΅ самоиндукции Π°Π½Π°Π»ΠΎΠ³ΠΈΡ‡Π½ΠΎ явлСнию ΠΈΠ½Π΅Ρ€Ρ†ΠΈΠΈ. Π˜Π½Π΄ΡƒΠΊΡ‚ΠΈΠ²Π½ΠΎΡΡ‚ΡŒ ΠΏΡ€ΠΈ ΠΈΠ·ΠΌΠ΅Π½Π΅Π½ΠΈΠΈ Ρ‚ΠΎΠΊΠ° ΠΈΠ³Ρ€Π°Π΅Ρ‚ Ρ‚Ρƒ ΠΆΠ΅ Ρ€ΠΎΠ»ΡŒ, Ρ‡Ρ‚ΠΎ ΠΈ масса ΠΏΡ€ΠΈ ΠΈΠ·ΠΌΠ΅Π½Π΅Π½ΠΈΠΈ скорости Ρ‚Π΅Π»Π°. Аналогом скорости являСтся сила Ρ‚ΠΎΠΊΠ°.

Π—Π½Π°Ρ‡ΠΈΡ‚ ΡΠ½Π΅Ρ€Π³ΠΈΡŽ ΠΌΠ°Π³Π½ΠΈΡ‚Π½ΠΎΠ³ΠΎ поля Ρ‚ΠΎΠΊΠ° ΠΌΠΎΠΆΠ½ΠΎ ΡΡ‡ΠΈΡ‚Π°Ρ‚ΡŒ Π²Π΅Π»ΠΈΡ‡ΠΈΠ½ΠΎΠΉ, ΠΏΠΎΠ΄ΠΎΠ±Π½ΠΎΠΉ кинСтичСской энСргии Ρ‚Π΅Π»Π° :

ΠŸΡ€Π΅Π΄ΠΏΠΎΠ»ΠΎΠΆΠΈΠΌ, Ρ‡Ρ‚ΠΎ послС ΠΎΡ‚ΠΊΠ»ΡŽΡ‡Π΅Π½ΠΈΡ ΠΊΠ°Ρ‚ΡƒΡˆΠΊΠΈ ΠΎΡ‚ источника,Ρ‚ΠΎΠΊ Π² Ρ†Π΅ΠΏΠΈ ΡƒΠ±Ρ‹Π²Π°Π΅Ρ‚ со Π²Ρ€Π΅ΠΌΠ΅Π½Π΅ΠΌ ΠΏΠΎ Π»ΠΈΠ½Π΅ΠΉΠ½ΠΎΠΌΡƒ Π·Π°ΠΊΠΎΠ½Ρƒ.

Π­Π”Π‘ самоиндукции ΠΈΠΌΠ΅Π΅Ρ‚ Π² этом случаС постоянноС Π·Π½Π°Ρ‡Π΅Π½ΠΈΠ΅:

Π³Π΄Π΅ I – Π½Π°Ρ‡Π°Π»ΡŒΠ½ΠΎΠ΅ Π·Π½Π°Ρ‡Π΅Π½ΠΈΠ΅ Ρ‚ΠΎΠΊΠ°, t – ΠΏΡ€ΠΎΠΌΠ΅ΠΆΡƒΡ‚ΠΎΠΊ Π²Ρ€Π΅ΠΌΠ΅Π½ΠΈ, Π·Π° ΠΊΠΎΡ‚ΠΎΡ€Ρ‹ΠΉ сила Ρ‚ΠΎΠΊΠ° ΡƒΠ±Ρ‹Π²Π°Π΅Ρ‚ ΠΎΡ‚ I Π΄ΠΎ 0.

Π—Π° врСмя t Π² Ρ†Π΅ΠΏΠΈ ΠΏΡ€ΠΎΡ…ΠΎΠ΄ΠΈΡ‚ элСктричСский заряд q = I cp t . Π’Π°ΠΊ ΠΊΠ°ΠΊ I cp = (I + 0)/2 = I/2 , Ρ‚ΠΎ q=It/2 . ΠŸΠΎΡΡ‚ΠΎΠΌΡƒ Ρ€Π°Π±ΠΎΡ‚Π° элСктричСского Ρ‚ΠΎΠΊΠ°:

Π­Ρ‚Π° Ρ€Π°Π±ΠΎΡ‚Π° ΡΠΎΠ²Π΅Ρ€ΡˆΠ°Π΅Ρ‚ΡΡ Π·Π° счСт энСргии ΠΌΠ°Π³Π½ΠΈΡ‚Π½ΠΎΠ³ΠΎ поля ΠΊΠ°Ρ‚ΡƒΡˆΠΊΠΈ. Π’Π°ΠΊΠΈΠΌ ΠΎΠ±Ρ€Π°Π·ΠΎΠΌ, снова ΠΏΠΎΠ»ΡƒΡ‡Π°Π΅ΠΌ:

ΠŸΡ€ΠΈΠΌΠ΅Ρ€. ΠžΠΏΡ€Π΅Π΄Π΅Π»ΠΈΡ‚Π΅ ΡΠ½Π΅Ρ€Π³ΠΈΡŽ ΠΌΠ°Π³Π½ΠΈΡ‚Π½ΠΎΠ³ΠΎ поля ΠΊΠ°Ρ‚ΡƒΡˆΠΊΠΈ, Π² ΠΊΠΎΡ‚ΠΎΡ€ΠΎΠΉ ΠΏΡ€ΠΈ Ρ‚ΠΎΠΊΠ΅ 7,5 А ΠΌΠ°Π³Π½ΠΈΡ‚Π½Ρ‹ΠΉ ΠΏΠΎΡ‚ΠΎΠΊ Ρ€Π°Π²Π΅Π½ 2,3*10 -3 Π’Π±. Как измСнится энСргия поля, Ссли сила Ρ‚ΠΎΠΊΠ° ΡƒΠΌΠ΅Π½ΡŒΡˆΠΈΡ‚ΡŒΡΡ Π²Π΄Π²ΠΎΠ΅?

ЭнСргия ΠΌΠ°Π³Π½ΠΈΡ‚Π½ΠΎΠ³ΠΎ поля ΠΊΠ°Ρ‚ΡƒΡˆΠΊΠΈ W 1 = LI 1 2 /2. По ΠΎΠΏΡ€Π΅Π΄Π΅Π»Π΅Π½ΠΈΡŽ, ΠΈΠ½Π΄ΡƒΠΊΡ‚ΠΈΠ²Π½ΠΎΡΡ‚ΡŒ ΠΊΠ°Ρ‚ΡƒΡˆΠΊΠΈ L = Π€/I 1 . Π‘Π»Π΅Π΄ΠΎΠ²Π°Ρ‚Π΅Π»ΡŒΠ½ΠΎ,

Π˜Π½Π΄ΡƒΠΊΡ†ΠΈΠΎΠ½Π½Ρ‹ΠΉ Ρ‚ΠΎΠΊ это Ρ‚Π°ΠΊΠΎΠΉ Ρ‚ΠΎΠΊ, ΠΊΠΎΡ‚ΠΎΡ€Ρ‹ΠΉ Π²ΠΎΠ·Π½ΠΈΠΊΠ°Π΅Ρ‚ Π² Π·Π°ΠΌΠΊΠ½ΡƒΡ‚ΠΎΠΌ проводящСм ΠΊΠΎΠ½Ρ‚ΡƒΡ€Π΅, находящСмся Π² ΠΏΠ΅Ρ€Π΅ΠΌΠ΅Π½Π½ΠΎΠΌ ΠΌΠ°Π³Π½ΠΈΡ‚Π½ΠΎΠΌ ΠΏΠΎΠ»Π΅. Π­Ρ‚ΠΎΡ‚ Ρ‚ΠΎΠΊ ΠΌΠΎΠΆΠ΅Ρ‚ Π²ΠΎΠ·Π½ΠΈΠΊΠ°Ρ‚ΡŒ Π² Π΄Π²ΡƒΡ… случаях. Если имССтся Π½Π΅ΠΏΠΎΠ΄Π²ΠΈΠΆΠ½Ρ‹ΠΉ ΠΊΠΎΠ½Ρ‚ΡƒΡ€, ΠΏΡ€ΠΎΠ½ΠΈΠ·Ρ‹Π²Π°Π΅ΠΌΡ‹ΠΉ ΠΈΠ·ΠΌΠ΅Π½ΡΡŽΡ‰ΠΈΠΌΡΡ ΠΏΠΎΡ‚ΠΎΠΊΠΎΠΌ ΠΌΠ°Π³Π½ΠΈΡ‚Π½ΠΎΠΉ ΠΈΠ½Π΄ΡƒΠΊΡ†ΠΈΠΈ. Π›ΠΈΠ±ΠΎ ΠΊΠΎΠ³Π΄Π° Π² Π½Π΅ΠΈΠ·ΠΌΠ΅Π½Π½ΠΎΠΌ ΠΌΠ°Π³Π½ΠΈΡ‚Π½ΠΎΠΌ ΠΏΠΎΠ»Π΅ двиТСтся проводящий ΠΊΠΎΠ½Ρ‚ΡƒΡ€, Ρ‡Ρ‚ΠΎ Ρ‚Π°ΠΊΠΆΠ΅ Π²Ρ‹Π·Ρ‹Π²Π°Π΅Ρ‚ ΠΈΠ·ΠΌΠ΅Π½Π΅Π½ΠΈΠ΅ ΠΌΠ°Π³Π½ΠΈΡ‚Π½ΠΎΠ³ΠΎ ΠΏΠΎΡ‚ΠΎΠΊΠ° ΠΏΡ€ΠΎΠ½ΠΈΠ·Ρ‹Π²Π°ΡŽΡ‰Π΅Π³ΠΎ ΠΊΠΎΠ½Ρ‚ΡƒΡ€.

Рисунок 1 β€” ΠŸΡ€ΠΎΠ²ΠΎΠ΄Π½ΠΈΠΊ пСрСмСщаСтся Π² Π½Π΅ΠΈΠ·ΠΌΠ΅Π½Π½ΠΎΠΌ ΠΌΠ°Π³Π½ΠΈΡ‚Π½ΠΎΠΌ ΠΏΠΎΠ»Π΅

ΠŸΡ€ΠΈΡ‡ΠΈΠ½ΠΎΠΉ возникновСния ΠΈΠ½Π΄ΡƒΠΊΡ†ΠΈΠΎΠ½Π½ΠΎΠ³ΠΎ Ρ‚ΠΎΠΊΠ° являСтся Π²ΠΈΡ…Ρ€Π΅Π²ΠΎΠ΅ элСктричСскоС ΠΏΠΎΠ»Π΅, ΠΊΠΎΡ‚ΠΎΡ€ΠΎΠ΅ пороТдаСтся ΠΌΠ°Π³Π½ΠΈΡ‚Π½Ρ‹ΠΌ ΠΏΠΎΠ»Π΅ΠΌ. Π­Ρ‚ΠΎ элСктричСскоС ΠΏΠΎΠ»Π΅ дСйствуСт Π½Π° свободныС заряды, находящиСся Π² ΠΏΡ€ΠΎΠ²ΠΎΠ΄Π½ΠΈΠΊΠ΅, ΠΏΠΎΠΌΠ΅Ρ‰Π΅Π½Π½ΠΎΠΌ Π² это Π²ΠΈΡ…Ρ€Π΅Π²ΠΎΠ΅ элСктричСскоС ΠΏΠΎΠ»Π΅.

Рисунок 2 β€” Π²ΠΈΡ…Ρ€Π΅Π²ΠΎΠ΅ элСктричСскоС ΠΏΠΎΠ»Π΅

Π’Π°ΠΊΠΆΠ΅ ΠΌΠΎΠΆΠ½ΠΎ Π²ΡΡ‚Ρ€Π΅Ρ‚ΠΈΡ‚ΡŒ ΠΈ Ρ‚Π°ΠΊΠΎΠ΅ ΠΎΠΏΡ€Π΅Π΄Π΅Π»Π΅Π½ΠΈΠ΅. Π˜Π½Π΄ΡƒΠΊΡ†ΠΈΠΎΠ½Π½Ρ‹ΠΉ Ρ‚ΠΎΠΊ это элСктричСский Ρ‚ΠΎΠΊ, ΠΊΠΎΡ‚ΠΎΡ€Ρ‹ΠΉ Π²ΠΎΠ·Π½ΠΈΠΊΠ°Π΅Ρ‚ вслСдствиС дСйствия элСктромагнитной ΠΈΠ½Π΄ΡƒΠΊΡ†ΠΈΠΈ. Если Π½Π΅ углубляСтся Π² тонкости Π·Π°ΠΊΠΎΠ½Π° элСктромагнитной ΠΈΠ½Π΄ΡƒΠΊΡ†ΠΈΠΈ, Ρ‚ΠΎ Π² Π΄Π²ΡƒΡ… словах Π΅Π΅ ΠΌΠΎΠΆΠ½ΠΎ ΠΎΠΏΠΈΡΠ°Ρ‚ΡŒ Ρ‚Π°ΠΊ. ЭлСктромагнитная индукция это явлСниС Π²ΠΎΠ·Π½ΠΈΠΊΠ½ΠΎΠ²Π΅Π½ΠΈΠ΅ Ρ‚ΠΎΠΊΠ° Π² проводящСм ΠΊΠΎΠ½Ρ‚ΡƒΡ€Π΅ ΠΏΠΎΠ΄ дСйствиС ΠΏΠ΅Ρ€Π΅ΠΌΠ΅Π½Π½ΠΎΠ³ΠΎ ΠΌΠ°Π³Π½ΠΈΡ‚Π½ΠΎΠ³ΠΎ поля.

Π‘ ΠΏΠΎΠΌΠΎΡ‰ΡŒΡŽ этого Π·Π°ΠΊΠΎΠ½Π° ΠΌΠΎΠΆΠ½ΠΎ ΠΎΠΏΡ€Π΅Π΄Π΅Π»ΠΈΡ‚ΡŒ ΠΈ Π²Π΅Π»ΠΈΡ‡ΠΈΠ½Ρƒ ΠΈΠ½Π΄ΡƒΠΊΡ†ΠΈΠΎΠ½Π½ΠΎΠ³ΠΎ Ρ‚ΠΎΠΊΠ°. Π’Π°ΠΊ ΠΊΠ°ΠΊ ΠΎΠ½ Π½Π°ΠΌ Π΄Π°Π΅Ρ‚ Π·Π½Π°Ρ‡Π΅Π½ΠΈΠ΅ Π­Π”Π‘, которая Π²ΠΎΠ·Π½ΠΈΠΊΠ°Π΅Ρ‚ Π² ΠΊΠΎΠ½Ρ‚ΡƒΡ€Π΅ ΠΏΠΎΠ΄ дСйствиС ΠΏΠ΅Ρ€Π΅ΠΌΠ΅Π½Π½ΠΎΠ³ΠΎ ΠΌΠ°Π³Π½ΠΈΡ‚Π½ΠΎΠ³ΠΎ поля.

Π€ΠΎΡ€ΠΌΡƒΠ»Π° 1 β€” Π­Π”Π‘ ΠΈΠ½Π΄ΡƒΠΊΡ†ΠΈΠΈ ΠΌΠ°Π³Π½ΠΈΡ‚Π½ΠΎΠ³ΠΎ поля .

Как Π²ΠΈΠ΄Π½ΠΎ ΠΈΠ· Ρ„ΠΎΡ€ΠΌΡƒΠ»Ρ‹ 1 Π²Π΅Π»ΠΈΡ‡ΠΈΠ½Π° Π­Π”Π‘ ΠΈΠ½Π΄ΡƒΠΊΡ†ΠΈΠΈ, Π° Π·Π½Π°Ρ‡ΠΈΡ‚ ΠΈ ΠΈΠ½Π΄ΡƒΠΊΡ†ΠΈΠΎΠ½Π½ΠΎΠ³ΠΎ Ρ‚ΠΎΠΊΠ° зависит ΠΎΡ‚ скорости измСнСния ΠΌΠ°Π³Π½ΠΈΡ‚Π½ΠΎΠ³ΠΎ ΠΏΠΎΡ‚ΠΎΠΊΠ° ΠΏΡ€ΠΎΠ½ΠΈΠ·Ρ‹Π²Π°ΡŽΡ‰Π΅Π³ΠΎ ΠΊΠΎΠ½Ρ‚ΡƒΡ€. Π’ΠΎ Π΅ΡΡ‚ΡŒ Ρ‡Π΅ΠΌ быстрСС Π±ΡƒΠ΄Π΅Ρ‚ ΠΌΠ΅Π½ΡΡ‚ΡŒΡΡ ΠΌΠ°Π³Π½ΠΈΡ‚Π½Ρ‹ΠΉ ΠΏΠΎΡ‚ΠΎΠΊ, Ρ‚Π΅ΠΌ больший ΠΈΠ½Π΄ΡƒΠΊΡ†ΠΈΠΎΠ½Π½Ρ‹ΠΉ Ρ‚ΠΎΠΊ ΠΌΠΎΠΆΠ½ΠΎ ΠΏΠΎΠ»ΡƒΡ‡ΠΈΡ‚ΡŒ. Π’ случаС, ΠΊΠΎΠ³Π΄Π° ΠΌΡ‹ ΠΈΠΌΠ΅Π΅ΠΌ постоянноС ΠΌΠ°Π³Π½ΠΈΡ‚Π½ΠΎΠ΅ ΠΏΠΎΠ»Π΅, Π² ΠΊΠΎΡ‚ΠΎΡ€ΠΎΠΌ двиТСтся проводящий ΠΊΠΎΠ½Ρ‚ΡƒΡ€, Ρ‚ΠΎ Π²Π΅Π»ΠΈΡ‡ΠΈΠ½Π° Π­Π”Π‘ Π±ΡƒΠ΄Π΅Ρ‚ Π·Π°Π²ΠΈΡΠ΅Ρ‚ΡŒ ΠΎΡ‚ скорости двиТСния ΠΊΠΎΠ½Ρ‚ΡƒΡ€Π°.

Π§Ρ‚ΠΎΠ±Ρ‹ ΠΎΠΏΡ€Π΅Π΄Π΅Π»ΠΈΡ‚ΡŒ Π½Π°ΠΏΡ€Π°Π²Π»Π΅Π½ΠΈΠ΅ ΠΈΠ½Π΄ΡƒΠΊΡ†ΠΈΠΎΠ½Π½ΠΎΠ³ΠΎ Ρ‚ΠΎΠΊΠ° ΠΈΡΠΏΠΎΠ»ΡŒΠ·ΡƒΡŽΡ‚ ΠΏΡ€Π°Π²ΠΈΠ»ΠΎ Π›Π΅Π½Ρ†Π°. ΠšΠΎΡ‚ΠΎΡ€ΠΎΠ΅ гласит Ρ‡Ρ‚ΠΎ, ΠΈΠ½Π΄ΡƒΠΊΡ†ΠΈΠΎΠ½Π½Ρ‹ΠΉ Ρ‚ΠΎΠΊ Π½Π°ΠΏΡ€Π°Π²Π»Π΅Π½ навстрСчу Ρ‚ΠΎΠΌΡƒ Ρ‚ΠΎΠΊΡƒ, ΠΊΠΎΡ‚ΠΎΡ€Ρ‹ΠΉ Π΅Π³ΠΎ Π²Ρ‹Π·Π²Π°Π». ΠžΡ‚ΡΡŽΠ΄Π° ΠΈ Π·Π½Π°ΠΊ минус Π² Ρ„ΠΎΡ€ΠΌΡƒΠ»Π΅ для опрСдСлСния Π­Π”Π‘ ΠΈΠ½Π΄ΡƒΠΊΡ†ΠΈΠΈ.

Π˜Π½Π΄ΡƒΠΊΡ†ΠΈΠΎΠ½Π½Ρ‹ΠΉ Ρ‚ΠΎΠΊ ΠΈΠ³Ρ€Π°Π΅Ρ‚ Π²Π°ΠΆΠ½ΡƒΡŽ Ρ€ΠΎΠ»ΡŒ Π² соврСмСнной элСктротСхникС. НапримСр, ΠΈΠ½Π΄ΡƒΠΊΡ†ΠΈΠΎΠ½Π½Ρ‹ΠΉ Ρ‚ΠΎΠΊ, Π²ΠΎΠ·Π½ΠΈΠΊΠ°ΡŽΡ‰ΠΈΠΉ Π² Ρ€ΠΎΡ‚ΠΎΡ€Π΅ асинхронного двигатСля, взаимодСйствуСт с Ρ‚ΠΎΠΊΠΎΠΌ, ΠΏΠΎΠ΄Π²ΠΎΠ΄ΠΈΠΌΡ‹ΠΌ ΠΎΡ‚ источника питания Π² Π΅Π³ΠΎ статорС, вслСдствиС Ρ‡Π΅Π³ΠΎ Ρ€ΠΎΡ‚ΠΎΡ€ вращаСтся. На этом ΠΏΡ€ΠΈΠ½Ρ†ΠΈΠΏΠ΅ построСны соврСмСнныС элСктродвигатСли.

Рисунок 3 β€” асинхронный Π΄Π²ΠΈΠ³Π°Ρ‚Π΅Π»ΡŒ.

Π’ трансформаторС ΠΆΠ΅ ΠΈΠ½Π΄ΡƒΠΊΡ†ΠΈΠΎΠ½Π½Ρ‹ΠΉ Ρ‚ΠΎΠΊ, Π²ΠΎΠ·Π½ΠΈΠΊΠ°ΡŽΡ‰ΠΈΠΉ Π²ΠΎ Π²Ρ‚ΠΎΡ€ΠΈΡ‡Π½ΠΎΠΉ ΠΎΠ±ΠΌΠΎΡ‚ΠΊΠ΅, ΠΈΡΠΏΠΎΠ»ΡŒΠ·ΡƒΠ΅Ρ‚ΡΡ для питания Ρ€Π°Π·Π»ΠΈΡ‡Π½Ρ‹Ρ… элСктротСхничСских ΠΏΡ€ΠΈΠ±ΠΎΡ€ΠΎΠ². Π’Π΅Π»ΠΈΡ‡ΠΈΠ½Π° этого Ρ‚ΠΎΠΊΠ° ΠΌΠΎΠΆΠ΅Ρ‚ Π±Ρ‹Ρ‚ΡŒ Π·Π°Π΄Π°Π½Π° ΠΏΠ°Ρ€Π°ΠΌΠ΅Ρ‚Ρ€Π°ΠΌΠΈ трансформатора.

Рисунок 4 β€” элСктричСский трансформатор.

И Π½Π°ΠΊΠΎΠ½Π΅Ρ†, ΠΈΠ½Π΄ΡƒΠΊΡ†ΠΈΠΎΠ½Π½Ρ‹Π΅ Ρ‚ΠΎΠΊΠΈ ΠΌΠΎΠ³ΡƒΡ‚ Π²ΠΎΠ·Π½ΠΈΠΊΠ°Ρ‚ΡŒ ΠΈ Π² массивных ΠΏΡ€ΠΎΠ²ΠΎΠ΄Π½ΠΈΠΊΠ°Ρ…. Π­Ρ‚ΠΎ Ρ‚Π°ΠΊ Π½Π°Π·Ρ‹Π²Π°Π΅ΠΌΡ‹Π΅ Ρ‚ΠΎΠΊΠΈ Π€ΡƒΠΊΠΎ. Благодаря ΠΈΠΌ ΠΌΠΎΠΆΠ½ΠΎ ΠΏΡ€ΠΎΠΈΠ·Π²ΠΎΠ΄ΠΈΡ‚ΡŒ ΠΈΠ½Π΄ΡƒΠΊΡ†ΠΈΠΎΠ½Π½ΡƒΡŽ ΠΏΠ»Π°Π²ΠΊΡƒ ΠΌΠ΅Ρ‚Π°Π»Π»ΠΎΠ². Π’ΠΎ Π΅ΡΡ‚ΡŒ Π²ΠΈΡ…Ρ€Π΅Π²Ρ‹Π΅ Ρ‚ΠΎΠΊΠΈ, Ρ‚Π΅ΠΊΡƒΡ‰ΠΈΠ΅ Π² ΠΏΡ€ΠΎΠ²ΠΎΠ΄Π½ΠΈΠΊΠ΅ Π²Ρ‹Π·Ρ‹Π²Π°ΡŽΡ‚ Π΅Π³ΠΎ Ρ€Π°Π·ΠΎΠ³Ρ€Π΅Π². Π’ зависимости ΠΎΡ‚ Π²Π΅Π»ΠΈΡ‡ΠΈΠ½Ρ‹ этих Ρ‚ΠΎΠΊΠΎΠ² ΠΏΡ€ΠΎΠ²ΠΎΠ΄Π½ΠΈΠΊ ΠΌΠΎΠΆΠ΅Ρ‚ Ρ€Π°Π·ΠΎΠ³Ρ€Π΅Π²Π°Ρ‚ΡŒΡΡ Π²Ρ‹ΡˆΠ΅ Ρ‚ΠΎΡ‡ΠΊΠΈ плавлСния.

Рисунок 5 β€” индукционная ΠΏΠ»Π°Π²ΠΊΠ° ΠΌΠ΅Ρ‚Π°Π»Π»ΠΎΠ².

Π˜Ρ‚Π°ΠΊ, ΠΌΡ‹ выяснили, Ρ‡Ρ‚ΠΎ ΠΈΠ½Π΄ΡƒΠΊΡ†ΠΈΠΎΠ½Π½Ρ‹ΠΉ Ρ‚ΠΎΠΊ ΠΌΠΎΠΆΠ΅Ρ‚ ΠΎΠΊΠ°Π·Ρ‹Π²Π°Ρ‚ΡŒ мСханичСскоС, элСктричСскоС ΠΈ Ρ‚Π΅ΠΏΠ»ΠΎΠ²ΠΎΠ΅ дСйствиС. ВсС эти эффСкты повсСмСстно ΠΈΡΠΏΠΎΠ»ΡŒΠ·ΡƒΡŽΡ‚ΡΡ Π² соврСмСнном ΠΌΠΈΡ€Π΅, ΠΊΠ°ΠΊ Π² ΠΏΡ€ΠΎΠΌΡ‹ΡˆΠ»Π΅Π½Π½Ρ‹Ρ… ΠΌΠ°ΡΡˆΡ‚Π°Π±Π°Ρ…, Ρ‚Π°ΠΊ ΠΈ Π½Π° Π±Ρ‹Ρ‚ΠΎΠ²ΠΎΠΌ ΡƒΡ€ΠΎΠ²Π½Π΅.

Если измСнСния ΠΌΠ°Π³Π½ΠΈΡ‚Π½ΠΎΠ³ΠΎ поля Π½Π΅ происходит, Ρ‚ΠΎ Π½Π΅ Π±ΡƒΠ΄Π΅Ρ‚ Π½ΠΈΠΊΠ°ΠΊΠΎΠ³ΠΎ элСктричСского Ρ‚ΠΎΠΊΠ°. Π”Π°ΠΆΠ΅ Ссли ΠΌΠ°Π³Π½ΠΈΡ‚Π½ΠΎΠ΅ ΠΏΠΎΠ»Π΅ сущСствуСт. ΠœΡ‹ ΠΌΠΎΠΆΠ΅ΠΌ ΡΠΊΠ°Π·Π°Ρ‚ΡŒ, Ρ‡Ρ‚ΠΎ ΠΈΠ½Π΄ΡƒΠΊΡ†ΠΈΠΎΠ½Π½Ρ‹ΠΉ элСктричСский Ρ‚ΠΎΠΊ прямо ΠΏΡ€ΠΎΠΏΠΎΡ€Ρ†ΠΈΠΎΠ½Π°Π»Π΅Π½, Π²ΠΎ-ΠΏΠ΅Ρ€Π²Ρ‹Ρ…, числу Π²ΠΈΡ‚ΠΊΠΎΠ², Π²ΠΎ-Π²Ρ‚ΠΎΡ€Ρ‹Ρ…, скорости ΠΌΠ°Π³Π½ΠΈΡ‚Π½ΠΎΠ³ΠΎ поля, с ΠΊΠΎΡ‚ΠΎΡ€ΠΎΠΉ измСняСтся это ΠΌΠ°Π³Π½ΠΈΡ‚Π½ΠΎΠ΅ ΠΏΠΎΠ»Π΅ ΠΎΡ‚Π½ΠΎΡΠΈΡ‚Π΅Π»ΡŒΠ½ΠΎ Π²ΠΈΡ‚ΠΊΠΎΠ² ΠΊΠ°Ρ‚ΡƒΡˆΠΊΠΈ.

Рис. 3. ΠžΡ‚ Ρ‡Π΅Π³ΠΎ зависит Π²Π΅Π»ΠΈΡ‡ΠΈΠ½Π° ΠΈΠ½Π΄ΡƒΠΊΡ†ΠΈΠΎΠ½Π½ΠΎΠ³ΠΎ Ρ‚ΠΎΠΊΠ°?

Для характСристики ΠΌΠ°Π³Π½ΠΈΡ‚Π½ΠΎΠ³ΠΎ поля ΠΈΡΠΏΠΎΠ»ΡŒΠ·ΡƒΠ΅Ρ‚ΡΡ Π²Π΅Π»ΠΈΡ‡ΠΈΠ½Π°, которая называСтся ΠΌΠ°Π³Π½ΠΈΡ‚Π½Ρ‹ΠΉ ΠΏΠΎΡ‚ΠΎΠΊ. Она Ρ…Π°Ρ€Π°ΠΊΡ‚Π΅Ρ€ΠΈΠ·ΡƒΠ΅Ρ‚ ΠΌΠ°Π³Π½ΠΈΡ‚Π½ΠΎΠ΅ ΠΏΠΎΠ»Π΅ Π² Ρ†Π΅Π»ΠΎΠΌ, ΠΌΡ‹ ΠΎΠ± этом Π±ΡƒΠ΄Π΅ΠΌ Π³ΠΎΠ²ΠΎΡ€ΠΈΡ‚ΡŒ Π½Π° ΡΠ»Π΅Π΄ΡƒΡŽΡ‰Π΅ΠΌ ΡƒΡ€ΠΎΠΊΠ΅. БСйчас ΠΎΡ‚ΠΌΠ΅Ρ‚ΠΈΠΌ лишь, Ρ‡Ρ‚ΠΎ ΠΈΠΌΠ΅Π½Π½ΠΎ ΠΈΠ·ΠΌΠ΅Π½Π΅Π½ΠΈΠ΅ ΠΌΠ°Π³Π½ΠΈΡ‚Π½ΠΎΠ³ΠΎ ΠΏΠΎΡ‚ΠΎΠΊΠ°, Ρ‚. Π΅. числа Π»ΠΈΠ½ΠΈΠΉ ΠΌΠ°Π³Π½ΠΈΡ‚Π½ΠΎΠ³ΠΎ поля, ΠΏΡ€ΠΎΠ½ΠΈΠ·Ρ‹Π²Π°ΡŽΡ‰ΠΈΡ… ΠΊΠΎΠ½Ρ‚ΡƒΡ€ с Ρ‚ΠΎΠΊΠΎΠΌ (ΠΊΠ°Ρ‚ΡƒΡˆΠΊΡƒ, Π½Π°ΠΏΡ€ΠΈΠΌΠ΅Ρ€), ΠΏΡ€ΠΈΠ²ΠΎΠ΄ΠΈΡ‚ ΠΊ возникновСнию Π² этом ΠΊΠΎΠ½Ρ‚ΡƒΡ€Π΅ ΠΈΠ½Π΄ΡƒΠΊΡ†ΠΈΠΎΠ½Π½ΠΎΠ³ΠΎ Ρ‚ΠΎΠΊΠ°.

Ѐизика. 9 класс

Π’Π΅ΠΌΠ°: Π­Π»Π΅ΠΊΡ‚Ρ€ΠΎΠΌΠ°Π³Π½ΠΈΡ‚Π½ΠΎΠ΅ ΠΏΠΎΠ»Π΅

Π£Ρ€ΠΎΠΊ 44.ΠœΠ°Π³Π½ΠΈΡ‚Π½Ρ‹ΠΉ ΠΏΠΎΡ‚ΠΎΠΊ

Π•Ρ€ΡŽΡ‚ΠΊΠΈΠ½ Π•.Π‘., ΡƒΡ‡ΠΈΡ‚Π΅Π»ΡŒ Ρ„ΠΈΠ·ΠΈΠΊΠΈ Π²Ρ‹ΡΡˆΠ΅ΠΉ ΠΊΠ°Ρ‚Π΅Π³ΠΎΡ€ΠΈΠΈ Π“ΠžΠ£ БОШ β„–1360

Π’Π²Π΅Π΄Π΅Π½ΠΈΠ΅. ΠžΠΏΡ‹Ρ‚Ρ‹ ЀарадСя

ΠŸΡ€ΠΎΠ΄ΠΎΠ»ΠΆΠ°Ρ ΠΈΠ·ΡƒΡ‡Π΅Π½ΠΈΠ΅ Ρ‚Π΅ΠΌΡ‹ «ЭлСктромагнитная индукция» Π΄Π°Π²Π°ΠΉΡ‚Π΅ ΠΏΠΎΠ΄Ρ€ΠΎΠ±Π½Π΅Π΅ ΠΎΡΡ‚Π°Π½ΠΎΠ²ΠΈΡ‚ΡŒΡΡ Π½Π° Ρ‚Π°ΠΊΠΎΠΌ понятии, ΠΊΠ°ΠΊ ΠΌΠ°Π³Π½ΠΈΡ‚Π½Ρ‹ΠΉ ΠΏΠΎΡ‚ΠΎΠΊ .

Π’Ρ‹ ΡƒΠΆΠ΅ Π·Π½Π°Π΅Ρ‚Π΅, ΠΊΠ°ΠΊ ΠΎΠ±Π½Π°Ρ€ΡƒΠΆΠΈΡ‚ΡŒ явлСниС элСктромагнитной ΠΈΠ½Π΄ΡƒΠΊΡ†ΠΈΠΈ – Ссли Π·Π°ΠΌΠΊΠ½ΡƒΡ‚Ρ‹ΠΉ ΠΏΡ€ΠΎΠ²ΠΎΠ΄Π½ΠΈΠΊ ΠΏΠ΅Ρ€Π΅ΡΠ΅ΠΊΠ°ΡŽΡ‚ ΠΌΠ°Π³Π½ΠΈΡ‚Π½Ρ‹Π΅ Π»ΠΈΠ½ΠΈΠΈ, Π² этом ΠΏΡ€ΠΎΠ²ΠΎΠ΄Π½ΠΈΠΊΠ΅ Π²ΠΎΠ·Π½ΠΈΠΊΠ°Π΅Ρ‚ элСктричСский Ρ‚ΠΎΠΊ. Π’Π°ΠΊΠΎΠΉ Ρ‚ΠΎΠΊ называСтся ΠΈΠ½Π΄ΡƒΠΊΡ†ΠΈΠΎΠ½Π½Ρ‹ΠΌ.

Π’Π΅ΠΏΠ΅Ρ€ΡŒ Π΄Π°Π²Π°ΠΉΡ‚Π΅ обсудим, Π·Π° счСт Ρ‡Π΅Π³ΠΎ образуСтся этот элСктричСский Ρ‚ΠΎΠΊ ΠΈ Ρ‡Ρ‚ΠΎ являСтся Π³Π»Π°Π²Π½Ρ‹ΠΌ для Ρ‚ΠΎΠ³ΠΎ, Ρ‡Ρ‚ΠΎΠ±Ρ‹ этот Ρ‚ΠΎΠΊ появился.

ΠŸΡ€Π΅ΠΆΠ΄Π΅ всСго, обратимся ΠΊ ΠΎΠΏΡ‹Ρ‚Ρƒ ЀарадСя ΠΈ посмотрим Π΅Ρ‰Π΅ Ρ€Π°Π· Π½Π° Π΅Π³ΠΎ Π²Π°ΠΆΠ½Ρ‹Π΅ особСнности.

Π˜Ρ‚Π°ΠΊ, Ρƒ нас Π² Π½Π°Π»ΠΈΡ‡ΠΈΠΈ Π΅ΡΡ‚ΡŒ Π°ΠΌΠΏΠ΅Ρ€ΠΌΠ΅Ρ‚Ρ€, ΠΊΠ°Ρ‚ΡƒΡˆΠΊΠ° с большим числом Π²ΠΈΡ‚ΠΊΠΎΠ², которая Π½Π°ΠΊΠΎΡ€ΠΎΡ‚ΠΊΠΎ ΠΏΡ€ΠΈΠΊΡ€Π΅ΠΏΠ»Π΅Π½Π° ΠΊ этому Π°ΠΌΠΏΠ΅Ρ€ΠΌΠ΅Ρ‚Ρ€Ρƒ.

Π‘Π΅Ρ€Π΅ΠΌ ΠΌΠ°Π³Π½ΠΈΡ‚, ΠΈ Ρ‚ΠΎΡ‡Π½ΠΎ Ρ‚Π°ΠΊ ΠΆΠ΅, ΠΊΠ°ΠΊ Π½Π° ΠΏΡ€Π΅Π΄Ρ‹Π΄ΡƒΡ‰Π΅ΠΌ ΡƒΡ€ΠΎΠΊΠ΅, опускаСм этот ΠΌΠ°Π³Π½ΠΈΡ‚ Π²Π½ΡƒΡ‚Ρ€ΡŒ ΠΊΠ°Ρ‚ΡƒΡˆΠΊΠΈ. Π‘Ρ‚Ρ€Π΅Π»ΠΊΠ° отклоняСтся, Ρ‚ΠΎ Π΅ΡΡ‚ΡŒ Π² Π΄Π°Π½Π½ΠΎΠΉ Ρ†Π΅ΠΏΠΈ сущСствуСт элСктричСский Ρ‚ΠΎΠΊ.

Рис. 1. ΠžΠΏΡ‹Ρ‚ ΠΏΠΎ ΠΎΠ±Π½Π°Ρ€ΡƒΠΆΠ΅Π½ΠΈΡŽ ΠΈΠ½Π΄ΡƒΠΊΡ†ΠΈΠΎΠ½Π½ΠΎΠ³ΠΎ Ρ‚ΠΎΠΊΠ°.

А Π²ΠΎΡ‚ ΠΊΠΎΠ³Π΄Π° ΠΌΠ°Π³Π½ΠΈΡ‚ находится Π²Π½ΡƒΡ‚Ρ€ΠΈ ΠΊΠ°Ρ‚ΡƒΡˆΠΊΠΈ элСктричСского Ρ‚ΠΎΠΊΠ° Π² Ρ†Π΅ΠΏΠΈ Π½Π΅Ρ‚. Но стоит Ρ‚ΠΎΠ»ΡŒΠΊΠΎ ΠΏΠΎΠΏΡ‹Ρ‚Π°Ρ‚ΡŒΡΡ этот ΠΌΠ°Π³Π½ΠΈΡ‚ Π΄ΠΎΡΡ‚Π°Ρ‚ΡŒ ΠΈΠ· ΠΊΠ°Ρ‚ΡƒΡˆΠΊΠΈ, ΠΊΠ°ΠΊ Π² Ρ†Π΅ΠΏΠΈ вновь появляСтся элСктричСский Ρ‚ΠΎΠΊ, Π½ΠΎ Π½Π°ΠΏΡ€Π°Π²Π»Π΅Π½ΠΈΠ΅ этого Ρ‚ΠΎΠΊΠ° измСняСтся Π½Π° ΠΏΡ€ΠΎΡ‚ΠΈΠ²ΠΎΠΏΠΎΠ»ΠΎΠΆΠ½ΠΎΠ΅.

ΠžΠ±Ρ€Π°Ρ‚ΠΈΡ‚Π΅ Π²Π½ΠΈΠΌΠ°Π½ΠΈΠ΅ Ρ‚Π°ΠΊΠΆΠ΅ Π½Π° Ρ‚ΠΎ, Ρ‡Ρ‚ΠΎ Π·Π½Π°Ρ‡Π΅Π½ΠΈΠ΅ элСктричСского Ρ‚ΠΎΠΊΠ°, ΠΊΠΎΡ‚ΠΎΡ€Ρ‹ΠΉ ΠΏΡ€ΠΎΡ‚Π΅ΠΊΠ°Π΅Ρ‚ Π² Ρ†Π΅ΠΏΠΈ, зависит Π΅Ρ‰Π΅ ΠΈ ΠΎΡ‚ свойств самого ΠΌΠ°Π³Π½ΠΈΡ‚Π°. Если Π²Π·ΡΡ‚ΡŒ Π΄Ρ€ΡƒΠ³ΠΎΠΉ ΠΌΠ°Π³Π½ΠΈΡ‚ ΠΈ ΠΏΡ€ΠΎΠ΄Π΅Π»Π°Ρ‚ΡŒ Ρ‚ΠΎΡ‚ ΠΆΠ΅ экспСримСнт, Π·Π½Π°Ρ‡Π΅Π½ΠΈΠ΅ Ρ‚ΠΎΠΊΠ° сущСствСнно мСняСтся, Π² Π΄Π°Π½Π½ΠΎΠΌ случаС Ρ‚ΠΎΠΊ становится мСньшС.

ΠŸΡ€ΠΎΠ²Π΅Π΄Ρ экспСримСнты, ΠΌΠΎΠΆΠ½ΠΎ ΡΠ΄Π΅Π»Π°Ρ‚ΡŒ Π²Ρ‹Π²ΠΎΠ΄ ΠΎ Ρ‚ΠΎΠΌ, Ρ‡Ρ‚ΠΎ элСктричСский Ρ‚ΠΎΠΊ, ΠΊΠΎΡ‚ΠΎΡ€Ρ‹ΠΉ Π²ΠΎΠ·Π½ΠΈΠΊΠ°Π΅Ρ‚ Π² Π·Π°ΠΌΠΊΠ½ΡƒΡ‚ΠΎΠΌ ΠΏΡ€ΠΎΠ²ΠΎΠ΄Π½ΠΈΠΊΠ΅ (Π² ΠΊΠ°Ρ‚ΡƒΡˆΠΊΠ΅), связан с ΠΌΠ°Π³Π½ΠΈΡ‚Π½Ρ‹ΠΌ ΠΏΠΎΠ»Π΅ΠΌ постоянного ΠΌΠ°Π³Π½ΠΈΡ‚Π°.

Π˜Π½Ρ‹ΠΌΠΈ словами, элСктричСский Ρ‚ΠΎΠΊ зависит ΠΎΡ‚ ΠΊΠ°ΠΊΠΎΠΉ-Ρ‚ΠΎ характСристики ΠΌΠ°Π³Π½ΠΈΡ‚Π½ΠΎΠ³ΠΎ поля. А ΠΌΡ‹ ΡƒΠΆΠ΅ Π²Π²Π΅Π»ΠΈ Ρ‚Π°ΠΊΡƒΡŽ характСристику – магнитная индукция .

Напомним, Ρ‡Ρ‚ΠΎ магнитная индукция обозначаСтся Π±ΡƒΠΊΠ²ΠΎΠΉ , это – вСкторная Π²Π΅Π»ΠΈΡ‡ΠΈΠ½Π°. И измСряСтся магнитная индукция Π² тСслах.

β‡’ – ВСсла – Π² Ρ‡Π΅ΡΡ‚ΡŒ СвропСйского ΠΈ амСриканского ΡƒΡ‡Π΅Π½ΠΎΠ³ΠΎ Николы ВСсла.

ΠœΠ°Π³Π½ΠΈΡ‚Π½Π°Ρ индукция Ρ…Π°Ρ€Π°ΠΊΡ‚Π΅Ρ€ΠΈΠ·ΡƒΠ΅Ρ‚ дСйствиС ΠΌΠ°Π³Π½ΠΈΡ‚Π½ΠΎΠ³ΠΎ поля Π½Π° ΠΏΡ€ΠΎΠ²ΠΎΠ΄Π½ΠΈΠΊ с Ρ‚ΠΎΠΊΠΎΠΌ, ΠΏΠΎΠΌΠ΅Ρ‰Π΅Π½Π½Ρ‹ΠΉ Π² это ΠΏΠΎΠ»Π΅.

Но, ΠΊΠΎΠ³Π΄Π° ΠΌΡ‹ Π³ΠΎΠ²ΠΎΡ€ΠΈΠΌ ΠΎΠ± элСктричСском Ρ‚ΠΎΠΊΠ΅, Ρ‚ΠΎ Π΄ΠΎΠ»ΠΆΠ½Ρ‹ ΠΏΠΎΠ½ΠΈΠΌΠ°Ρ‚ΡŒ, Ρ‡Ρ‚ΠΎ элСктричСский Ρ‚ΠΎΠΊ, ΠΈ это Π²Ρ‹ Π·Π½Π°Π΅Ρ‚Π΅ ΠΈΠ· 8 класса, Π²ΠΎΠ·Π½ΠΈΠΊΠ°Π΅Ρ‚ ΠΏΠΎΠ΄ дСйствиСм элСктричСского поля.

Π‘Π»Π΅Π΄ΠΎΠ²Π°Ρ‚Π΅Π»ΡŒΠ½ΠΎ, ΠΌΠΎΠΆΠ½ΠΎ ΡΠ΄Π΅Π»Π°Ρ‚ΡŒ Π²Ρ‹Π²ΠΎΠ΄ ΠΎ Ρ‚ΠΎΠΌ, Ρ‡Ρ‚ΠΎ элСктричСский ΠΈΠ½Π΄ΡƒΠΊΡ†ΠΈΠΎΠ½Π½Ρ‹ΠΉ Ρ‚ΠΎΠΊ появляСтся Π·Π° счСт элСктричСского поля, ΠΊΠΎΡ‚ΠΎΡ€Ρ‹ΠΉ Π² свою ΠΎΡ‡Π΅Ρ€Π΅Π΄ΡŒ образуСтся Π² Ρ€Π΅Π·ΡƒΠ»ΡŒΡ‚Π°Ρ‚Π΅ дСйствия ΠΌΠ°Π³Π½ΠΈΡ‚Π½ΠΎΠ³ΠΎ поля. И такая взаимосвязь ΠΊΠ°ΠΊ Ρ€Π°Π· осущСствляСтся Π·Π° счСт ΠΌΠ°Π³Π½ΠΈΡ‚Π½ΠΎΠ³ΠΎ ΠΏΠΎΡ‚ΠΎΠΊΠ° .

Π’Π·Π°ΠΈΠΌΠΎΡΠ²ΡΠ·ΡŒ элСктричСских ΠΈ ΠΌΠ°Π³Π½ΠΈΡ‚Π½Ρ‹Ρ… ΠΏΠΎΠ»Π΅ΠΉ Π·Π°ΠΌΠ΅Ρ‡Π΅Π½Π° ΠΎΡ‡Π΅Π½ΡŒ Π΄Π°Π²Π½ΠΎ. Π”Π°Π½Π½ΡƒΡŽ связь Π΅Ρ‰Π΅ Π² 19 Π²Π΅ΠΊΠ΅ ΠΎΠ±Π½Π°Ρ€ΡƒΠΆΠΈΠ» английский ΡƒΡ‡Π΅Π½Ρ‹ΠΉ-Ρ„ΠΈΠ·ΠΈΠΊ Π€Π°Ρ€Π°Π΄Π΅ΠΉ ΠΈ Π΄Π°Π» Π΅ΠΌΡƒ Π½Π°Π·Π²Π°Π½ΠΈΠ΅ . Она появляСтся Π² Ρ‚ΠΎΡ‚ ΠΌΠΎΠΌΠ΅Π½Ρ‚, ΠΊΠΎΠ³Π΄Π° ΠΌΠ°Π³Π½ΠΈΡ‚Π½Ρ‹ΠΉ ΠΏΠΎΡ‚ΠΎΠΊ ΠΏΡ€ΠΎΠ½ΠΈΠ·Ρ‹Π²Π°Π΅Ρ‚ ΠΏΠΎΠ²Π΅Ρ€Ρ…Π½ΠΎΡΡ‚ΡŒ Π·Π°ΠΌΠΊΠ½ΡƒΡ‚ΠΎΠ³ΠΎ ΠΊΠΎΠ½Ρ‚ΡƒΡ€Π°. ПослС Ρ‚ΠΎΠ³ΠΎ ΠΊΠ°ΠΊ происходит ΠΈΠ·ΠΌΠ΅Π½Π΅Π½ΠΈΠ΅ ΠΌΠ°Π³Π½ΠΈΡ‚Π½ΠΎΠ³ΠΎ ΠΏΠΎΡ‚ΠΎΠΊΠ° Π² Ρ‚Π΅Ρ‡Π΅Π½ΠΈΠ΅ ΠΎΠΏΡ€Π΅Π΄Π΅Π»Π΅Π½Π½ΠΎΠ³ΠΎ Π²Ρ€Π΅ΠΌΠ΅Π½ΠΈ, Π² этом ΠΊΠΎΠ½Ρ‚ΡƒΡ€Π΅ Π½Π°Π±Π»ΡŽΠ΄Π°Π΅Ρ‚ΡΡ появлСниС элСктричСского Ρ‚ΠΎΠΊΠ°.

Π’Π·Π°ΠΈΠΌΠΎΡΠ²ΡΠ·ΡŒ элСктромагнитной ΠΈΠ½Π΄ΡƒΠΊΡ†ΠΈΠΈ ΠΈ ΠΌΠ°Π³Π½ΠΈΡ‚Π½ΠΎΠ³ΠΎ ΠΏΠΎΡ‚ΠΎΠΊΠ°

Π‘ΡƒΡ‚ΡŒ ΠΌΠ°Π³Π½ΠΈΡ‚Π½ΠΎΠ³ΠΎ ΠΏΠΎΡ‚ΠΎΠΊΠ° отобраТаСтся извСстной Ρ„ΠΎΡ€ΠΌΡƒΠ»ΠΎΠΉ: Π€ = BS cos Ξ±. Π’ Π½Π΅ΠΉ Π€ являСтся ΠΌΠ°Π³Π½ΠΈΡ‚Π½Ρ‹ΠΌ ΠΏΠΎΡ‚ΠΎΠΊΠΎΠΌ, S – ΠΏΠΎΠ²Π΅Ρ€Ρ…Π½ΠΎΡΡ‚ΡŒ ΠΊΠΎΠ½Ρ‚ΡƒΡ€Π° (ΠΏΠ»ΠΎΡ‰Π°Π΄ΡŒ), Π’ – Π²Π΅ΠΊΡ‚ΠΎΡ€ ΠΌΠ°Π³Π½ΠΈΡ‚Π½ΠΎΠΉ ΠΈΠ½Π΄ΡƒΠΊΡ†ΠΈΠΈ. Π£Π³ΠΎΠ» Ξ± образуСтся Π·Π° счСт направлСния Π²Π΅ΠΊΡ‚ΠΎΡ€Π° ΠΌΠ°Π³Π½ΠΈΡ‚Π½ΠΎΠΉ ΠΈΠ½Π΄ΡƒΠΊΡ†ΠΈΠΈ ΠΈ Π½ΠΎΡ€ΠΌΠ°Π»ΠΈ ΠΊ повСрхности ΠΊΠΎΠ½Ρ‚ΡƒΡ€Π°. ΠžΡ‚ΡΡŽΠ΄Π° слСдуСт, Ρ‡Ρ‚ΠΎ максимального ΠΏΠΎΡ€ΠΎΠ³Π° ΠΌΠ°Π³Π½ΠΈΡ‚Π½Ρ‹ΠΉ ΠΏΠΎΡ‚ΠΎΠΊ достигнСт ΠΏΡ€ΠΈ cos Ξ± = 1, Π° минимального – ΠΏΡ€ΠΈ cos Ξ± = 0.

Π’ΠΎ Π²Ρ‚ΠΎΡ€ΠΎΠΌ Π²Π°Ρ€ΠΈΠ°Π½Ρ‚Π΅ Π²Π΅ΠΊΡ‚ΠΎΡ€ Π’ Π±ΡƒΠ΄Π΅Ρ‚ пСрпСндикулярСн ΠΊ Π½ΠΎΡ€ΠΌΠ°Π»ΠΈ. ΠŸΠΎΠ»ΡƒΡ‡Π°Π΅Ρ‚ΡΡ, Ρ‡Ρ‚ΠΎ Π»ΠΈΠ½ΠΈΠΈ ΠΏΠΎΡ‚ΠΎΠΊΠ° Π½Π΅ ΠΏΠ΅Ρ€Π΅ΡΠ΅ΠΊΠ°ΡŽΡ‚ ΠΊΠΎΠ½Ρ‚ΡƒΡ€, Π° лишь ΡΠΊΠΎΠ»ΡŒΠ·ΡΡ‚ ΠΏΠΎ Π΅Π³ΠΎ плоскости. Π‘Π»Π΅Π΄ΠΎΠ²Π°Ρ‚Π΅Π»ΡŒΠ½ΠΎ, ΠΎΠΏΡ€Π΅Π΄Π΅Π»ΡΡ‚ΡŒ характСристики Π±ΡƒΠ΄ΡƒΡ‚ Π»ΠΈΠ½ΠΈΠΈ Π²Π΅ΠΊΡ‚ΠΎΡ€Π° Π’, ΠΏΠ΅Ρ€Π΅ΡΠ΅ΠΊΠ°ΡŽΡ‰ΠΈΠ΅ ΠΏΠΎΠ²Π΅Ρ€Ρ…Π½ΠΎΡΡ‚ΡŒ ΠΊΠΎΠ½Ρ‚ΡƒΡ€Π°. Для расчСта Π² качСствС Π΅Π΄ΠΈΠ½ΠΈΡ†Ρ‹ измСрСния ΠΈΡΠΏΠΎΠ»ΡŒΠ·ΡƒΠ΅Ρ‚ΡΡ Π²Π΅Π±Π΅Ρ€: 1 Π²Π± = 1Π² Ρ… 1с (Π²ΠΎΠ»ΡŒΡ‚-сСкунда). Π•Ρ‰Π΅ ΠΎΠ΄Π½ΠΎΠΉ, Π±ΠΎΠ»Π΅Π΅ ΠΌΠ΅Π»ΠΊΠΎΠΉ Π΅Π΄ΠΈΠ½ΠΈΡ†Π΅ΠΉ измСрСния слуТит максвСлл (мкс). Он составляСт: 1 Π²Π± = 108 мкс, Ρ‚ΠΎ Π΅ΡΡ‚ΡŒ 1 мкс = 10-8 Π²Π±.

Для исслСдования Π€Π°Ρ€Π°Π΄Π΅Π΅ΠΌ Π±Ρ‹Π»ΠΈ ΠΈΡΠΏΠΎΠ»ΡŒΠ·ΠΎΠ²Π°Π½Ρ‹ Π΄Π²Π΅ ΠΏΡ€ΠΎΠ²ΠΎΠ»ΠΎΡ‡Π½Ρ‹Π΅ спирали, ΠΈΠ·ΠΎΠ»ΠΈΡ€ΠΎΠ²Π°Π½Π½Ρ‹Π΅ ΠΌΠ΅ΠΆΠ΄Ρƒ собой ΠΈ Ρ€Π°Π·ΠΌΠ΅Ρ‰Π΅Π½Π½Ρ‹Π΅ Π½Π° ΠΊΠ°Ρ‚ΡƒΡˆΠΊΠ΅ ΠΈΠ· Π΄Π΅Ρ€Π΅Π²Π°. Одна ΠΈΠ· Π½ΠΈΡ… соСдинялась с источником энСргии, Π° другая – с Π³Π°Π»ΡŒΠ²Π°Π½ΠΎΠΌΠ΅Ρ‚Ρ€ΠΎΠΌ, ΠΏΡ€Π΅Π΄Π½Π°Π·Π½Π°Ρ‡Π΅Π½Π½Ρ‹ΠΌ для рСгистрации ΠΌΠ°Π»Ρ‹Ρ… Ρ‚ΠΎΠΊΠΎΠ². Π’ Ρ‚ΠΎΡ‚ ΠΌΠΎΠΌΠ΅Π½Ρ‚, ΠΊΠΎΠ³Π΄Π° Ρ†Π΅ΠΏΡŒ ΠΏΠ΅Ρ€Π²ΠΎΠ½Π°Ρ‡Π°Π»ΡŒΠ½ΠΎΠΉ спирали Π·Π°ΠΌΡ‹ΠΊΠ°Π»Π°ΡΡŒ ΠΈ Ρ€Π°Π·ΠΌΡ‹ΠΊΠ°Π»Π°ΡΡŒ, Π² Π΄Ρ€ΡƒΠ³ΠΎΠΉ Ρ†Π΅ΠΏΠΈ стрСлка ΠΈΠ·ΠΌΠ΅Ρ€ΠΈΡ‚Π΅Π»ΡŒΠ½ΠΎΠ³ΠΎ устройства ΠΎΡ‚ΠΊΠ»ΠΎΠ½ΡΠ»Π°ΡΡŒ.

ΠŸΡ€ΠΎΠ²Π΅Π΄Π΅Π½ΠΈΠ΅ исслСдований явлСния ΠΈΠ½Π΄ΡƒΠΊΡ†ΠΈΠΈ

Π’ ΠΏΠ΅Ρ€Π²ΠΎΠΉ сСрии ΠΎΠΏΡ‹Ρ‚ΠΎΠ² Майкл Π€Π°Ρ€Π°Π΄Π΅ΠΉ вставлял Π½Π°ΠΌΠ°Π³Π½ΠΈΡ‡Π΅Π½Π½Ρ‹ΠΉ мСталличСский брусок Π² ΠΊΠ°Ρ‚ΡƒΡˆΠΊΡƒ, ΠΏΠΎΠ΄ΠΊΠ»ΡŽΡ‡Π΅Π½Π½ΡƒΡŽ ΠΊ Ρ‚ΠΎΠΊΡƒ, Π° Π·Π°Ρ‚Π΅ΠΌ Π²Ρ‹Π½ΠΈΠΌΠ°Π» Π΅Π³ΠΎ Π½Π°Ρ€ΡƒΠΆΡƒ (рис. 1, 2).

1 2

Π’ случаС помСщСния ΠΌΠ°Π³Π½ΠΈΡ‚Π° Π² ΠΊΠ°Ρ‚ΡƒΡˆΠΊΡƒ, ΠΏΠΎΠ΄ΠΊΠ»ΡŽΡ‡Π΅Π½Π½ΡƒΡŽ ΠΊ ΠΈΠ·ΠΌΠ΅Ρ€ΠΈΡ‚Π΅Π»ΡŒΠ½ΠΎΠΌΡƒ ΠΏΡ€ΠΈΠ±ΠΎΡ€Ρƒ, Π² Ρ†Π΅ΠΏΠΈ Π½Π°Ρ‡ΠΈΠ½Π°Π΅Ρ‚ ΠΏΡ€ΠΎΡ‚Π΅ΠΊΠ°Ρ‚ΡŒ ΠΈΠ½Π΄ΡƒΠΊΡ†ΠΈΠΎΠ½Π½Ρ‹ΠΉ Ρ‚ΠΎΠΊ. Если ΠΌΠ°Π³Π½ΠΈΡ‚Π½Ρ‹ΠΉ брусок удаляСтся ΠΈΠ· ΠΊΠ°Ρ‚ΡƒΡˆΠΊΠΈ, ΠΈΠ½Π΄ΡƒΠΊΡ†ΠΈΠΎΠ½Π½Ρ‹ΠΉ Ρ‚ΠΎΠΊ всС Ρ€Π°Π²Π½ΠΎ появляСтся, Π½ΠΎ Π΅Π³ΠΎ Π½Π°ΠΏΡ€Π°Π²Π»Π΅Π½ΠΈΠ΅ становится ΡƒΠΆΠ΅ ΠΏΡ€ΠΎΡ‚ΠΈΠ²ΠΎΠΏΠΎΠ»ΠΎΠΆΠ½Ρ‹ΠΌ. Π‘Π»Π΅Π΄ΠΎΠ²Π°Ρ‚Π΅Π»ΡŒΠ½ΠΎ, ΠΏΠ°Ρ€Π°ΠΌΠ΅Ρ‚Ρ€Ρ‹ ΠΈΠ½Π΄ΡƒΠΊΡ†ΠΈΠΎΠ½Π½ΠΎΠ³ΠΎ Ρ‚ΠΎΠΊΠ° Π±ΡƒΠ΄ΡƒΡ‚ ΠΈΠ·ΠΌΠ΅Π½Π΅Π½Ρ‹ ΠΏΠΎ Π½Π°ΠΏΡ€Π°Π²Π»Π΅Π½ΠΈΡŽ двиТСния бруска ΠΈ Π² зависимости ΠΎΡ‚ полюса, ΠΊΠΎΡ‚ΠΎΡ€Ρ‹ΠΌ ΠΎΠ½ помСщаСтся Π² ΠΊΠ°Ρ‚ΡƒΡˆΠΊΡƒ. На силу Ρ‚ΠΎΠΊΠ° ΠΎΠΊΠ°Π·Ρ‹Π²Π°Π΅Ρ‚ влияниС быстрота пСрСмСщСния ΠΌΠ°Π³Π½ΠΈΡ‚Π°.

Π’ΠΎ Π²Ρ‚ΠΎΡ€ΠΎΠΉ сСрии ΠΎΠΏΡ‹Ρ‚ΠΎΠ² подтвСрТдаСтся явлСниС, ΠΏΡ€ΠΈ ΠΊΠΎΡ‚ΠΎΡ€ΠΎΠΌ ΠΈΠ·ΠΌΠ΅Π½ΡΡŽΡ‰ΠΈΠΉΡΡ Ρ‚ΠΎΠΊ Π² ΠΎΠ΄Π½ΠΎΠΉ ΠΊΠ°Ρ‚ΡƒΡˆΠΊΠ΅, Π²Ρ‹Π·Ρ‹Π²Π°Π΅Ρ‚ ΠΈΠ½Π΄ΡƒΠΊΡ†ΠΈΠΎΠ½Π½Ρ‹ΠΉ Ρ‚ΠΎΠΊ Π² Π΄Ρ€ΡƒΠ³ΠΎΠΉ ΠΊΠ°Ρ‚ΡƒΡˆΠΊΠ΅ (рис. 3, 4, 5). Π­Ρ‚ΠΎ происходит Π² ΠΌΠΎΠΌΠ΅Π½Ρ‚Ρ‹ замыкания ΠΈ размыкания Ρ†Π΅ΠΏΠΈ. ΠžΡ‚ Ρ‚ΠΎΠ³ΠΎ, замыкаСтся ΠΈΠ»ΠΈ размыкаСтся элСктричСская Ρ†Π΅ΠΏΡŒ, Π±ΡƒΠ΄Π΅Ρ‚ Π·Π°Π²ΠΈΡΠ΅Ρ‚ΡŒ ΠΈ Π½Π°ΠΏΡ€Π°Π²Π»Π΅Π½ΠΈΠ΅ Ρ‚ΠΎΠΊΠ°. ΠšΡ€ΠΎΠΌΠ΅ Ρ‚ΠΎΠ³ΠΎ, эти дСйствия Π΅ΡΡ‚ΡŒ Π½ΠΈ Ρ‡Ρ‚ΠΎ ΠΈΠ½ΠΎΠ΅, ΠΊΠ°ΠΊ способы измСнСния ΠΌΠ°Π³Π½ΠΈΡ‚Π½ΠΎΠ³ΠΎ ΠΏΠΎΡ‚ΠΎΠΊΠ°. ΠŸΡ€ΠΈ Π·Π°ΠΌΡ‹ΠΊΠ°Π½ΠΈΠΈ Ρ†Π΅ΠΏΠΈ ΠΎΠ½ Π±ΡƒΠ΄Π΅Ρ‚ ΡƒΠ²Π΅Π»ΠΈΡ‡ΠΈΠ²Π°Ρ‚ΡŒΡΡ, Π° ΠΏΡ€ΠΈ Ρ€Π°Π·ΠΌΡ‹ΠΊΠ°Π½ΠΈΠΈ – ΡƒΠΌΠ΅Π½ΡŒΡˆΠ°Ρ‚ΡŒΡΡ, ΠΎΠ΄Π½ΠΎΠ²Ρ€Π΅ΠΌΠ΅Π½Π½ΠΎ пронизывая ΠΏΠ΅Ρ€Π²ΡƒΡŽ ΠΊΠ°Ρ‚ΡƒΡˆΠΊΡƒ.

3 4

5

Π’ Ρ€Π΅Π·ΡƒΠ»ΡŒΡ‚Π°Ρ‚Π΅ ΠΎΠΏΡ‹Ρ‚ΠΎΠ² Π±Ρ‹Π»ΠΎ установлСно, Ρ‡Ρ‚ΠΎ Π²ΠΎΠ·Π½ΠΈΠΊΠ½ΠΎΠ²Π΅Π½ΠΈΠ΅ элСктричСского Ρ‚ΠΎΠΊΠ° Π²Π½ΡƒΡ‚Ρ€ΠΈ Π·Π°ΠΌΠΊΠ½ΡƒΡ‚ΠΎΠ³ΠΎ проводящСго ΠΊΠΎΠ½Ρ‚ΡƒΡ€Π° Π²ΠΎΠ·ΠΌΠΎΠΆΠ½ΠΎ лишь Π² Ρ‚ΠΎΠΌ случаС, ΠΊΠΎΠ³Π΄Π° ΠΎΠ½ΠΈ ΠΏΠΎΠΌΠ΅Ρ‰Π°ΡŽΡ‚ΡΡ Π² ΠΏΠ΅Ρ€Π΅ΠΌΠ΅Π½Π½ΠΎΠ΅ ΠΌΠ°Π³Π½ΠΈΡ‚Π½ΠΎΠ΅ ΠΏΠΎΠ»Π΅. ΠŸΡ€ΠΈ этом, ΠΏΠΎΡ‚ΠΎΠΊ ΠΌΠΎΠΆΠ΅Ρ‚ ΠΈΠ·ΠΌΠ΅Π½ΡΡ‚ΡŒΡΡ Π²ΠΎ Π²Ρ€Π΅ΠΌΠ΅Π½ΠΈ Π»ΡŽΠ±Ρ‹ΠΌΠΈ способами.

ЭлСктричСский Ρ‚ΠΎΠΊ, ΠΏΠΎΡΠ²Π»ΡΡŽΡ‰ΠΈΠΉΡΡ ΠΏΠΎΠ΄ дСйствиСм элСктромагнитной ΠΈΠ½Π΄ΡƒΠΊΡ†ΠΈΠΈ, ΠΏΠΎΠ»ΡƒΡ‡ΠΈΠ» Π½Π°Π·Π²Π°Π½ΠΈΠ΅ ΠΈΠ½Π΄ΡƒΠΊΡ†ΠΈΠΎΠ½Π½ΠΎΠ³ΠΎ, хотя это ΠΈ Π½Π΅ Π±ΡƒΠ΄Π΅Ρ‚ Ρ‚ΠΎΠΊΠΎΠΌ Π² общСпринятом ΠΏΠΎΠ½ΠΈΠΌΠ°Π½ΠΈΠΈ. Когда Π·Π°ΠΌΠΊΠ½ΡƒΡ‚Ρ‹ΠΉ ΠΊΠΎΠ½Ρ‚ΡƒΡ€ оказываСтся Π² ΠΌΠ°Π³Π½ΠΈΡ‚Π½ΠΎΠΌ ΠΏΠΎΠ»Π΅, происходит гСнСрация Π­Π”Π‘ с Ρ‚ΠΎΡ‡Π½Ρ‹ΠΌ Π·Π½Π°Ρ‡Π΅Π½ΠΈΠ΅ΠΌ, Π° Π½Π΅ Ρ‚ΠΎΠΊΠ°, зависящСго ΠΎΡ‚ Ρ€Π°Π·Π½Ρ‹Ρ… сопротивлСний.

Π”Π°Π½Π½ΠΎΠ΅ явлСниС ΠΏΠΎΠ»ΡƒΡ‡ΠΈΠ»ΠΎ Π½Π°Π·Π²Π°Π½ΠΈΠ΅ Π­Π”Π‘ ΠΈΠ½Π΄ΡƒΠΊΡ†ΠΈΠΈ, ΠΊΠΎΡ‚ΠΎΡ€ΡƒΡŽ ΠΎΡ‚Ρ€Π°ΠΆΠ°Π΅Ρ‚ Ρ„ΠΎΡ€ΠΌΡƒΠ»Π°: Π•ΠΈΠ½Π΄ = – βˆ†Π€/βˆ†t. Π•Π΅ Π·Π½Π°Ρ‡Π΅Π½ΠΈΠ΅ совпадаСт с быстротой ΠΈΠ·ΠΌΠ΅Π½Π΅Π½ΠΈΠΉ ΠΌΠ°Π³Π½ΠΈΡ‚Π½ΠΎΠ³ΠΎ ΠΏΠΎΡ‚ΠΎΠΊΠ°, ΠΏΡ€ΠΎΠ½ΠΈΠ·Ρ‹Π²Π°ΡŽΡ‰Π΅Π³ΠΎ ΠΏΠΎΠ²Π΅Ρ€Ρ…Π½ΠΎΡΡ‚ΡŒ Π·Π°ΠΌΠΊΠ½ΡƒΡ‚ΠΎΠ³ΠΎ ΠΊΠΎΠ½Ρ‚ΡƒΡ€Π°, взятого с ΠΎΡ‚Ρ€ΠΈΡ†Π°Ρ‚Π΅Π»ΡŒΠ½Ρ‹ΠΌ Π·Π½Π°Ρ‡Π΅Π½ΠΈΠ΅ΠΌ. ΠœΠΈΠ½ΡƒΡ, ΠΏΡ€ΠΈΡΡƒΡ‚ΡΡ‚Π²ΡƒΡŽΡ‰ΠΈΠΉ Π² Π΄Π°Π½Π½ΠΎΠΌ Π²Ρ‹Ρ€Π°ΠΆΠ΅Π½ΠΈΠΈ, являСтся ΠΎΡ‚Ρ€Π°ΠΆΠ΅Π½ΠΈΠ΅ΠΌ ΠΏΡ€Π°Π²ΠΈΠ»Π° Π›Π΅Π½Ρ†Π°.

ΠŸΡ€Π°Π²ΠΈΠ»ΠΎ Π›Π΅Π½Ρ†Π° Π² ΠΎΡ‚Π½ΠΎΡˆΠ΅Π½ΠΈΠΈ ΠΌΠ°Π³Π½ΠΈΡ‚Π½ΠΎΠ³ΠΎ ΠΏΠΎΡ‚ΠΎΠΊΠ°

Π˜Π·Π²Π΅ΡΡ‚Π½ΠΎΠ΅ ΠΏΡ€Π°Π²ΠΈΠ»ΠΎ Π±Ρ‹Π»ΠΎ Π²Ρ‹Π²Π΅Π΄Π΅Π½ΠΎ послС провСдСния Ρ†ΠΈΠΊΠ»Π° исслСдований Π² 30-Ρ… Π³ΠΎΠ΄Π°Ρ… 19 Π²Π΅ΠΊΠ°. Оно сформулировано Π² ΡΠ»Π΅Π΄ΡƒΡŽΡ‰Π΅ΠΌ Π²ΠΈΠ΄Π΅:

НаправлСниС ΠΈΠ½Π΄ΡƒΠΊΡ†ΠΈΠΎΠ½Π½ΠΎΠ³ΠΎ Ρ‚ΠΎΠΊΠ°, Π²ΠΎΠ·Π±ΡƒΠΆΠ΄Π°Π΅ΠΌΠΎΠ³ΠΎ Π² Π·Π°ΠΌΠΊΠ½ΡƒΡ‚ΠΎΠΌ ΠΊΠΎΠ½Ρ‚ΡƒΡ€Π΅ ΠΈΠ·ΠΌΠ΅Π½ΡΡŽΡ‰ΠΈΠΌΡΡ ΠΌΠ°Π³Π½ΠΈΡ‚Π½Ρ‹ΠΌ ΠΏΠΎΡ‚ΠΎΠΊΠΎΠΌ, ΠΎΠΊΠ°Π·Ρ‹Π²Π°Π΅Ρ‚ влияниС Π½Π° создаваСмоС ΠΈΠΌ ΠΌΠ°Π³Π½ΠΈΡ‚Π½ΠΎΠ΅ ΠΏΠΎΠ»Π΅ Ρ‚Π°ΠΊΠΈΠΌ ΠΎΠ±Ρ€Π°Π·ΠΎΠΌ, Ρ‡Ρ‚ΠΎ ΠΎΠ½ΠΎ Π² свою ΠΎΡ‡Π΅Ρ€Π΅Π΄ΡŒ создаСт прСпятствиС ΠΌΠ°Π³Π½ΠΈΡ‚Π½ΠΎΠΌΡƒ ΠΏΠΎΡ‚ΠΎΠΊΡƒ, Π²Ρ‹Π·Ρ‹Π²Π°ΡŽΡ‰Π΅ΠΌΡƒ появлСниС ΠΈΠ½Π΄ΡƒΠΊΡ†ΠΈΠΎΠ½Π½ΠΎΠ³ΠΎ Ρ‚ΠΎΠΊΠ°.

Когда ΠΌΠ°Π³Π½ΠΈΡ‚Π½Ρ‹ΠΉ ΠΏΠΎΡ‚ΠΎΠΊ увСличиваСтся, Ρ‚ΠΎ Π΅ΡΡ‚ΡŒ становится Π€ > 0, Π° Π­Π”Π‘ ΠΈΠ½Π΄ΡƒΠΊΡ†ΠΈΠΈ сниТаСтся ΠΈ становится Π•ΠΈΠ½Π΄

Если ΠΏΠΎΡ‚ΠΎΠΊ сниТаСтся, Ρ‚ΠΎ наступаСт ΠΎΠ±Ρ€Π°Ρ‚Π½Ρ‹ΠΉ процСсс, ΠΊΠΎΠ³Π΄Π° Π€ 0, Ρ‚ΠΎ Π΅ΡΡ‚ΡŒ дСйствиС ΠΌΠ°Π³Π½ΠΈΡ‚Π½ΠΎΠ³ΠΎ поля ΠΈΠ½Π΄ΡƒΠΊΡ†ΠΈΠΎΠ½Π½ΠΎΠ³ΠΎ Ρ‚ΠΎΠΊΠ°, происходит ΡƒΠ²Π΅Π»ΠΈΡ‡Π΅Π½ΠΈΠ΅ ΠΌΠ°Π³Π½ΠΈΡ‚Π½ΠΎΠ³ΠΎ ΠΏΠΎΡ‚ΠΎΠΊΠ°, проходящСго Ρ‡Π΅Ρ€Π΅Π· ΠΊΠΎΠ½Ρ‚ΡƒΡ€.

ЀизичСский смысл ΠΏΡ€Π°Π²ΠΈΠ»Π° Π›Π΅Π½Ρ†Π° Π·Π°ΠΊΠ»ΡŽΡ‡Π°Π΅Ρ‚ΡΡ Π² ΠΎΡ‚Ρ€Π°ΠΆΠ΅Π½ΠΈΠΈ Π·Π°ΠΊΠΎΠ½Π° сохранСния энСргии, ΠΊΠΎΠ³Π΄Π° ΠΏΡ€ΠΈ ΡƒΠΌΠ΅Π½ΡŒΡˆΠ΅Π½ΠΈΠΈ ΠΎΠ΄Π½ΠΎΠΉ Π²Π΅Π»ΠΈΡ‡ΠΈΠ½Ρ‹, другая увСличиваСтся, ΠΈ, Π½Π°ΠΎΠ±ΠΎΡ€ΠΎΡ‚, ΠΏΡ€ΠΈ ΡƒΠ²Π΅Π»ΠΈΡ‡Π΅Π½ΠΈΠΈ ΠΎΠ΄Π½ΠΎΠΉ Π²Π΅Π»ΠΈΡ‡ΠΈΠ½Ρ‹ другая Π±ΡƒΠ΄Π΅Ρ‚ ΡƒΠΌΠ΅Π½ΡŒΡˆΠ°Ρ‚ΡŒΡΡ. Π Π°Π·Π»ΠΈΡ‡Π½Ρ‹Π΅ Ρ„Π°ΠΊΡ‚ΠΎΡ€Ρ‹ Π²Π»ΠΈΡΡŽΡ‚ ΠΈ Π½Π° Π­Π”Π‘ ΠΈΠ½Π΄ΡƒΠΊΡ†ΠΈΠΈ. ΠŸΡ€ΠΈ Π²Π²ΠΎΠ΄Π΅ Π² ΠΊΠ°Ρ‚ΡƒΡˆΠΊΡƒ ΠΏΠΎΠΎΡ‡Π΅Ρ€Π΅Π΄Π½ΠΎ сильного ΠΈ слабого ΠΌΠ°Π³Π½ΠΈΡ‚Π°, ΠΏΡ€ΠΈΠ±ΠΎΡ€ соотвСтствСнно Π±ΡƒΠ΄Π΅Ρ‚ ΠΏΠΎΠΊΠ°Π·Ρ‹Π²Π°Ρ‚ΡŒ Π² ΠΏΠ΅Ρ€Π²ΠΎΠΌ случаС Π±ΠΎΠ»Π΅Π΅ высокоС, Π° Π²ΠΎ Π²Ρ‚ΠΎΡ€ΠΎΠΌ – Π±ΠΎΠ»Π΅Π΅ Π½ΠΈΠ·ΠΊΠΎΠ΅ Π·Π½Π°Ρ‡Π΅Π½ΠΈΠ΅. Π’ΠΎ ΠΆΠ΅ самоС происходит, ΠΊΠΎΠ³Π΄Π° измСняСтся ΡΠΊΠΎΡ€ΠΎΡΡ‚ΡŒ двиТСния ΠΌΠ°Π³Π½ΠΈΡ‚Π°.

На прСдставлСнном рисункС Π²ΠΈΠ΄Π½ΠΎ, ΠΊΠ°ΠΊ опрСдСляСтся Π½Π°ΠΏΡ€Π°Π²Π»Π΅Π½ΠΈΠ΅ ΠΈΠ½Π΄ΡƒΠΊΡ†ΠΈΠΎΠ½Π½ΠΎΠ³ΠΎ Ρ‚ΠΎΠΊΠ° с ΠΏΡ€ΠΈΠΌΠ΅Π½Π΅Π½ΠΈΠ΅ΠΌ ΠΏΡ€Π°Π²ΠΈΠ»Π° Π›Π΅Π½Ρ†Π°. Π‘ΠΈΠ½ΠΈΠΉ Ρ†Π²Π΅Ρ‚ соотвСтствуСт силовым линиям ΠΌΠ°Π³Π½ΠΈΡ‚Π½Ρ‹Ρ… ΠΏΠΎΠ»Π΅ΠΉ ΠΈΠ½Π΄ΡƒΠΊΡ†ΠΈΠΎΠ½Π½ΠΎΠ³ΠΎ Ρ‚ΠΎΠΊΠ° ΠΈ постоянного ΠΌΠ°Π³Π½ΠΈΡ‚Π°. Они располоТСны Π² Π½Π°ΠΏΡ€Π°Π²Π»Π΅Π½ΠΈΠΈ полюсов ΠΎΡ‚ сСвСра ΠΊ ΡŽΠ³Ρƒ, ΠΊΠΎΡ‚ΠΎΡ€Ρ‹Π΅ ΠΈΠΌΠ΅ΡŽΡ‚ΡΡ Π² ΠΊΠ°ΠΆΠ΄ΠΎΠΌ ΠΌΠ°Π³Π½ΠΈΡ‚Π΅.

Π˜Π·ΠΌΠ΅Π½ΡΡŽΡ‰ΠΈΠΉΡΡ ΠΌΠ°Π³Π½ΠΈΡ‚Π½Ρ‹ΠΉ ΠΏΠΎΡ‚ΠΎΠΊ ΠΏΡ€ΠΈΠ²ΠΎΠ΄ΠΈΡ‚ ΠΊ возникновСнию ΠΈΠ½Π΄ΡƒΠΊΡ†ΠΈΠΎΠ½Π½ΠΎΠ³ΠΎ элСктричСского Ρ‚ΠΎΠΊΠ°, Π½Π°ΠΏΡ€Π°Π²Π»Π΅Π½ΠΈΠ΅ ΠΊΠΎΡ‚ΠΎΡ€ΠΎΠ³ΠΎ Π²Ρ‹Π·Ρ‹Π²Π°Π΅Ρ‚ противодСйствиС со стороны Π΅Π³ΠΎ ΠΌΠ°Π³Π½ΠΈΡ‚Π½ΠΎΠ³ΠΎ поля, ΠΏΡ€Π΅ΠΏΡΡ‚ΡΡ‚Π²ΡƒΡŽΡ‰Π΅Π΅ измСнСниям ΠΌΠ°Π³Π½ΠΈΡ‚Π½ΠΎΠ³ΠΎ ΠΏΠΎΡ‚ΠΎΠΊΠ°. Π‘Π²ΠΈΡ‚Ρ‹ΠΉ Π² ΠΊΠ°Ρ‚ΡƒΡˆΠΊΡƒ ΠΏΡ€ΠΎΠ²ΠΎΠ΄Π½ΠΈΠΊ замыкаСтся Π½Π° Π³Π°Π»ΡŒΠ²Π°Π½ΠΎΠΌΠ΅Ρ‚Ρ€Π΅ (рис. 3.19). Если Π²Π΄Π²ΠΈΠ³Π°Ρ‚ΡŒ Π² ΠΊΠ°Ρ‚ΡƒΡˆΠΊΡƒ постоянный ΠΌΠ°Π³Π½ΠΈΡ‚, Ρ‚ΠΎ Π³Π°Π»ΡŒΠ²Π°Π½ΠΎΠΌΠ΅Ρ‚Ρ€ ΠΏΠΎΠΊΠ°ΠΆΠ΅Ρ‚ Π½Π°Π»ΠΈΡ‡ΠΈΠ΅ Ρ‚ΠΎΠΊΠ° Π² Ρ‚Π΅Ρ‡Π΅Π½ΠΈΠ΅ всСго ΠΏΡ€ΠΎΠΌΠ΅ΠΆΡƒΡ‚ΠΊΠ° Π²Ρ€Π΅ΠΌΠ΅Π½ΠΈ, ΠΏΠΎΠΊΠ° ΠΌΠ°Π³Π½ΠΈΡ‚ пСрСмСщаСтся ΠΎΡ‚Π½ΠΎΡΠΈΡ‚Π΅Π»ΡŒΠ½ΠΎ ΠΊΠ°Ρ‚ΡƒΡˆΠΊΠΈ. ΠŸΡ€ΠΈ Π²Ρ‹Π΄Π΅Ρ€Π³ΠΈΠ²Π°Π½ΠΈΠΈ ΠΌΠ°Π³Π½ΠΈΡ‚Π° ΠΈΠ· ΠΊΠ°Ρ‚ΡƒΡˆΠΊΠΈ Π³Π°Π»ΡŒΠ²Π°Π½ΠΎΠΌΠ΅Ρ‚Ρ€ ΠΏΠΎΠΊΠ°Π·Ρ‹Π²Π°Π΅Ρ‚ Π½Π°Π»ΠΈΡ‡ΠΈΠ΅ Ρ‚ΠΎΠΊΠ° ΠΏΡ€ΠΎΡ‚ΠΈΠ²ΠΎΠΏΠΎΠ»ΠΎΠΆΠ½ΠΎΠ³ΠΎ направлСния. ИзмСнСния направлСния Ρ‚ΠΎΠΊΠ° происходит ΠΏΡ€ΠΈ ΠΈΠ·ΠΌΠ΅Π½Π΅Π½ΠΈΠΈ Π²Π΄Π²ΠΈΠ³Π°Π΅ΠΌΠΎΠ³ΠΎ ΠΈΠ»ΠΈ Π²Ρ‹Π΄Π²ΠΈΠ³Π°Π΅ΠΌΠΎΠ³ΠΎ полюса ΠΌΠ°Π³Π½ΠΈΡ‚Π°.

АналогичныС Ρ€Π΅Π·ΡƒΠ»ΡŒΡ‚Π°Ρ‚Ρ‹ наблюдались ΠΏΡ€ΠΈ Π·Π°ΠΌΠ΅Π½Π΅ постоянного ΠΌΠ°Π³Π½ΠΈΡ‚Π° элСктромагнитом (ΠΊΠ°Ρ‚ΡƒΡˆΠΊΠΎΠΉ с Ρ‚ΠΎΠΊΠΎΠΌ). Если ΠΎΠ±Π΅ ΠΊΠ°Ρ‚ΡƒΡˆΠΊΠΈ Π·Π°ΠΊΡ€Π΅ΠΏΠΈΡ‚ΡŒ Π½Π΅ΠΏΠΎΠ΄Π²ΠΈΠΆΠ½ΠΎ, Π½ΠΎ Π² ΠΎΠ΄Π½ΠΎΠΉ ΠΈΠ· Π½ΠΈΡ… ΠΌΠ΅Π½ΡΡ‚ΡŒ Π·Π½Π°Ρ‡Π΅Π½ΠΈΠ΅ Ρ‚ΠΎΠΊΠ°, Ρ‚ΠΎ Π² этот ΠΌΠΎΠΌΠ΅Π½Ρ‚ Π² Π΄Ρ€ΡƒΠ³ΠΎΠΉ ΠΊΠ°Ρ‚ΡƒΡˆΠΊΠ΅ Π½Π°Π±Π»ΡŽΠ΄Π°Π΅Ρ‚ΡΡ ΠΈΠ½Π΄ΡƒΠΊΡ†ΠΈΠΎΠ½Π½Ρ‹ΠΉ Ρ‚ΠΎΠΊ.

Π―Π’Π›Π•ΠΠ˜Π• Π­Π›Π•ΠšΠ’Π ΠžΠœΠΠ“ΠΠ˜Π’ΠΠžΠ™ Π˜ΠΠ”Π£ΠšΠ¦Π˜Π˜ состоит Π² Π²ΠΎΠ·Π½ΠΈΠΊΠ½ΠΎΠ²Π΅Π½ΠΈΠΈ элСктродвиТущСй силы (э.Π΄.с.) ΠΈΠ½Π΄ΡƒΠΊΡ†ΠΈΠΈ Π² проводящСм ΠΊΠΎΠ½Ρ‚ΡƒΡ€Π΅, Ρ‡Π΅Ρ€Π΅Π· ΠΊΠΎΡ‚ΠΎΡ€Ρ‹ΠΉ мСняСтся ΠΏΠΎΡ‚ΠΎΠΊ Π²Π΅ΠΊΡ‚ΠΎΡ€Π° ΠΌΠ°Π³Π½ΠΈΡ‚Π½ΠΎΠΉ ΠΈΠ½Π΄ΡƒΠΊΡ†ΠΈΠΈ. Если ΠΊΠΎΠ½Ρ‚ΡƒΡ€ являСтся Π·Π°ΠΌΠΊΠ½ΡƒΡ‚Ρ‹ΠΌ, Ρ‚ΠΎ Π² Π½Π΅ΠΌ Π²ΠΎΠ·Π½ΠΈΠΊΠ°Π΅Ρ‚ ΠΈΠ½Π΄ΡƒΠΊΡ†ΠΈΠΎΠ½Π½Ρ‹ΠΉ Ρ‚ΠΎΠΊ.

ΠžΡ‚ΠΊΡ€Ρ‹Ρ‚ΠΈΠ΅ явлСния элСктромагнитной ΠΈΠ½Π΄ΡƒΠΊΡ†ΠΈΠΈ:

1) ΠΏΠΎΠΊΠ°Π·Π°Π»ΠΎ взаимосвязь ΠΌΠ΅ΠΆΠ΄Ρƒ элСктричСским ΠΈ ΠΌΠ°Π³Π½ΠΈΡ‚Π½Ρ‹ΠΌ ΠΏΠΎΠ»Π΅ΠΌ ;

2) ΠΏΡ€Π΅Π΄Π»ΠΎΠΆΠΈΠ»ΠΎ способ получСния элСктричСского Ρ‚ΠΎΠΊΠ° с ΠΏΠΎΠΌΠΎΡ‰ΡŒΡŽ ΠΌΠ°Π³Π½ΠΈΡ‚Π½ΠΎΠ³ΠΎ поля.

ΠžΡΠ½ΠΎΠ²Π½Ρ‹Π΅ свойства ΠΈΠ½Π΄ΡƒΠΊΡ†ΠΈΠΎΠ½Π½ΠΎΠ³ΠΎ Ρ‚ΠΎΠΊΠ° :

1. Π˜Π½Π΄ΡƒΠΊΡ†ΠΈΠΎΠ½Π½Ρ‹ΠΉ Ρ‚ΠΎΠΊ Π²ΠΎΠ·Π½ΠΈΠΊΠ°Π΅Ρ‚ всСгда, ΠΊΠΎΠ³Π΄Π° происходит ΠΈΠ·ΠΌΠ΅Π½Π΅Π½ΠΈΠ΅ сцСплСнного с ΠΊΠΎΠ½Ρ‚ΡƒΡ€ΠΎΠΌ ΠΏΠΎΡ‚ΠΎΠΊΠ° ΠΌΠ°Π³Π½ΠΈΡ‚Π½ΠΎΠΉ ΠΈΠ½Π΄ΡƒΠΊΡ†ΠΈΠΈ.

2. Π‘ΠΈΠ»Π° ΠΈΠ½Π΄ΡƒΠΊΡ†ΠΈΠΎΠ½Π½ΠΎΠ³ΠΎ Ρ‚ΠΎΠΊΠ° Π½Π΅ зависит ΠΎΡ‚ способа измСнСния ΠΏΠΎΡ‚ΠΎΠΊΠ° ΠΌΠ°Π³Π½ΠΈΡ‚Π½ΠΎΠΉ ΠΈΠ½Π΄ΡƒΠΊΡ†ΠΈΠΈ, Π° опрСдСляСтся лишь ΡΠΊΠΎΡ€ΠΎΡΡ‚ΡŒΡŽ Π΅Π³ΠΎ измСнСния.

ΠžΠΏΡ‹Ρ‚Π°ΠΌΠΈ ЀарадСя Π±Ρ‹Π»ΠΎ установлСно, Ρ‡Ρ‚ΠΎ Π²Π΅Π»ΠΈΡ‡ΠΈΠ½Π° элСктродвиТущСй силы ΠΈΠ½Π΄ΡƒΠΊΡ†ΠΈΠΈ ΠΏΡ€ΠΎΠΏΠΎΡ€Ρ†ΠΈΠΎΠ½Π°Π»ΡŒΠ½Π° скорости измСнСния ΠΌΠ°Π³Π½ΠΈΡ‚Π½ΠΎΠ³ΠΎ ΠΏΠΎΡ‚ΠΎΠΊΠ°, ΠΏΡ€ΠΎΠ½ΠΈΠ·Ρ‹Π²Π°ΡŽΡ‰Π΅Π³ΠΎ ΠΊΠΎΠ½Ρ‚ΡƒΡ€ ΠΏΡ€ΠΎΠ²ΠΎΠ΄Π½ΠΈΠΊΠ° (Π·Π°ΠΊΠΎΠ½ элСктромагнитной ΠΈΠ½Π΄ΡƒΠΊΡ†ΠΈΠΈ ЀарадСя)

Или , (3.46)

Π³Π΄Π΅ (dF) – ΠΈΠ·ΠΌΠ΅Π½Π΅Π½ΠΈΠ΅ ΠΏΠΎΡ‚ΠΎΠΊΠ° Π² Ρ‚Π΅Ρ‡Π΅Π½ΠΈΠΈ Π²Ρ€Π΅ΠΌΠ΅Π½ΠΈ (dt).ΠœΠΠ“ΠΠ˜Π’ΠΠ«Πœ ПОВОКОМ ΠΈΠ»ΠΈ ПОВОКОМ ΠœΠΠ“ΠΠ˜Π’ΠΠžΠ™ Π˜ΠΠ”Π£ΠšΠ¦Π˜Π˜ называСтся Π²Π΅Π»ΠΈΡ‡ΠΈΠ½Π°, которая опрСдСляСтся Π½Π° основС ΡΠ»Π΅Π΄ΡƒΡŽΡ‰Π΅Π³ΠΎ ΡΠΎΠΎΡ‚Π½ΠΎΡˆΠ΅Π½ΠΈΡ: (ΠΌΠ°Π³Π½ΠΈΡ‚Π½Ρ‹ΠΉ ΠΏΠΎΡ‚ΠΎΠΊ Ρ‡Π΅Ρ€Π΅Π· ΠΏΠΎΠ²Π΅Ρ€Ρ…Π½ΠΎΡΡ‚ΡŒ ΠΏΠ»ΠΎΡ‰Π°Π΄ΡŒΡŽ S ): Π€=Π’ScosΞ±, (3.45), ΡƒΠ³ΠΎΠ» a – ΡƒΠ³ΠΎΠ» ΠΌΠ΅ΠΆΠ΄Ρƒ Π½ΠΎΡ€ΠΌΠ°Π»ΡŒΡŽ ΠΊ рассматриваСмой повСрхности ΠΈ Π½Π°ΠΏΡ€Π°Π²Π»Π΅Π½ΠΈΠ΅ΠΌ Π²Π΅ΠΊΡ‚ΠΎΡ€Π° ΠΈΠ½Π΄ΡƒΠΊΡ†ΠΈΠΈ ΠΌΠ°Π³Π½ΠΈΡ‚Π½ΠΎΠ³ΠΎ поля

Π΅Π΄ΠΈΠ½ΠΈΡ†Π° ΠΌΠ°Π³Π½ΠΈΡ‚Π½ΠΎΠ³ΠΎ ΠΏΠΎΡ‚ΠΎΠΊΠ° Π² систСмС БИ носит Π½Π°Π·Π²Π°Π½ΠΈΠ΅ Π²Π΅Π±Π΅Ρ€ – [Π’Π±=Π’Π»Γ—ΠΌ 2 ].

Π—Π½Π°ΠΊ «–» Π² Ρ„ΠΎΡ€ΠΌΡƒΠ»Π΅ ΠΎΠ·Π½Π°Ρ‡Π°Π΅Ρ‚, Ρ‡Ρ‚ΠΎ э.Π΄.с. ΠΈΠ½Π΄ΡƒΠΊΡ†ΠΈΠΈ Π²Ρ‹Π·Ρ‹Π²Π°Π΅Ρ‚ ΠΈΠ½Π΄ΡƒΠΊΡ†ΠΈΠΎΠ½Π½Ρ‹ΠΉ Ρ‚ΠΎΠΊ, ΠΌΠ°Π³Π½ΠΈΡ‚Π½ΠΎΠ΅ ΠΏΠΎΠ»Π΅ ΠΊΠΎΡ‚ΠΎΡ€ΠΎΠ³ΠΎ противодСйствуСт всякому измСнСнию ΠΌΠ°Π³Π½ΠΈΡ‚Π½ΠΎΠ³ΠΎ ΠΏΠΎΡ‚ΠΎΠΊΠ°, Ρ‚.Π΅. ΠΏΡ€ΠΈ >0 э.Π΄.с. ΠΈΠ½Π΄ΡƒΠΊΡ†ΠΈΠΈ e И

э.Π΄.с. ΠΈΠ½Π΄ΡƒΠΊΡ†ΠΈΠΈ измСряСтся Π² Π²ΠΎΠ»ΡŒΡ‚Π°Ρ…

Для нахоТдСния направлСния ΠΈΠ½Π΄ΡƒΠΊΡ†ΠΈΠΎΠ½Π½ΠΎΠ³ΠΎ Ρ‚ΠΎΠΊΠ° сущСствуСт ΠΏΡ€Π°Π²ΠΈΠ»ΠΎ Π›Π΅Π½Ρ†Π° (ΠΏΡ€Π°Π²ΠΈΠ»ΠΎ установлСно Π² 1833 Π³.): ΠΈΠ½Π΄ΡƒΠΊΡ†ΠΈΠΎΠ½Π½Ρ‹ΠΉ Ρ‚ΠΎΠΊ ΠΈΠΌΠ΅Π΅Ρ‚ Ρ‚Π°ΠΊΠΎΠ΅ Π½Π°ΠΏΡ€Π°Π²Π»Π΅Π½ΠΈΠ΅, Ρ‡Ρ‚ΠΎ создаваСмоС ΠΈΠΌ ΠΌΠ°Π³Π½ΠΈΡ‚Π½ΠΎΠ΅ ΠΏΠΎΠ»Π΅ стрСмится ΠΊΠΎΠΌΠΏΠ΅Π½ΡΠΈΡ€ΠΎΠ²Π°Ρ‚ΡŒ ΠΈΠ·ΠΌΠ΅Π½Π΅Π½ΠΈΠ΅ ΠΌΠ°Π³Π½ΠΈΡ‚Π½ΠΎΠ³ΠΎ ΠΏΠΎΡ‚ΠΎΠΊΠ°, Π²Ρ‹Π·Π²Π°Π²ΡˆΠ΅Π΅ этот ΠΈΠ½Π΄ΡƒΠΊΡ†ΠΈΠΎΠ½Π½Ρ‹ΠΉ Ρ‚ΠΎΠΊ.

НапримСр, Ссли Π²Π΄Π²ΠΈΠ³Π°Ρ‚ΡŒ сСвСрный полюс ΠΌΠ°Π³Π½ΠΈΡ‚Π° Π² ΠΊΠ°Ρ‚ΡƒΡˆΠΊΡƒ, Ρ‚. Π΅. ΡƒΠ²Π΅Π»ΠΈΡ‡ΠΈΠ²Π°Ρ‚ΡŒ ΠΌΠ°Π³Π½ΠΈΡ‚Π½Ρ‹ΠΉ ΠΏΠΎΡ‚ΠΎΠΊ Ρ‡Π΅Ρ€Π΅Π· Π΅Π³ΠΎ Π²ΠΈΡ‚ΠΊΠΈ, Π² ΠΊΠ°Ρ‚ΡƒΡˆΠΊΠ΅ Π²ΠΎΠ·Π½ΠΈΠΊΠ°Π΅Ρ‚ ΠΈΠ½Π΄ΡƒΠΊΡ†ΠΈΠΎΠ½Π½Ρ‹ΠΉ Ρ‚ΠΎΠΊ Ρ‚Π°ΠΊΠΎΠ³ΠΎ направлСния, Ρ‡Ρ‚ΠΎ Π½Π° блиТайшСм ΠΊ ΠΌΠ°Π³Π½ΠΈΡ‚Ρƒ ΠΊΠΎΠ½Ρ†Π΅ ΠΊΠ°Ρ‚ΡƒΡˆΠΊΠΈ Π²ΠΎΠ·Π½ΠΈΠΊΠ°Π΅Ρ‚ сСвСрный полюс (рис.3.20). Π˜Ρ‚Π°ΠΊ, ΠΌΠ°Π³Π½ΠΈΡ‚Π½ΠΎΠ΅ ΠΏΠΎΠ»Π΅ ΠΈΠ½Π΄ΡƒΠΊΡ†ΠΈΠΎΠ½Π½ΠΎΠ³ΠΎ Ρ‚ΠΎΠΊΠ° стрСмится Π½Π΅ΠΉΡ‚Ρ€Π°Π»ΠΈΠ·ΠΎΠ²Π°Ρ‚ΡŒ Π²Ρ‹Π·Π²Π°Π²ΡˆΠ΅Π΅ Π΅Π³ΠΎ ΠΈΠ·ΠΌΠ΅Π½Π΅Π½ΠΈΠ΅ ΠΌΠ°Π³Π½ΠΈΡ‚Π½ΠΎΠ³ΠΎ ΠΏΠΎΡ‚ΠΎΠΊΠ°.

НС Ρ‚ΠΎΠ»ΡŒΠΊΠΎ ΠΏΠ΅Ρ€Π΅ΠΌΠ΅Π½Π½ΠΎΠ΅ ΠΌΠ°Π³Π½ΠΈΡ‚Π½ΠΎΠ΅ ΠΏΠΎΠ»Π΅ ΠΏΠΎΡ€ΠΎΠΆΠ΄Π°Π΅Ρ‚ ΠΈΠ½Π΄ΡƒΠΊΡ†ΠΈΠΎΠ½Π½Ρ‹ΠΉ Ρ‚ΠΎΠΊ Π² Π·Π°ΠΌΠΊΠ½ΡƒΡ‚ΠΎΠΌ ΠΏΡ€ΠΎΠ²ΠΎΠ΄Π½ΠΈΠΊΠ΅, Π½ΠΎ ΠΈ ΠΏΡ€ΠΈ Π΄Π²ΠΈΠΆΠ΅Π½ΠΈΠΈ Π·Π°ΠΌΠΊΠ½ΡƒΡ‚ΠΎΠ³ΠΎ ΠΏΡ€ΠΎΠ²ΠΎΠ΄Π½ΠΈΠΊΠ° Π΄Π»ΠΈΠ½ΠΎΠΉ l Π² постоянном ΠΌΠ°Π³Π½ΠΈΡ‚Π½ΠΎΠΌ ΠΏΠΎΠ»Π΅ (Π’) со ΡΠΊΠΎΡ€ΠΎΡΡ‚ΡŒΡŽ v Π² ΠΏΡ€ΠΎΠ²ΠΎΠ΄Π½ΠΈΠΊΠ΅ Π²ΠΎΠ·Π½ΠΈΠΊΠ°Π΅Ρ‚ эдс:

a (B Γ™v) (3. 47)

Как Π²Ρ‹ ΡƒΠΆΠ΅ Π·Π½Π°Π΅Ρ‚Π΅, элСктродвиТущая сила Π² цСпи– это Ρ€Π΅Π·ΡƒΠ»ΡŒΡ‚Π°Ρ‚ дСйствия сторонних сил. ΠŸΡ€ΠΈ Π΄Π²ΠΈΠΆΠ΅Π½ΠΈΠΈ ΠΏΡ€ΠΎΠ²ΠΎΠ΄Π½ΠΈΠΊΠ° Π² ΠΌΠ°Π³Π½ΠΈΡ‚Π½ΠΎΠΌ ΠΏΠΎΠ»Π΅ Ρ€ΠΎΠ»ΡŒ сторонних сил выполняСт сила Π›ΠΎΡ€Π΅Π½Ρ†Π° (которая дСйствуСт со стороны ΠΌΠ°Π³Π½ΠΈΡ‚Π½ΠΎΠ³ΠΎ поля Π½Π° двиТущийся элСктричСский заряд). Под дСйствиСм этой силы происходит Ρ€Π°Π·Π΄Π΅Π»Π΅Π½ΠΈΠ΅ зарядов ΠΈ Π½Π° ΠΊΠΎΠ½Ρ†Π°Ρ… ΠΏΡ€ΠΎΠ²ΠΎΠ΄Π½ΠΈΠΊΠ° Π²ΠΎΠ·Π½ΠΈΠΊΠ°Π΅Ρ‚ Ρ€Π°Π·Π½ΠΎΡΡ‚ΡŒ ΠΏΠΎΡ‚Π΅Π½Ρ†ΠΈΠ°Π»ΠΎΠ². Π­.Π΄.с. ΠΈΠ½Π΄ΡƒΠΊΡ†ΠΈΠΈ Π² ΠΏΡ€ΠΎΠ²ΠΎΠ΄Π½ΠΈΠΊΠ΅ являСтся Ρ€Π°Π±ΠΎΡ‚ΠΎΠΉ ΠΏΠΎ ΠΏΠ΅Ρ€Π΅ΠΌΠ΅Ρ‰Π΅Π½ΠΈΡŽ Π΅Π΄ΠΈΠ½ΠΈΡ‡Π½Ρ‹Ρ… зарядов вдоль ΠΏΡ€ΠΎΠ²ΠΎΠ΄Π½ΠΈΠΊΠ°.

НаправлСниС ΠΈΠ½Π΄ΡƒΠΊΡ†ΠΈΠΎΠ½Π½ΠΎΠ³ΠΎ Ρ‚ΠΎΠΊΠ° ΠΌΠΎΠΆΠ½ΠΎ ΠΎΠΏΡ€Π΅Π΄Π΅Π»ΠΈΡ‚ΡŒΠΏΠΎ ΠΏΡ€Π°Π²ΠΈΠ»Ρƒ ΠΏΡ€Π°Π²ΠΎΠΉ Ρ€ΡƒΠΊΠΈ: Π’Π΅ΠΊΡ‚ΠΎΡ€ Π’ Π²Ρ…ΠΎΠ΄ΠΈΡ‚ Π² ладонь, ΠΎΡ‚Π²Π΅Π΄Π΅Π½Π½Ρ‹ΠΉ большой ΠΏΠ°Π»Π΅Ρ† совпадаСт с Π½Π°ΠΏΡ€Π°Π²Π»Π΅Π½ΠΈΠ΅ΠΌ скорости ΠΏΡ€ΠΎΠ²ΠΎΠ΄Π½ΠΈΠΊΠ°, Π° 4 ΠΏΠ°Π»ΡŒΡ†Π° ΡƒΠΊΠ°ΠΆΡƒΡ‚ Π½Π°ΠΏΡ€Π°Π²Π»Π΅Π½ΠΈΠ΅ ΠΈΠ½Π΄ΡƒΠΊΡ†ΠΈΠΎΠ½Π½ΠΎΠ³ΠΎ Ρ‚ΠΎΠΊΠ°.

Π’Π°ΠΊΠΈΠΌ ΠΎΠ±Ρ€Π°Π·ΠΎΠΌ ΠΏΠ΅Ρ€Π΅ΠΌΠ΅Π½Π½ΠΎΠ΅ ΠΌΠ°Π³Π½ΠΈΡ‚Π½ΠΎΠ΅ ΠΏΠΎΠ»Π΅ Π²Ρ‹Π·Ρ‹Π²Π°Π΅Ρ‚ появлСниС ΠΈΠ½Π΄ΡƒΡ†ΠΈΡ€ΠΎΠ²Π°Π½Π½ΠΎΠ³ΠΎ элСктричСского поля. Оно Π½Π΅ ΠΏΠΎΡ‚Π΅Π½Ρ†ΠΈΠ°Π»ΡŒΠ½ΠΎ (Π² ΠΎΡ‚Π»ΠΈΡ‡ΠΈΠ΅ ΠΎΡ‚ элСктростатичСского), Ρ‚.ΠΊ. Ρ€Π°Π±ΠΎΡ‚Π° ΠΏΠΎ ΠΏΠ΅Ρ€Π΅ΠΌΠ΅Ρ‰Π΅Π½ΠΈΡŽ Π΅Π΄ΠΈΠ½ΠΈΡ‡Π½ΠΎΠ³ΠΎ ΠΏΠΎΠ»ΠΎΠΆΠΈΡ‚Π΅Π»ΡŒΠ½ΠΎΠ³ΠΎ заряда Ρ€Π°Π²Π½Π° э.Π΄.с. ΠΈΠ½Π΄ΡƒΠΊΡ†ΠΈΠΈ , Π° Π½Π΅ Π½ΡƒΠ»ΡŽ.

Π’Π°ΠΊΠΈΠ΅ поля Π½Π°Π·Ρ‹Π²Π°ΡŽΡ‚ΡΡ Π²ΠΈΡ…Ρ€Π΅Π²Ρ‹ΠΌΠΈ. Π‘ΠΈΠ»ΠΎΠ²Ρ‹Π΅ Π»ΠΈΠ½ΠΈΠΈ Π²ΠΈΡ…Ρ€Π΅Π²ΠΎΠ³ΠΎ элСктричСского поля – Π·Π°ΠΌΠΊΠ½ΡƒΡ‚Ρ‹ сами Π½Π° сСбя, Π² ΠΎΡ‚Π»ΠΈΡ‡ΠΈΠ΅ ΠΎΡ‚ Π»ΠΈΠ½ΠΈΠΉ напряТСнности элСктростатичСского поля.

Π­.Π΄.с. ΠΈΠ½Π΄ΡƒΠΊΡ†ΠΈΠΈ Π²ΠΎΠ·Π½ΠΈΠΊΠ°Π΅Ρ‚ Π½Π΅ Ρ‚ΠΎΠ»ΡŒΠΊΠΎ Π² сосСдних ΠΏΡ€ΠΎΠ²ΠΎΠ΄Π½ΠΈΠΊΠ°Ρ…, Π½ΠΎ ΠΈ Π² самом ΠΏΡ€ΠΎΠ²ΠΎΠ΄Π½ΠΈΠΊΠ΅ ΠΏΡ€ΠΈ ΠΈΠ·ΠΌΠ΅Π½Π΅Π½ΠΈΠΈ ΠΌΠ°Π³Π½ΠΈΡ‚Π½ΠΎΠ³ΠΎ поля Ρ‚ΠΎΠΊΠ°, ΠΈΠ΄ΡƒΡ‰Π΅Π³ΠΎ ΠΏΠΎ ΠΏΡ€ΠΎΠ²ΠΎΠ΄Π½ΠΈΠΊΡƒ. Π’ΠΎΠ·Π½ΠΈΠΊΠ½ΠΎΠ²Π΅Π½ΠΈΠ΅ э.Π΄.с. Π² ΠΊΠ°ΠΊΠΎΠΌ-Π»ΠΈΠ±ΠΎ ΠΏΡ€ΠΎΠ²ΠΎΠ΄Π½ΠΈΠΊΠ΅ ΠΏΡ€ΠΈ ΠΈΠ·ΠΌΠ΅Π½Π΅Π½ΠΈΠΈ Π² Π½Π΅ΠΌ самом силы Ρ‚ΠΎΠΊΠ° (ΡΠ»Π΅Π΄ΠΎΠ²Π°Ρ‚Π΅Π»ΡŒΠ½ΠΎ, ΠΌΠ°Π³Π½ΠΈΡ‚Π½ΠΎΠ³ΠΎ ΠΏΠΎΡ‚ΠΎΠΊΠ° Π² ΠΏΡ€ΠΎΠ²ΠΎΠ΄Π½ΠΈΠΊΠ΅) называСтся самоиндукциСй, Π° Ρ‚ΠΎΠΊ, ΠΈΠ½Π΄ΡƒΡ†ΠΈΡ€ΡƒΠ΅ΠΌΡ‹ΠΉ Π² этом ΠΏΡ€ΠΎΠ²ΠΎΠ΄Π½ΠΈΠΊΠ΅, – Ρ‚ΠΎΠΊΠΎΠΌ самоиндукции.

Π’ΠΎΠΊ Π² Π·Π°ΠΌΠΊΠ½ΡƒΡ‚ΠΎΠΌ ΠΊΠΎΠ½Ρ‚ΡƒΡ€Π΅ создаСт Π² ΠΎΠΊΡ€ΡƒΠΆΠ°ΡŽΡ‰Π΅ΠΌ пространствС ΠΌΠ°Π³Π½ΠΈΡ‚Π½ΠΎΠ΅ ΠΏΠΎΠ»Π΅, Π½Π°ΠΏΡ€ΡΠΆΠ΅Π½Π½ΠΎΡΡ‚ΡŒ ΠΊΠΎΡ‚ΠΎΡ€ΠΎΠ³ΠΎ ΠΏΡ€ΠΎΠΏΠΎΡ€Ρ†ΠΈΠΎΠ½Π°Π»ΡŒΠ½Π° силС Ρ‚ΠΎΠΊΠ° I. ΠŸΠΎΡΡ‚ΠΎΠΌΡƒ ΠΌΠ°Π³Π½ΠΈΡ‚Π½Ρ‹ΠΉ ΠΏΠΎΡ‚ΠΎΠΊ Π€, ΠΏΡ€ΠΎΠ½ΠΈΠ·Ρ‹Π²Π°ΡŽΡ‰ΠΈΠΉ ΠΊΠΎΠ½Ρ‚ΡƒΡ€, ΠΏΡ€ΠΎΠΏΠΎΡ€Ρ†ΠΈΠΎΠ½Π°Π»Π΅Π½ силС Ρ‚ΠΎΠΊΠ° Π² ΠΊΠΎΠ½Ρ‚ΡƒΡ€Π΅

Π€=LΓ—I, (3.48).

L – коэффициСнт ΠΏΡ€ΠΎΠΏΠΎΡ€Ρ†ΠΈΠΎΠ½Π°Π»ΡŒΠ½ΠΎΡΡ‚ΠΈ, ΠΊΠΎΡ‚ΠΎΡ€Ρ‹ΠΉ носит Π½Π°Π·Π²Π°Π½ΠΈΠ΅ коэффициСнта самоиндукции, ΠΈΠ»ΠΈ, просто, индуктивности. Π˜Π½Π΄ΡƒΠΊΡ‚ΠΈΠ²Π½ΠΎΡΡ‚ΡŒ зависит ΠΎΡ‚ Ρ€Π°Π·ΠΌΠ΅Ρ€ΠΎΠ² ΠΈ Ρ„ΠΎΡ€ΠΌΡ‹ ΠΊΠΎΠ½Ρ‚ΡƒΡ€Π°, Π° Ρ‚Π°ΠΊΠΆΠ΅ ΠΎΡ‚ ΠΌΠ°Π³Π½ΠΈΡ‚Π½ΠΎΠΉ проницаСмости срСды, ΠΎΠΊΡ€ΡƒΠΆΠ°ΡŽΡ‰Π΅ΠΉ ΠΊΠΎΠ½Ρ‚ΡƒΡ€.

Π’ этом смыслС ΠΈΠ½Π΄ΡƒΠΊΡ‚ΠΈΠ²Π½ΠΎΡΡ‚ΡŒ ΠΊΠΎΠ½Ρ‚ΡƒΡ€Π° – Π°Π½Π°Π»ΠΎΠ³ элСктричСской Смкости ΡƒΠ΅Π΄ΠΈΠ½Π΅Π½Π½ΠΎΠ³ΠΎ ΠΏΡ€ΠΎΠ²ΠΎΠ΄Π½ΠΈΠΊΠ°, которая Ρ‚Π°ΠΊΠΆΠ΅ зависит Ρ‚ΠΎΠ»ΡŒΠΊΠΎ ΠΎΡ‚ Ρ„ΠΎΡ€ΠΌΡ‹ ΠΏΡ€ΠΎΠ²ΠΎΠ΄Π½ΠΈΠΊΠ°, Π΅Π³ΠΎ Ρ€Π°Π·ΠΌΠ΅Ρ€ΠΎΠ² ΠΈ диэлСктричСской проницаСмости срСды.

Π•Π΄ΠΈΠ½ΠΈΡ†Π° индуктивности – Π³Π΅Π½Ρ€ΠΈ (Π“Π½) : 1Π“Π½ – ΠΈΠ½Π΄ΡƒΠΊΡ‚ΠΈΠ²Π½ΠΎΡΡ‚ΡŒ Ρ‚Π°ΠΊΠΎΠ³ΠΎ ΠΊΠΎΠ½Ρ‚ΡƒΡ€Π°, ΠΌΠ°Π³Π½ΠΈΡ‚Π½Ρ‹ΠΉ ΠΏΠΎΡ‚ΠΎΠΊ самоиндукции ΠΊΠΎΡ‚ΠΎΡ€ΠΎΠ³ΠΎ ΠΏΡ€ΠΈ Ρ‚ΠΎΠΊΠ΅ Π² 1А Ρ€Π°Π²Π΅Π½ 1Π’Π± (1Π“Π½=1Π’Π±/А=1В·с/А).

Если L=const, Ρ‚ΠΎ э.Π΄.с. самоиндукции ΠΌΠΎΠΆΠ½ΠΎ ΠΏΡ€Π΅Π΄ΡΡ‚Π°Π²ΠΈΡ‚ΡŒ Π² ΡΠ»Π΅Π΄ΡƒΡŽΡ‰Π΅ΠΌ Π²ΠΈΠ΄Π΅:

, ΠΈΠ»ΠΈ , (3.49)

Π³Π΄Π΅ DI (dI) – ΠΈΠ·ΠΌΠ΅Π½Π΅Π½ΠΈΠ΅ Ρ‚ΠΎΠΊΠ° Π² Ρ†Π΅ΠΏΠΈ, содСрТащСй ΠΊΠ°Ρ‚ΡƒΡˆΠΊΡƒ индуктивности (ΠΈΠ»ΠΈ ΠΊΠΎΠ½Ρ‚ΡƒΡ€) L, Π·Π° врСмя Dt (dt). Π—Π½Π°ΠΊ «–» Π² этом Π²Ρ‹Ρ€Π°ΠΆΠ΅Π½ΠΈΠΈ ΠΎΠ·Π½Π°Ρ‡Π°Π΅Ρ‚, Ρ‡Ρ‚ΠΎ э.Π΄.с. самоиндукции прСпятствуСт измСнСнию Ρ‚ΠΎΠΊΠ° (Ρ‚. Π΅. Ссли Ρ‚ΠΎΠΊ Π² Π·Π°ΠΌΠΊΠ½ΡƒΡ‚ΠΎΠΌ ΠΊΠΎΠ½Ρ‚ΡƒΡ€Π΅ ΡƒΠΌΠ΅Π½ΡŒΡˆΠ°Π΅Ρ‚ΡΡ, Ρ‚ΠΎ э.Π΄.с. самоиндукции ΠΏΡ€ΠΈΠ²ΠΎΠ΄ΠΈΡ‚ ΠΊ возникновСнию Ρ‚ΠΎΠΊΠ° Ρ‚ΠΎΠ³ΠΎ ΠΆΠ΅ направлСния ΠΈ Π½Π°ΠΎΠ±ΠΎΡ€ΠΎΡ‚).

Одним ΠΈΠ· проявлСний элСктромагнитной ΠΈΠ½Π΄ΡƒΠΊΡ†ΠΈΠΈ являСтся Π²ΠΎΠ·Π½ΠΈΠΊΠ½ΠΎΠ²Π΅Π½ΠΈΠ΅ Π·Π°ΠΌΠΊΠ½ΡƒΡ‚Ρ‹Ρ… ΠΈΠ½Π΄ΡƒΠΊΡ†ΠΈΠΎΠ½Π½Ρ‹Ρ… Ρ‚ΠΎΠΊΠΎΠ² Π² ΡΠΏΠ»ΠΎΡˆΠ½Ρ‹Ρ… проводящих срСдах: мСталличСских Ρ‚Π΅Π»Π°Ρ…, растворах элСктролитов, биологичСских ΠΎΡ€Π³Π°Π½Π°Ρ… ΠΈ Ρ‚. Π΄. Π’Π°ΠΊΠΈΠ΅ Ρ‚ΠΎΠΊΠΈ носят Π½Π°Π·Π²Π°Π½ΠΈΠ΅ Π²ΠΈΡ…Ρ€Π΅Π²Ρ‹Ρ… Ρ‚ΠΎΠΊΠΎΠ² ΠΈΠ»ΠΈ Ρ‚ΠΎΠΊΠΎΠ² Π€ΡƒΠΊΠΎ. Π­Ρ‚ΠΈ Ρ‚ΠΎΠΊΠΈ Π²ΠΎΠ·Π½ΠΈΠΊΠ°ΡŽΡ‚ ΠΏΡ€ΠΈ ΠΏΠ΅Ρ€Π΅ΠΌΠ΅Ρ‰Π΅Π½ΠΈΠΈ проводящСго Ρ‚Π΅Π»Π° Π² ΠΌΠ°Π³Π½ΠΈΡ‚Π½ΠΎΠΌ ΠΏΠΎΠ»Π΅ ΠΈ/ΠΈΠ»ΠΈ ΠΏΡ€ΠΈ ΠΈΠ·ΠΌΠ΅Π½Π΅Π½ΠΈΠΈ со Π²Ρ€Π΅ΠΌΠ΅Π½Π΅ΠΌ ΠΈΠ½Π΄ΡƒΠΊΡ†ΠΈΠΈ поля, Π² ΠΊΠΎΡ‚ΠΎΡ€ΠΎΠ΅ ΠΏΠΎΠΌΠ΅Ρ‰Π΅Π½Ρ‹ Ρ‚Π΅Π»Π°. Π‘ΠΈΠ»Π° Ρ‚ΠΎΠΊΠΎΠ² Π€ΡƒΠΊΠΎ зависит ΠΎΡ‚ элСктричСского сопротивлСния Ρ‚Π΅Π», Π° Ρ‚Π°ΠΊΠΆΠ΅ ΠΎΡ‚ скорости измСнСния ΠΌΠ°Π³Π½ΠΈΡ‚Π½ΠΎΠ³ΠΎ поля.

Π’ΠΎΠΊΠΈ Π€ΡƒΠΊΠΎ Ρ‚Π°ΠΊΠΆΠ΅ ΠΏΠΎΠ΄Ρ‡ΠΈΠ½ΡΡŽΡ‚ΡΡ ΠΏΡ€Π°Π²ΠΈΠ»Ρƒ Π›Π΅Π½Ρ†Π° : ΠΈΡ… ΠΌΠ°Π³Π½ΠΈΡ‚Π½ΠΎΠ΅ ΠΏΠΎΠ»Π΅ Π½Π°ΠΏΡ€Π°Π²Π»Π΅Π½ΠΎ Ρ‚Π°ΠΊ, Ρ‡Ρ‚ΠΎΠ±Ρ‹ ΠΏΡ€ΠΎΡ‚ΠΈΠ²ΠΎΠ΄Π΅ΠΉΡΡ‚Π²ΠΎΠ²Π°Ρ‚ΡŒ измСнСнию ΠΌΠ°Π³Π½ΠΈΡ‚Π½ΠΎΠ³ΠΎ ΠΏΠΎΡ‚ΠΎΠΊΠ°, ΠΈΠ½Π΄ΡƒΡ†ΠΈΡ€ΡƒΡŽΡ‰Π΅ΠΌΡƒ Π²ΠΈΡ…Ρ€Π΅Π²Ρ‹Π΅ Ρ‚ΠΎΠΊΠΈ.

ΠŸΠΎΡΡ‚ΠΎΠΌΡƒ массивныС ΠΏΡ€ΠΎΠ²ΠΎΠ΄Π½ΠΈΠΊΠΈ тормозятся Π² ΠΌΠ°Π³Π½ΠΈΡ‚Π½ΠΎΠΌ ΠΏΠΎΠ»Π΅. Π’ элСктричСских ΠΌΠ°ΡˆΠΈΠ½Π°Ρ…, для Ρ‚ΠΎΠ³ΠΎ Ρ‡Ρ‚ΠΎΠ±Ρ‹ ΠΌΠΈΠ½ΠΈΠΌΠΈΠ·ΠΈΡ€ΠΎΠ²Π°Ρ‚ΡŒ влияниС Ρ‚ΠΎΠΊΠΎΠ² Π€ΡƒΠΊΠΎ, сСрдСчники трансформаторов ΠΈ ΠΌΠ°Π³Π½ΠΈΡ‚Π½Ρ‹Π΅ Ρ†Π΅ΠΏΠΈ элСктричСских машин ΡΠΎΠ±ΠΈΡ€Π°ΡŽΡ‚ ΠΈΠ· Ρ‚ΠΎΠ½ΠΊΠΈΡ… пластин, ΠΈΠ·ΠΎΠ»ΠΈΡ€ΠΎΠ²Π°Π½Π½Ρ‹Ρ… Π΄Ρ€ΡƒΠ³ ΠΎΡ‚ Π΄Ρ€ΡƒΠ³Π° ΡΠΏΠ΅Ρ†ΠΈΠ°Π»ΡŒΠ½Ρ‹ΠΌ Π»Π°ΠΊΠΎΠΌ ΠΈΠ»ΠΈ ΠΎΠΊΠ°Π»ΠΈΠ½ΠΎΠΉ.

Π’ΠΈΡ…Ρ€Π΅Π²Ρ‹Π΅ Ρ‚ΠΎΠΊΠΈ Π²Ρ‹Π·Ρ‹Π²Π°ΡŽΡ‚ сильноС Π½Π°Π³Ρ€Π΅Π²Π°Π½ΠΈΠ΅ ΠΏΡ€ΠΎΠ²ΠΎΠ΄Π½ΠΈΠΊΠΎΠ². Π”ΠΆΠΎΡƒΠ»Π΅Π²ΠΎ Ρ‚Π΅ΠΏΠ»ΠΎ, выдСляСмоС Ρ‚ΠΎΠΊΠ°ΠΌΠΈ Π€ΡƒΠΊΠΎ , ΠΈΡΠΏΠΎΠ»ΡŒΠ·ΡƒΠ΅Ρ‚ΡΡ Π² ΠΈΠ½Π΄ΡƒΠΊΡ†ΠΈΠΎΠ½Π½Ρ‹Ρ… мСталлургичСских ΠΏΠ΅Ρ‡Π°Ρ… для ΠΏΠ»Π°Π²ΠΊΠΈ ΠΌΠ΅Ρ‚Π°Π»Π»ΠΎΠ², согласно Π·Π°ΠΊΠΎΠ½Ρƒ ДТоуля-Π›Π΅Π½Ρ†Π° .

Π Π΅ΠΊΠΎΠΌΠ΅Π½Π΄ΡƒΠ΅ΠΌ Ρ‚Π°ΠΊΠΆΠ΅

По ΠΊΠ°ΠΊΠΎΠΉ Ρ„ΠΎΡ€ΠΌΡƒΠ»Π΅ рассчитываСтся Π·Π°ΠΊΠΎΠ½ элСктромагнитной ΠΈΠ½Π΄ΡƒΠΊΡ†ΠΈΠΈ. Π—Π°ΠΊΠΎΠ½ элСктромагнитной ΠΈΠ½Π΄ΡƒΠΊΡ†ΠΈΠΈ Ρ„ΠΎΡ€ΠΌΡƒΠ»Π°

Π’ 1831 Π³ΠΎΠ΄Ρƒ ΠΌΠΈΡ€ Π²ΠΏΠ΅Ρ€Π²Ρ‹Π΅ ΡƒΠ·Π½Π°Π» ΠΎ понятии элСктромагнитной ΠΈΠ½Π΄ΡƒΠΊΡ†ΠΈΠΈ. ИмСнно Ρ‚ΠΎΠ³Π΄Π° Майкл Π€Π°Ρ€Π°Π΄Π΅ΠΉ ΠΎΠ±Π½Π°Ρ€ΡƒΠΆΠΈΠ» это явлСниС, ΡΡ‚Π°Π²ΡˆΠ΅Π΅ Π² ΠΈΡ‚ΠΎΠ³Π΅ ваТнСйшим ΠΎΡ‚ΠΊΡ€Ρ‹Ρ‚ΠΈΠ΅ΠΌ Π² элСктродинамикС.

Π˜ΡΡ‚ΠΎΡ€ΠΈΡ развития ΠΈ ΠΎΠΏΡ‹Ρ‚Ρ‹ ЀарадСя

Π”ΠΎ сСрСдины XIX Π²Π΅ΠΊΠ° ΡΡ‡ΠΈΡ‚Π°Π»ΠΎΡΡŒ, Ρ‡Ρ‚ΠΎ элСктричСскоС ΠΈ ΠΌΠ°Π³Π½ΠΈΡ‚Π½ΠΎΠ΅ ΠΏΠΎΠ»Π΅ Π½Π΅ ΠΈΠΌΠ΅ΡŽΡ‚ Π½ΠΈΠΊΠ°ΠΊΠΎΠΉ связи, ΠΈ ΠΏΡ€ΠΈΡ€ΠΎΠ΄Π° ΠΈΡ… сущСствования Ρ€Π°Π·Π»ΠΈΡ‡Π½Π°. Но М. Π€Π°Ρ€Π°Π΄Π΅ΠΉ Π±Ρ‹Π» ΡƒΠ²Π΅Ρ€Π΅Π½ Π² Π΅Π΄ΠΈΠ½ΠΎΠΉ ΠΏΡ€ΠΈΡ€ΠΎΠ΄Π΅ этих ΠΏΠΎΠ»Π΅ΠΉ ΠΈ ΠΈΡ… свойств. Π―Π²Π»Π΅Π½ΠΈΠ΅ элСктромагнитной ΠΈΠ½Π΄ΡƒΠΊΡ†ΠΈΠΈ, ΠΎΠ±Π½Π°Ρ€ΡƒΠΆΠ΅Π½Π½ΠΎΠ΅ ΠΈΠΌ, впослСдствии стало Ρ„ΡƒΠ½Π΄Π°ΠΌΠ΅Π½Ρ‚ΠΎΠΌ для устройства Π³Π΅Π½Π΅Ρ€Π°Ρ‚ΠΎΡ€ΠΎΠ² всСх элСктростанций. Благодаря этому ΠΎΡ‚ΠΊΡ€Ρ‹Ρ‚ΠΈΡŽ знания чСловСчСства ΠΎ элСктромагнСтизмС ΡˆΠ°Π³Π½ΡƒΠ»ΠΈ Π΄Π°Π»Π΅ΠΊΠΎ Π²ΠΏΠ΅Ρ€Π΅Π΄.

Π€Π°Ρ€Π°Π΄Π΅ΠΉ ΠΏΡ€ΠΎΠ΄Π΅Π»Π°Π» ΡΠ»Π΅Π΄ΡƒΡŽΡ‰ΠΈΠΉ ΠΎΠΏΡ‹Ρ‚: ΠΎΠ½ Π·Π°ΠΌΡ‹ΠΊΠ°Π» Ρ†Π΅ΠΏΡŒ Π² ΠΊΠ°Ρ‚ΡƒΡˆΠΊΠ΅ I ΠΈ Π²ΠΎΠΊΡ€ΡƒΠ³ Π½Π΅Π΅ возрастало ΠΌΠ°Π³Π½ΠΈΡ‚Π½ΠΎΠ΅ ΠΏΠΎΠ»Π΅. Π”Π°Π»Π΅Π΅ Π»ΠΈΠ½ΠΈΠΈ ΠΈΠ½Π΄ΡƒΠΊΡ†ΠΈΠΈ Π΄Π°Π½Π½ΠΎΠ³ΠΎ ΠΌΠ°Π³Π½ΠΈΡ‚Π½ΠΎΠ³ΠΎ поля пСрСсСкали ΠΊΠ°Ρ‚ΡƒΡˆΠΊΡƒ II, Π² ΠΊΠΎΡ‚ΠΎΡ€ΠΎΠΉ Π²ΠΎΠ·Π½ΠΈΠΊΠ°Π» ΠΈΠ½Π΄ΡƒΠΊΡ†ΠΈΠΎΠ½Π½Ρ‹ΠΉ Ρ‚ΠΎΠΊ.

Рис. 1. Π‘Ρ…Π΅ΠΌΠ° ΠΎΠΏΡ‹Ρ‚Π° ЀарадСя

На самом Π΄Π΅Π»Π΅, ΠΎΠ΄Π½ΠΎΠ²Ρ€Π΅ΠΌΠ΅Π½Π½ΠΎ с Π€Π°Ρ€Π°Π΄Π΅Π΅ΠΌ, Π½ΠΎ нСзависимо ΠΎΡ‚ Π½Π΅Π³ΠΎ, Π΄Ρ€ΡƒΠ³ΠΎΠΉ ΡƒΡ‡Π΅Π½Ρ‹ΠΉ Π”ΠΆΠΎΠ·Π΅Ρ„ Π“Π΅Π½Ρ€ΠΈ ΠΎΠ±Π½Π°Ρ€ΡƒΠΆΠΈΠ» это явлСниС. Однако Π€Π°Ρ€Π°Π΄Π΅ΠΉ ΠΎΠΏΡƒΠ±Π»ΠΈΠΊΠΎΠ²Π°Π» свои исслСдования Ρ€Π°Π½ΡŒΡˆΠ΅. Π’Π°ΠΊΠΈΠΌ ΠΎΠ±Ρ€Π°Π·ΠΎΠΌ, Π°Π²Ρ‚ΠΎΡ€ΠΎΠΌ Π·Π°ΠΊΠΎΠ½Π° элСктромагнитной ΠΈΠ½Π΄ΡƒΠΊΡ†ΠΈΠΈ стал Майкл Π€Π°Ρ€Π°Π΄Π΅ΠΉ.

Бколько Π±Ρ‹ экспСримСнтов Π½Π΅ ΠΏΡ€ΠΎΠ²ΠΎΠ΄ΠΈΠ» Π€Π°Ρ€Π°Π΄Π΅ΠΉ, Π½Π΅ΠΈΠ·ΠΌΠ΅Π½Π½Ρ‹ΠΌ ΠΎΡΡ‚Π°Π²Π°Π»ΠΎΡΡŒ ΠΎΠ΄Π½ΠΎ условиС: для образования ΠΈΠ½Π΄ΡƒΠΊΡ†ΠΈΠΎΠ½Π½ΠΎΠ³ΠΎ Ρ‚ΠΎΠΊΠ° Π²Π°ΠΆΠ½Ρ‹ΠΌ являСтся ΠΈΠ·ΠΌΠ΅Π½Π΅Π½ΠΈΠ΅ ΠΌΠ°Π³Π½ΠΈΡ‚Π½ΠΎΠ³ΠΎ ΠΏΠΎΡ‚ΠΎΠΊΠ°, ΠΏΡ€ΠΎΠ½ΠΈΠ·Ρ‹Π²Π°ΡŽΡ‰Π΅Π³ΠΎ Π·Π°ΠΌΠΊΠ½ΡƒΡ‚Ρ‹ΠΉ проводящий ΠΊΠΎΠ½Ρ‚ΡƒΡ€ (ΠΊΠ°Ρ‚ΡƒΡˆΠΊΡƒ).

Π—Π°ΠΊΠΎΠ½ ЀарадСя

Π―Π²Π»Π΅Π½ΠΈΠ΅ элСктромагнитной ΠΈΠ½Π΄ΡƒΠΊΡ†ΠΈΠΈ опрСдСляСтся Π²ΠΎΠ·Π½ΠΈΠΊΠ½ΠΎΠ²Π΅Π½ΠΈΠ΅ΠΌ элСктричСского Ρ‚ΠΎΠΊΠ° Π² Π·Π°ΠΌΠΊΠ½ΡƒΡ‚ΠΎΠΌ элСктропроводящСм ΠΊΠΎΠ½Ρ‚ΡƒΡ€Π΅ ΠΏΡ€ΠΈ ΠΈΠ·ΠΌΠ΅Π½Π΅Π½ΠΈΠΈ ΠΌΠ°Π³Π½ΠΈΡ‚Π½ΠΎΠ³ΠΎ ΠΏΠΎΡ‚ΠΎΠΊΠ° Ρ‡Π΅Ρ€Π΅Π· ΠΏΠ»ΠΎΡ‰Π°Π΄ΡŒ этого ΠΊΠΎΠ½Ρ‚ΡƒΡ€Π°.

Основной Π·Π°ΠΊΠΎΠ½ ЀарадСя Π·Π°ΠΊΠ»ΡŽΡ‡Π°Π΅Ρ‚ΡΡ Π² Ρ‚ΠΎΠΌ, Ρ‡Ρ‚ΠΎ элСктродвиТущая сила (Π­Π”Π‘) прямо ΠΏΡ€ΠΎΠΏΠΎΡ€Ρ†ΠΈΠΎΠ½Π°Π»ΡŒΠ½Π° скорости измСнСния ΠΌΠ°Π³Π½ΠΈΡ‚Π½ΠΎΠ³ΠΎ ΠΏΠΎΡ‚ΠΎΠΊΠ°.

Π€ΠΎΡ€ΠΌΡƒΠ»Π° Π·Π°ΠΊΠΎΠ½Π° элСктромагнитной ΠΈΠ½Π΄ΡƒΠΊΡ†ΠΈΠΈ ЀарадСя выглядит ΡΠ»Π΅Π΄ΡƒΡŽΡ‰ΠΈΠΌ ΠΎΠ±Ρ€Π°Π·ΠΎΠΌ:

Рис. 2. Π€ΠΎΡ€ΠΌΡƒΠ»Π° Π·Π°ΠΊΠΎΠ½Π° элСктромагнитной ΠΈΠ½Π΄ΡƒΠΊΡ†ΠΈΠΈ

И Ссли сама Ρ„ΠΎΡ€ΠΌΡƒΠ»Π°, исходя ΠΈΠ· Π²Ρ‹ΡˆΠ΅ΡΠΊΠ°Π·Π°Π½Π½Ρ‹Ρ… объяснСний Π½Π΅ ΠΏΠΎΡ€ΠΎΠΆΠ΄Π°Π΅Ρ‚ вопросов, Ρ‚ΠΎ Π·Π½Π°ΠΊ Β«-Β» ΠΌΠΎΠΆΠ΅Ρ‚ Π²Ρ‹Π·Π²Π°Ρ‚ΡŒ сомнСния. ΠžΠΊΠ°Π·Ρ‹Π²Π°Π΅Ρ‚ΡΡ сущСствуСт ΠΏΡ€Π°Π²ΠΈΠ»ΠΎ Π›Π΅Π½Ρ†Π° – русского ΡƒΡ‡Π΅Π½ΠΎΠ³ΠΎ, ΠΊΠΎΡ‚ΠΎΡ€Ρ‹ΠΉ ΠΏΡ€ΠΎΠ²ΠΎΠ΄ΠΈΠ» свои исслСдования, ΠΎΡΠ½ΠΎΠ²Ρ‹Π²Π°ΡΡΡŒ Π½Π° постулатах ЀарадСя. По Π›Π΅Π½Ρ†Ρƒ Π·Π½Π°ΠΊ Β«-Β» ΡƒΠΊΠ°Π·Ρ‹Π²Π°Π΅Ρ‚ Π½Π° Π½Π°ΠΏΡ€Π°Π²Π»Π΅Π½ΠΈΠ΅ Π²ΠΎΠ·Π½ΠΈΠΊΠ°ΡŽΡ‰Π΅ΠΉ Π­Π”Π‘, Ρ‚.Π΅. ΠΈΠ½Π΄ΡƒΠΊΡ†ΠΈΠΎΠ½Π½Ρ‹ΠΉ Ρ‚ΠΎΠΊ Π½Π°ΠΏΡ€Π°Π²Π»Π΅Π½ Ρ‚Π°ΠΊ, Ρ‡Ρ‚ΠΎ ΠΌΠ°Π³Π½ΠΈΡ‚Π½Ρ‹ΠΉ ΠΏΠΎΡ‚ΠΎΠΊ, ΠΊΠΎΡ‚ΠΎΡ€Ρ‹ΠΉ ΠΎΠ½ создаСт, Ρ‡Π΅Ρ€Π΅Π· ΠΏΠ»ΠΎΡ‰Π°Π΄ΡŒ, ΠΎΠ³Ρ€Π°Π½ΠΈΡ‡Π΅Π½Π½ΡƒΡŽ ΠΊΠΎΠ½Ρ‚ΡƒΡ€ΠΎΠΌ, стрСмится ΠΏΡ€Π΅ΠΏΡΡ‚ΡΡ‚Π²ΠΎΠ²Π°Ρ‚ΡŒ Ρ‚ΠΎΠΌΡƒ измСнСнию ΠΏΠΎΡ‚ΠΎΠΊΠ°, ΠΊΠΎΡ‚ΠΎΡ€ΠΎΠ΅ Π²Ρ‹Π·Ρ‹Π²Π°Π΅Ρ‚ Π΄Π°Π½Π½Ρ‹ΠΉ Ρ‚ΠΎΠΊ.

Π—Π°ΠΊΠΎΠ½ ЀарадСя-МаксвСлла

Π’ 1873 Π”ΠΆ.К.МаксвСлл ΠΏΠΎ-Π½ΠΎΠ²ΠΎΠΌΡƒ ΠΈΠ·Π»ΠΎΠΆΠΈΠ» Ρ‚Π΅ΠΎΡ€ΠΈΡŽ элСктромагнитного поля. УравнСния, ΠΊΠΎΡ‚ΠΎΡ€Ρ‹Π΅ ΠΎΠ½ Π²Ρ‹Π²Π΅Π», Π»Π΅Π³Π»ΠΈ Π² основу соврСмСнной Ρ€Π°Π΄ΠΈΠΎΡ‚Π΅Ρ…Π½ΠΈΠΊΠΈ ΠΈ элСктротСхники. Они Π²Ρ‹Ρ€Π°ΠΆΠ°ΡŽΡ‚ΡΡ ΡΠ»Π΅Π΄ΡƒΡŽΡ‰ΠΈΠΌ ΠΎΠ±Ρ€Π°Π·ΠΎΠΌ:

  • Edl = -dΠ€/dt – ΡƒΡ€Π°Π²Π½Π΅Π½ΠΈΠ΅ элСктродвиТущСй силы
  • Hdl = -dN/dt – ΡƒΡ€Π°Π²Π½Π΅Π½ΠΈΠ΅ ΠΌΠ°Π³Π½ΠΈΡ‚ΠΎΠ΄Π²ΠΈΠΆΡƒΡ‰Π΅ΠΉ силы.

Π“Π΄Π΅ E – Π½Π°ΠΏΡ€ΡΠΆΠ΅Π½Π½ΠΎΡΡ‚ΡŒ элСктричСского поля Π½Π° участкС dl; H – Π½Π°ΠΏΡ€ΡΠΆΠ΅Π½Π½ΠΎΡΡ‚ΡŒ ΠΌΠ°Π³Π½ΠΈΡ‚Π½ΠΎΠ³ΠΎ поля Π½Π° участкС dl; N – ΠΏΠΎΡ‚ΠΎΠΊ элСктричСской ΠΈΠ½Π΄ΡƒΠΊΡ†ΠΈΠΈ, t – врСмя.

Π‘ΠΈΠΌΠΌΠ΅Ρ‚Ρ€ΠΈΡ‡Π½Ρ‹ΠΉ Ρ…Π°Ρ€Π°ΠΊΡ‚Π΅Ρ€ Π΄Π°Π½Π½Ρ‹Ρ… ΡƒΡ€Π°Π²Π½Π΅Π½ΠΈΠΉ устанавливаСт связь элСктричСских ΠΈ ΠΌΠ°Π³Π½ΠΈΡ‚Π½Ρ‹Ρ… явлСний, Π° Ρ‚Π°ΠΊΠΆΠ΅ ΠΌΠ°Π³Π½ΠΈΡ‚Π½Ρ‹Ρ… с элСктричСскими. физичСский смысл, ΠΊΠΎΡ‚ΠΎΡ€Ρ‹ΠΌ ΠΎΠΏΡ€Π΅Π΄Π΅Π»ΡΡŽΡ‚ΡΡ эти уравнСния, ΠΌΠΎΠΆΠ½ΠΎ Π²Ρ‹Ρ€Π°Π·ΠΈΡ‚ΡŒ ΡΠ»Π΅Π΄ΡƒΡŽΡ‰ΠΈΠΌΠΈ полоТСниями:

  • Ссли элСктричСскоС ΠΏΠΎΠ»Π΅ измСняСтся, Ρ‚ΠΎ это ΠΈΠ·ΠΌΠ΅Π½Π΅Π½ΠΈΠ΅ всСгда сопровоТдаСтся ΠΌΠ°Π³Π½ΠΈΡ‚Π½Ρ‹ΠΌ ΠΏΠΎΠ»Π΅ΠΌ.
  • Ссли ΠΌΠ°Π³Π½ΠΈΡ‚Π½ΠΎΠ΅ ΠΏΠΎΠ»Π΅ измСняСтся, Ρ‚ΠΎ это ΠΈΠ·ΠΌΠ΅Π½Π΅Π½ΠΈΠ΅ всСгда сопровоТдаСтся элСктричСским ΠΏΠΎΠ»Π΅ΠΌ.

Рис. 3. Π’ΠΎΠ·Π½ΠΈΠΊΠ½ΠΎΠ²Π΅Π½ΠΈΠ΅ Π²ΠΈΡ…Ρ€Π΅Π²ΠΎΠ³ΠΎ ΠΌΠ°Π³Π½ΠΈΡ‚Π½ΠΎΠ³ΠΎ поля

Π’Π°ΠΊΠΆΠ΅ МаксвСлл установил, Ρ‡Ρ‚ΠΎ распространСниС элСктромагнитного поля Ρ€Π°Π²Π½Π° скорости распространСния свСта.

ВсСго ΠΏΠΎΠ»ΡƒΡ‡Π΅Π½ΠΎ ΠΎΡ†Π΅Π½ΠΎΠΊ: 134.

Π’ 1831 Π³ΠΎΠ΄Ρƒ английский ΡƒΡ‡Π΅Π½Ρ‹ΠΉ Ρ„ΠΈΠ·ΠΈΠΊ Π² своих ΠΎΠΏΡ‹Ρ‚Π°Ρ… М.Π€Π°Ρ€Π°Π΄Π΅ΠΉ ΠΎΡ‚ΠΊΡ€Ρ‹Π» явлСниС элСктромагнитной ΠΈΠ½Π΄ΡƒΠΊΡ†ΠΈΠΈ . Π—Π°Ρ‚Π΅ΠΌ ΠΈΠ·ΡƒΡ‡Π΅Π½ΠΈΠ΅ΠΌ этого явлСния занимались русскиС ΡƒΡ‡Π΅Π½Ρ‹ΠΉ Π­.Π₯. Π›Π΅Π½Ρ† ΠΈ Π‘.Π‘.Π―ΠΊΠΎΠ±ΠΈ.

Π’ настоящСС врСмя, Π² основС ΠΌΠ½ΠΎΠ³ΠΈΡ… устройств Π»Π΅ΠΆΠΈΡ‚ явлСниС элСктромагнитной ΠΈΠ½Π΄ΡƒΠΊΡ†ΠΈΠΈ, Π½Π°ΠΏΡ€ΠΈΠΌΠ΅Ρ€ Π² Π΄Π²ΠΈΠ³Π°Ρ‚Π΅Π»Π΅ ΠΈΠ»ΠΈ Π³Π΅Π½Π΅Ρ€Π°Ρ‚ΠΎΡ€Π΅ элСктричСского Ρ‚ΠΎΠΊΠ° Ρ‚ΠΎΠΊΠ°, Π² трансформаторах, Ρ€Π°Π΄ΠΈΠΎΠΏΡ€ΠΈΠ΅ΠΌΠ½ΠΈΠΊΠ°Ρ…, ΠΈ ΠΌΠ½ΠΎΠ³ΠΈΡ… Π΄Ρ€ΡƒΠ³ΠΈΡ… устройствах.

ЭлСктромагнитная индукция – это явлСниС возникновСния Ρ‚ΠΎΠΊΠ° Π² Π·Π°ΠΌΠΊΠ½ΡƒΡ‚ΠΎΠΌ ΠΏΡ€ΠΎΠ²ΠΎΠ΄Π½ΠΈΠΊΠ΅, ΠΏΡ€ΠΈ ΠΏΡ€ΠΎΡ…ΠΎΠΆΠ΄Π΅Π½ΠΈΠΈ Ρ‡Π΅Ρ€Π΅Π· Π½Π΅Π³ΠΎ ΠΌΠ°Π³Π½ΠΈΡ‚Π½ΠΎΠ³ΠΎ ΠΏΠΎΡ‚ΠΎΠΊΠ°. Π’ΠΎ Π΅ΡΡ‚ΡŒ, благодаря этому явлСнию ΠΌΡ‹ ΠΌΠΎΠΆΠ΅ΠΌ ΠΏΡ€Π΅ΠΎΠ±Ρ€Π°Π·ΠΎΠ²Ρ‹Π²Π°Ρ‚ΡŒ ΠΌΠ΅Ρ…Π°Π½ΠΈΡ‡Π΅ΡΠΊΡƒΡŽ ΡΠ½Π΅Ρ€Π³ΠΈΡŽ Π² ΡΠ»Π΅ΠΊΡ‚Ρ€ΠΈΡ‡Π΅ΡΠΊΡƒΡŽ – ΠΈ это Π·Π°ΠΌΠ΅Ρ‡Π°Ρ‚Π΅Π»ΡŒΠ½ΠΎ. Π’Π΅Π΄ΡŒ Π΄ΠΎ открытия этого явлСния люди Π½Π΅ Π·Π½Π°Π»ΠΈ ΠΎ ΠΌΠ΅Ρ‚ΠΎΠ΄Π°Ρ… получСния элСктричСского Ρ‚ΠΎΠΊΠ° , ΠΊΡ€ΠΎΠΌΠ΅ гальваники.

Когда ΠΏΡ€ΠΎΠ²ΠΎΠ΄Π½ΠΈΠΊ оказываСтся ΠΏΠΎΠ΄ дСйствиСм ΠΌΠ°Π³Π½ΠΈΡ‚Π½ΠΎΠ³ΠΎ поля, Π² Π½Π΅ΠΌ Π²ΠΎΠ·Π½ΠΈΠΊΠ°Π΅Ρ‚ Π­Π”Π‘, ΠΊΠΎΡ‚ΠΎΡ€ΡƒΡŽ количСствСнно ΠΌΠΎΠΆΠ½ΠΎ Π²Ρ‹Ρ€Π°Π·ΠΈΡ‚ΡŒ Ρ‡Π΅Ρ€Π΅Π· Π·Π°ΠΊΠΎΠ½ элСктромагнитной ΠΈΠ½Π΄ΡƒΠΊΡ†ΠΈΠΈ.

Π—Π°ΠΊΠΎΠ½ элСктромагнитной ΠΈΠ½Π΄ΡƒΠΊΡ†ΠΈΠΈ

ЭлСктродвиТущая сила, индуцируСмая Π² проводящСм ΠΊΠΎΠ½Ρ‚ΡƒΡ€Π΅, Ρ€Π°Π²Π½Π° скорости измСнСния ΠΌΠ°Π³Π½ΠΈΡ‚Π½ΠΎΠ³ΠΎ ΠΏΠΎΡ‚ΠΎΠΊΠ°, ΡΡ†Π΅ΠΏΠ»ΡΡŽΡ‰Π΅Π³ΠΎΡΡ с этим ΠΊΠΎΠ½Ρ‚ΡƒΡ€ΠΎΠΌ.

Π’ ΠΊΠ°Ρ‚ΡƒΡˆΠΊΠ΅, которая ΠΈΠΌΠ΅Π΅Ρ‚ нСсколько Π²ΠΈΡ‚ΠΊΠΎΠ², общая Π­Π”Π‘ зависит ΠΎΡ‚ количСства Π²ΠΈΡ‚ΠΊΠΎΠ² n:

Но Π² ΠΎΠ±Ρ‰Π΅ΠΌ случаС, ΠΏΡ€ΠΈΠΌΠ΅Π½ΡΡŽΡ‚ Ρ„ΠΎΡ€ΠΌΡƒΠ»Ρƒ Π­Π”Π‘ с ΠΎΠ±Ρ‰ΠΈΠΌ потокосцСплСниСм:

Π­Π”Π‘ возбуТдаСмая Π² ΠΊΠΎΠ½Ρ‚ΡƒΡ€Π΅, создаСт Ρ‚ΠΎΠΊ. НаиболСС простым ΠΏΡ€ΠΈΠΌΠ΅Ρ€ΠΎΠΌ появлСния Ρ‚ΠΎΠΊΠ° Π² ΠΏΡ€ΠΎΠ²ΠΎΠ΄Π½ΠΈΠΊΠ΅ являСтся ΠΊΠ°Ρ‚ΡƒΡˆΠΊΠ°, Ρ‡Π΅Ρ€Π΅Π· ΠΊΠΎΡ‚ΠΎΡ€ΡƒΡŽ ΠΏΡ€ΠΎΡ…ΠΎΠ΄ΠΈΡ‚ постоянный ΠΌΠ°Π³Π½ΠΈΡ‚ . НаправлСниС ΠΈΠ½Π΄ΡƒΡ†ΠΈΡ€ΡƒΠ΅ΠΌΠΎΠ³ΠΎ Ρ‚ΠΎΠΊΠ° ΠΌΠΎΠΆΠ½ΠΎ ΠΎΠΏΡ€Π΅Π΄Π΅Π»ΠΈΡ‚ΡŒ с ΠΏΠΎΠΌΠΎΡ‰ΡŒΡŽ ΠΏΡ€Π°Π²ΠΈΠ»Π° Π›Π΅Π½Ρ†Π° .


ΠŸΡ€Π°Π²ΠΈΠ»ΠΎ Π›Π΅Π½Ρ†Π°

Π’ΠΎΠΊ, ΠΈΠ½Π΄ΡƒΡ†ΠΈΡ€ΡƒΠ΅ΠΌΡ‹ΠΉ ΠΏΡ€ΠΈ ΠΈΠ·ΠΌΠ΅Π½Π΅Π½ΠΈΠΈ ΠΌΠ°Π³Π½ΠΈΡ‚Π½ΠΎΠ³ΠΎ поля проходящСго Ρ‡Π΅Ρ€Π΅Π· ΠΊΠΎΠ½Ρ‚ΡƒΡ€, своим ΠΌΠ°Π³Π½ΠΈΡ‚Π½Ρ‹ΠΌ ΠΏΠΎΠ»Π΅ΠΌ прСпятствуСт этому измСнСнию.

Π’ Ρ‚ΠΎΠΌ случаС, ΠΊΠΎΠ³Π΄Π° ΠΌΡ‹ Π²Π²ΠΎΠ΄ΠΈΠΌ ΠΌΠ°Π³Π½ΠΈΡ‚ Π² ΠΊΠ°Ρ‚ΡƒΡˆΠΊΡƒ, ΠΌΠ°Π³Π½ΠΈΡ‚Π½Ρ‹ΠΉ ΠΏΠΎΡ‚ΠΎΠΊ Π² ΠΊΠΎΠ½Ρ‚ΡƒΡ€Π΅ увСличиваСтся, Π° Π·Π½Π°Ρ‡ΠΈΡ‚ ΠΌΠ°Π³Π½ΠΈΡ‚Π½ΠΎΠ΅ ΠΏΠΎΠ»Π΅, создаваСмоС ΠΈΠ½Π΄ΡƒΡ†ΠΈΡ€ΡƒΠ΅ΠΌΡ‹ΠΌ Ρ‚ΠΎΠΊΠΎΠΌ, ΠΏΠΎ ΠΏΡ€Π°Π²ΠΈΠ»Ρƒ Π›Π΅Π½Ρ†Π°, Π½Π°ΠΏΡ€Π°Π²Π»Π΅Π½ΠΎ ΠΏΡ€ΠΎΡ‚ΠΈΠ² увСличСния поля ΠΌΠ°Π³Π½ΠΈΡ‚Π°. Π§Ρ‚ΠΎΠ±Ρ‹ ΠΎΠΏΡ€Π΅Π΄Π΅Π»ΠΈΡ‚ΡŒ Π½Π°ΠΏΡ€Π°Π²Π»Π΅Π½ΠΈΠ΅ Ρ‚ΠΎΠΊΠ°, Π½ΡƒΠΆΠ½ΠΎ ΠΏΠΎΡΠΌΠΎΡ‚Ρ€Π΅Ρ‚ΡŒ Π½Π° ΠΌΠ°Π³Π½ΠΈΡ‚ со стороны сСвСрного полюса. Π‘ этой ΠΏΠΎΠ·ΠΈΡ†ΠΈΠΈ ΠΌΡ‹ Π±ΡƒΠ΄Π΅ΠΌ Π²ΠΊΡ€ΡƒΡ‡ΠΈΠ²Π°Ρ‚ΡŒ Π±ΡƒΡ€Π°Π²Ρ‡ΠΈΠΊ ΠΏΠΎ Π½Π°ΠΏΡ€Π°Π²Π»Π΅Π½ΠΈΡŽ ΠΌΠ°Π³Π½ΠΈΡ‚Π½ΠΎΠ³ΠΎ поля Ρ‚ΠΎΠΊΠ°, Ρ‚ΠΎ Π΅ΡΡ‚ΡŒ навстрСчу сСвСрному ΠΏΠΎΠ»ΡŽΡΡƒ. Π’ΠΎΠΊ Π±ΡƒΠ΄Π΅Ρ‚ Π΄Π²ΠΈΠ³Π°Ρ‚ΡŒΡΡ ΠΏΠΎ Π½Π°ΠΏΡ€Π°Π²Π»Π΅Π½ΠΈΡŽ вращСния Π±ΡƒΡ€Π°Π²Ρ‡ΠΈΠΊΠ°, Ρ‚ΠΎ Π΅ΡΡ‚ΡŒ ΠΏΠΎ часовой стрСлкС.

Π’ Ρ‚ΠΎΠΌ случаС, ΠΊΠΎΠ³Π΄Π° ΠΌΡ‹ Π²Ρ‹Π²ΠΎΠ΄ΠΈΠΌ ΠΌΠ°Π³Π½ΠΈΡ‚ ΠΈΠ· ΠΊΠ°Ρ‚ΡƒΡˆΠΊΠΈ, ΠΌΠ°Π³Π½ΠΈΡ‚Π½Ρ‹ΠΉ ΠΏΠΎΡ‚ΠΎΠΊ Π² ΠΊΠΎΠ½Ρ‚ΡƒΡ€Π΅ ΡƒΠΌΠ΅Π½ΡŒΡˆΠ°Π΅Ρ‚ΡΡ, Π° Π·Π½Π°Ρ‡ΠΈΡ‚ ΠΌΠ°Π³Π½ΠΈΡ‚Π½ΠΎΠ΅ ΠΏΠΎΠ»Π΅, создаваСмоС ΠΈΠ½Π΄ΡƒΡ†ΠΈΡ€ΡƒΠ΅ΠΌΡ‹ΠΌ Ρ‚ΠΎΠΊΠΎΠΌ, Π½Π°ΠΏΡ€Π°Π²Π»Π΅Π½ΠΎ ΠΏΡ€ΠΎΡ‚ΠΈΠ² ΡƒΠΌΠ΅Π½ΡŒΡˆΠ΅Π½ΠΈΡ поля ΠΌΠ°Π³Π½ΠΈΡ‚Π°. Π§Ρ‚ΠΎΠ±Ρ‹ ΠΎΠΏΡ€Π΅Π΄Π΅Π»ΠΈΡ‚ΡŒ Π½Π°ΠΏΡ€Π°Π²Π»Π΅Π½ΠΈΠ΅ Ρ‚ΠΎΠΊΠ°, Π½ΡƒΠΆΠ½ΠΎ Π²Ρ‹ΠΊΡ€ΡƒΡ‡ΠΈΠ²Π°Ρ‚ΡŒ Π±ΡƒΡ€Π°Π²Ρ‡ΠΈΠΊ, Π½Π°ΠΏΡ€Π°Π²Π»Π΅Π½ΠΈΠ΅ вращСния Π±ΡƒΡ€Π°Π²Ρ‡ΠΈΠΊΠ° ΡƒΠΊΠ°ΠΆΠ΅Ρ‚ Π½Π°ΠΏΡ€Π°Π²Π»Π΅Π½ΠΈΠ΅ Ρ‚ΠΎΠΊΠ° Π² ΠΏΡ€ΠΎΠ²ΠΎΠ΄Π½ΠΈΠΊΠ΅ – ΠΏΡ€ΠΎΡ‚ΠΈΠ² часовой стрСлки.

>>Π€ΠΈΠ·ΠΈΠΊΠ° ΠΈ астрономия >>Π€ΠΈΠ·ΠΈΠΊΠ° 11 класс >> Π—Π°ΠΊΠΎΠ½ элСктромагнитной ΠΈΠ½Π΄ΡƒΠΊΡ†ΠΈΠΈ

Π—Π°ΠΊΠΎΠ½ ЀарадСя. Π˜Π½Π΄ΡƒΠΊΡ†ΠΈΡ

Π­Π»Π΅ΠΊΡ‚Ρ€ΠΎΠΌΠ°Π³Π½ΠΈΡ‚Π½ΠΎΠΉ ΠΈΠ½Π΄ΡƒΠΊΡ†ΠΈΠ΅ΠΉ Π½Π°Π·Ρ‹Π²Π°ΡŽΡ‚ Ρ‚Π°ΠΊΠΎΠ΅ явлСниС, ΠΊΠ°ΠΊ Π²ΠΎΠ·Π½ΠΈΠΊΠ½ΠΎΠ²Π΅Π½ΠΈΠ΅ элСктричСского Ρ‚ΠΎΠΊΠ° Π² Π·Π°ΠΌΠΊΠ½ΡƒΡ‚ΠΎΠΌ ΠΊΠΎΠ½Ρ‚ΡƒΡ€Π΅, ΠΏΡ€ΠΈ условии измСнСния ΠΌΠ°Π³Π½ΠΈΡ‚Π½ΠΎΠ³ΠΎ ΠΏΠΎΡ‚ΠΎΠΊΠ°, ΠΊΠΎΡ‚ΠΎΡ€Ρ‹ΠΉ ΠΏΡ€ΠΎΡ…ΠΎΠ΄ΠΈΡ‚ Ρ‡Π΅Ρ€Π΅Π· этот ΠΊΠΎΠ½Ρ‚ΡƒΡ€.

Π—Π°ΠΊΠΎΠ½ элСктромагнитной ΠΈΠ½Π΄ΡƒΠΊΡ†ΠΈΠΈ ЀарадСя записываСтся Ρ‚Π°ΠΊΠΎΠΉ Ρ„ΠΎΡ€ΠΌΡƒΠ»ΠΎΠΉ:

И гласит, Ρ‡Ρ‚ΠΎ:



Каким ΠΆΠ΅ ΠΎΠ±Ρ€Π°Π·ΠΎΠΌ ΡƒΡ‡Π΅Π½Ρ‹ΠΌ ΡƒΠ΄Π°Π»ΠΎΡΡŒ вывСсти Ρ‚Π°ΠΊΡƒΡŽ Ρ„ΠΎΡ€ΠΌΡƒΠ»Ρƒ ΠΈ ΡΡ„ΠΎΡ€ΠΌΡƒΠ»ΠΈΡ€ΠΎΠ²Π°Ρ‚ΡŒ этот Π·Π°ΠΊΠΎΠ½? ΠœΡ‹ с Π²Π°ΠΌΠΈ ΡƒΠΆΠ΅ Π·Π½Π°Π΅ΠΌ, Ρ‡Ρ‚ΠΎ Π²ΠΎΠΊΡ€ΡƒΠ³ ΠΏΡ€ΠΎΠ²ΠΎΠ΄Π½ΠΈΠΊΠ° с Ρ‚ΠΎΠΊΠΎΠΌ всСгда сущСствуСт ΠΌΠ°Π³Π½ΠΈΡ‚Π½ΠΎΠ΅ ΠΏΠΎΠ»Π΅, Π° элСктричСство ΠΎΠ±Π»Π°Π΄Π°Π΅Ρ‚ ΠΌΠ°Π³Π½ΠΈΡ‚Π½ΠΎΠΉ силой. ΠŸΠΎΡΡ‚ΠΎΠΌΡƒ Π² Π½Π°Ρ‡Π°Π»Π΅ 19Π³ΠΎ Π²Π΅ΠΊΠ° ΠΈ Π²ΠΎΠ·Π½ΠΈΠΊΠ»Π° Π·Π°Π΄Π°Ρ‡Π° ΠΎ нСобходимости подтвСрТдСния влияния ΠΌΠ°Π³Π½ΠΈΡ‚Π½Ρ‹Ρ… явлСний Π½Π° элСктричСскиС, ΠΊΠΎΡ‚ΠΎΡ€ΡƒΡŽ ΠΏΡ‹Ρ‚Π°Π»ΠΈΡΡŒ Ρ€Π΅ΡˆΠΈΡ‚ΡŒ ΠΌΠ½ΠΎΠ³ΠΈΠ΅ ΡƒΡ‡Π΅Π½Ρ‹Π΅, ΠΈ английский ΡƒΡ‡Π΅Π½Ρ‹ΠΉ Майкл Π€Π°Ρ€Π°Π΄Π΅ΠΉ Π±Ρ‹Π» Π² ΠΈΡ… числС. ΠŸΠΎΡ‡Ρ‚ΠΈ 10 Π»Π΅Ρ‚, начиная с 1822 Π³ΠΎΠ΄Π°, ΠΎΠ½ ΠΏΠΎΡ‚Ρ€Π°Ρ‚ΠΈΠ» Π½Π° Ρ€Π°Π·Π»ΠΈΡ‡Π½Ρ‹Π΅ ΠΎΠΏΡ‹Ρ‚Ρ‹, Π½ΠΎ Π±Π΅Π·ΡƒΡΠΏΠ΅ΡˆΠ½ΠΎ. И Ρ‚ΠΎΠ»ΡŒΠΊΠΎ 29 августа 1831 Π³ΠΎΠ΄Π° наступил Ρ‚Ρ€ΠΈΡƒΠΌΡ„.

ПослС напряТСнных поисков, исслСдований ΠΈ ΠΎΠΏΡ‹Ρ‚ΠΎΠ², Π€Π°Ρ€Π°Π΄Π΅ΠΉ ΠΏΡ€ΠΈΡˆΠ΅Π» ΠΊ Π²Ρ‹Π²ΠΎΠ΄Ρƒ, Ρ‡Ρ‚ΠΎ Ρ‚ΠΎΠ»ΡŒΠΊΠΎ ΠΌΠ΅Π½ΡΡŽΡ‰Π΅Π΅ΡΡ со Π²Ρ€Π΅ΠΌΠ΅Π½Π΅ΠΌ ΠΌΠ°Π³Π½ΠΈΡ‚Π½ΠΎΠ΅ ΠΏΠΎΠ»Π΅ ΠΌΠΎΠΆΠ΅Ρ‚ ΡΠΎΠ·Π΄Π°Ρ‚ΡŒ элСктричСский Ρ‚ΠΎΠΊ.

ΠžΠΏΡ‹Ρ‚Ρ‹ Π€Π°Ρ€Π°Π΄Π΅ΠΉ

ΠžΠΏΡ‹Ρ‚Ρ‹ Π€Π°Ρ€Π°Π΄Π΅ΠΉ состояли Π² ΡΠ»Π΅Π΄ΡƒΡŽΡ‰Π΅ΠΌ:

Π’ΠΎ-ΠΏΠ΅Ρ€Π²Ρ‹Ρ…, Ссли Π²Π·ΡΡ‚ΡŒ постоянный ΠΌΠ°Π³Π½ΠΈΡ‚ ΠΈ Π΄Π²ΠΈΠ³Π°Ρ‚ΡŒ Π΅Π³ΠΎ Π²Π½ΡƒΡ‚Ρ€ΠΈ ΠΊΠ°Ρ‚ΡƒΡˆΠΊΠΈ, ΠΊ ΠΊΠΎΡ‚ΠΎΡ€ΠΎΠΉ присоСдинСн Π³Π°Π»ΡŒΠ²Π°Π½ΠΎΠΌΠ΅Ρ‚Ρ€, Ρ‚ΠΎ Π² Ρ†Π΅ΠΏΠΈ Π²ΠΎΠ·Π½ΠΈΠΊΠ°Π» элСктричСский Ρ‚ΠΎΠΊ.
Π’ΠΎ-Π²Ρ‚ΠΎΡ€Ρ‹Ρ…, Ссли этот ΠΌΠ°Π³Π½ΠΈΡ‚ Π²Ρ‹Π΄Π²ΠΈΠ³Π°Ρ‚ΡŒ ΠΈΠ· ΠΊΠ°Ρ‚ΡƒΡˆΠΊΠΈ, Ρ‚ΠΎ ΠΌΡ‹ наблюдаСм, Ρ‡Ρ‚ΠΎ Π³Π°Π»ΡŒΠ²Π°Π½ΠΎΠΌΠ΅Ρ‚Ρ€ Ρ‚Π°ΠΊ ΠΆΠ΅ ΠΏΠΎΠΊΠ°Π·Ρ‹Π²Π°Π΅Ρ‚ Ρ‚ΠΎΠΊ, Π½ΠΎ этот Ρ‚ΠΎΠΊ ΠΈΠΌΠ΅Π΅Ρ‚ ΠΏΡ€ΠΎΡ‚ΠΈΠ²ΠΎΠΏΠΎΠ»ΠΎΠΆΠ½ΠΎΠ΅ Π½Π°ΠΏΡ€Π°Π²Π»Π΅Π½ΠΈΠ΅.



А Ρ‚Π΅ΠΏΠ΅Ρ€ΡŒ Π΄Π°Π²Π°ΠΉΡ‚Π΅ ΠΏΠΎΠΏΡ€ΠΎΠ±ΡƒΠ΅ΠΌ этот ΠΎΠΏΡ‹Ρ‚ Π½Π΅ΠΌΠ½ΠΎΠ³ΠΎ ΠΈΠ·ΠΌΠ΅Π½ΠΈΡ‚ΡŒ. Для этого ΠΌΡ‹ ΠΏΠΎΠΏΡ€ΠΎΠ±ΡƒΠ΅ΠΌ Π½Π° Π½Π΅ΠΏΠΎΠ΄Π²ΠΈΠΆΠ½Ρ‹ΠΉ ΠΌΠ°Π³Π½ΠΈΡ‚ ΠΎΠ΄Π΅Π²Π°Ρ‚ΡŒ ΠΈ ΡΠ½ΠΈΠΌΠ°Ρ‚ΡŒ ΠΊΠ°Ρ‚ΡƒΡˆΠΊΡƒ. И Ρ‡Ρ‚ΠΎ ΠΌΡ‹ Π² ΠΈΡ‚ΠΎΠ³Π΅ Π²ΠΈΠ΄ΠΈΠΌ? А ΠΌΡ‹ с Π²Π°ΠΌΠΈ наблюдаСм Ρ‚ΠΎ, Ρ‡Ρ‚ΠΎ Π²ΠΎ врСмя двиТСния ΠΊΠ°Ρ‚ΡƒΡˆΠΊΠΈ ΠΎΡ‚Π½ΠΎΡΠΈΡ‚Π΅Π»ΡŒΠ½ΠΎ ΠΌΠ°Π³Π½ΠΈΡ‚Π° Π² Ρ†Π΅ΠΏΠΈ снова появляСтся Ρ‚ΠΎΠΊ. А Ссли Π² ΠΊΠ°Ρ‚ΡƒΡˆΠΊΠ΅ ΠΏΡ€Π΅ΠΊΡ€Π°Ρ‚ΠΈΠ»ΠΎΡΡŒ, Ρ‚ΠΎ ΠΈ Ρ‚ΠΎΠΊ сразу ΠΆΠ΅ исчСзаСт.



Π’Π΅ΠΏΠ΅Ρ€ΡŒ Π΄Π°Π²Π°ΠΉΡ‚Π΅ ΠΏΡ€ΠΎΠ΄Π΅Π»Π°Π΅ΠΌ Π΅Ρ‰Π΅ ΠΎΠ΄ΠΈΠ½ ΠΎΠΏΡ‹Ρ‚. Для этого ΠΌΡ‹ с Π²Π°ΠΌΠΈ возьмСм ΠΈ помСстим Π² ΠΌΠ°Π³Π½ΠΈΡ‚Π½ΠΎΠ΅ ΠΏΠΎΠ»Π΅ плоский ΠΊΠΎΠ½Ρ‚ΡƒΡ€ Π±Π΅Π· ΠΏΡ€ΠΎΠ²ΠΎΠ΄Π½ΠΈΠΊΠ°, Π° Π΅Π³ΠΎ ΠΊΠΎΠ½Ρ†Ρ‹ ΠΏΠΎΠΏΡ€ΠΎΠ±ΡƒΠ΅ΠΌ ΡΠΎΠ΅Π΄ΠΈΠ½ΠΈΡ‚ΡŒ с Π³Π°Π»ΡŒΠ²Π°Π½ΠΎΠΌΠ΅Ρ‚Ρ€ΠΎΠΌ. И Ρ‡Ρ‚ΠΎ ΠΌΡ‹ наблюдаСм? Как Ρ‚ΠΎΠ»ΡŒΠΊΠΎ ΠΊΠΎΠ½Ρ‚ΡƒΡ€ Π³Π°Π»ΡŒΠ²Π°Π½ΠΎΠΌΠ΅Ρ‚Ρ€ поворачиваСтся, Ρ‚ΠΎ ΠΌΡ‹ наблюдаСм появлСниС Π² Π½Π΅ΠΌ ΠΈΠ½Π΄ΡƒΠΊΡ†ΠΈΠΎΠ½Π½ΠΎΠ³ΠΎ Ρ‚ΠΎΠΊΠ°. А Ссли ΠΏΠΎΠΏΡ€ΠΎΠ±ΠΎΠ²Π°Ρ‚ΡŒ Π²Ρ€Π°Ρ‰Π°Ρ‚ΡŒ ΠΌΠ°Π³Π½ΠΈΡ‚ Π²Π½ΡƒΡ‚Ρ€ΠΈ Π½Π΅Π³ΠΎ ΠΈ рядом с ΠΊΠΎΠ½Ρ‚ΡƒΡ€ΠΎΠΌ, Ρ‚ΠΎ Π² этом случаС Ρ‚Π°ΠΊΠΆΠ΅ появится Ρ‚ΠΎΠΊ.



Π”ΡƒΠΌΠ°ΡŽ, Π²Ρ‹ ΡƒΠΆΠ΅ Π·Π°ΠΌΠ΅Ρ‚ΠΈΠ»ΠΈ, Ρ‚ΠΎΠΊ появляСтся Π² ΠΊΠ°Ρ‚ΡƒΡˆΠΊΠ΅ Ρ‚ΠΎΠ³Π΄Π°, ΠΊΠΎΠ³Π΄Π° измСняСтся ΠΌΠ°Π³Π½ΠΈΡ‚Π½Ρ‹ΠΉ ΠΏΠΎΡ‚ΠΎΠΊ, ΠΊΠΎΡ‚ΠΎΡ€Ρ‹ΠΉ ΠΏΡ€ΠΎΠ½ΠΈΠ·Ρ‹Π²Π°Π΅Ρ‚ эту ΠΊΠ°Ρ‚ΡƒΡˆΠΊΡƒ.

И Ρ‚ΡƒΡ‚ Π²ΠΎΠ·Π½ΠΈΠΊΠ°Π΅Ρ‚ вопрос, ΠΏΡ€ΠΈ всяких Π»ΠΈ двиТСниях ΠΌΠ°Π³Π½ΠΈΡ‚Π° ΠΈ ΠΊΠ°Ρ‚ΡƒΡˆΠΊΠΈ, ΠΌΠΎΠΆΠ΅Ρ‚ Π²ΠΎΠ·Π½ΠΈΠΊΠ½ΡƒΡ‚ΡŒ элСктричСский Ρ‚ΠΎΠΊ? ΠžΠΊΠ°Π·Ρ‹Π²Π°Π΅Ρ‚ΡΡ Π½Π΅ всСгда. Π’ΠΎΠΊ Π½Π΅ Π²ΠΎΠ·Π½ΠΈΠΊΠ½Π΅Ρ‚ Π² Ρ‚ΠΎΠΌ случаС, ΠΊΠΎΠ³Π΄Π° ΠΌΠ°Π³Π½ΠΈΡ‚ вращаСтся Π²ΠΎΠΊΡ€ΡƒΠ³ Π²Π΅Ρ€Ρ‚ΠΈΠΊΠ°Π»ΡŒΠ½ΠΎΠΉ оси.

А ΠΈΠ· этого слСдуСт, Ρ‡Ρ‚ΠΎ ΠΏΡ€ΠΈ любом ΠΈΠ·ΠΌΠ΅Π½Π΅Π½ΠΈΠΈ ΠΌΠ°Π³Π½ΠΈΡ‚Π½ΠΎΠ³ΠΎ ΠΏΠΎΡ‚ΠΎΠΊΠ°, ΠΌΡ‹ наблюдаСм Ρ‚ΠΎ, Ρ‡Ρ‚ΠΎ Π² этом ΠΏΡ€ΠΎΠ²ΠΎΠ΄Π½ΠΈΠΊΠ΅ Π²ΠΎΠ·Π½ΠΈΠΊΠ°Π΅Ρ‚ элСктричСский Ρ‚ΠΎΠΊ, ΠΊΠΎΡ‚ΠΎΡ€Ρ‹ΠΉ сущСствовал Π² Ρ‚Π΅Ρ‡Π΅Π½ΠΈΠΈ всСго процСсса, ΠΏΠΎΠΊΠ° происходили измСнСния ΠΌΠ°Π³Π½ΠΈΡ‚Π½ΠΎΠ³ΠΎ ΠΏΠΎΡ‚ΠΎΠΊΠ°. ИмСнно Π² этом ΠΈ Π·Π°ΠΊΠ»ΡŽΡ‡Π°Π΅Ρ‚ΡΡ явлСниС элСктромагнитной ΠΈΠ½Π΄ΡƒΠΊΡ†ΠΈΠΈ. А ΠΈΠ½Π΄ΡƒΠΊΡ†ΠΈΠΎΠ½Π½Ρ‹ΠΌ Ρ‚ΠΎΠΊΠΎΠΌ являСтся Ρ‚ΠΎΡ‚ Ρ‚ΠΎΠΊ, ΠΊΠΎΡ‚ΠΎΡ€Ρ‹ΠΉ Π±Ρ‹Π» ΠΏΠΎΠ»ΡƒΡ‡Π΅Π½ Π΄Π°Π½Π½Ρ‹ΠΌ ΠΌΠ΅Ρ‚ΠΎΠ΄ΠΎΠΌ.

Если ΠΌΡ‹ с Π²Π°ΠΌΠΈ ΠΏΡ€ΠΎΠ°Π½Π°Π»ΠΈΠ·ΠΈΡ€ΡƒΠ΅ΠΌ Π΄Π°Π½Π½Ρ‹ΠΉ ΠΎΠΏΡ‹Ρ‚, Ρ‚ΠΎ ΡƒΠ²ΠΈΠ΄ΠΈΠΌ, Ρ‡Ρ‚ΠΎ Π·Π½Π°Ρ‡Π΅Π½ΠΈΠ΅ ΠΈΠ½Π΄ΡƒΠΊΡ†ΠΈΠΎΠ½Π½ΠΎΠ³ΠΎ Ρ‚ΠΎΠΊΠ° ΡΠΎΠ²Π΅Ρ€ΡˆΠ΅Π½Π½ΠΎ Π½Π΅ зависит ΠΎΡ‚ ΠΏΡ€ΠΈΡ‡ΠΈΠ½Ρ‹ измСнСния ΠΌΠ°Π³Π½ΠΈΡ‚Π½ΠΎΠ³ΠΎ ΠΏΠΎΡ‚ΠΎΠΊΠ°. Π’ Π΄Π°Π½Π½ΠΎΠΌ случаС, пСрвостСпСнноС Π·Π½Π°Ρ‡Π΅Π½ΠΈΠ΅ ΠΈΠΌΠ΅Π΅Ρ‚ лишь ΡΠΊΠΎΡ€ΠΎΡΡ‚ΡŒ, которая влияСт Π½Π° измСнСния ΠΌΠ°Π³Π½ΠΈΡ‚Π½ΠΎΠ³ΠΎ ΠΏΠΎΡ‚ΠΎΠΊΠ°. Из ΠΎΠΏΡ‹Ρ‚ΠΎΠ² ЀарадСя слСдуСт, Ρ‡Ρ‚ΠΎ Ρ‡Π΅ΠΌ быстрСС двигаСтся ΠΌΠ°Π³Π½ΠΈΡ‚ Π² ΠΊΠ°Ρ‚ΡƒΡˆΠΊΠ΅, Ρ‚Π΅ΠΌ большС отклоняСтся стрСлка Π³Π°Π»ΡŒΠ²Π°Π½ΠΎΠΌΠ΅Ρ‚Ρ€Π°.



Π’Π΅ΠΏΠ΅Ρ€ΡŒ ΠΌΡ‹ ΠΌΠΎΠΆΠ΅ΠΌ подвСсти ΠΈΡ‚ΠΎΠ³ Π΄Π°Π½Π½ΠΎΠ³ΠΎ ΡƒΡ€ΠΎΠΊΠ° ΠΈ ΡΠ΄Π΅Π»Π°Ρ‚ΡŒ Π²Ρ‹Π²ΠΎΠ΄, Ρ‡Ρ‚ΠΎ Π·Π°ΠΊΠΎΠ½ элСктромагнитной ΠΈΠ½Π΄ΡƒΠΊΡ†ΠΈΠΈ являСтся ΠΎΠ΄Π½ΠΈΠΌ ΠΈΠ· основных Π·Π°ΠΊΠΎΠ½ΠΎΠΌ элСктродинамики. Благодаря ΠΈΠ·ΡƒΡ‡Π΅Π½ΠΈΡŽ явлСний элСктромагнитной ΠΈΠ½Π΄ΡƒΠΊΡ†ΠΈΠΈ, ΡƒΡ‡Π΅Π½Ρ‹ΠΌΠΈ Ρ€Π°Π·Π½Ρ‹Ρ… стран Π±Ρ‹Π»ΠΈ созданы Ρ€Π°Π·Π»ΠΈΡ‡Π½Ρ‹Π΅ элСктродвигатСли ΠΈ ΠΌΠΎΡ‰Π½Ρ‹Π΅ Π³Π΅Π½Π΅Ρ€Π°Ρ‚ΠΎΡ€Ρ‹. ΠžΠ³Ρ€ΠΎΠΌΠ½Ρ‹ΠΉ Π²ΠΊΠ»Π°Π΄ Π² Ρ€Π°Π·Π²ΠΈΡ‚ΠΈΠ΅ элСктротСхники внСсли ΠΈ Ρ‚Π°ΠΊΠΈΠ΅ извСстныС ΡƒΡ‡Π΅Π½Ρ‹Π΅, ΠΊΠ°ΠΊ Π›Π΅Π½Ρ†, Π―ΠΊΠΎΠ±ΠΈ, ΠΈ Π΄Ρ€ΡƒΠ³ΠΈΠ΅.

Π§Ρ‚ΠΎ ΠΌΠΎΠΆΠ΅Ρ‚ Π±Ρ‹Ρ‚ΡŒ Π»ΡƒΡ‡ΡˆΠ΅, Ρ‡Π΅ΠΌ Π²Π΅Ρ‡Π΅Ρ€ΠΎΠΌ понСдСльника ΠΏΠΎΡ‡ΠΈΡ‚Π°Ρ‚ΡŒ ΠΏΡ€ΠΎ основы элСктродинамики . ΠŸΡ€Π°Π²ΠΈΠ»ΡŒΠ½ΠΎ, ΠΌΠΎΠΆΠ½ΠΎ Π½Π°ΠΉΡ‚ΠΈ мноТСство Π²Π΅Ρ‰Π΅ΠΉ, ΠΊΠΎΡ‚ΠΎΡ€Ρ‹Π΅ Π±ΡƒΠ΄ΡƒΡ‚ Π»ΡƒΡ‡ΡˆΠ΅. Π’Π΅ΠΌ Π½Π΅ ΠΌΠ΅Π½Π΅Π΅, ΠΌΡ‹ всС Ρ€Π°Π²Π½ΠΎ ΠΏΡ€Π΅Π΄Π»Π°Π³Π°Π΅ΠΌ Π’Π°ΠΌ ΠΏΡ€ΠΎΡ‡Π΅ΡΡ‚ΡŒ эту ΡΡ‚Π°Ρ‚ΡŒΡŽ. Π’Ρ€Π΅ΠΌΠ΅Π½ΠΈ Π·Π°Π½ΠΈΠΌΠ°Π΅Ρ‚ Π½Π΅ ΠΌΠ½ΠΎΠ³ΠΎ, Π° полСзная информация останСтся Π² подсознании. НапримСр, Π½Π° экзамСнС, Π² условиях стрСсса, ΠΌΠΎΠΆΠ½ΠΎ Π±ΡƒΠ΄Π΅Ρ‚ ΡƒΡΠΏΠ΅ΡˆΠ½ΠΎ ΠΈΠ·Π²Π»Π΅Ρ‡ΡŒ ΠΈΠ· Π½Π΅Π΄Ρ€ памяти Π·Π°ΠΊΠΎΠ½ ЀарадСя. Π’Π°ΠΊ ΠΊΠ°ΠΊ Π·Π°ΠΊΠΎΠ½ΠΎΠ² ЀарадСя нСсколько, ΡƒΡ‚ΠΎΡ‡Π½ΠΈΠΌ, Ρ‡Ρ‚ΠΎ здСсь ΠΌΡ‹ Π³ΠΎΠ²ΠΎΡ€ΠΈΠΌ ΠΎ Π·Π°ΠΊΠΎΠ½Π΅ ΠΈΠ½Π΄ΡƒΠΊΡ†ΠΈΠΈ ЀарадСя.

Π­Π»Π΅ΠΊΡ‚Ρ€ΠΎΠ΄ΠΈΠ½Π°ΠΌΠΈΠΊΠ° – Ρ€Π°Π·Π΄Π΅Π» Ρ„ΠΈΠ·ΠΈΠΊΠΈ, ΠΈΠ·ΡƒΡ‡Π°ΡŽΡ‰ΠΈΠΉ элСктромагнитноС ΠΏΠΎΠ»Π΅ Π²ΠΎ всСх Π΅Π³ΠΎ проявлСниях.

Π­Ρ‚ΠΎ ΠΈ взаимодСйствиС элСктричСского ΠΈ ΠΌΠ°Π³Π½ΠΈΡ‚Π½ΠΎΠ³ΠΎ ΠΏΠΎΠ»Π΅ΠΉ, элСктричСский Ρ‚ΠΎΠΊ, элСктро-ΠΌΠ°Π³Π½ΠΈΡ‚Π½ΠΎΠ΅ ΠΈΠ·Π»ΡƒΡ‡Π΅Π½ΠΈΠ΅, влияниС поля Π½Π° заряТСнныС Ρ‚Π΅Π»Π°.

Π—Π΄Π΅ΡΡŒ ΠΌΡ‹ Π½Π΅ ставим Ρ†Π΅Π»ΡŒΡŽ Ρ€Π°ΡΡΠΌΠΎΡ‚Ρ€Π΅Ρ‚ΡŒ всю элСктродинамику. Упаси Π‘ΠΎΠΆΠ΅! Рассмотрим Π»ΡƒΡ‡ΡˆΠ΅ ΠΎΠ΄ΠΈΠ½ ΠΈΠ· основных Π΅Π΅ Π·Π°ΠΊΠΎΠ½ΠΎΠ², ΠΊΠΎΡ‚ΠΎΡ€Ρ‹ΠΉ называСтся Π·Π°ΠΊΠΎΠ½ΠΎΠΌ элСктромагнитной ΠΈΠ½Π΄ΡƒΠΊΡ†ΠΈΠΈ ЀарадСя .

Π˜ΡΡ‚ΠΎΡ€ΠΈΡ ΠΈ ΠΎΠΏΡ€Π΅Π΄Π΅Π»Π΅Π½ΠΈΠ΅

Π€Π°Ρ€Π°Π΄Π΅ΠΉ, ΠΏΠ°Ρ€Π°Π»Π»Π΅Π»ΡŒΠ½ΠΎ с Π“Π΅Π½Ρ€ΠΈ, ΠΎΡ‚ΠΊΡ€Ρ‹Π» явлСниС элСктромагнитной ΠΈΠ½Π΄ΡƒΠΊΡ†ΠΈΠΈ Π² 1831 Π³ΠΎΠ΄Ρƒ. ΠŸΡ€Π°Π²Π΄Π°, успСл ΠΎΠΏΡƒΠ±Π»ΠΈΠΊΠΎΠ²Π°Ρ‚ΡŒ Ρ€Π΅Π·ΡƒΠ»ΡŒΡ‚Π°Ρ‚Ρ‹ Ρ€Π°Π½ΡŒΡˆΠ΅. Π—Π°ΠΊΠΎΠ½ ЀарадСя повсСмСстно ΠΈΡΠΏΠΎΠ»ΡŒΠ·ΡƒΠ΅Ρ‚ΡΡ Π² Ρ‚Π΅Ρ…Π½ΠΈΠΊΠ΅, Π² элСктродвигатСлях, трансформаторах, Π³Π΅Π½Π΅Ρ€Π°Ρ‚ΠΎΡ€Π°Ρ… ΠΈ дроссСлях. Π’ Ρ‡Π΅ΠΌ ΡΡƒΡ‚ΡŒ Π·Π°ΠΊΠΎΠ½Π° ЀарадСя для элСктромагнитной ΠΈΠ½Π΄ΡƒΠΊΡ†ΠΈΠΈ, Ссли Π³ΠΎΠ²ΠΎΡ€ΠΈΡ‚ΡŒ просто? А Π²ΠΎΡ‚ Π² Ρ‡Π΅ΠΌ!

ΠŸΡ€ΠΈ ΠΈΠ·ΠΌΠ΅Π½Π΅Π½ΠΈΠΈ ΠΌΠ°Π³Π½ΠΈΡ‚Π½ΠΎΠ³ΠΎ ΠΏΠΎΡ‚ΠΎΠΊΠ° Ρ‡Π΅Ρ€Π΅Π· Π·Π°ΠΌΠΊΠ½ΡƒΡ‚Ρ‹ΠΉ проводящий ΠΊΠΎΠ½Ρ‚ΡƒΡ€, Π² ΠΊΠΎΠ½Ρ‚ΡƒΡ€Π΅ Π²ΠΎΠ·Π½ΠΈΠΊΠ°Π΅Ρ‚ элСктричСский Ρ‚ΠΎΠΊ. Π’ΠΎ Π΅ΡΡ‚ΡŒ, Ссли ΠΌΡ‹ скрутим ΠΈΠ· ΠΏΡ€ΠΎΠ²ΠΎΠ»ΠΎΠΊΠΈ Ρ€Π°ΠΌΠΊΡƒ ΠΈ помСстим Π΅Π΅ Π² ΠΈΠ·ΠΌΠ΅Π½ΡΡŽΡ‰Π΅Π΅ΡΡ ΠΌΠ°Π³Π½ΠΈΡ‚Π½ΠΎΠ΅ ΠΏΠΎΠ»Π΅ (возьмСм ΠΌΠ°Π³Π½ΠΈΡ‚, ΠΈ Π±ΡƒΠ΄Π΅ΠΌ ΠΊΡ€ΡƒΡ‚ΠΈΡ‚ΡŒ Π΅Π³ΠΎ Π²ΠΎΠΊΡ€ΡƒΠ³ Ρ€Π°ΠΌΠΊΠΈ), ΠΏΠΎ Ρ€Π°ΠΌΠΊΠ΅ ΠΏΠΎΡ‚Π΅Ρ‡Π΅Ρ‚ Ρ‚ΠΎΠΊ!

Π­Ρ‚ΠΎΡ‚ Ρ‚ΠΎΠΊ Π€Π°Ρ€Π°Π΄Π΅ΠΉ Π½Π°Π·Π²Π°Π» ΠΈΠ½Π΄ΡƒΠΊΡ†ΠΈΠΎΠ½Π½Ρ‹ΠΌ, Π° само явлСниС окрСстил элСктромагнитной ΠΈΠ½Π΄ΡƒΠΊΡ†ΠΈΠ΅ΠΉ.

ЭлСктромагнитная индукция – Π²ΠΎΠ·Π½ΠΈΠΊΠ½ΠΎΠ²Π΅Π½ΠΈΠ΅ Π² Π·Π°ΠΌΠΊΠ½ΡƒΡ‚ΠΎΠΌ ΠΊΠΎΠ½Ρ‚ΡƒΡ€Π΅ элСктричСского Ρ‚ΠΎΠΊΠ° ΠΏΡ€ΠΈ ΠΈΠ·ΠΌΠ΅Π½Π΅Π½ΠΈΠΈ ΠΌΠ°Π³Π½ΠΈΡ‚Π½ΠΎΠ³ΠΎ ΠΏΠΎΡ‚ΠΎΠΊΠ°, проходящСго Ρ‡Π΅Ρ€Π΅Π· ΠΊΠΎΠ½Ρ‚ΡƒΡ€.

Π€ΠΎΡ€ΠΌΡƒΠ»ΠΈΡ€ΠΎΠ²ΠΊΠ° основного Π·Π°ΠΊΠΎΠ½Π° элСктродинамики – Π·Π°ΠΊΠΎΠ½Π° элСктромагнитной ΠΈΠ½Π΄ΡƒΠΊΡ†ΠΈΠΈ ЀарадСя, выглядит ΠΈ Π·Π²ΡƒΡ‡ΠΈΡ‚ ΡΠ»Π΅Π΄ΡƒΡŽΡ‰ΠΈΠΌ ΠΎΠ±Ρ€Π°Π·ΠΎΠΌ:

Π­Π”Π‘ , Π²ΠΎΠ·Π½ΠΈΠΊΠ°ΡŽΡ‰Π°Ρ Π² ΠΊΠΎΠ½Ρ‚ΡƒΡ€Π΅, ΠΏΡ€ΠΎΠΏΠΎΡ€Ρ†ΠΈΠΎΠ½Π°Π»ΡŒΠ½Π° скорости измСнСния ΠΌΠ°Π³Π½ΠΈΡ‚Π½ΠΎΠ³ΠΎ ΠΏΠΎΡ‚ΠΎΠΊΠ° Π€ Ρ‡Π΅Ρ€Π΅Π· ΠΊΠΎΠ½Ρ‚ΡƒΡ€.

А ΠΎΡ‚ΠΊΡƒΠ΄Π° Π² Ρ„ΠΎΡ€ΠΌΡƒΠ»Π΅ минус, спроситС Π’Ρ‹. Для объяснСния Π·Π½Π°ΠΊΠ° минус Π² этой Ρ„ΠΎΡ€ΠΌΡƒΠ»Π΅ Π΅ΡΡ‚ΡŒ ΡΠΏΠ΅Ρ†ΠΈΠ°Π»ΡŒΠ½ΠΎΠ΅ ΠΏΡ€Π°Π²ΠΈΠ»ΠΎ Π›Π΅Π½Ρ†Π° . Оно гласит, Ρ‡Ρ‚ΠΎ Π·Π½Π°ΠΊ минус, Π² Π΄Π°Π½Π½ΠΎΠΌ случаС, ΡƒΠΊΠ°Π·Ρ‹Π²Π°Π΅Ρ‚ Π½Π° Ρ‚ΠΎ, ΠΊΠ°ΠΊ Π½Π°ΠΏΡ€Π°Π²Π»Π΅Π½Π° Π²ΠΎΠ·Π½ΠΈΠΊΠ°ΡŽΡ‰Π°Ρ Π­Π”Π‘. Π”Π΅Π»ΠΎ Π² Ρ‚ΠΎΠΌ, Ρ‡Ρ‚ΠΎ создаваСмоС ΠΈΠ½Π΄ΡƒΠΊΡ†ΠΈΠΎΠ½Π½Ρ‹ΠΌ Ρ‚ΠΎΠΊΠΎΠΌ ΠΌΠ°Π³Π½ΠΈΡ‚Π½ΠΎΠ΅ ΠΏΠΎΠ»Π΅ Π½Π°ΠΏΡ€Π°Π²Π»Π΅Π½ΠΎ Ρ‚Π°ΠΊ, Ρ‡Ρ‚ΠΎ прСпятствуСт измСнСнию ΠΌΠ°Π³Π½ΠΈΡ‚Π½ΠΎΠ³ΠΎ ΠΏΠΎΡ‚ΠΎΠΊΠ°, ΠΊΠΎΡ‚ΠΎΡ€Ρ‹ΠΉ Π²Ρ‹Π·Π²Π°Π» ΠΈΠ½Π΄ΡƒΠΊΡ†ΠΈΠΎΠ½Π½Ρ‹ΠΉ Ρ‚ΠΎΠΊ.

ΠŸΡ€ΠΈΠΌΠ΅Ρ€Ρ‹ Ρ€Π΅ΡˆΠ΅Π½ΠΈΡ Π·Π°Π΄Π°Ρ‡

Π’ΠΎΡ‚ Π²Ρ€ΠΎΠ΄Π΅ Π±Ρ‹ ΠΈ всС. Π—Π½Π°Ρ‡Π΅Π½ΠΈΠ΅ Π·Π°ΠΊΠΎΠ½Π° ЀарадСя Ρ„ΡƒΠ½Π΄Π°ΠΌΠ΅Π½Ρ‚Π°Π»ΡŒΠ½ΠΎ, вСдь Π½Π° использовании Π΄Π°Π½Π½ΠΎΠ³ΠΎ Π·Π°ΠΊΠΎΠ½Π° построСна основа ΠΏΠΎΡ‡Ρ‚ΠΈ всСй элСктричСской ΠΏΡ€ΠΎΠΌΡ‹ΡˆΠ»Π΅Π½Π½ΠΎΡΡ‚ΠΈ. Π§Ρ‚ΠΎΠ±Ρ‹ ΠΏΠΎΠ½ΠΈΠΌΠ°Π½ΠΈΠ΅ ΠΏΡ€ΠΈΡˆΠ»ΠΎ быстрСС, рассмотрим ΠΏΡ€ΠΈΠΌΠ΅Ρ€ Ρ€Π΅ΡˆΠ΅Π½ΠΈΡ Π·Π°Π΄Π°Ρ‡ΠΈ Π½Π° Π·Π°ΠΊΠΎΠ½ ЀарадСя.

И ΠΏΠΎΠΌΠ½ΠΈΡ‚Π΅, Π΄Ρ€ΡƒΠ·ΡŒΡ! Если Π·Π°Π΄Π°Ρ‡Π° засСла, ΠΊΠ°ΠΊ ΠΊΠΎΡΡ‚ΡŒ Π² Π³ΠΎΡ€Π»Π΅, ΠΈ Π½Π΅Ρ‚ большС сил Π΅Π΅ Ρ‚Π΅Ρ€ΠΏΠ΅Ρ‚ΡŒ – ΠΎΠ±Ρ€Π°Ρ‚ΠΈΡ‚Π΅ΡΡŒ ΠΊ нашим Π°Π²Ρ‚ΠΎΡ€Π°ΠΌ! Π’Π΅ΠΏΠ΅Ρ€ΡŒ Π²Ρ‹ Π·Π½Π°Π΅Ρ‚Π΅ . ΠœΡ‹ быстро прСдоставим ΠΏΠΎΠ΄Ρ€ΠΎΠ±Π½ΠΎΠ΅ Ρ€Π΅ΡˆΠ΅Π½ΠΈΠ΅ ΠΈ Ρ€Π°Π·ΡŠΡΡΠ½ΠΈΠΌ всС вопросы!

Π’ Ρ€Π΅Π·ΡƒΠ»ΡŒΡ‚Π°Ρ‚Π΅ многочислСнных ΠΎΠΏΡ‹Ρ‚ΠΎΠ² Π€Π°Ρ€Π°Π΄Π΅ΠΉ установил основной количСствСнный Π·Π°ΠΊΠΎΠ½ элСктромагнитной ΠΈΠ½Π΄ΡƒΠΊΡ†ΠΈΠΈ. Он ΠΏΠΎΠΊΠ°Π·Π°Π», Ρ‡Ρ‚ΠΎ всякий Ρ€Π°Π·, ΠΊΠΎΠ³Π΄Π° происходит ΠΈΠ·ΠΌΠ΅Π½Π΅Π½ΠΈΠ΅ сцСплСнного с ΠΊΠΎΠ½Ρ‚ΡƒΡ€ΠΎΠΌ ΠΏΠΎΡ‚ΠΎΠΊΠ° ΠΌΠ°Π³Π½ΠΈΡ‚Π½ΠΎΠΉ ΠΈΠ½Π΄ΡƒΠΊΡ†ΠΈΠΈ, Π² ΠΊΠΎΠ½Ρ‚ΡƒΡ€Π΅ Π²ΠΎΠ·Π½ΠΈΠΊΠ°Π΅Ρ‚ ΠΈΠ½Π΄ΡƒΠΊΡ†ΠΈΠΎΠ½Π½Ρ‹ΠΉ Ρ‚ΠΎΠΊ. Π’ΠΎΠ·Π½ΠΈΠΊΠ½ΠΎΠ²Π΅Π½ΠΈΠ΅ ΠΈΠ½Π΄ΡƒΠΊΡ†ΠΈΠΎΠ½Π½ΠΎΠ³ΠΎ Ρ‚ΠΎΠΊΠ° ΡƒΠΊΠ°Π·Ρ‹Π²Π°Π΅Ρ‚ Π½Π° Π½Π°Π»ΠΈΡ‡ΠΈΠ΅ Π² Ρ†Π΅ΠΏΠΈ элСктродвиТущСй силы, Π½Π°Π·Ρ‹Π²Π°Π΅ΠΌΠΎΠΉ элСктродвиТущСй силой элСктромагнитной ΠΈΠ½Π΄ΡƒΠΊΡ†ΠΈΠΈ. Π€Π°Ρ€Π°Π΄Π΅ΠΉ установил, Ρ‡Ρ‚ΠΎ Π·Π½Π°Ρ‡Π΅Π½ΠΈΠ΅ Π­Π”Π‘ элСктромагнитной ΠΈΠ½Π΄ΡƒΠΊΡ†ΠΈΠΈ E i ΠΏΡ€ΠΎΠΏΠΎΡ€Ρ†ΠΈΠΎΠ½Π°Π»ΡŒΠ½ΠΎ скорости измСнСния ΠΌΠ°Π³Π½ΠΈΡ‚Π½ΠΎΠ³ΠΎ ΠΏΠΎΡ‚ΠΎΠΊΠ°:

E i = -К , (27.1)

Π³Π΄Π΅ К – коэффициСнт ΠΏΡ€ΠΎΠΏΠΎΡ€Ρ†ΠΈΠΎΠ½Π°Π»ΡŒΠ½ΠΎΡΡ‚ΠΈ, зависящий Ρ‚ΠΎΠ»ΡŒΠΊΠΎ ΠΎΡ‚ Π²Ρ‹Π±ΠΎΡ€Π° Π΅Π΄ΠΈΠ½ΠΈΡ† измСрСния.

Π’ систСмС Π΅Π΄ΠΈΠ½ΠΈΡ† БИ коэффициСнт К = 1, Ρ‚.Π΅.

E i = – . (27.2)

Π­Ρ‚Π° Ρ„ΠΎΡ€ΠΌΡƒΠ»Π° ΠΈ прСдставляСт собой Π·Π°ΠΊΠΎΠ½ элСктромагнитной ΠΈΠ½Π΄ΡƒΠΊΡ†ΠΈΠΈ ЀарадСя. Π—Π½Π°ΠΊ минус Π² этой Ρ„ΠΎΡ€ΠΌΡƒΠ»Π΅ соотвСтствуСт ΠΏΡ€Π°Π²ΠΈΠ»Ρƒ (Π·Π°ΠΊΠΎΠ½Ρƒ) Π›Π΅Π½Ρ†Π°.

Π—Π°ΠΊΠΎΠ½ ЀарадСя ΠΌΠΎΠΆΠ½ΠΎ ΡΡ„ΠΎΡ€ΠΌΡƒΠ»ΠΈΡ€ΠΎΠ²Π°Ρ‚ΡŒ Π΅Ρ‰Π΅ Ρ‚Π°ΠΊΠΈΠΌ ΠΎΠ±Ρ€Π°Π·ΠΎΠΌ: Π­Π”Π‘ элСктромагнитной ΠΈΠ½Π΄ΡƒΠΊΡ†ΠΈΠΈ E i Π² ΠΊΠΎΠ½Ρ‚ΡƒΡ€Π΅ числСнно Ρ€Π°Π²Π½Π° ΠΈ ΠΏΡ€ΠΎΡ‚ΠΈΠ²ΠΎΠΏΠΎΠ»ΠΎΠΆΠ½Π° ΠΏΠΎ Π·Π½Π°ΠΊΡƒ скорости измСнСния ΠΌΠ°Π³Π½ΠΈΡ‚Π½ΠΎΠ³ΠΎ ΠΏΠΎΡ‚ΠΎΠΊΠ° сквозь ΠΏΠΎΠ²Π΅Ρ€Ρ…Π½ΠΎΡΡ‚ΡŒ, ΠΎΠ³Ρ€Π°Π½ΠΈΡ‡Π΅Π½Π½ΡƒΡŽ этим ΠΊΠΎΠ½Ρ‚ΡƒΡ€ΠΎΠΌ. Π­Ρ‚ΠΎΡ‚ Π·Π°ΠΊΠΎΠ½ являСтся ΡƒΠ½ΠΈΠ²Π΅Ρ€ΡΠ°Π»ΡŒΠ½Ρ‹ΠΌ: Π­Π”Π‘ E i Π½Π΅ зависит ΠΎΡ‚ способа измСнСния ΠΌΠ°Π³Π½ΠΈΡ‚Π½ΠΎΠ³ΠΎ ΠΏΠΎΡ‚ΠΎΠΊΠ°.

Π—Π½Π°ΠΊ минус Π² (27.2) ΠΏΠΎΠΊΠ°Π·Ρ‹Π²Π°Π΅Ρ‚, Ρ‡Ρ‚ΠΎ ΡƒΠ²Π΅Π»ΠΈΡ‡Π΅Π½ΠΈΠ΅ ΠΏΠΎΡ‚ΠΎΠΊΠ° ( > 0) Π²Ρ‹Π·Ρ‹Π²Π°Π΅Ρ‚ Π­Π”Π‘ E i 0 Ρ‚. Π΅. направлСния ΠΌΠ°Π³Π½ΠΈΡ‚Π½ΠΎΠ³ΠΎ ΠΏΠΎΡ‚ΠΎΠΊΠ° ΠΈΠ½Π΄ΡƒΠΊΡ†ΠΈΠΎΠ½Π½ΠΎΠ³ΠΎ Ρ‚ΠΎΠΊΠ° ΠΈ ΠΏΠΎΡ‚ΠΎΠΊΠ°, Π²Ρ‹Π·Π²Π°Π²ΡˆΠ΅Π³ΠΎ Π΅Π³ΠΎ, ΡΠΎΠ²ΠΏΠ°Π΄Π°ΡŽΡ‚. Π—Π½Π°ΠΊ минус Π² Ρ„ΠΎΡ€ΠΌΡƒΠ»Π΅ (27.2) являСтся матСматичСским Π²Ρ‹Ρ€Π°ΠΆΠ΅Π½ΠΈΠ΅ΠΌ ΠΏΡ€Π°Π²ΠΈΠ»Π° Π›Π΅Π½Ρ†Π° – ΠΎΠ±Ρ‰Π΅Π³ΠΎ ΠΏΡ€Π°Π²ΠΈΠ»Π° для нахоТдСния направлСния ΠΈΠ½Π΄ΡƒΠΊΡ†ΠΈΠΎΠ½Π½ΠΎΠ³ΠΎ Ρ‚ΠΎΠΊΠ° (Π° Π·Π½Π°Ρ‡ΠΈΡ‚ ΠΈ Π·Π½Π°ΠΊΠ° ΠΈ Π­Π”Π‘ ΠΈΠ½Π΄ΡƒΠΊΡ†ΠΈΠΈ), Π²Ρ‹Π²Π΅Π΄Π΅Π½Π½ΠΎΠ³ΠΎ Π² 1833 Π³. ΠŸΡ€Π°Π²ΠΈΠ»ΠΎ Π›Π΅Π½Ρ†Π°: ΠΈΠ½Π΄ΡƒΠΊΡ†ΠΈΠΎΠ½Π½Ρ‹ΠΉ Ρ‚ΠΎΠΊ всСгда Π½Π°ΠΏΡ€Π°Π²Π»Π΅Π½ Ρ‚Π°ΠΊ, Ρ‡Ρ‚ΠΎΠ±Ρ‹ ΠΏΡ€ΠΎΡ‚ΠΈΠ²ΠΎΠ΄Π΅ΠΉΡΡ‚Π²ΠΎΠ²Π°Ρ‚ΡŒ ΠΏΡ€ΠΈΡ‡ΠΈΠ½Π΅, Π΅Π³ΠΎ Π²Ρ‹Π·Ρ‹Π²Π°ΡŽΡ‰Π΅ΠΉ. Π˜Π½Π°Ρ‡Π΅ говоря, ΠΈΠ½Π΄ΡƒΠΊΡ†ΠΈΠΎΠ½Π½Ρ‹ΠΉ Ρ‚ΠΎΠΊ создаСт ΠΌΠ°Π³Π½ΠΈΡ‚Π½Ρ‹ΠΉ ΠΏΠΎΡ‚ΠΎΠΊ, ΠΏΡ€Π΅ΠΏΡΡ‚ΡΡ‚Π²ΡƒΡŽΡ‰ΠΈΠΉ измСнСнию ΠΌΠ°Π³Π½ΠΈΡ‚Π½ΠΎΠ³ΠΎ ΠΏΠΎΡ‚ΠΎΠΊΠ°, Π²Ρ‹Π·Ρ‹Π²Π°ΡŽΡ‰Π΅Π³ΠΎ Π­Π”Π‘ ΠΈΠ½Π΄ΡƒΠΊΡ†ΠΈΠΈ.

Π­Π”Π‘ ΠΈΠ½Π΄ΡƒΠΊΡ†ΠΈΠΈ выраТаСтся Π² Π²ΠΎΠ»ΡŒΡ‚Π°Ρ… (Π’). Π”Π΅ΠΉΡΡ‚Π²ΠΈΡ‚Π΅Π»ΡŒΠ½ΠΎ, учитывая, Ρ‡Ρ‚ΠΎ Π΅Π΄ΠΈΠ½ΠΈΡ†Π΅ΠΉ ΠΌΠ°Π³Π½ΠΈΡ‚Π½ΠΎΠ³ΠΎ ΠΏΠΎΡ‚ΠΎΠΊΠ° являСтся Π²Π΅Π±Π΅Ρ€ (Π’Π±), ΠΏΠΎΠ»ΡƒΡ‡ΠΈΠΌ:

Если Π·Π°ΠΌΠΊΠ½ΡƒΡ‚Ρ‹ΠΉ ΠΊΠΎΠ½Ρ‚ΡƒΡ€, Π² ΠΊΠΎΡ‚ΠΎΡ€ΠΎΠΌ индуцируСтся Π­Π”Π‘ ΠΈΠ½Π΄ΡƒΠΊΡ†ΠΈΠΈ, состоит ΠΈΠ· N Π²ΠΈΡ‚ΠΊΠΎΠ², Ρ‚ΠΎ E i Π±ΡƒΠ΄Π΅Ρ‚ Ρ€Π°Π²Π½Π° суммС Π­Π”Π‘, ΠΈΠ½Π΄ΡƒΡ†ΠΈΡ€ΡƒΠ΅ΠΌΡ‹Ρ… Π² ΠΊΠ°ΠΆΠ΄ΠΎΠΌ ΠΈΠ· Π²ΠΈΡ‚ΠΊΠΎΠ². И Ссли ΠΌΠ°Π³Π½ΠΈΡ‚Π½Ρ‹ΠΉ ΠΏΠΎΡ‚ΠΎΠΊ, ΠΎΡ…Π²Π°Ρ‚Ρ‹Π²Π°Π΅ΠΌΡ‹ΠΉ ΠΊΠ°ΠΆΠ΄Ρ‹ΠΌ Π²ΠΈΡ‚ΠΊΠΎΠΌ, ΠΎΠ΄ΠΈΠ½Π°ΠΊΠΎΠ² ΠΈ Ρ€Π°Π²Π΅Π½ Π€, Ρ‚ΠΎ суммарный ΠΏΠΎΡ‚ΠΎΠΊ сквозь ΠΏΠΎΠ²Π΅Ρ€Ρ…Π½ΠΎΡΡ‚ΡŒ N Π²ΠΈΡ‚ΠΊΠΎΠ², Ρ€Π°Π²Π΅Π½ (NΠ€) – ΠΏΠΎΠ»Π½Ρ‹ΠΉ ΠΌΠ°Π³Π½ΠΈΡ‚Π½Ρ‹ΠΉ ΠΏΠΎΡ‚ΠΎΠΊ (потокосцСплСниС). Π’ этом случаС Π­Π”Π‘ ΠΈΠ½Π΄ΡƒΠΊΡ†ΠΈΠΈ Ρ€Π°Π²Π½Π°:

E i = -NΓ— , (27.3)

Π€ΠΎΡ€ΠΌΡƒΠ»Π° (27.2) Π²Ρ‹Ρ€Π°ΠΆΠ°Π΅Ρ‚ Π·Π°ΠΊΠΎΠ½ элСктромагнитной ΠΈΠ½Π΄ΡƒΠΊΡ†ΠΈΠΈ Π² ΠΎΠ±Ρ‰Π΅ΠΉ Ρ„ΠΎΡ€ΠΌΠ΅. Она ΠΏΡ€ΠΈΠΌΠ΅Π½ΠΈΠΌΠ° ΠΊΠ°ΠΊ ΠΊ Π½Π΅ΠΏΠΎΠ΄Π²ΠΈΠΆΠ½Ρ‹ΠΌ ΠΊΠΎΠ½Ρ‚ΡƒΡ€Π°ΠΌ, Ρ‚Π°ΠΊ ΠΈ ΠΊ двиТущимся ΠΏΡ€ΠΎΠ²ΠΎΠ΄Π½ΠΈΠΊΠ°ΠΌ Π² ΠΌΠ°Π³Π½ΠΈΡ‚Π½ΠΎΠΌ ΠΏΠΎΠ»Π΅. Входящая Π² Π½Π΅Π΅ производная ΠΎΡ‚ ΠΌΠ°Π³Π½ΠΈΡ‚Π½ΠΎΠ³ΠΎ ΠΏΠΎΡ‚ΠΎΠΊΠ° ΠΏΠΎ Π²Ρ€Π΅ΠΌΠ΅Π½ΠΈ Π² ΠΎΠ±Ρ‰Π΅ΠΌ случаС состоит ΠΈΠ· Π΄Π²ΡƒΡ… частСй, ΠΎΠ΄Π½Π° ΠΈΠ· ΠΊΠΎΡ‚ΠΎΡ€Ρ‹Ρ… обусловлСна ΠΈΠ·ΠΌΠ΅Π½Π΅Π½ΠΈΠ΅ΠΌ ΠΌΠ°Π³Π½ΠΈΡ‚Π½ΠΎΠΉ ΠΈΠ½Π΄ΡƒΠΊΡ†ΠΈΠΈ Π²ΠΎ Π²Ρ€Π΅ΠΌΠ΅Π½ΠΈ, Π° другая – Π΄Π²ΠΈΠΆΠ΅Π½ΠΈΠ΅ΠΌ ΠΊΠΎΠ½Ρ‚ΡƒΡ€Π° ΠΎΡ‚Π½ΠΎΡΠΈΡ‚Π΅Π»ΡŒΠ½ΠΎ ΠΌΠ°Π³Π½ΠΈΡ‚Π½ΠΎΠ³ΠΎ поля (ΠΈΠ»ΠΈ Π΅Π³ΠΎ Π΄Π΅Ρ„ΠΎΡ€ΠΌΠ°Ρ†ΠΈΠ΅ΠΉ). Рассмотрим Π½Π΅ΠΊΠΎΡ‚ΠΎΡ€Ρ‹Π΅ ΠΏΡ€ΠΈΠΌΠ΅Ρ€Ρ‹ примСнСния этого Π·Π°ΠΊΠΎΠ½Π°.

ΠŸΡ€ΠΈΠΌΠ΅Ρ€ 1. ΠŸΡ€ΡΠΌΠΎΠ»ΠΈΠ½Π΅ΠΉΠ½Ρ‹ΠΉ ΠΏΡ€ΠΎΠ²ΠΎΠ΄Π½ΠΈΠΊ Π΄Π»ΠΈΠ½ΠΎΠΉ l двиТСтся ΠΏΠ°Ρ€Π°Π»Π»Π΅Π»ΡŒΠ½ΠΎ самому сСбС Π² ΠΎΠ΄Π½ΠΎΡ€ΠΎΠ΄Π½ΠΎΠΌ ΠΌΠ°Π³Π½ΠΈΡ‚Π½ΠΎΠΌ ΠΏΠΎΠ»Π΅ (рисунок 38). Π­Ρ‚ΠΎΡ‚ ΠΏΡ€ΠΎΠ²ΠΎΠ΄Π½ΠΈΠΊ ΠΌΠΎΠΆΠ΅Ρ‚ Π²Ρ…ΠΎΠ΄ΠΈΡ‚ΡŒ Π² состав Π·Π°ΠΌΠΊΠ½ΡƒΡ‚ΠΎΠΉ Ρ†Π΅ΠΏΠΈ, ΠΎΡΡ‚Π°Π»ΡŒΠ½Ρ‹Π΅ части ΠΊΠΎΡ‚ΠΎΡ€ΠΎΠΉ Π½Π΅ΠΏΠΎΠ΄Π²ΠΈΠΆΠ½Ρ‹. НайдСм Π­Π”Π‘, Π²ΠΎΠ·Π½ΠΈΠΊΠ°ΡŽΡ‰ΡƒΡŽ Π² ΠΏΡ€ΠΎΠ²ΠΎΠ΄Π½ΠΈΠΊΠ΅.

Если ΠΌΠ³Π½ΠΎΠ²Π΅Π½Π½ΠΎΠ΅ Π·Π½Π°Ρ‡Π΅Π½ΠΈΠ΅ скорости ΠΏΡ€ΠΎΠ²ΠΎΠ΄Π½ΠΈΠΊΠ° Π΅ΡΡ‚ΡŒ v , Ρ‚ΠΎ Π·Π° врСмя dt ΠΎΠ½ ΠΎΠΏΠΈΡˆΠ΅Ρ‚ ΠΏΠ»ΠΎΡ‰Π°Π΄ΡŒ dS = lΓ—v Γ—dt ΠΈ Π·Π° это врСмя пСрСсСчСт всС Π»ΠΈΠ½ΠΈΠΈ ΠΌΠ°Π³Π½ΠΈΡ‚Π½ΠΎΠΉ ΠΈΠ½Π΄ΡƒΠΊΡ†ΠΈΠΈ, проходящиС Ρ‡Π΅Ρ€Π΅Π· dS. ΠŸΠΎΡΡ‚ΠΎΠΌΡƒ ΠΈΠ·ΠΌΠ΅Π½Π΅Π½ΠΈΠ΅ ΠΌΠ°Π³Π½ΠΈΡ‚Π½ΠΎΠ³ΠΎ ΠΏΠΎΡ‚ΠΎΠΊΠ° Ρ‡Π΅Ρ€Π΅Π· ΠΊΠΎΠ½Ρ‚ΡƒΡ€, Π² состав ΠΊΠΎΡ‚ΠΎΡ€ΠΎΠ³ΠΎ Π²Ρ…ΠΎΠ΄ΠΈΡ‚ двиТущийся ΠΏΡ€ΠΎΠ²ΠΎΠ΄Π½ΠΈΠΊ, Π±ΡƒΠ΄Π΅Ρ‚ dΠ€ = B n Γ—lΓ—v Γ—dt. Π—Π΄Π΅ΡΡŒ B n – ΡΠΎΡΡ‚Π°Π²Π»ΡΡŽΡ‰Π°Ρ ΠΌΠ°Π³Π½ΠΈΡ‚Π½ΠΎΠΉ ΠΈΠ½Π΄ΡƒΠΊΡ†ΠΈΠΈ, пСрпСндикулярная ΠΊ dS. ΠŸΠΎΠ΄ΡΡ‚Π°Π²Π»ΡΡ это Π² Ρ„ΠΎΡ€ΠΌΡƒΠ»Ρƒ (27.2) ΠΏΠΎΠ»ΡƒΡ‡Π°Π΅ΠΌ Π²Π΅Π»ΠΈΡ‡ΠΈΠ½Ρƒ Π­Π”Π‘:

E i = B n Γ—lΓ—v . (27.4)

НаправлСниС ΠΈΠ½Π΄ΡƒΠΊΡ†ΠΈΠΎΠ½Π½ΠΎΠ³ΠΎ Ρ‚ΠΎΠΊΠ° ΠΈ Π·Π½Π°ΠΊ Π­Π”Π‘ ΠΎΠΏΡ€Π΅Π΄Π΅Π»ΡΡŽΡ‚ΡΡ ΠΏΡ€Π°Π²ΠΈΠ»ΠΎΠΌ Π›Π΅Π½Ρ†Π°: ΠΈΠ½Π΄ΡƒΠΊΡ†ΠΈΠΎΠ½Π½Ρ‹ΠΉ Ρ‚ΠΎΠΊ Π² ΠΊΠΎΠ½Ρ‚ΡƒΡ€Π΅ всСгда ΠΈΠΌΠ΅Π΅Ρ‚ Ρ‚Π°ΠΊΠΎΠ΅ Π½Π°ΠΏΡ€Π°Π²Π»Π΅Π½ΠΈΠ΅, Ρ‡Ρ‚ΠΎ создаваСмоС ΠΈΠΌ ΠΌΠ°Π³Π½ΠΈΡ‚Π½ΠΎΠ΅ ΠΏΠΎΠ»Π΅ прСпятствуСт измСнСнию ΠΌΠ°Π³Π½ΠΈΡ‚Π½ΠΎΠ³ΠΎ ΠΏΠΎΡ‚ΠΎΠΊΠ°, Π²Ρ‹Π·Π²Π°Π²ΡˆΠ΅Π³ΠΎ этот ΠΈΠ½Π΄ΡƒΠΊΡ†ΠΈΠΎΠ½Π½Ρ‹ΠΉ Ρ‚ΠΎΠΊ. Π’ Π½Π΅ΠΊΠΎΡ‚ΠΎΡ€Ρ‹Ρ… случаях Π²ΠΎΠ·ΠΌΠΎΠΆΠ½ΠΎ ΠΎΠΏΡ€Π΅Π΄Π΅Π»Π΅Π½ΠΈΠ΅ направлСния ΠΈΠ½Π΄ΡƒΠΊΡ†ΠΈΠΎΠ½Π½ΠΎΠ³ΠΎ Ρ‚ΠΎΠΊΠ° (полярности Π­Π”Π‘ ΠΈΠ½Π΄ΡƒΠΊΡ†ΠΈΠΈ) согласно Π΄Ρ€ΡƒΠ³ΠΎΠΉ Ρ„ΠΎΡ€ΠΌΡƒΠ»ΠΈΡ€ΠΎΠ²ΠΊΠ΅ ΠΏΡ€Π°Π²ΠΈΠ»Π° Π›Π΅Π½Ρ†Π°: ΠΈΠ½Π΄ΡƒΠΊΡ†ΠΈΠΎΠ½Π½Ρ‹ΠΉ Ρ‚ΠΎΠΊ Π² двиТущСмся ΠΏΡ€ΠΎΠ²ΠΎΠ΄Π½ΠΈΠΊΠ΅ Π½Π°ΠΏΡ€Π°Π²Π»Π΅Π½ Ρ‚Π°ΠΊΠΈΠΌ ΠΎΠ±Ρ€Π°Π·ΠΎΠΌ, Ρ‡Ρ‚ΠΎ Π²ΠΎΠ·Π½ΠΈΠΊΠ°ΡŽΡ‰Π°Ρ ΠΏΡ€ΠΈ этом сила АмпСра ΠΏΡ€ΠΎΡ‚ΠΈΠ²ΠΎΠΏΠΎΠ»ΠΎΠΆΠ½Π° Π²Π΅ΠΊΡ‚ΠΎΡ€Ρƒ скорости (Ρ‚ΠΎΡ€ΠΌΠΎΠ·ΠΈΡ‚ Π΄Π²ΠΈΠΆΠ΅Π½ΠΈΠ΅).

Π Π°Π·Π±Π΅Ρ€Π΅ΠΌ числСнный ΠΏΡ€ΠΈΠΌΠ΅Ρ€. Π’Π΅Ρ€Ρ‚ΠΈΠΊΠ°Π»ΡŒΠ½Ρ‹ΠΉ ΠΏΡ€ΠΎΠ²ΠΎΠ΄Π½ΠΈΠΊ (Π°Π²Ρ‚ΠΎΠΌΠΎΠ±ΠΈΠ»ΡŒΠ½Π°Ρ Π°Π½Ρ‚Π΅Π½Π½Π°) Π΄Π»ΠΈΠ½ΠΎΠΉ l = 2 ΠΌ двиТСтся с востока Π½Π° Π·Π°ΠΏΠ°Π΄ Π² ΠΌΠ°Π³Π½ΠΈΡ‚Π½ΠΎΠΌ ΠΏΠΎΠ»Π΅ Π—Π΅ΠΌΠ»ΠΈ со ΡΠΊΠΎΡ€ΠΎΡΡ‚ΡŒΡŽ v = 72 ΠΊΠΌ/час = 20 ΠΌ/с. Вычислим напряТСниС ΠΌΠ΅ΠΆΠ΄Ρƒ ΠΊΠΎΠ½Ρ†Π°ΠΌΠΈ ΠΏΡ€ΠΎΠ²ΠΎΠ΄Π½ΠΈΠΊΠ°. Π’Π°ΠΊ ΠΊΠ°ΠΊ ΠΏΡ€ΠΎΠ²ΠΎΠ΄Π½ΠΈΠΊ Ρ€Π°Π·ΠΎΠΌΠΊΠ½ΡƒΡ‚, Ρ‚ΠΎ Ρ‚ΠΎΠΊΠ° Π² Π½Π΅ΠΌ Π½Π΅ Π±ΡƒΠ΄Π΅Ρ‚ ΠΈ напряТСниС Π½Π° ΠΊΠΎΠ½Ρ†Π°Ρ… Π±ΡƒΠ΄Π΅Ρ‚ Ρ€Π°Π²Π½ΠΎ Π­Π”Π‘ ΠΈΠ½Π΄ΡƒΠΊΡ†ΠΈΠΈ. Учитывая, Ρ‡Ρ‚ΠΎ Π³ΠΎΡ€ΠΈΠ·ΠΎΠ½Ρ‚Π°Π»ΡŒΠ½Π°Ρ ΡΠΎΡΡ‚Π°Π²Π»ΡΡŽΡ‰Π°Ρ ΠΌΠ°Π³Π½ΠΈΡ‚Π½ΠΎΠΉ ΠΈΠ½Π΄ΡƒΠΊΡ†ΠΈΠΈ поля Π—Π΅ΠΌΠ»ΠΈ (Ρ‚.Π΅. ΡΠΎΡΡ‚Π°Π²Π»ΡΡŽΡ‰Π°Ρ, пСрпСндикулярная ΠΊ Π½Π°ΠΏΡ€Π°Π²Π»Π΅Π½ΠΈΡŽ двиТСния) для срСдних ΡˆΠΈΡ€ΠΎΡ‚ Ρ€Π°Π²Π½Π° 2Γ—10 -5 Π’Π», ΠΏΠΎ Ρ„ΠΎΡ€ΠΌΡƒΠ»Π΅ (27.4) Π½Π°Ρ…ΠΎΠ΄ΠΈΠΌ

U = B n Γ—lΓ—v = 2Γ—10 -5 Γ—2Γ—20 = 0,8Γ—10 -3 Π’,

Ρ‚.Π΅. ΠΎΠΊΠΎΠ»ΠΎ 1 ΠΌΠ’. ΠœΠ°Π³Π½ΠΈΡ‚Π½ΠΎΠ΅ ΠΏΠΎΠ»Π΅ Π—Π΅ΠΌΠ»ΠΈ Π½Π°ΠΏΡ€Π°Π²Π»Π΅Π½ΠΎ с юга Π½Π° сСвСр. ΠŸΠΎΡΡ‚ΠΎΠΌΡƒ ΠΌΡ‹ Π½Π°Ρ…ΠΎΠ΄ΠΈΠΌ, Ρ‡Ρ‚ΠΎ Π­Π”Π‘ Π½Π°ΠΏΡ€Π°Π²Π»Π΅Π½Π° свСрху Π²Π½ΠΈΠ·. Π­Ρ‚ΠΎ Π·Π½Π°Ρ‡ΠΈΡ‚, Ρ‡Ρ‚ΠΎ Π½ΠΈΠΆΠ½ΠΈΠΉ ΠΊΠΎΠ½Π΅Ρ† ΠΏΡ€ΠΎΠ²ΠΎΠ΄Π° Π±ΡƒΠ΄Π΅Ρ‚ ΠΈΠΌΠ΅Ρ‚ΡŒ Π±ΠΎΠ»Π΅Π΅ высокий ΠΏΠΎΡ‚Π΅Π½Ρ†ΠΈΠ°Π» (зарядится ΠΏΠΎΠ»ΠΎΠΆΠΈΡ‚Π΅Π»ΡŒΠ½ΠΎ), Π° Π²Π΅Ρ€Ρ…Π½ΠΈΠΉ – Π±ΠΎΠ»Π΅Π΅ Π½ΠΈΠ·ΠΊΠΈΠΉ (зарядится ΠΎΡ‚Ρ€ΠΈΡ†Π°Ρ‚Π΅Π»ΡŒΠ½ΠΎ).

ΠŸΡ€ΠΈΠΌΠ΅Ρ€ 2. Π’ ΠΌΠ°Π³Π½ΠΈΡ‚Π½ΠΎΠΌ ΠΏΠΎΠ»Π΅ находится Π·Π°ΠΌΠΊΠ½ΡƒΡ‚Ρ‹ΠΉ ΠΏΡ€ΠΎΠ²ΠΎΠ»ΠΎΡ‡Π½Ρ‹ΠΉ ΠΊΠΎΠ½Ρ‚ΡƒΡ€, ΠΏΡ€ΠΎΠ½ΠΈΠ·Ρ‹Π²Π°Π΅ΠΌΡ‹ΠΉ ΠΌΠ°Π³Π½ΠΈΡ‚Π½Ρ‹ΠΌ ΠΏΠΎΡ‚ΠΎΠΊΠΎΠΌ Π€. ΠŸΡ€Π΅Π΄ΠΏΠΎΠ»ΠΎΠΆΠΈΠΌ, Ρ‡Ρ‚ΠΎ этот ΠΏΠΎΡ‚ΠΎΠΊ ΡƒΠΌΠ΅Π½ΡŒΡˆΠ°Π΅Ρ‚ΡΡ Π΄ΠΎ нуля, ΠΈ вычислим ΠΏΠΎΠ»Π½ΡƒΡŽ Π²Π΅Π»ΠΈΡ‡ΠΈΠ½Ρƒ заряда, ΠΏΡ€ΠΎΡˆΠ΅Π΄ΡˆΠ΅Π³ΠΎ ΠΏΠΎ Ρ†Π΅ΠΏΠΈ. МгновСнноС Π·Π½Π°Ρ‡Π΅Π½ΠΈΠ΅ Π­Π”Π‘ Π² процСссС исчСзновСния ΠΌΠ°Π³Π½ΠΈΡ‚Π½ΠΎΠ³ΠΎ ΠΏΠΎΡ‚ΠΎΠΊΠ° выраТаСтся Ρ„ΠΎΡ€ΠΌΡƒΠ»ΠΎΠΉ (27.2). Π‘Π»Π΅Π΄ΠΎΠ²Π°Ρ‚Π΅Π»ΡŒΠ½ΠΎ, согласно Π·Π°ΠΊΠΎΠ½Ρƒ Ома ΠΌΠ³Π½ΠΎΠ²Π΅Π½Π½ΠΎΠ΅ Π·Π½Π°Ρ‡Π΅Π½ΠΈΠ΅ силы Ρ‚ΠΎΠΊΠ° Π΅ΡΡ‚ΡŒ

Π³Π΄Π΅ R – ΠΏΠΎΠ»Π½ΠΎΠ΅ сопротивлСниС Ρ†Π΅ΠΏΠΈ.

Π’Π΅Π»ΠΈΡ‡ΠΈΠ½Π° ΠΏΡ€ΠΎΡˆΠ΅Π΄ΡˆΠ΅Π³ΠΎ заряда Ρ€Π°Π²Π½Π°

q = = – = . (27.6)

ΠŸΠΎΠ»ΡƒΡ‡Π΅Π½Π½ΠΎΠ΅ ΡΠΎΠΎΡ‚Π½ΠΎΡˆΠ΅Π½ΠΈΠ΅ Π²Ρ‹Ρ€Π°ΠΆΠ°Π΅Ρ‚ Π·Π°ΠΊΠΎΠ½ элСктромагнитной ΠΈΠ½Π΄ΡƒΠΊΡ†ΠΈΠΈ Π² Ρ„ΠΎΡ€ΠΌΠ΅, Π½Π°ΠΉΠ΄Π΅Π½Π½ΠΎΠΉ Π€Π°Ρ€Π°Π΄Π΅Π΅ΠΌ, ΠΊΠΎΡ‚ΠΎΡ€Ρ‹ΠΉ ΠΈΠ· своих ΠΎΠΏΡ‹Ρ‚ΠΎΠ² Π·Π°ΠΊΠ»ΡŽΡ‡ΠΈΠ», Ρ‡Ρ‚ΠΎ Π²Π΅Π»ΠΈΡ‡ΠΈΠ½Π° заряда, ΠΏΡ€ΠΎΡˆΠ΅Π΄ΡˆΠ΅Π³ΠΎ ΠΏΠΎ Ρ†Π΅ΠΏΠΈ, ΠΏΡ€ΠΎΠΏΠΎΡ€Ρ†ΠΈΠΎΠ½Π°Π»ΡŒΠ½Π° ΠΏΠΎΠ»Π½ΠΎΠΌΡƒ числу Π»ΠΈΠ½ΠΈΠΉ ΠΌΠ°Π³Π½ΠΈΡ‚Π½ΠΎΠΉ ΠΈΠ½Π΄ΡƒΠΊΡ†ΠΈΠΈ, пСрСсСчСнных ΠΏΡ€ΠΎΠ²ΠΎΠ΄Π½ΠΈΠΊΠΎΠΌ (Ρ‚.Π΅. измСнСнию ΠΌΠ°Π³Π½ΠΈΡ‚Π½ΠΎΠ³ΠΎ ΠΏΠΎΡ‚ΠΎΠΊΠ° Π€ 1 -Π€ 2), ΠΈ ΠΎΠ±Ρ€Π°Ρ‚Π½ΠΎ ΠΏΡ€ΠΎΠΏΠΎΡ€Ρ†ΠΈΠΎΠ½Π°Π»ΡŒΠ½Π° ΡΠΎΠΏΡ€ΠΎΡ‚ΠΈΠ²Π»Π΅Π½ΠΈΡŽ Ρ†Π΅ΠΏΠΈ R. Π‘ΠΎΠΎΡ‚Π½ΠΎΡˆΠ΅Π½ΠΈΠ΅ (27.6) позволяСт Π΄Π°Ρ‚ΡŒ ΠΎΠΏΡ€Π΅Π΄Π΅Π»Π΅Π½ΠΈΠ΅ Π΅Π΄ΠΈΠ½ΠΈΡ†Ρ‹ ΠΌΠ°Π³Π½ΠΈΡ‚Π½ΠΎΠ³ΠΎ ΠΏΠΎΡ‚ΠΎΠΊΠ° Π² систСмС БИ: Π²Π΅Π±Π΅Ρ€ – ΠΌΠ°Π³Π½ΠΈΡ‚Π½Ρ‹ΠΉ ΠΏΠΎΡ‚ΠΎΠΊ, ΠΏΡ€ΠΈ ΡƒΠ±Ρ‹Π²Π°Π½ΠΈΠΈ ΠΊΠΎΡ‚ΠΎΡ€ΠΎΠ³ΠΎ Π΄ΠΎ нуля Π² сцСплСнном с Π½ΠΈΠΌ ΠΊΠΎΠ½Ρ‚ΡƒΡ€Π΅ сопротивлСниСм 1 Ом ΠΏΡ€ΠΎΡ…ΠΎΠ΄ΠΈΡ‚ заряд 1 Кл.

Богласно Π·Π°ΠΊΠΎΠ½Ρƒ ЀарадСя, Π²ΠΎΠ·Π½ΠΈΠΊΠ½ΠΎΠ²Π΅Π½ΠΈΠ΅ Π­Π”Π‘ элСктромагнитной ΠΈΠ½Π΄ΡƒΠΊΡ†ΠΈΠΈ Π²ΠΎΠ·ΠΌΠΎΠΆΠ½ΠΎ ΠΈ Π² случаС Π½Π΅ΠΏΠΎΠ΄Π²ΠΈΠΆΠ½ΠΎΠ³ΠΎ ΠΊΠΎΠ½Ρ‚ΡƒΡ€Π°, находящСгося Π² ΠΏΠ΅Ρ€Π΅ΠΌΠ΅Π½Π½ΠΎΠΌ ΠΌΠ°Π³Π½ΠΈΡ‚Π½ΠΎΠΌ ΠΏΠΎΠ»Π΅. Однако сила Π›ΠΎΡ€Π΅Π½Ρ†Π° Π½Π° Π½Π΅ΠΏΠΎΠ΄Π²ΠΈΠΆΠ½Ρ‹Π΅ заряды Π½Π΅ дСйствуСт, поэтому Π² Π΄Π°Π½Π½ΠΎΠΌ случаС ΠΎΠ½Π° Π½Π΅ ΠΌΠΎΠΆΠ΅Ρ‚ Π±Ρ‹Ρ‚ΡŒ ΠΏΡ€ΠΈΡ‡ΠΈΠ½ΠΎΠΉ возникновСния Π­Π”Π‘ ΠΈΠ½Π΄ΡƒΠΊΡ†ΠΈΠΈ. МаксвСлл для объяснСния Π­Π”Π‘ ΠΈΠ½Π΄ΡƒΠΊΡ†ΠΈΠΈ Π² Π½Π΅ΠΏΠΎΠ΄Π²ΠΈΠΆΠ½Ρ‹Ρ… ΠΏΡ€ΠΎΠ²ΠΎΠ΄Π½ΠΈΠΊΠ°Ρ… ΠΏΡ€Π΅Π΄ΠΏΠΎΠ»ΠΎΠΆΠΈΠ», Ρ‡Ρ‚ΠΎ всякоС ΠΏΠ΅Ρ€Π΅ΠΌΠ΅Π½Π½ΠΎΠ΅ ΠΌΠ°Π³Π½ΠΈΡ‚Π½ΠΎΠ΅ ΠΏΠΎΠ»Π΅ Π²ΠΎΠ·Π±ΡƒΠΆΠ΄Π°Π΅Ρ‚ Π² ΠΎΠΊΡ€ΡƒΠΆΠ°ΡŽΡ‰Π΅ΠΌ пространствС Π²ΠΈΡ…Ρ€Π΅Π²ΠΎΠ΅ элСктричСскоС ΠΏΠΎΠ»Π΅, ΠΊΠΎΡ‚ΠΎΡ€ΠΎΠ΅ ΠΈ являСтся ΠΏΡ€ΠΈΡ‡ΠΈΠ½ΠΎΠΉ возникновСния ΠΈΠ½Π΄ΡƒΠΊΡ†ΠΈΠΎΠ½Π½ΠΎΠ³ΠΎ Ρ‚ΠΎΠΊΠ° Π² ΠΏΡ€ΠΎΠ²ΠΎΠ΄Π½ΠΈΠΊΠ΅. Циркуляция Π²Π΅ΠΊΡ‚ΠΎΡ€Π° напряТСнности этого поля ΠΏΠΎ Π»ΡŽΠ±ΠΎΠΌΡƒ Π½Π΅ΠΏΠΎΠ΄Π²ΠΈΠΆΠ½ΠΎΠΌΡƒ ΠΊΠΎΠ½Ρ‚ΡƒΡ€Ρƒ L ΠΏΡ€ΠΎΠ²ΠΎΠ΄Π½ΠΈΠΊΠ° прСдставляСт собой Π­Π”Π‘ элСктромагнитной ΠΈΠ½Π΄ΡƒΠΊΡ†ΠΈΠΈ:

E i = = – . (27.7)

Π›ΠΈΠ½ΠΈΠΈ напряТСнности Π²ΠΈΡ…Ρ€Π΅Π²ΠΎΠ³ΠΎ элСктричСского поля ΠΏΡ€Π΅Π΄ΡΡ‚Π°Π²Π»ΡΡŽΡ‚ собой Π·Π°ΠΌΠΊΠ½ΡƒΡ‚Ρ‹Π΅ ΠΊΡ€ΠΈΠ²Ρ‹Π΅, поэтому ΠΏΡ€ΠΈ ΠΏΠ΅Ρ€Π΅ΠΌΠ΅Ρ‰Π΅Π½ΠΈΠΈ заряда Π² Π²ΠΈΡ…Ρ€Π΅Π²ΠΎΠΌ элСктричСском ΠΏΠΎΠ»Π΅ ΠΏΠΎ Π·Π°ΠΌΠΊΠ½ΡƒΡ‚ΠΎΠΌΡƒ ΠΊΠΎΠ½Ρ‚ΡƒΡ€Ρƒ ΡΠΎΠ²Π΅Ρ€ΡˆΠ°Π΅Ρ‚ΡΡ отличная ΠΎΡ‚ нуля Ρ€Π°Π±ΠΎΡ‚Π°. Π’ этом Π·Π°ΠΊΠ»ΡŽΡ‡Π°Π΅Ρ‚ΡΡ ΠΎΡ‚Π»ΠΈΡ‡ΠΈΠ΅ Π²ΠΈΡ…Ρ€Π΅Π²ΠΎΠ³ΠΎ элСктричСского поля ΠΎΡ‚ элСктростатичСского, Π»ΠΈΠ½ΠΈΠΈ напряТСнности ΠΊΠΎΡ‚ΠΎΡ€ΠΎΠ³ΠΎ Π½Π°Ρ‡ΠΈΠ½Π°ΡŽΡ‚ΡΡ ΠΈ Π·Π°ΠΊΠ°Π½Ρ‡ΠΈΠ²Π°ΡŽΡ‚ΡΡ Π½Π° зарядах.

ЭлСктромагнитная индукция. Π€ΠΎΡ€ΠΌΡƒΠ»Π°. ΠœΠ°Π³Π½ΠΈΡ‚Π½ΠΎΠ΅ ΠΏΠΎΠ»Π΅. ЭлСктричСскоС ΠΏΠΎΠ»Π΅. Π’ΠΎΠΊ смСщСния. РСлятивистский эффСкт. Заряд.

ο»Ώ

ЭлСктромагнитная индукция. Π€ΠΎΡ€ΠΌΡƒΠ»Π°. ΠœΠ°Π³Π½ΠΈΡ‚Π½ΠΎΠ΅ ΠΏΠΎΠ»Π΅. ЭлСктричСскоС ΠΏΠΎΠ»Π΅. Π’ΠΎΠΊ смСщСния. РСлятивистский эффСкт. Заряд.

ЭлСктродинамичСскоС взаимодСйствиС, ΠΎΠ±Ρ€Π°Π·ΠΎΠ²Π°Π½Π½ΠΎΠ΅ Ρ‚ΠΎΠΊΠ°ΠΌΠΈ элСктричСского смСщСния поля.

«ВсякоС Π²ΠΎΠ·ΠΌΡƒΡ‰Π΅Π½ΠΈΠ΅ Π² пространствС распространяСтся со ΡΠΊΠΎΡ€ΠΎΡΡ‚ΡŒΡŽ Π½Π΅ Π²Ρ‹ΡˆΠ΅ скорости свСта. Π’ частности, элСктричСскоС ΠΏΠΎΠ»Π΅ ΠΏΡ€ΠΈ смСщСнии Ρ‚ΠΎΡ‡Π΅Ρ‡Π½ΠΎΠ³ΠΎ заряда Π½Π΅ просто пСрСмСстится вмСстС с зарядом, ΠΊΠ°ΠΊ Π² случаС бСсконСчно большой скорости распространСния поля, Π° мСняСтся Π±ΠΎΠ»Π΅Π΅ слоТным ΠΎΠ±Ρ€Π°Π·ΠΎΠΌ. Π’ΠΎΠ·Π½ΠΈΠΊΠ°ΡŽΡ‚ эффСкты, связанныС с Π·Π°ΠΏΠ°Π·Π΄Ρ‹Π²Π°Π½ΠΈΠ΅ΠΌ появлСния поля Π½Π° Π±ΠΎΠ»ΡŒΡˆΠΈΡ… расстояниях ΠΎΡ‚ заряда, ΠΊΠΎΡ‚ΠΎΡ€Ρ‹Π΅ ΠΌΠΎΠ³ΡƒΡ‚ Π±Ρ‹Ρ‚ΡŒ описаны Π²Π²Π΅Π΄Π΅Π½ΠΈΠ΅ΠΌ ΠΈΠ½Π΄ΡƒΠΊΡ†ΠΈΠΈ ΠΌΠ°Π³Π½ΠΈΡ‚Π½ΠΎΠ³ΠΎ поля.Β»
ΠžΡΠ½ΠΎΠ²Ρ‹ Ρ„ΠΈΠ·ΠΈΠΊΠΈ. Π›.А.Π“Ρ€ΠΈΠ±ΠΎΠ², Н.И.ΠŸΡ€ΠΎΠΊΠΎΡ„ΡŒΠ΅Π². 1995. Π‘.300.
Β 
ΠœΠ°Π³Π½ΠΈΡ‚Π½ΠΎΠ΅ ΠΏΠΎΠ»Π΅ (ΠΏΠΎΡ‚ΠΎΠΊ) ΠΌΠΎΠΆΠ½ΠΎ Ρ€Π°ΡΡΠΌΠ°Ρ‚Ρ€ΠΈΠ²Π°Ρ‚ΡŒ ΠΊΠ°ΠΊ рСлятивистский эффСкт (эффСкт двиТСния), связанный с Π·Π°ΠΏΠ°Π·Π΄Ρ‹Π²Π°Π½ΠΈΠ΅ΠΌ распространСния элСктричСского смСщСния поля. Богласно Ρ„ΠΎΡ€ΠΌΡƒΠ»Π΅ прСобразования ΠΏΠΎΠ»Π΅ΠΉ BΒ =Β ΞΌ0[vD], магнитная индукция Π²ΠΎΠ·Π½ΠΈΠΊΠ°Π΅Ρ‚ ΠΏΡ€ΠΈ Π΄Π²ΠΈΠΆΠ΅Π½ΠΈΠΈ элСктричСского ΠΏΠΎΡ‚ΠΎΠΊΠ°. ΠœΠ°Π³Π½ΠΈΡ‚Π½ΡƒΡŽ ΡΠ½Π΅Ρ€Π³ΠΈΡŽ ΠΌΠΎΠΆΠ½ΠΎ Ρ‚Ρ€Π°ΠΊΡ‚ΠΎΠ²Π°Ρ‚ΡŒ ΠΊΠ°ΠΊ ΠΊΠΈΠ½Π΅Ρ‚ΠΈΡ‡Π΅ΡΠΊΡƒΡŽ ΡΠ½Π΅Ρ€Π³ΠΈΡŽ двиТущихся элСктричСских ΠΏΠΎΡ‚ΠΎΠΊΠΎΠ² WΠΌΒ =Β Mэv2Β sin2a, Π³Π΄Π΅ Mэ – масса элСктричСского ΠΏΠΎΡ‚ΠΎΠΊΠ°, v – ΡΠΊΠΎΡ€ΠΎΡΡ‚ΡŒ двиТСния, a – ΡƒΠ³ΠΎΠ» ΠΌΠ΅ΠΆΠ΄Ρƒ Π½Π°ΠΏΡ€Π°Π²Π»Π΅Π½ΠΈΠ΅ΠΌ двиТСния ΠΈ Π²Π΅ΠΊΡ‚ΠΎΡ€ΠΎΠΌ D.
Β 
Β« BΒ =Β [vE]/c2 Β»
Π­Π»Π΅ΠΊΡ‚Ρ€ΠΎΠΌΠ°Π³Π½Π΅Ρ‚ΠΈΠ·ΠΌ. И.Π•.Π˜Ρ€ΠΎΠ΄ΠΎΠ². 1991. Π‘.135.
Β«Π’ Ρ€Π΅Π·ΡƒΠ»ΡŒΡ‚Π°Ρ‚Π΅ ΠΌΠ°Π³Π½ΠΈΡ‚Π½ΠΎΠ΅ ΠΏΠΎΠ»Π΅ ΠΌΠΎΠΆΠ½ΠΎ Ρ€Π°ΡΡΠΌΠ°Ρ‚Ρ€ΠΈΠ²Π°Ρ‚ΡŒ ΠΊΠ°ΠΊ Π½Π΅ΠΈΠ·Π±Π΅ΠΆΠ½Ρ‹ΠΉ рСлятивистский Ρ€Π΅Π·ΡƒΠ»ΡŒΡ‚Π°Ρ‚ двиТСния элСктричСских зарядов (Ρ‚ΠΎΠΊΠ°) ΠΈ нСстационарности создаваСмого ΠΈΠΌΠΈ элСктричСского поля (Ρ‚ΠΎΠΊΠ° смСщСния).Β»
ЀизичСская энциклопСдия. Π­Π›Π•ΠšΠ’Π ΠžΠ”Π˜ΠΠΠœΠ˜ΠšΠ.
Β 
ΠœΠ°Π³Π½ΠΈΡ‚Π½ΠΎΠ΅ ΠΏΠΎΠ»Π΅ Π²ΠΎΠ·Π½ΠΈΠΊΠ°Π΅Ρ‚ ΠΊΠ°ΠΊ Ρ€Π΅Π·ΡƒΠ»ΡŒΡ‚Π°Ρ‚ двиТСния элСктричСских зарядов (ΠΏΠΎΡ‚ΠΎΠΊΠΎΠ²), Π½ΠΎ Π΅Ρ‰Π΅ нСдостаточно рассмотрСн сам ΠΌΠ΅Ρ…Π°Π½ΠΈΠ·ΠΌ Π΅Π³ΠΎ возникновСния. ΠŸΠΎΡΡ‚Π°Ρ€Π°ΡŽΡΡŒ ΠΏΡ€ΠΎΠ°Π½Π°Π»ΠΈΠ·ΠΈΡ€ΠΎΠ²Π°Ρ‚ΡŒ ΠΈ ΠΎΠΏΠΈΡΠ°Ρ‚ΡŒ элСктродинамичСскиС процСссы, ΠΏΡ€ΠΎΡ‚Π΅ΠΊΠ°ΡŽΡ‰ΠΈΠ΅ ΠΏΡ€ΠΈ Π΄Π²ΠΈΠΆΠ΅Π½ΠΈΠΈ элСктричСских зарядов.
Β 
ВозмущСния поля Π½Π΅ Ρ€Π°ΡΠΏΡ€ΠΎΡΡ‚Ρ€Π°Π½ΡΡŽΡ‚ΡΡ ΠΌΠ³Π½ΠΎΠ²Π΅Π½Π½ΠΎ, для возникновСния возмущСния трСбуСтся ΠΎΠΏΡ€Π΅Π΄Π΅Π»Π΅Π½Π½ΠΎΠ΅ врСмя. ΠŸΡ€ΠΈ Π΄Π²ΠΈΠΆΠ΅Π½ΠΈΠΈ заряда Π²ΠΎΠ·ΠΌΡƒΡ‰Π΅Π½ΠΈΠ΅ поля, возникая Π² Ρ‚ΠΎΠΌ мСстС, ΠΊΡƒΠ΄Π° пСрСмСстился заряд, ΠΈ ΠΎΠ΄Π½ΠΎΠ²Ρ€Π΅ΠΌΠ΅Π½Π½ΠΎ исчСзая Π² Ρ‚ΠΎΠΌ мСстС, ΠΎΡ‚ΠΊΡƒΠ΄Π° ΠΎΠ½ пСрСмСстился, ΠΎΠ±Ρ€Π°Π·ΡƒΠ΅Ρ‚ Π² пространствС Ρ‚ΠΎΠΊΠΈ элСктричСского смСщСния, ΠΊΠΎΡ‚ΠΎΡ€Ρ‹Π΅ ΠΈΠΌΠ΅ΡŽΡ‚ ΠΎΠ±Ρ€Π°Ρ‚Π½ΠΎΠ΅ Π½Π°ΠΏΡ€Π°Π²Π»Π΅Π½ΠΈΠ΅. ΠŸΡ€ΠΈΠΌΠ΅Ρ€Ρ‹ расчСтов ΠΎΠ±Ρ€Π°Ρ‚Π½Ρ‹Ρ… Ρ‚ΠΎΠΊΠΎΠ² смСщСния ΠΏΡ€ΠΈΠ²Π΅Π΄Π΅Π½Ρ‹ Π² ΡƒΡ‡Π΅Π±Π½ΠΈΠΊΠ°Ρ….
Β 
Β«ΠŸΡ€ΠΈΠΌΠ΅Ρ€. Π’ΠΎΡ‡Π΅Ρ‡Π½Ρ‹ΠΉ заряд q двиТСтся Ρ€Π°Π²Π½ΠΎΠΌΠ΅Ρ€Π½ΠΎ ΠΈ прямолинСйно с нСрСлятивистской ΡΠΊΠΎΡ€ΠΎΡΡ‚ΡŒΡŽ v. Найти Π²Π΅ΠΊΡ‚ΠΎΡ€ плотности Ρ‚ΠΎΠΊΠ° смСщСния Π² Ρ‚ΠΎΡ‡ΠΊΠ΅ P, находящСйся Π½Π° расстоянии r ΠΎΡ‚ заряда Π½Π° прямой, пСрпСндикулярной Π΅Π³ΠΎ Ρ‚Ρ€Π°Π΅ΠΊΡ‚ΠΎΡ€ΠΈΠΈ ΠΈ проходящСй Ρ‡Π΅Ρ€Π΅Π· заряд. РСшСниС: jсм =Β -qv/4Ο€r3.Β»
Π­Π»Π΅ΠΊΡ‚Ρ€ΠΎΠΌΠ°Π³Π½Π΅Ρ‚ΠΈΠ·ΠΌ. И.Π•.Π˜Ρ€ΠΎΠ΄ΠΎΠ². 2000. Π‘.302.
Β 
ΠžΡ‚ΡΡŽΠ΄Π°, Π·Π° ΠΏΡ€Π΅Π΄Π΅Π»Π°ΠΌΠΈ радиуса r ΠΎΡ‚ двиТущСгося Ρ‚ΠΎΡ‡Π΅Ρ‡Π½ΠΎΠ³ΠΎ заряда Ρ‚Π΅Ρ‡Π΅Ρ‚ ΠΎΠ±Ρ€Π°Ρ‚Π½Ρ‹ΠΉ Ρ‚ΠΎΠΊ смСщСния:

Iсм = -qv/2r.

ΠŸΠΎΠ»ΡƒΡ‡Π°Π΅Ρ‚ΡΡ, Ссли заряд Π² 1 Кл двиТСтся со ΡΠΊΠΎΡ€ΠΎΡΡ‚ΡŒΡŽ 2Β ΠΌ/с, Ρ‚ΠΎ Π·Π° ΠΏΡ€Π΅Π΄Π΅Π»Π°ΠΌΠΈ радиуса Π² 1Β ΠΌ Ρ‚Π΅Ρ‡Π΅Ρ‚ ΠΎΠ±Ρ€Π°Ρ‚Π½Ρ‹ΠΉ Ρ‚ΠΎΠΊ смСщСния силой Π² 1 А, ΠΏΠ»ΠΎΡ‚Π½ΠΎΡΡ‚ΡŒ ΠΆΠ΅ ΠΎΠ±Ρ€Π°Ρ‚Π½ΠΎΠ³ΠΎ Ρ‚ΠΎΠΊΠ° смСщСния Π½Π° расстоянии 1Β ΠΌ Ρ€Π°Π²Π½Π° 0.16 А/ΠΌ2. Π’.Π΅., Ссли двиТСтся заряТСнный ΡˆΠ°Ρ€ радиусом Π² 1Β ΠΌ, с зарядом Π² 1 Кл ΠΈ со ΡΠΊΠΎΡ€ΠΎΡΡ‚ΡŒΡŽ 2Β ΠΌ/с, Ρ‚ΠΎ ΠΏΠΎΠ»Π½Ρ‹ΠΉ ΠΎΠ±Ρ€Π°Ρ‚Π½Ρ‹ΠΉ Ρ‚ΠΎΠΊ смСщСния, ΠΊΠΎΡ‚ΠΎΡ€Ρ‹ΠΉ ΠΎΠ½ создаСт, Ρ€Π°Π²Π΅Π½ 1 А. ΠžΡ‚ΡΡŽΠ΄Π° слСдуСт, Ρ‡Ρ‚ΠΎ Ρ‚ΠΎΠΊ смСщСния Π½Π΅ влияСт Π½Π° Π²Π΅Π»ΠΈΡ‡ΠΈΠ½Ρƒ ΠΌΠ°Π³Π½ΠΈΡ‚Π½ΠΎΠΉ ΠΈΠ½Π΄ΡƒΠΊΡ†ΠΈΠΈ, Ρ‚Π°ΠΊ ΠΊΠ°ΠΊ нСзависимо ΠΎΡ‚ Π²Π΅Π»ΠΈΡ‡ΠΈΠ½Ρ‹ Ρ‚ΠΎΠΊΠ° смСщСния (Ρ€Π°Π·ΠΌΠ΅Ρ€Π° ΡˆΠ°Ρ€Π°) магнитная индукция Π²ΠΎΠΊΡ€ΡƒΠ³ ΡˆΠ°Ρ€Π° всСгда Ρ€Π°Π²Π½Π° BΒ =Β ΞΌ0q[vr]/4Ο€r3. Π’ΠΎ, Ρ‡Ρ‚ΠΎ Ρ‚ΠΎΠΊ элСктричСского смСщСния Π½Π΅ создаСт ΠΌΠ°Π³Π½ΠΈΡ‚Π½ΡƒΡŽ ΠΈΠ½Π΄ΡƒΠΊΡ†ΠΈΡŽ, Π²Ρ…ΠΎΠ΄ΠΈΡ‚ Π² ΠΏΡ€ΠΎΡ‚ΠΈΠ²ΠΎΡ€Π΅Ρ‡ΠΈΠ΅ с постулатом, ΡƒΡ‚Π²Π΅Ρ€ΠΆΠ΄Π°ΡŽΡ‰ΠΈΠΌ, Ρ‡Ρ‚ΠΎ Ρ‚ΠΎΠΊ смСщСния создаСт Ρ‚Π°ΠΊΠΎΠ΅ ΠΆΠ΅ ΠΌΠ°Π³Π½ΠΈΡ‚Π½ΠΎΠ΅ ΠΏΠΎΠ»Π΅, ΠΊΠ°ΠΊ ΠΈ Ρ‚ΠΎΠΊ проводимости. ΠŸΡ€ΠΈΡ‡ΠΈΠ½Π° возникновСния Ρ‚Π°ΠΊΠΎΠ³ΠΎ противорСчия Π² Ρ‚ΠΎΠΌ, Ρ‡Ρ‚ΠΎ Π½Π° Ρ‚ΠΎΠΊ смСщСния, Π±Π΅Π· ΡΠΊΡΠΏΠ΅Ρ€ΠΈΠΌΠ΅Π½Ρ‚Π°Π»ΡŒΠ½ΠΎΠΉ ΠΏΡ€ΠΎΠ²Π΅Ρ€ΠΊΠΈ, Π±Ρ‹Π»ΠΈ пСрСнСсСны ΠΌΠ°Π³Π½ΠΈΡ‚Π½Ρ‹Π΅ свойства Ρ‚ΠΎΠΊΠ° проводимости. Π’.Π΅. Π±Ρ‹Π» ΡƒΠΏΡƒΡ‰Π΅Π½ Ρ‚ΠΎΡ‚ Ρ„Π°ΠΊΡ‚, Ρ‡Ρ‚ΠΎ ΠΌΠ°Π³Π½ΠΈΡ‚Π½ΠΎΠ΅ ΠΏΠΎΠ»Π΅, согласно элСктродинамикС, прСдставляСт двиТущийся элСктричСский ΠΏΠΎΡ‚ΠΎΠΊ BΒ =Β ΞΌ0[vD], Π° Π½Π΅ Ρ‚ΠΎΠΊ, Ρ‚Π°ΠΊ ΠΊΠ°ΠΊ Π² Ρ„ΠΎΡ€ΠΌΡƒΠ»Π΅ D – это ΠΏΠ»ΠΎΡ‚Π½ΠΎΡΡ‚ΡŒ элСктричСского ΠΏΠΎΡ‚ΠΎΠΊΠ°. БоотвСтствСнно, для опрСдСлСния ΠΌΠ°Π³Π½ΠΈΡ‚Π½ΠΎΠΉ ΠΈΠ½Π΄ΡƒΠΊΡ†ΠΈΠΈ Π½Π΅ΠΎΠ±Ρ…ΠΎΠ΄ΠΈΠΌΠΎ Ρ€Π°ΡΡΠΌΠ°Ρ‚Ρ€ΠΈΠ²Π°Ρ‚ΡŒ Π½Π΅ Ρ‚ΠΎ, ΠΊΠ°ΠΊ Ρ‚Π΅Ρ‡Π΅Ρ‚ Ρ‚ΠΎΠΊ проводимости ΠΈΠ»ΠΈ смСщСния, Π° ΠΊΠ°ΠΊ двиТСтся элСктричСский ΠΏΠΎΡ‚ΠΎΠΊ. Данная Ρ„ΠΎΡ€ΠΌΡƒΠ»Π°, прСдставляя Π·Π°ΠΊΠΎΠ½ ΠΌΠ°Π³Π½ΠΈΡ‚Π½ΠΎΠΉ ΠΈΠ½Π΄ΡƒΠΊΡ†ΠΈΠΈ, Π»Π΅ΠΆΠΈΡ‚ Π² основС всСх ΠΎΡΡ‚Π°Π»ΡŒΠ½Ρ‹Ρ… Ρ„ΠΎΡ€ΠΌΡƒΠ». НапримСр, вмСстС с двиТущимся зарядом двиТСтся Π΅Π³ΠΎ элСктричСский ΠΏΠΎΡ‚ΠΎΠΊ. Зная ΠΏΠ»ΠΎΡ‚Π½ΠΎΡΡ‚ΡŒ связанного с зарядом двиТущСгося элСктричСского ΠΏΠΎΡ‚ΠΎΠΊΠ° DΒ =Β qr/4Ο€r3, ΠΌΠΎΠΆΠ½ΠΎ, согласно BΒ =Β ΞΌ0[vD], Π²Ρ‹Ρ‡ΠΈΡΠ»ΠΈΡ‚ΡŒ ΠΏΠ»ΠΎΡ‚Π½ΠΎΡΡ‚ΡŒ ΠΌΠ°Π³Π½ΠΈΡ‚Π½ΠΎΠ³ΠΎ ΠΏΠΎΡ‚ΠΎΠΊΠ° Π²ΠΎΠΊΡ€ΡƒΠ³ заряда BΒ =Β ΞΌ0q[vr]/4Ο€r3. Аналогичным ΠΎΠ±Ρ€Π°Π·ΠΎΠΌ, зная, Ρ‡Ρ‚ΠΎ с ΠΊΠ°ΠΆΠ΄Ρ‹ΠΌ двиТущимся зарядом связан двиТущийся элСктричСский ΠΏΠΎΡ‚ΠΎΠΊ, выводятся ΠΈ Π΄Ρ€ΡƒΠ³ΠΈΠ΅ Ρ„ΠΎΡ€ΠΌΡƒΠ»Ρ‹ для расчСта ΠΌΠ°Π³Π½ΠΈΡ‚Π½ΠΎΠΉ ΠΈΠ½Π΄ΡƒΠΊΡ†ΠΈΠΈ. НапримСр, ΠΏΠ»ΠΎΡ‚Π½ΠΎΡΡ‚ΡŒ двиТущСгося элСктричСского ΠΏΠΎΡ‚ΠΎΠΊΠ° Π²ΠΎΠΊΡ€ΡƒΠ³ прямого бСсконСчного ΠΏΡ€ΠΎΠ²ΠΎΠ΄Π° с Ρ‚ΠΎΠΊΠΎΠΌ DΒ =Β P/2Ο€rΒ =Β q/2Ο€rL, Π³Π΄Π΅ P – ΠΏΠ»ΠΎΡ‚Π½ΠΎΡΡ‚ΡŒ двиТущихся зарядов Π² ΠΏΡ€ΠΎΠ²ΠΎΠ΄Π΅ (PΒ =Β q/L), r – расстояниС ΠΎΡ‚ ΠΏΡ€ΠΎΠ²ΠΎΠ΄Π°. Богласно BΒ =Β ΞΌ0[vD], ΠΏΠΎΠ»ΡƒΡ‡ΠΈΠΌ BΒ =Β ΞΌ0qv/2Ο€rLΒ =Β ΞΌ0I/2Ο€r, Π³Π΄Π΅ I – сила Ρ‚ΠΎΠΊΠ° (IΒ =Β PvΒ =Β qv/L). ВсС это относится ΠΊΠ°ΠΊ ΠΊ Ρ‚ΠΎΠΊΡƒ проводимости, Ρ‚Π°ΠΊ ΠΈ ΠΊ Ρ‚ΠΎΠΊΡƒ смСщСния, Ρ‚.Π΅. сначала Π½Π°Π΄ΠΎ Ρ€Π°ΡΡΠΌΠΎΡ‚Ρ€Π΅Ρ‚ΡŒ Π΄Π²ΠΈΠΆΠ΅Π½ΠΈΠ΅ элСктричСских ΠΏΠΎΡ‚ΠΎΠΊΠΎΠ², Π° ΠΏΠΎΡ‚ΠΎΠΌ, согласно BΒ =Β ΞΌ0[vD], ΠΎΠΏΡ€Π΅Π΄Π΅Π»ΠΈΡ‚ΡŒ ΠΌΠ°Π³Π½ΠΈΡ‚Π½ΡƒΡŽ ΠΈΠ½Π΄ΡƒΠΊΡ†ΠΈΡŽ.


Β 
Β«… ΠΏΠΎΠ»Π΅ B Ρ‚ΠΎΡ‡Π΅Ρ‡Π½ΠΎΠ³ΠΎ заряда q, двиТущСгося с постоянной нСрСлятивистской ΡΠΊΠΎΡ€ΠΎΡΡ‚ΡŒΡŽ v. … BΒ =Β ΞΌ0q[vr]/4Ο€r3Β …Β»
Π­Π»Π΅ΠΊΡ‚Ρ€ΠΎΠΌΠ°Π³Π½Π΅Ρ‚ΠΈΠ·ΠΌ. И.Π•.Π˜Ρ€ΠΎΠ΄ΠΎΠ². 2000. Π‘.155.
Β«… магнитная индукция поля прямого Ρ‚ΠΎΠΊΠ° BΒ =Β ΞΌ0I/2Ο€r.Β»
ΠšΡƒΡ€Ρ Ρ„ΠΈΠ·ΠΈΠΊΠΈ. Π’.И.Π’Ρ€ΠΎΡ„ΠΈΠΌΠΎΠ²Π°. 1998. Π‘.208.
Β 
Π—Π½Π°ΠΊ минус Π² Ρ„ΠΎΡ€ΠΌΡƒΠ»Π΅ Iсм =Β -qv/2r ΠΎΠ·Π½Π°Ρ‡Π°Π΅Ρ‚, Ρ‡Ρ‚ΠΎ Ρ‚ΠΎΠΊ смСщСния Ρ‚Π΅Ρ‡Π΅Ρ‚ Π² ΠΎΠ±Ρ€Π°Ρ‚Π½ΠΎΠΌ Π½Π°ΠΏΡ€Π°Π²Π»Π΅Π½ΠΈΠΈ. ΠŸΡ€ΠΈ этом Ρ‚ΠΎΠΊ смСщСния Π²ΠΎΠ·Π½ΠΈΠΊΠ°Π΅Ρ‚ нСзависимо ΠΎΡ‚ Ρ‚ΠΎΠ³ΠΎ, двиТСтся Π»ΠΈ заряд ΡΠ°ΠΌΠΎΡΡ‚ΠΎΡΡ‚Π΅Π»ΡŒΠ½ΠΎ ΠΈΠ»ΠΈ, Π½Π°ΠΏΡ€ΠΈΠΌΠ΅Ρ€, ΠΏΠΎ ΠΏΡ€ΠΎΠ²ΠΎΠ΄Π½ΠΈΠΊΡƒ, Π³Π΄Π΅ Ρ‚ΠΎΠΊ смСщСния распространяСтся Π² пространствС Π·Π° ΠΏΡ€Π΅Π΄Π΅Π»Π°ΠΌΠΈ ΠΏΡ€ΠΎΠ²ΠΎΠ΄Π½ΠΈΠΊΠ° ΠΈ, Ссли рядом находится Π΄Ρ€ΡƒΠ³ΠΎΠΉ ΠΏΡ€ΠΎΠ²ΠΎΠ΄Π½ΠΈΠΊ, Ρ‚ΠΎ Π² Π½Π΅ΠΌ ΠΎΠ±Ρ€Π°Ρ‚Π½Ρ‹ΠΉ Ρ‚ΠΎΠΊ смСщСния Π±ΡƒΠ΄Π΅Ρ‚ ΠΏΠ΅Ρ€Π΅Ρ…ΠΎΠ΄ΠΈΡ‚ΡŒ Π² Ρ‚ΠΎΠΊ проводимости. Π’.Π΅. Ρ‚ΠΎΠΊ смСщСния Π±ΡƒΠ΄Π΅Ρ‚ Π·Π°ΠΌΡ‹ΠΊΠ°Ρ‚ΡŒΡΡ Ρ‚ΠΎΠΊΠΎΠΌ проводимости – это явлСниС называСтся элСктромагнитной ΠΈΠ½Π΄ΡƒΠΊΡ†ΠΈΠ΅ΠΉ.
Β 
«Для ΠΌΠ°Π³Π½ΠΈΡ‚Π½ΠΎΠ³ΠΎ поля, Ρ‚Π°ΠΊ ΠΆΠ΅ ΠΊΠ°ΠΊ для элСктричСского, справСдлив ΠΏΡ€ΠΈΠ½Ρ†ΠΈΠΏ супСрпозиции;Β …Β»
Π‘ΠΏΡ€Π°Π²ΠΎΡ‡Π½ΠΈΠΊ ΠΏΠΎ Ρ„ΠΈΠ·ΠΈΠΊΠ΅. Π‘.М.Яворский, А.А.Π”Π΅Ρ‚Π»Π°Ρ„. 1996. Π‘.236.
Β«… Ρ‚ΠΎΠΊ смСщСния ΠΏΠΎ своСй сути – это ΠΈΠ·ΠΌΠ΅Π½ΡΡŽΡ‰Π΅Π΅ΡΡ со Π²Ρ€Π΅ΠΌΠ΅Π½Π΅ΠΌ элСктричСскоС ΠΏΠΎΠ»Π΅.Β»
ΠšΡƒΡ€Ρ Ρ„ΠΈΠ·ΠΈΠΊΠΈ. Π’.И.Π’Ρ€ΠΎΡ„ΠΈΠΌΠΎΠ²Π°. 1998. Π‘.250.
Β 
Для Ρ‚ΠΎΠΊΠ° смСщСния, ΠΊΠ°ΠΊ ΠΈ для поля, дСйствуСт ΠΏΡ€ΠΈΠ½Ρ†ΠΈΠΏ супСрпозиции (для любого Ρ‚ΠΎΠΊΠ° дСйствуСт ΠΏΡ€ΠΈΠ½Ρ†ΠΈΠΏ супСрпозиции), Ρ‚.Π΅., Ссли двиТутся нСсколько зарядов, Ρ‚ΠΎ ΠΈΡ… ΠΎΠ±Ρ€Π°Ρ‚Π½Ρ‹Π΅ Ρ‚ΠΎΠΊΠΈ смСщСния ΡΠΊΠ»Π°Π΄Ρ‹Π²Π°ΡŽΡ‚ΡΡ Π² пространствС согласно ΠΏΡ€ΠΈΠ½Ρ†ΠΈΠΏΡƒ супСрпозиции. НапримСр, ΠΏΠ»ΠΎΡ‚Π½ΠΎΡΡ‚ΡŒ ΠΎΠ±Ρ€Π°Ρ‚Π½ΠΎΠ³ΠΎ постоянного Ρ‚ΠΎΠΊΠ° смСщСния Π²ΠΎΠΊΡ€ΡƒΠ³ Ρ‚ΠΎΠ½ΠΊΠΎΠ³ΠΎ прямого ΠΏΡ€ΠΎΠ²ΠΎΠ΄Π° бСсконСчной Π΄Π»ΠΈΠ½Ρ‹:

jсм =Β -I/2Ο€r2,

Π³Π΄Π΅ r – расстояниС ΠΎΡ‚ оси ΠΏΡ€ΠΎΠ²ΠΎΠ΄Π°, I - постоянный Ρ‚ΠΎΠΊ Π² ΠΏΡ€ΠΎΠ²ΠΎΠ΄Π΅. Π’.Π΅. Π½Π΅ Ρ‚ΠΎΠ»ΡŒΠΊΠΎ Π²ΠΎΠΊΡ€ΡƒΠ³ двиТущСгося заряда, Π½ΠΎ ΠΈ Π²ΠΎΠΊΡ€ΡƒΠ³ ΠΏΡ€ΠΎΠ²ΠΎΠ΄Π½ΠΈΠΊΠ° с Ρ‚ΠΎΠΊΠΎΠΌ Ρ‚Π΅Ρ‡Π΅Ρ‚ ΠΎΠ±Ρ€Π°Ρ‚Π½Ρ‹ΠΉ Ρ‚ΠΎΠΊ смСщСния.


Β 
Β«… ΠΊΠ°ΠΆΠ΄Ρ‹ΠΉ заряд Π²ΠΎΠ·Π±ΡƒΠΆΠ΄Π°Π΅Ρ‚ ΠΏΠΎΠ»Π΅, ΡΠΎΠ²Π΅Ρ€ΡˆΠ΅Π½Π½ΠΎ Π½Π΅ зависящСС ΠΎΡ‚ наличия Π΄Ρ€ΡƒΠ³ΠΈΡ… зарядов.Β»
ΠžΠ±Ρ‰ΠΈΠΉ курс Ρ„ΠΈΠ·ΠΈΠΊΠΈ. ЭлСктричСство. Π”.Π’.Π‘ΠΈΠ²ΡƒΡ…ΠΈΠ½. 1996. Π’.3. Π§.2. Π‘.204.
Β 
НСзависимо ΠΎΡ‚ Ρ‚ΠΎΠ³ΠΎ двиТСтся заряд ΡΠ°ΠΌΠΎΡΡ‚ΠΎΡΡ‚Π΅Π»ΡŒΠ½ΠΎ ΠΈΠ»ΠΈ, Π½Π°ΠΏΡ€ΠΈΠΌΠ΅Ρ€, Π² ΠΏΡ€ΠΎΠ²ΠΎΠ΄Π½ΠΈΠΊΠ΅, всСгда Π² ΠΎΠΊΡ€ΡƒΠΆΠ°ΡŽΡ‰Π΅ΠΌ пространствС вмСстС с Π½ΠΈΠΌ двиТСтся элСктричСский ΠΏΠΎΡ‚ΠΎΠΊ. Π’Π°ΠΌ, Π³Π΄Π΅ происходит Π΄Π²ΠΈΠΆΠ΅Π½ΠΈΠ΅ элСктричСских ΠΏΠΎΡ‚ΠΎΠΊΠΎΠ², всСгда Ρ‚Π΅Ρ‡Π΅Ρ‚ Ρ‚ΠΎΠΊ смСщСния, Π΄Π°ΠΆΠ΅ Ссли Π½Π΅ измСняСтся элСктричСскоС ΠΏΠΎΠ»Π΅ (элСктричСская Π½Π°ΠΏΡ€ΡΠΆΠ΅Π½Π½ΠΎΡΡ‚ΡŒ). НапримСр, Ссли двиТСтся заряд, Ρ‚ΠΎ Π²ΠΎΠΊΡ€ΡƒΠ³ Π½Π΅Π³ΠΎ Π²ΠΎΠ·Π½ΠΈΠΊΠ°Π΅Ρ‚ элСктричСский Ρ‚ΠΎΠΊ смСщСния, ΠΊΠΎΠ³Π΄Π° ΠΆΠ΅ ΠΏΠΎ ΠΏΡ€ΠΎΠ²ΠΎΠ΄Π½ΠΈΠΊΡƒ двиТСтся мноТСство зарядов, прСдставляя постоянный Ρ‚ΠΎΠΊ проводимости, Ρ‚ΠΎ элСктричСскоС ΠΏΠΎΠ»Π΅ Π² пространствС Π½Π΅ измСняСтся, Π½ΠΎ супСрпозиция Ρ‚ΠΎΠΊΠΎΠ² смСщСния ΠΎΡ‚ всСх двиТущихся зарядов прСдставляСт постоянный Ρ‚ΠΎΠΊ смСщСния, ΠΊΠΎΡ‚ΠΎΡ€Ρ‹ΠΉ Ρ‚Π΅Ρ‡Π΅Ρ‚ Π² ΠΎΠ±Ρ€Π°Ρ‚Π½ΠΎΠΌ Π½Π°ΠΏΡ€Π°Π²Π»Π΅Π½ΠΈΠΈ.
Β 
Β«Π’ΠΎΠΊ смСщСния Π²Ρ…ΠΎΠ΄ΠΈΡ‚ Π² МаксвСлла уравнСния Π½Π° Ρ€Π°Π²Π½Ρ‹Ρ… ΠΏΡ€Π°Π²Π°Ρ… с Ρ‚ΠΎΠΊΠΎΠΌ, обусловлСнным Π΄Π²ΠΈΠΆΠ΅Π½ΠΈΠ΅ΠΌ зарядов.Β»
ЀизичСский энциклопСдичСский ΡΠ»ΠΎΠ²Π°Ρ€ΡŒ. Π­Π›Π•ΠšΠ’Π Π˜Π§Π•Π‘ΠšΠ˜Π™ ВОК.
Β«Π’ΠΎΠΊ смСщСния, Π² ΠΎΡ‚Π»ΠΈΡ‡ΠΈΠ΅ ΠΎΡ‚ Ρ‚ΠΎΠΊΠ° проводимости, Π½Π΅ сопровоТдаСтся Π²Ρ‹Π΄Π΅Π»Π΅Π½ΠΈΠ΅ΠΌ Ρ‚Π΅ΠΏΠ»ΠΎΡ‚Ρ‹.Β»
Π‘ΠΏΡ€Π°Π²ΠΎΡ‡Π½ΠΈΠΊ ΠΏΠΎ Ρ„ΠΈΠ·ΠΈΠΊΠ΅. Π‘.М.Яворский, А.А.Π”Π΅Ρ‚Π»Π°Ρ„. 1996. Π‘.290.
Β 
ЭлСктричСский Ρ‚ΠΎΠΊ смСщСния поля Ρ‚Π΅Ρ‡Π΅Ρ‚ Π±Π΅Π· сопротивлСния, Π° Ρ€Π°Π· Π½Π΅Ρ‚ сопротивлСния, Ρ‚ΠΎ Π½Π΅Ρ‚ ΠΈ напряТСнности (Π·Π°ΠΊΠΎΠ½ Ома). ЭлСктричСская Π½Π°ΠΏΡ€ΡΠΆΠ΅Π½Π½ΠΎΡΡ‚ΡŒ поля Π²ΠΎΠ·Π½ΠΈΠΊΠ°Π΅Ρ‚ Ρ‚ΠΎΠ»ΡŒΠΊΠΎ ΠΏΡ€ΠΈ ΠΈΠ·ΠΌΠ΅Π½Π΅Π½ΠΈΠΈ Ρ‚ΠΎΠΊΠ° смСщСния ΠΊΠ°ΠΊ Π²ΠΈΡ…Ρ€Π΅Π²ΠΎΠ΅ элСктричСскоС ΠΏΠΎΠ»Π΅. НапримСр, Π²ΠΎΠ·Π½ΠΈΠΊΠ½ΠΎΠ²Π΅Π½ΠΈΠ΅ Π²ΠΈΡ…Ρ€Π΅Π²ΠΎΠ³ΠΎ элСктричСского поля ΠΏΡ€ΠΈ Π²ΠΊΠ»ΡŽΡ‡Π΅Π½ΠΈΠΈ ΠΈ Π²Ρ‹ΠΊΠ»ΡŽΡ‡Π΅Π½ΠΈΠΈ элСктромагнита Π³ΠΎΠ²ΠΎΡ€ΠΈΡ‚ ΠΎ Ρ‚ΠΎΠΌ, Ρ‡Ρ‚ΠΎ Ρ‚ΠΎΠΊ смСщСния поля ΠΏΡ€ΠΈ Π²ΠΊΠ»ΡŽΡ‡Π΅Π½ΠΈΠΈ возрастаСт, Π° ΠΏΡ€ΠΈ Π²Ρ‹ΠΊΠ»ΡŽΡ‡Π΅Π½ΠΈΠΈ ΡƒΠΌΠ΅Π½ΡŒΡˆΠ°Π΅Ρ‚ΡΡ. Π’ ΠΏΠ΅Ρ€ΠΈΠΎΠ΄, ΠΊΠΎΠ³Π΄Π° ΠΌΠ°Π³Π½ΠΈΡ‚Π½ΠΎΠ΅ ΠΏΠΎΠ»Π΅ Π½Π΅ измСняСтся, ΠΏΠ»ΠΎΡ‚Π½ΠΎΡΡ‚ΡŒ Ρ‚ΠΎΠΊΠ° смСщСния Ρ‚Π°ΠΊΠΆΠ΅ Π½Π΅ измСняСтся ΠΈ, соотвСтствСнно, Π½Π΅Ρ‚ Π²ΠΈΡ…Ρ€Π΅Π²ΠΎΠ³ΠΎ элСктричСского поля, поэтому постоянноС ΠΌΠ°Π³Π½ΠΈΡ‚Π½ΠΎΠ΅ ΠΏΠΎΠ»Π΅ Π½Π΅ дСйствуСт Π½Π° покоящиСся элСктричСскиС заряды. Линиями Ρ‚ΠΎΠΊΠ° смСщСния поля ΠΌΠΎΠΆΠ½ΠΎ наглядно ΠΏΡ€Π΅Π΄ΡΡ‚Π°Π²ΠΈΡ‚ΡŒ элСктродинамику ΠΏΠΎΠ»Π΅Π²Ρ‹Ρ… процСссов, Ρ‚Π°ΠΊ ΠΊΠ°ΠΊ Π»ΡŽΠ±Ρ‹Π΅ измСнСния поля всСгда связаны с Ρ‚ΠΎΠΊΠ°ΠΌΠΈ смСщСния. Π’ ΠΊΠ½ΠΈΠ³Π°Ρ… ΠΏΠΎ элСктродинамикС хотя ΠΈ говорится, Ρ‡Ρ‚ΠΎ ΠΏΡ€ΠΈ Π΄Π²ΠΈΠΆΠ΅Π½ΠΈΠΈ зарядов Π² ΠΎΠΊΡ€ΡƒΠΆΠ°ΡŽΡ‰Π΅ΠΌ пространствС Ρ‚Π΅ΠΊΡƒΡ‚ Ρ‚ΠΎΠΊΠΈ смСщСния, Π½ΠΎ, ΠΊ соТалСнию, Π½ΠΈ ΠΎΠ΄Π½ΠΎΠ³ΠΎ рисунка, наглядно ΠΈΠ·ΠΎΠ±Ρ€Π°ΠΆΠ°ΡŽΡ‰Π΅Π³ΠΎ этот процСсс, Ρ‚Π°ΠΊ ΠΈ Π½Π΅ ΡƒΠ΄Π°Π»ΠΎΡΡŒ Π½Π°ΠΉΡ‚ΠΈ.
Β 
Рассмотрим Ρ‚ΠΎΠΊΠΈ смСщСния, Π²ΠΎΠ·Π½ΠΈΠΊΠ°ΡŽΡ‰ΠΈΠ΅ ΠΏΡ€ΠΈ Π΄Π²ΠΈΠΆΠ΅Π½ΠΈΠΈ элСктричСских зарядов.

На рисункС Π·Π½Π°ΠΊΠΎΠΌ (+) ΠΎΠ±ΠΎΠ·Π½Π°Ρ‡Π΅Π½Π° ΠΎΠ±Π»Π°ΡΡ‚ΡŒ, ΠΊΡƒΠ΄Π° пСрСмСстился ΠΏΠΎΠ»ΠΎΠΆΠΈΡ‚Π΅Π»ΡŒΠ½Ρ‹ΠΉ заряд ΠΈ Π³Π΄Π΅ Π²ΠΎΠ·Π½ΠΈΠΊΠ°Π΅Ρ‚ Π²ΠΎΠ·ΠΌΡƒΡ‰Π΅Π½ΠΈΠ΅ (элСктричСскоС смСщСниС поля), Ρ‚.Π΅. распространяСтся ΠΏΠΎΠ»ΠΎΠΆΠΈΡ‚Π΅Π»ΡŒΠ½ΠΎΠ΅ элСктричСскоС Π²ΠΎΠ·ΠΌΡƒΡ‰Π΅Π½ΠΈΠ΅ поля. Π—Π½Π°ΠΊΠΎΠΌ (-) ΠΎΠ±ΠΎΠ·Π½Π°Ρ‡Π΅Π½Π° ΠΎΠ±Π»Π°ΡΡ‚ΡŒ, Π³Π΄Π΅ Ρ€Π°Π½ΡŒΡˆΠ΅ Π±Ρ‹Π» заряд ΠΈ Π³Π΄Π΅ исчСзаСт Π²ΠΎΠ·ΠΌΡƒΡ‰Π΅Π½ΠΈΠ΅, Ρ‚.Π΅. распространяСтся ΠΎΡ‚Ρ€ΠΈΡ†Π°Ρ‚Π΅Π»ΡŒΠ½ΠΎΠ΅ Π²ΠΎΠ·ΠΌΡƒΡ‰Π΅Π½ΠΈΠ΅. Линиями ΠΈΠ·ΠΎΠ±Ρ€Π°ΠΆΠ΅Π½ ΠΎΠ±Ρ€Π°Ρ‚Π½Ρ‹ΠΉ Ρ‚ΠΎΠΊ смСщСния поля, стрСлками – Π½Π°ΠΏΡ€Π°Π²Π»Π΅Π½ΠΈΠ΅ Ρ‚ΠΎΠΊΠ°. Надо Π·Π°ΠΌΠ΅Ρ‚ΠΈΡ‚ΡŒ, Ρ‡Ρ‚ΠΎ Ρ‚ΠΎΠΊ смСщСния “стСкаСт” Π² (-)-ΠΎΠ±Π»Π°ΡΡ‚ΡŒ, хотя Π²ΠΎΠ·ΠΌΡƒΡ‰Π΅Π½ΠΈΠ΅ распространяСтся ΠΈΠ· (-)-области (аналогия с Ρ‚ΠΎΠΊΠΎΠΌ проводимости, Π³Π΄Π΅ ΠΎΡ‚Ρ€ΠΈΡ†Π°Ρ‚Π΅Π»ΡŒΠ½ΠΎ заряТСнныС элСктроны двиТутся Π² ΠΎΠ΄Π½Ρƒ сторону, Π½ΠΎ принято ΡΡ‡ΠΈΡ‚Π°Ρ‚ΡŒ, Ρ‡Ρ‚ΠΎ Ρ‚ΠΎΠΊ Ρ‚Π΅Ρ‡Π΅Ρ‚ Π² ΠΎΠ±Ρ€Π°Ρ‚Π½ΠΎΠΌ Π½Π°ΠΏΡ€Π°Π²Π»Π΅Π½ΠΈΠΈ). РаспространСниС возмущСния ΠΈΠ· (+)-области совпадаСт с Π½Π°ΠΏΡ€Π°Π²Π»Π΅Π½ΠΈΠ΅ΠΌ Ρ‚ΠΎΠΊΠ° смСщСния. Когда двиТутся нСсколько зарядов, Π½Π°Π΄ΠΎ ΠΎΡ‚Π΄Π΅Π»ΡŒΠ½ΠΎ Ρ€Π°ΡΡΠΌΠ°Ρ‚Ρ€ΠΈΠ²Π°Ρ‚ΡŒ ΠΊΠ°ΠΆΠ΄Ρ‹ΠΉ двиТущийся заряд, Π° ΠΏΠΎΡ‚ΠΎΠΌ ΡΡƒΠΌΠΌΠΈΡ€ΠΎΠ²Π°Ρ‚ΡŒ всС Ρ‚ΠΎΠΊΠΈ смСщСния, ΠΊΠΎΡ‚ΠΎΡ€Ρ‹Π΅ ΠΈΡ… ΡΠΎΠΏΡ€ΠΎΠ²ΠΎΠΆΠ΄Π°ΡŽΡ‚, Π½Π° основС ΠΏΡ€ΠΈΠ½Ρ†ΠΈΠΏΠ° супСрпозиции. ΠŸΡ€ΠΈ Π΄Π²ΠΈΠΆΠ΅Π½ΠΈΠΈ Ρ†Π΅ΠΏΠΎΡ‡ΠΊΠΈ зарядов ΠΏΠΎΠΏΠ΅Ρ€Π΅Ρ‡Π½Ρ‹Π΅ Ρ‚ΠΎΠΊΠΈ смСщСния, ΠΈΠΌΠ΅ΡŽΡ‰ΠΈΠ΅ встрСчноС Π½Π°ΠΏΡ€Π°Π²Π»Π΅Π½ΠΈΠ΅, Π²Π·Π°ΠΈΠΌΠΎΠ½Π΅ΠΉΡ‚Ρ€Π°Π»ΠΈΠ·ΡƒΡŽΡ‚ΡΡ ΠΈ образуСтся постоянный ΠΎΠ±Ρ€Π°Ρ‚Π½Ρ‹ΠΉ Ρ‚ΠΎΠΊ смСщСния.

Π’ΠΎΠΊ проводимости прСдставляСт Π΄Π²ΠΈΠΆΠ΅Π½ΠΈΠ΅ зарядов, поэтому Π² ΠΎΠΊΡ€ΡƒΠΆΠ°ΡŽΡ‰Π΅ΠΌ пространствС Π²ΠΎΠ·Π½ΠΈΠΊΠ°Π΅Ρ‚ ΠΎΠ±Ρ€Π°Ρ‚Π½Ρ‹ΠΉ Ρ‚ΠΎΠΊ смСщСния. Когда Ρ‚ΠΎΠΊ Ρ‚Π΅Ρ‡Π΅Ρ‚ ΠΏΠΎ Π²ΠΈΡ‚ΠΊΡƒ, Ρ‚ΠΎ Π² ΠΎΠΊΡ€ΡƒΠΆΠ°ΡŽΡ‰Π΅ΠΌ пространствС Π²ΠΎΠ·Π½ΠΈΠΊΠ°Π΅Ρ‚ ΠΊΡ€ΡƒΠ³ΠΎΠ²ΠΎΠΉ Ρ‚ΠΎΠΊ смСщСния, ΠΈΠΌΠ΅ΡŽΡ‰ΠΈΠΉ ΠΎΠ±Ρ€Π°Ρ‚Π½ΠΎΠ΅ Π½Π°ΠΏΡ€Π°Π²Π»Π΅Π½ΠΈΠ΅. ΠŸΡ€ΠΈ ΠΈΠ·ΠΌΠ΅Π½Π΅Π½ΠΈΠΈ Ρ‚ΠΎΠΊΠ° смСщСния образуСтся Π²ΠΈΡ…Ρ€Π΅Π²ΠΎΠ΅ элСктричСскоС ΠΏΠΎΠ»Π΅. Если рядом с Π²ΠΈΡ‚ΠΊΠΎΠΌ Ρ‚ΠΎΠΊΠ° Ρ€Π°ΡΠΏΠΎΠ»ΠΎΠΆΠΈΡ‚ΡŒ, Π½Π°ΠΏΡ€ΠΈΠΌΠ΅Ρ€, свСрхпроводящий ΠΊΠΎΠ½Ρ‚ΡƒΡ€, Ρ‚ΠΎ Π² Π½Π΅ΠΌ Π·Π° счСт ΠΎΠ±Ρ€Π°Ρ‚Π½ΠΎΠ³ΠΎ Ρ‚ΠΎΠΊΠ° элСктричСского смСщСния синхронно, Π½ΠΎ Π² ΠΎΠ±Ρ€Π°Ρ‚Π½ΠΎΠΌ Π½Π°ΠΏΡ€Π°Π²Π»Π΅Π½ΠΈΠΈ Π²ΠΎΠ·Π½ΠΈΠΊΠ°Π΅Ρ‚ ΠΈΠ½Π΄ΡƒΠΊΡ†ΠΈΠΎΠ½Π½Ρ‹ΠΉ Ρ‚ΠΎΠΊ. Π’ΠΎΠΊ смСщСния ΠΊΠ°ΠΊ Π±Ρ‹ замыкаСтся Ρ‡Π΅Ρ€Π΅Π· свСрхпроводящий ΠΏΡ€ΠΎΠ²ΠΎΠ΄Π½ΠΈΠΊ.
Β 
Β«… Π²ΠΈΡ…Ρ€Π΅Π²ΠΎΠ΅ ΠΏΠΎΠ»Π΅ Π±Π΅Π· ΠΊΠ°ΠΊΠΈΡ… Π±Ρ‹ Ρ‚ΠΎ Π½ΠΈ Π±Ρ‹Π»ΠΎ Π΄ΠΎΠ±Π°Π²ΠΎΡ‡Π½Ρ‹Ρ… сил ΠΌΠΎΠΆΠ΅Ρ‚ Π²Ρ‹Π·Π²Π°Ρ‚ΡŒ Π½Π΅ΠΏΡ€Π΅Ρ€Ρ‹Π²Π½ΠΎΠ΅ Ρ‚Π΅Ρ‡Π΅Π½ΠΈΠ΅ элСктричСства ΠΏΠΎ Π·Π°ΠΌΠΊΠ½ΡƒΡ‚Ρ‹ΠΌ ΠΏΡ€ΠΎΠ²ΠΎΠ΄Π°ΠΌ. Π­Ρ‚ΠΎ Ρ‚Π΅Ρ‡Π΅Π½ΠΈΠ΅ ΠΈ Π½Π°Π±Π»ΡŽΠ΄Π°Π΅Ρ‚ΡΡ Π² Π²ΠΈΠ΄Π΅ ΠΈΠ½Π΄ΡƒΠΊΡ†ΠΈΠΎΠ½Π½Ρ‹Ρ… Ρ‚ΠΎΠΊΠΎΠ².Β»
ΠžΠ±Ρ‰ΠΈΠΉ курс Ρ„ΠΈΠ·ΠΈΠΊΠΈ. ЭлСктричСство. Π”.Π’.Π‘ΠΈΠ²ΡƒΡ…ΠΈΠ½. 1996. Π’.3. Π§.1. Π‘.252.
Β 
Π’Π°ΠΊΠΆΠ΅ самоиндукция связана с ΠΎΠ±Ρ€Π°Ρ‚Π½Ρ‹ΠΌΠΈ Ρ‚ΠΎΠΊΠ°ΠΌΠΈ смСщСния, с Π·Π°ΠΏΠ°Π·Π΄Ρ‹Π²Π°Π½ΠΈΠ΅ΠΌ распространСния Π²ΠΎΠ·ΠΌΡƒΡ‰Π΅Π½ΠΈΠΉ. ΠŸΡ€ΠΈ остановкС зарядов ΠΎΠ±Ρ€Π°Ρ‚Π½Ρ‹Π΅ Ρ‚ΠΎΠΊΠΈ смСщСния, Π΅Ρ‰Π΅ Π½Π΅ΠΊΠΎΡ‚ΠΎΡ€ΠΎΠ΅ врСмя продолТая Ρ‚Π΅Ρ‡ΡŒ (ΠΊΠ°ΠΊ возмущСния поля), Π²ΠΎΠ·Π΄Π΅ΠΉΡΡ‚Π²ΡƒΡŽΡ‚ Π½Π° заряды.
Β 
«МаксвСлл приписал Ρ‚ΠΎΠΊΡƒ смСщСния лишь ΠΎΠ΄Π½ΠΎ – ΡΠΏΠΎΡΠΎΠ±Π½ΠΎΡΡ‚ΡŒ ΡΠΎΠ·Π΄Π°Π²Π°Ρ‚ΡŒ Π² ΠΎΠΊΡ€ΡƒΠΆΠ°ΡŽΡ‰Π΅ΠΌ пространствС ΠΌΠ°Π³Π½ΠΈΡ‚Π½ΠΎΠ΅ ΠΏΠΎΠ»Π΅. Β»
ΠšΡƒΡ€Ρ Ρ„ΠΈΠ·ΠΈΠΊΠΈ. Π’.И.Π’Ρ€ΠΎΡ„ΠΈΠΌΠΎΠ²Π°. 1998. Π‘.250.
Β 
Π’ΠΎΡ‡Π½Π΅Π΅, Ρ‚ΠΎΠΊ смСщСния сам прСдставляСт ΠΌΠ°Π³Π½ΠΈΡ‚Π½ΠΎΠ΅ ΠΏΠΎΠ»Π΅. ΠœΠ°Π³Π½ΠΈΡ‚Π½Π°Ρ сила притяТСния Π²ΠΎΠ·Π½ΠΈΠΊΠ°Π΅Ρ‚ ΠΌΠ΅ΠΆΠ΄Ρƒ двумя ΠΏΡ€ΠΎΠ²ΠΎΠ΄Π½ΠΈΠΊΠ°ΠΌΠΈ с Ρ‚ΠΎΠΊΠΎΠΌ, ΠΊΠΎΠ³Π΄Π° ΠΎΠ±Ρ€Π°Ρ‚Π½Ρ‹Π΅ Ρ‚ΠΎΠΊΠΈ смСщСния ΠΏΠΎΠ»Π΅Π²ΠΎΠΉ срСды Ρ‚Π΅ΠΊΡƒΡ‚ Π² ΠΎΠ΄Π½ΠΎΠΌ Π½Π°ΠΏΡ€Π°Π²Π»Π΅Π½ΠΈΠΈ, – Ρ‚ΠΎΠΊΠΈ стрСмятся ΡΠ»ΠΈΡ‚ΡŒΡΡ. ΠžΡ‚Ρ‚Π°Π»ΠΊΠΈΠ²Π°Π½ΠΈΠ΅ ΠΆΠ΅ Π²ΠΎΠ·Π½ΠΈΠΊΠ°Π΅Ρ‚, ΠΊΠΎΠ³Π΄Π° ΠΎΠ±Ρ€Π°Ρ‚Π½Ρ‹Π΅ Ρ‚ΠΎΠΊΠΈ смСщСния ΠΏΠΎΠ»Π΅Π²ΠΎΠΉ срСды Ρ‚Π΅ΠΊΡƒΡ‚ навстрСчу Π΄Ρ€ΡƒΠ³ Π΄Ρ€ΡƒΠ³Ρƒ, – Ρ‚ΠΎΠΊΠΈ стрСмятся Ρ€Π°Π·ΠΎΠΉΡ‚ΠΈΡΡŒ. Π’Π°ΠΊΠΈΠΌ ΠΎΠ±Ρ€Π°Π·ΠΎΠΌ, магнитная сила – это обычная гидродинамичСская сила, Π²ΠΎΠ·Π½ΠΈΠΊΠ°ΡŽΡ‰Π°Ρ ΠΌΠ΅ΠΆΠ΄Ρƒ Ρ‚Π΅ΠΊΡƒΡ‰ΠΈΠΌΠΈ ΠΏΠΎΡ‚ΠΎΠΊΠ°ΠΌΠΈ срСды. НапримСр, Ссли Π΄Π²Π° диска Π²Ρ€Π°Ρ‰Π°ΡŽΡ‚ΡΡ Π² ΠΎΠ΄Π½ΠΎΠΌ Π½Π°ΠΏΡ€Π°Π²Π»Π΅Π½ΠΈΠΈ, Ρ‚ΠΎ ΠΌΠ΅ΠΆΠ΄Ρƒ Π½ΠΈΠΌΠΈ Π²ΠΎΠ·Π½ΠΈΠΊΠ°Π΅Ρ‚ сила притяТСния. Если ΠΆΠ΅ ΠΎΠ½ΠΈ Π²Ρ€Π°Ρ‰Π°ΡŽΡ‚ΡΡ Π² ΠΏΡ€ΠΎΡ‚ΠΈΠ²ΠΎΠΏΠΎΠ»ΠΎΠΆΠ½Ρ‹Ρ… направлСниях, Ρ‚ΠΎ, Π½Π°ΠΎΠ±ΠΎΡ€ΠΎΡ‚, Π²ΠΎΠ·Π½ΠΈΠΊΠ°Π΅Ρ‚ сила отталкивания. Π­Ρ‚ΠΎ происходит ΠΏΠΎΡ‚ΠΎΠΌΡƒ, Ρ‡Ρ‚ΠΎ своим Π²Ρ€Π°Ρ‰Π΅Π½ΠΈΠ΅ΠΌ диски ΡƒΠ²Π»Π΅ΠΊΠ°ΡŽΡ‚ Π²ΠΎΠ·Π΄ΡƒΡ…, ΠΏΠΎΡ‚ΠΎΠΊΠΈ ΠΊΠΎΡ‚ΠΎΡ€ΠΎΠ³ΠΎ ΡΠΎΠ·Π΄Π°ΡŽΡ‚ Π³ΠΈΠ΄Ρ€ΠΎΠ΄ΠΈΠ½Π°ΠΌΠΈΡ‡Π΅ΡΠΊΡƒΡŽ силу притяТСния ΠΈΠ»ΠΈ отталкивания. На покоящийся заряд ΠΌΠ°Π³Π½ΠΈΡ‚Π½ΠΎΠ΅ ΠΏΠΎΠ»Π΅ Π½Π΅ дСйствуСт, Ρ‚Π°ΠΊ ΠΊΠ°ΠΊ Π²ΠΎΠΊΡ€ΡƒΠ³ Π½Π΅Π³ΠΎ Π½Π΅ Ρ‚Π΅ΠΊΡƒΡ‚ Ρ‚ΠΎΠΊΠΈ элСктричСского смСщСния поля. На ΠΏΡ€Π°ΠΊΡ‚ΠΈΠΊΠ΅ ΠΆΠ΅ Π½Π΅ Ρ€Π°ΡΡΠΌΠ°Ρ‚Ρ€ΠΈΠ²Π°ΡŽΡ‚ Ρ€Π°ΡΠΏΡ€ΠΎΡΡ‚Ρ€Π°Π½ΡΡŽΡ‰ΠΈΠ΅ΡΡ смСщСния поля, ΠΏΡ€Π΅Π΄ΡΡ‚Π°Π²Π»ΡΡŽΡ‰ΠΈΠ΅ двиТущиСся элСктричСскиС ΠΏΠΎΡ‚ΠΎΠΊΠΈ, Π° ΠΏΠΎΠ»ΡŒΠ·ΡƒΡŽΡ‚ΡΡ линиями ΠΌΠ°Π³Π½ΠΈΡ‚Π½ΠΎΠΉ ΠΈΠ½Π΄ΡƒΠΊΡ†ΠΈΠΈ, рассматривая взаимодСйствиС с Π½ΠΈΠΌΠΈ элСктричСских Ρ‚ΠΎΠΊΠΎΠ². Π›ΠΈΠ½ΠΈΠΈ ΠΌΠ°Π³Π½ΠΈΡ‚Π½ΠΎΠΉ ΠΈΠ½Π΄ΡƒΠΊΡ†ΠΈΠΈ Π½Π΅ ΡΠ²Π»ΡΡŽΡ‚ΡΡ силовыми линиями (линиями дСйствия силы), Π½Π°ΠΏΡ€ΠΈΠΌΠ΅Ρ€, Π½Π°ΠΏΡ€Π°Π²Π»Π΅Π½ΠΈΠ΅ Π²Π΅ΠΊΡ‚ΠΎΡ€Π° ΠΌΠ°Π³Π½ΠΈΡ‚Π½ΠΎΠΉ силы, Π²ΠΎΠ·Π½ΠΈΠΊΠ°ΡŽΡ‰Π΅ΠΉ ΠΌΠ΅ΠΆΠ΄Ρƒ ΠΏΠ°Ρ€Π°Π»Π»Π΅Π»ΡŒΠ½Ρ‹ΠΌΠΈ ΠΏΡ€ΠΎΠ²ΠΎΠ΄Π½ΠΈΠΊΠ°ΠΌΠΈ с постоянным Ρ‚ΠΎΠΊΠΎΠΌ, Π½Π΅ совпадаСт с Π½Π°ΠΏΡ€Π°Π²Π»Π΅Π½ΠΈΠ΅ΠΌ Π»ΠΈΠ½ΠΈΠΉ ΠΌΠ°Π³Π½ΠΈΡ‚Π½ΠΎΠΉ ΠΈΠ½Π΄ΡƒΠΊΡ†ΠΈΠΈ. Π’.Π΅. ΠΈΠ½Π΄ΡƒΠΊΡ†ΠΈΠΎΠ½Π½Ρ‹Π΅ Π»ΠΈΠ½ΠΈΠΈ ΠΌΠ°Π³Π½ΠΈΡ‚Π½Ρ‹Ρ… ΠΏΠΎΠ»Π΅ΠΉ силовыми Π½Π΅ ΡΠ²Π»ΡΡŽΡ‚ΡΡ, Ρ‚Π°ΠΊ ΠΊΠ°ΠΊ Π½Π΅ ΠΏΠΎΠΊΠ°Π·Ρ‹Π²Π°ΡŽΡ‚ Π½Π°ΠΏΡ€Π°Π²Π»Π΅Π½ΠΈΠ΅ силы, Π΄Π΅ΠΉΡΡ‚Π²ΡƒΡŽΡ‰Π΅ΠΉ Π½Π° элСмСнтарный Ρ‚ΠΎΠΊ. Π’Π°ΠΊΠΆΠ΅ Π² Π΄Π°Π½Π½ΠΎΠΌ ΠΏΡ€ΠΈΠΌΠ΅Ρ€Π΅ Π²ΠΈΠ΄Π½ΠΎ, Ρ‡Ρ‚ΠΎ ΠΌΠ°Π³Π½ΠΈΡ‚Π½ΠΎΠ΅ ΠΏΠΎΠ»Π΅ Π½Π΅ являСтся Π²ΠΈΡ…Ρ€Π΅Π²Ρ‹ΠΌ, Ρ‚Π°ΠΊ ΠΊΠ°ΠΊ Ρƒ Π²ΠΈΡ…Ρ€Π΅Π²Ρ‹Ρ… ΠΏΠΎΠ»Π΅ΠΉ Ρ€Π°Π±ΠΎΡ‚Π° сил ΠΏΡ€ΠΈ Π΄Π²ΠΈΠΆΠ΅Π½ΠΈΠΈ ΠΏΠΎ Π·Π°ΠΌΠΊΠ½ΡƒΡ‚ΠΎΠΉ Π»ΠΈΠ½ΠΈΠΈ ΠΌΠΎΠΆΠ΅Ρ‚ Π±Ρ‹Ρ‚ΡŒ ΠΎΡ‚Π»ΠΈΡ‡Π½Π° ΠΎΡ‚ нуля, Ρ‡Ρ‚ΠΎ являСтся ΠΏΡ€ΠΈΠ·Π½Π°ΠΊΠΎΠΌ Π²ΠΈΡ…Ρ€Π΅Π²ΠΎΠ³ΠΎ поля. Π’ΠΈΡ…Ρ€Π΅Π²Ρ‹Π΅ поля ΠΌΠΎΠ³ΡƒΡ‚ Π²ΠΎΠ·Π±ΡƒΠΆΠ΄Π°Ρ‚ΡŒ Π²ΠΈΡ…Ρ€Π΅Π²Ρ‹Π΅ элСктричСскиС Ρ‚ΠΎΠΊΠΈ. Π’Π°ΠΊΠΈΠΌ ΠΎΠ±Ρ€Π°Π·ΠΎΠΌ, постоянноС ΠΌΠ°Π³Π½ΠΈΡ‚Π½ΠΎΠ΅ ΠΏΠΎΠ»Π΅ являСтся ΡΠΎΠ»Π΅Π½ΠΎΠΈΠ΄Π°Π»ΡŒΠ½Ρ‹ΠΌ, Π½ΠΎ Π½Π΅ Π²ΠΈΡ…Ρ€Π΅Π²Ρ‹ΠΌ.
Β 
Β«ΠŸΡ€ΡΠΌΠ°Ρ, вдоль ΠΊΠΎΡ‚ΠΎΡ€ΠΎΠΉ Π½Π°ΠΏΡ€Π°Π²Π»Π΅Π½Π° сила, называСтся Π»ΠΈΠ½ΠΈΠ΅ΠΉ дСйствия силы. Β»
ЀизичСская энциклопСдия. Π‘Π˜Π›Π.
Β«Π Π°Π±ΠΎΡ‚Π° сил Π²ΠΈΡ…Ρ€Π΅Π²ΠΎΠ³ΠΎ элСктричСского поля ΠΏΡ€ΠΈ Π΄Π²ΠΈΠΆΠ΅Π½ΠΈΠΈ элСктричСского заряда ΠΏΠΎ Π·Π°ΠΌΠΊΠ½ΡƒΡ‚ΠΎΠΉ Π»ΠΈΠ½ΠΈΠΈ ΠΌΠΎΠΆΠ΅Ρ‚ Π±Ρ‹Ρ‚ΡŒ ΠΎΡ‚Π»ΠΈΡ‡Π½Π° ΠΎΡ‚ нуля.Β»
Π€ΠΈΠ·ΠΈΠΊΠ°. О.Π€.ΠšΠ°Π±Π°Ρ€Π΄ΠΈΠ½. 1991. Π‘.189.
Β 
Π Π°Π±ΠΎΡ‚Π° сил Π²ΠΈΡ…Ρ€Π΅Π²ΠΎΠ³ΠΎ элСктричСского поля ΠΈΠ»ΠΈ Π²ΠΈΡ…Ρ€Π΅Π²ΠΎΠ³ΠΎ ΠΌΠ°Π³Π½ΠΈΡ‚Π½ΠΎΠ³ΠΎ поля ΠΏΡ€ΠΈ Π΄Π²ΠΈΠΆΠ΅Π½ΠΈΠΈ элСктричСского заряда ΠΈΠ»ΠΈ ΠΌΠ°Π³Π½ΠΈΡ‚Π° ΠΏΠΎ Π·Π°ΠΌΠΊΠ½ΡƒΡ‚ΠΎΠΉ Π»ΠΈΠ½ΠΈΠΈ ΠΌΠΎΠΆΠ΅Ρ‚ Π±Ρ‹Ρ‚ΡŒ ΠΎΡ‚Π»ΠΈΡ‡Π½Π° ΠΎΡ‚ нуля. НапримСр, Π² элСктромагнитных Π²ΠΎΠ»Π½Π°Ρ… элСктричСскиС ΠΈ ΠΌΠ°Π³Π½ΠΈΡ‚Π½Ρ‹Π΅ ΠΏΠΎΡ‚ΠΎΠΊΠΈ ΡΠ²Π»ΡΡŽΡ‚ΡΡ Π²ΠΈΡ…Ρ€Π΅Π²Ρ‹ΠΌΠΈ.
Β 
Β«… ΠΌΠ°Π³Π½ΠΈΡ‚Π½ΠΎΠ΅ ΠΆΠ΅ ΠΏΠΎΠ»Π΅ – солСноидальноС.Β»
Π­Π»Π΅ΠΊΡ‚Ρ€ΠΎΠΌΠ°Π³Π½Π΅Ρ‚ΠΈΠ·ΠΌ. И.Π•.Π˜Ρ€ΠΎΠ΄ΠΎΠ². 2000. Π‘.170.
Β«… ΡƒΡΠΊΠΎΡ€ΠΈΡ‚Π΅Π»ΡŒ, ΠΈΡΠΏΠΎΠ»ΡŒΠ·ΡƒΡŽΡ‰ΠΈΠΉ Π²ΠΈΡ…Ρ€Π΅Π²ΠΎΠ΅ ΠΌΠ°Π³Π½ΠΈΡ‚Π½ΠΎΠ΅ ΠΏΠΎΠ»Π΅.Β»
ЀизичСская энциклопСдия. Π‘Π•Π’ΠΠ’Π ΠžΠ.
Β 
ΠœΠ°Π³Π½ΠΈΡ‚Π½ΠΎΠ΅ ΠΏΠΎΠ»Π΅, хотя солСноидально, Π½ΠΎ Π½Π΅ всСгда являСтся Π²ΠΈΡ…Ρ€Π΅Π²Ρ‹ΠΌ. Надо Π·Π°ΠΌΠ΅Ρ‚ΠΈΡ‚ΡŒ, Ρ‡Ρ‚ΠΎ Π½Π΅ΠΊΠΎΡ‚ΠΎΡ€Ρ‹Π΅ Π°Π²Ρ‚ΠΎΡ€Ρ‹ ΠΊΠ½ΠΈΠ³ ΠΏΠΎ элСктродинамикС ΠΏΡƒΡ‚Π°ΡŽΡ‚ ΡΠΎΠ»Π΅Π½ΠΎΠΈΠ΄Π°Π»ΡŒΠ½Ρ‹Π΅ поля с Π²ΠΈΡ…Ρ€Π΅Π²Ρ‹ΠΌΠΈ, ΠΈΠ½Π΄ΡƒΠΊΡ†ΠΈΠΎΠ½Π½Ρ‹Π΅ Π»ΠΈΠ½ΠΈΠΈ с силовыми. Π£ элСктричСского поля, Π΄Π΅ΠΉΡΡ‚Π²ΠΈΡ‚Π΅Π»ΡŒΠ½ΠΎ, ΠΈΠ½Π΄ΡƒΠΊΡ†ΠΈΠΎΠ½Π½Ρ‹Π΅ Π»ΠΈΠ½ΠΈΠΈ ΡΠΎΠ²ΠΏΠ°Π΄Π°ΡŽΡ‚ с силовыми, Π½ΠΎ это Π½ΠΈΠΊΠ°ΠΊ Π½Π΅ относится ΠΊ ΠΌΠ°Π³Π½ΠΈΡ‚Π½ΠΎΠΌΡƒ полю, Π³Π΄Π΅ ΠΈΠ½Π΄ΡƒΠΊΡ†ΠΈΠΎΠ½Π½Ρ‹Π΅ Π»ΠΈΠ½ΠΈΠΈ Π½Π΅ всСгда ΡΠΎΠ²ΠΏΠ°Π΄Π°ΡŽΡ‚ с силовыми линиями дСйствия поля.
Β 
Π’Π°ΠΊΠΆΠ΅ ΠΏΠΎ линиям ΠΌΠ°Π³Π½ΠΈΡ‚Π½ΠΎΠΉ ΠΈΠ½Π΄ΡƒΠΊΡ†ΠΈΠΈ, Π½Π°ΠΏΡ€ΠΈΠΌΠ΅Ρ€, Π½Π΅Π²ΠΎΠ·ΠΌΠΎΠΆΠ½ΠΎ ΠΎΠΏΡ€Π΅Π΄Π΅Π»ΠΈΡ‚ΡŒ Π½Π°ΠΏΡ€Π°Π²Π»Π΅Π½ΠΈΠ΅ силы, Π΄Π΅ΠΉΡΡ‚Π²ΡƒΡŽΡ‰Π΅ΠΉ Π½Π° покоящийся элСктричСский заряд Π² ΠΌΠΎΠΌΠ΅Π½Ρ‚ Π²ΠΊΠ»ΡŽΡ‡Π΅Π½ΠΈΡ элСктромагнита Π² случаС, ΠΊΠΎΠ³Π΄Π° ΠΌΠ°Π³Π½ΠΈΡ‚ ΠΈ заряд находятся Π² ΠΏΠΎΠΊΠΎΠ΅, Ρ‚.Π΅. ΠΏΠΎ линиям ΠΌΠ°Π³Π½ΠΈΡ‚Π½ΠΎΠΉ ΠΈΠ½Π΄ΡƒΠΊΡ†ΠΈΠΈ Π½Π΅Π²ΠΎΠ·ΠΌΠΎΠΆΠ½ΠΎ ΠΎΠΏΡ€Π΅Π΄Π΅Π»ΠΈΡ‚ΡŒ Π½Π°ΠΏΡ€Π°Π²Π»Π΅Π½ΠΈΠ΅ силы, Π΄Π΅ΠΉΡΡ‚Π²ΡƒΡŽΡ‰Π΅ΠΉ Π½Π° покоящийся заряд Π² ΠΏΠ΅Ρ€Π΅ΠΌΠ΅Π½Π½ΠΎΠΌ ΠΌΠ°Π³Π½ΠΈΡ‚Π½ΠΎΠΌ ΠΏΠΎΠ»Π΅. ΠŸΡ€Π΅Π΄ΡΡ‚Π°Π²Π»ΡΡ ΠΌΠ°Π³Π½ΠΈΡ‚Π½ΠΎΠ΅ ΠΏΠΎΠ»Π΅ линиями Ρ‚ΠΎΠΊΠΎΠ² смСщСния, Ρ‚Π°ΠΊΠΈΡ… ΠΏΡ€ΠΎΠ±Π»Π΅ΠΌ Π½Π΅ Π²ΠΎΠ·Π½ΠΈΠΊΠ°Π΅Ρ‚. По силС, Π΄Π΅ΠΉΡΡ‚Π²ΡƒΡŽΡ‰Π΅ΠΉ Π½Π° покоящийся элСктричСский заряд Π² ΠΌΠΎΠΌΠ΅Π½Ρ‚ Π²ΠΊΠ»ΡŽΡ‡Π΅Π½ΠΈΡ элСктромагнита, ΠΌΠΎΠΆΠ½ΠΎ ΠΎΠΏΡ€Π΅Π΄Π΅Π»ΠΈΡ‚ΡŒ Π½Π°ΠΏΡ€Π°Π²Π»Π΅Π½ΠΈΠ΅ Ρ‚ΠΎΠΊΠ° смСщСния Π² ΠΊΠΎΠ½ΠΊΡ€Π΅Ρ‚Π½ΠΎΠΉ Ρ‚ΠΎΡ‡ΠΊΠ΅ ΠΌΠ°Π³Π½ΠΈΡ‚Π½ΠΎΠ³ΠΎ поля. ИзмСнСниС любого элСктричСского Ρ‚ΠΎΠΊΠ° всСгда связано с элСктричСской Π½Π°ΠΏΡ€ΡΠΆΠ΅Π½Π½ΠΎΡΡ‚ΡŒΡŽ.
Β 
Β«ΠœΠ°Π³Π½ΠΈΡ‚Π½ΠΎΠ΅ ΠΏΠΎΠ»Π΅, нСпостоянноС Π²ΠΎ Π²Ρ€Π΅ΠΌΠ΅Π½ΠΈ, ΠΎΠΊΠ°Π·Ρ‹Π²Π°Π΅Ρ‚ силовоС дСйствиС Π½Π° покоящиСся элСктричСскиС заряды ΠΈ ΠΏΡ€ΠΈΠ²ΠΎΠ΄ΠΈΡ‚ ΠΈΡ… Π² Π΄Π²ΠΈΠΆΠ΅Π½ΠΈΠ΅;Β …Β»
ЀизичСский энциклопСдичСский ΡΠ»ΠΎΠ²Π°Ρ€ΡŒ. ΠœΠΠ“ΠΠ˜Π’ΠΠžΠ• ΠŸΠžΠ›Π•.
Β 
Π”Π°Π½Π½ΠΎΠ΅ ΠΏΡ€Π°Π²ΠΈΠ»ΠΎ Π½Π΅ Π΄Π°Π΅Ρ‚ ΠΎΡ‚Π²Π΅Ρ‚Π° Π½Π° Π³Π»Π°Π²Π½Ρ‹ΠΉ вопрос: ΠΊΡƒΠ΄Π° дСйствуСт сила? И ΠΏΠΎ сути ΠΎΠ½ΠΎ являСтся Π½Π΅Π²Π΅Ρ€Π½Ρ‹ΠΌ, Ρ‚Π°ΠΊ ΠΊΠ°ΠΊ Π½Π΅ ΡƒΡ‡ΠΈΡ‚Ρ‹Π²Π°ΡŽΡ‚ΡΡ Ρ‚ΠΎΠΊΠΈ смСщСния (ΠΌΠ°Π³Π½ΠΈΡ‚Π½ΠΎΠ΅ ΠΏΠΎΠ»Π΅ Π²ΠΎΠΎΠ±Ρ‰Π΅ Π½Π΅ дСйствуСт Π½Π° покоящиСся заряды). ΠŸΡ€Π°Π²ΠΈΠ»ΡŒΠ½ΠΎΠΉ ΠΆΠ΅ являСтся такая Ρ„ΠΎΡ€ΠΌΡƒΠ»ΠΈΡ€ΠΎΠ²ΠΊΠ°: ΠΏΠ΅Ρ€Π΅ΠΌΠ΅Π½Π½ΠΎΠ΅ ΠΌΠ°Π³Π½ΠΈΡ‚Π½ΠΎΠ΅ ΠΏΠΎΠ»Π΅ прСдставляСт ΠΏΠ΅Ρ€Π΅ΠΌΠ΅Π½Π½Ρ‹ΠΉ Ρ‚ΠΎΠΊ элСктричСского смСщСния, ΠΊΠΎΡ‚ΠΎΡ€Ρ‹ΠΉ проявляСтся ΠΊΠ°ΠΊ Π²ΠΈΡ…Ρ€Π΅Π²ΠΎΠ΅ элСктричСскоС ΠΏΠΎΠ»Π΅ ΠΈ ΠΎΠΊΠ°Π·Ρ‹Π²Π°Π΅Ρ‚ силовоС дСйствиС Π½Π° покоящиСся элСктричСскиС заряды. НапримСр, Ссли покоящийся заряд находится Π² Ρ†Π΅Π½Ρ‚Ρ€Π΅ солСноида, Ρ‚ΠΎ ΠΏΡ€ΠΈ Π²ΠΊΠ»ΡŽΡ‡Π΅Π½ΠΈΠΈ ΠΈΠ»ΠΈ Π²Ρ‹ΠΊΠ»ΡŽΡ‡Π΅Π½ΠΈΠΈ Ρ‚ΠΎΠΊΠ° Π² солСноидС Π½Π° заряд Π½Π΅ дСйствуСт сила, нСсмотря Π½Π° Ρ‚ΠΎ, Ρ‡Ρ‚ΠΎ измСняСтся ΠΏΠΎΡ‚ΠΎΠΊ ΠΌΠ°Π³Π½ΠΈΡ‚Π½ΠΎΠΉ ΠΈΠ½Π΄ΡƒΠΊΡ†ΠΈΠΈ, Ρ‚Π°ΠΊ ΠΊΠ°ΠΊ Π² Ρ†Π΅Π½Ρ‚Ρ€Π΅ солСноида Ρ‚ΠΎΠΊ смСщСния отсутствуСт ΠΈ, соотвСтствСнно, отсутствуСт Π²ΠΈΡ…Ρ€Π΅Π²ΠΎΠ΅ элСктричСскоС ΠΏΠΎΠ»Π΅. Достаточно Π²Π·Π³Π»ΡΠ½ΡƒΡ‚ΡŒ Π½Π° ΠΏΡ€ΠΈΠΌΠ΅Ρ€Ρ‹ Π² ΡƒΡ‡Π΅Π±Π½ΠΈΠΊΠ°Ρ…, ΠΈΠ· ΠΊΠΎΡ‚ΠΎΡ€Ρ‹Ρ… Π²ΠΈΠ΄Π½ΠΎ, Ρ‡Ρ‚ΠΎ Ρ‚ΠΎΠΊ смСщСния Π² Ρ†Π΅Π½Ρ‚Ρ€Π΅ солСноида отсутствуСт.
Β 
Β«ΠŸΡ€ΠΈΠΌΠ΅Ρ€. Найти ΠΏΠ»ΠΎΡ‚Π½ΠΎΡΡ‚ΡŒ Ρ‚ΠΎΠΊΠ° смСщСния ΠΊΠ°ΠΊ Ρ„ΡƒΠ½ΠΊΡ†ΠΈΡŽ расстояния r ΠΎΡ‚ оси солСноида.Β»
Π­Π»Π΅ΠΊΡ‚Ρ€ΠΎΠΌΠ°Π³Π½Π΅Ρ‚ΠΈΠ·ΠΌ. И.Π•.Π˜Ρ€ΠΎΠ΄ΠΎΠ². 2000. Π‘.303.
Β 
Π’Π°ΠΊΠΈΠΌ ΠΎΠ±Ρ€Π°Π·ΠΎΠΌ, Ρ„ΡƒΠ½Π΄Π°ΠΌΠ΅Π½Ρ‚Π°Π»ΡŒΠ½ΠΎΠ΅ ΠΏΡ€Π°Π²ΠΈΠ»ΠΎ ΠΈΠΌΠ΅Π΅Ρ‚ ΠΈΡΠΊΠ»ΡŽΡ‡Π΅Π½ΠΈΠ΅: Π² Ρ†Π΅Π½Ρ‚Ρ€Π΅ солСноида ΠΏΠ΅Ρ€Π΅ΠΌΠ΅Π½Π½ΠΎΠ΅ ΠΌΠ°Π³Π½ΠΈΡ‚Π½ΠΎΠ΅ ΠΏΠΎΠ»Π΅ Π½Π΅ ΠΎΠΊΠ°Π·Ρ‹Π²Π°Π΅Ρ‚ силового дСйствия Π½Π° покоящиСся элСктричСскиС заряды ΠΈ Π½Π΅ ΠΏΡ€ΠΈΠ²ΠΎΠ΄ΠΈΡ‚ ΠΈΡ… Π² Π΄Π²ΠΈΠΆΠ΅Π½ΠΈΠ΅. Ось солСноида – это “мСртвая” линия ΠΌΠ°Π³Π½ΠΈΡ‚Π½ΠΎΠ³ΠΎ поля, Π²ΠΎΠΊΡ€ΡƒΠ³ ΠΊΠΎΡ‚ΠΎΡ€ΠΎΠΉ Ρ‚Π΅ΠΊΡƒΡ‚ элСктричСскиС Ρ‚ΠΎΠΊΠΈ смСщСния. Вакая “мСртвая” линия имССтся Ρƒ любого ΠΌΠ°Π³Π½ΠΈΡ‚Π°.
Β 
Для ΠΏΡ€ΠΈΠΌΠ΅Ρ€Π° рассмотрим экспСримСнт, Π³Π΄Π΅ элСктромагнитная индукция Π²ΠΎΠ·Π½ΠΈΠΊΠ°Π΅Ρ‚ “Π±Π΅Π· ΠΌΠ°Π³Π½ΠΈΡ‚Π½ΠΎΠ³ΠΎ поля”. Π’ Ρ†Π΅Π½Ρ‚Ρ€Π°Π»ΡŒΠ½ΠΎΠΉ Ρ‚ΠΎΡ‡ΠΊΠ΅ ΠΌΠ΅ΠΆΠ΄Ρƒ двумя элСктромагнитами, Π³Π΄Π΅ ΠΌΠ°Π³Π½ΠΈΡ‚Π½ΠΎΠ΅ ΠΏΠΎΠ»Π΅, согласно ΠΏΡ€ΠΈΠ½Ρ†ΠΈΠΏΡƒ супСрпозиции ΠΏΠΎΠ»Π΅ΠΉ, Ρ€Π°Π²Π½ΠΎ Π½ΡƒΠ»ΡŽ, установлСн ΠΏΡ€ΠΎΠ±Π½Ρ‹ΠΉ элСктричСский заряд.

[N] (+) [S]

[N] ΠΈ [S] – полюса Π΄Π²ΡƒΡ… элСктромагнитов, (+) - ΠΏΡ€ΠΎΠ±Π½Ρ‹ΠΉ ΠΏΠΎΠ»ΠΎΠΆΠΈΡ‚Π΅Π»ΡŒΠ½Ρ‹ΠΉ элСктричСский заряд.
Β 
Если элСктромагниты Π²Ρ‹ΠΊΠ»ΡŽΡ‡Π°Ρ‚ΡŒ ΠΏΠΎ ΠΎΡ‚Π΄Π΅Π»ΡŒΠ½ΠΎΡΡ‚ΠΈ, Ρ‚ΠΎ Π½Π° заряд Π±ΡƒΠ΄Π΅Ρ‚ Π΄Π΅ΠΉΡΡ‚Π²ΠΎΠ²Π°Ρ‚ΡŒ сила, направлСнная Π²Π²Π΅Ρ€Ρ….
Β 
«ЭлСктромагнитная индукция – Π²ΠΎΠ·Π½ΠΈΠΊΠ½ΠΎΠ²Π΅Π½ΠΈΠ΅ элСктричСского поля, элСктричСского Ρ‚ΠΎΠΊΠ° ΠΈΠ»ΠΈ элСктричСской поляризации ΠΏΡ€ΠΈ ΠΈΠ·ΠΌΠ΅Π½Π΅Π½ΠΈΠΈ Π²ΠΎ Π²Ρ€Π΅ΠΌΠ΅Π½ΠΈ ΠΌΠ°Π³Π½ΠΈΡ‚Π½ΠΎΠ³ΠΎ поля ΠΈΠ»ΠΈ ΠΏΡ€ΠΈ Π΄Π²ΠΈΠΆΠ΅Π½ΠΈΠΈ ΠΌΠ°Ρ‚Π΅Ρ€ΠΈΠ°Π»ΡŒΠ½Ρ‹Ρ… срСд Π² ΠΌΠ°Π³Π½ΠΈΡ‚Π½ΠΎΠΌ ΠΏΠΎΠ»Π΅. Β»
ЀизичСская энциклопСдия. Π­Π›Π•ΠšΠ’Π ΠžΠœΠΠ“ΠΠ˜Π’ΠΠΠ― Π˜ΠΠ”Π£ΠšΠ¦Π˜Π―.
Β«… ΠΈΠ·ΠΌΠ΅Π½ΡΡŽΡ‰Π΅Π΅ΡΡ Π²ΠΎ Π²Ρ€Π΅ΠΌΠ΅Π½ΠΈ ΠΌΠ°Π³Π½ΠΈΡ‚Π½ΠΎΠ΅ ΠΏΠΎΠ»Π΅ ΠΏΠΎΡ€ΠΎΠΆΠ΄Π°Π΅Ρ‚ элСктричСскоС ΠΏΠΎΠ»Π΅Β …Β»
ΠšΡƒΡ€Ρ Ρ„ΠΈΠ·ΠΈΠΊΠΈ. Π’.И.Π’Ρ€ΠΎΡ„ΠΈΠΌΠΎΠ²Π°. 1998. Π‘.248.
Β 
ΠŸΡ€ΠΈ ΠΎΠ΄Π½ΠΎΠ²Ρ€Π΅ΠΌΠ΅Π½Π½ΠΎΠΌ Π²Ρ‹ΠΊΠ»ΡŽΡ‡Π΅Π½ΠΈΠΈ элСктромагнитов Π½Π° заряд Ρ‚Π°ΠΊΠΆΠ΅ Π±ΡƒΠ΄Π΅Ρ‚ Π΄Π΅ΠΉΡΡ‚Π²ΠΎΠ²Π°Ρ‚ΡŒ сила, направлСнная Π²Π²Π΅Ρ€Ρ…, хотя ΠΌΠ°Π³Π½ΠΈΡ‚Π½ΠΎΠ΅ ΠΏΠΎΠ»Π΅ Π² Ρ‚ΠΎΡ‡ΠΊΠ΅, Π³Π΄Π΅ находится заряд, всСгда Π±ΡƒΠ΄Π΅Ρ‚ ΠΎΡΡ‚Π°Π²Π°Ρ‚ΡŒΡΡ Ρ€Π°Π²Π½Ρ‹ΠΌ Π½ΡƒΠ»ΡŽ. Π’.Π΅. Π² Ρ‚ΠΎΡ‡ΠΊΠ΅, Π³Π΄Π΅ находится заряд, ΠΌΠ°Π³Π½ΠΈΡ‚Π½ΠΎΠ΅ ΠΏΠΎΠ»Π΅ Π½Π΅ измСняСтся ΠΈ всСгда Ρ€Π°Π²Π½ΠΎ Π½ΡƒΠ»ΡŽ, Π½ΠΎ Ρ‚ΠΎΠ³Π΄Π° ΠΏΠΎΡ‡Π΅ΠΌΡƒ Π½Π° заряд дСйствуСт сила? ΠŸΠ°Ρ€Π°Π΄ΠΎΠΊΡ с элСктромагнитной ΠΈΠ½Π΄ΡƒΠΊΡ†ΠΈΠ΅ΠΉ ΠΌΠΎΠΆΠ½ΠΎ ΠΎΠ±ΡŠΡΡΠ½ΠΈΡ‚ΡŒ присутствиСм Ρ‚ΠΎΠΊΠΎΠ² смСщСния, ΠΊΠΎΡ‚ΠΎΡ€Ρ‹Π΅ Ρ‚Π΅ΠΊΡƒΡ‚ Π² ΠΎΠ΄Π½ΠΎΠΌ Π½Π°ΠΏΡ€Π°Π²Π»Π΅Π½ΠΈΠΈ ΠΈ ΡΠΊΠ»Π°Π΄Ρ‹Π²Π°ΡŽΡ‚ΡΡ согласно ΠΏΡ€ΠΈΠ½Ρ†ΠΈΠΏΡƒ супСрпозиции. ΠžΠ±Π½Π°Ρ€ΡƒΠΆΠΈΡ‚ΡŒ Ρ‚ΠΎΠΊΠΈ смСщСния ΠΌΠΎΠΆΠ½ΠΎ ΠΏΠΎ силС Π΄Π΅ΠΉΡΡ‚Π²ΡƒΡŽΡ‰Π΅ΠΉ Π½Π° заряд Π² ΠΌΠΎΠΌΠ΅Π½Ρ‚ Π²ΠΊΠ»ΡŽΡ‡Π΅Π½ΠΈΡ ΠΈΠ»ΠΈ Π²Ρ‹ΠΊΠ»ΡŽΡ‡Π΅Π½ΠΈΡ элСктромагнита. Π”Π°Π½Π½Ρ‹ΠΉ ΠΏΡ€ΠΈΠΌΠ΅Ρ€ ΠΏΠΎΠΊΠ°Π·Ρ‹Π²Π°Π΅Ρ‚, Ρ‡Ρ‚ΠΎ ΠΏΠ΅Ρ€Π΅ΠΌΠ΅Π½Π½Ρ‹ΠΉ Ρ‚ΠΎΠΊ смСщСния, дСйствуСт Π½Π° покоящийся элСктричСский заряд Π΄Π°ΠΆΠ΅ Π² Ρ‚Π΅Ρ… Ρ‚ΠΎΡ‡ΠΊΠ°Ρ… поля, Π³Π΄Π΅ Π½Π΅Ρ‚ ΠΌΠ°Π³Π½ΠΈΡ‚Π½ΠΎΠΉ ΠΈΠ½Π΄ΡƒΠΊΡ†ΠΈΠΈ. Π’.Π΅. Ρ„ΡƒΠ½Π΄Π°ΠΌΠ΅Π½Ρ‚Π°Π»ΡŒΠ½ΠΎΠ΅ ΡƒΡ‚Π²Π΅Ρ€ΠΆΠ΄Π΅Π½ΠΈΠ΅, Ρ‡Ρ‚ΠΎ ΠΈΠ·ΠΌΠ΅Π½ΡΡŽΡ‰Π΅Π΅ΡΡ Π²ΠΎ Π²Ρ€Π΅ΠΌΠ΅Π½ΠΈ ΠΌΠ°Π³Π½ΠΈΡ‚Π½ΠΎΠ΅ ΠΏΠΎΠ»Π΅ ΠΏΠΎΡ€ΠΎΠΆΠ΄Π°Π΅Ρ‚ элСктричСскоС, Π½Π΅ всСгда соотвСтствуСт Π΄Π΅ΠΉΡΡ‚Π²ΠΈΡ‚Π΅Π»ΡŒΠ½ΠΎΡΡ‚ΠΈ, Ρ‚Π°ΠΊ ΠΊΠ°ΠΊ Π²ΠΈΡ…Ρ€Π΅Π²ΠΎΠ΅ элСктричСскоС ΠΏΠΎΠ»Π΅ ΠΌΠΎΠΆΠ΅Ρ‚ Π²ΠΎΠ·Π½ΠΈΠΊΠ°Ρ‚ΡŒ Π΄Π°ΠΆΠ΅ Π² Ρ‚Π΅Ρ… Ρ‚ΠΎΡ‡ΠΊΠ°Ρ…, Π³Π΄Π΅ Π½Π΅Ρ‚ ΠΌΠ°Π³Π½ΠΈΡ‚Π½ΠΎΠΉ ΠΈΠ½Π΄ΡƒΠΊΡ†ΠΈΠΈ. Π’ ΠΏΡ€ΠΈΠ²Π΅Π΄Π΅Π½Π½ΠΎΠΌ ΠΏΡ€ΠΈΠΌΠ΅Ρ€Π΅ элСктромагниты ΠΌΠΎΠΆΠ½ΠΎ Π·Π°ΠΌΠ΅Π½ΠΈΡ‚ΡŒ Π½Π° постоянныС ΠΌΠ°Π³Π½ΠΈΡ‚Ρ‹, ΠΊΠΎΡ‚ΠΎΡ€Ρ‹Π΅ Ρ€Π°Π·Π΄Π²ΠΈΠ³Π°ΡŽΡ‚ΡΡ симмСтрично ΠΎΡ‚Π½ΠΎΡΠΈΡ‚Π΅Π»ΡŒΠ½ΠΎ покоящСгося заряда. Π’Π°ΠΊΠΆΠ΅ ΠΌΠΎΠΆΠ½ΠΎ привСсти ΠΈ Π΄Ρ€ΡƒΠ³ΠΈΠ΅ ΠΏΡ€ΠΈΠΌΠ΅Ρ€Ρ‹, Π½Π°ΠΏΡ€ΠΈΠΌΠ΅Ρ€, Π²ΠΎΠ·Π½ΠΈΠΊΠ½ΠΎΠ²Π΅Π½ΠΈΠ΅ ΠΈΠ½Π΄ΡƒΠΊΡ†ΠΈΠΎΠ½Π½ΠΎΠ³ΠΎ Ρ‚ΠΎΠΊΠ° Π²Π½ΡƒΡ‚Ρ€ΠΈ Ρ‚Ρ€ΡƒΠ±ΠΊΠΈ, ΠΏΠΎ ΠΊΠΎΡ‚ΠΎΡ€ΠΎΠΉ Ρ‚Π΅Ρ‡Π΅Ρ‚ ΠΏΠ΅Ρ€Π΅ΠΌΠ΅Π½Π½Ρ‹ΠΉ Ρ‚ΠΎΠΊ, хотя магнитная индукция Π²Π½ΡƒΡ‚Ρ€ΠΈ Ρ‚Ρ€ΡƒΠ±ΠΊΠΈ отсутствуСт. Π’.Π΅., рассматривая ΠΏΠ΅Ρ€Π΅ΠΌΠ΅Π½Π½Ρ‹Π΅ ΠΌΠ°Π³Π½ΠΈΡ‚Π½Ρ‹Π΅ поля, Π½Π΅ΠΎΠ±Ρ…ΠΎΠ΄ΠΈΠΌΠΎ ΡƒΡ‡ΠΈΡ‚Ρ‹Π²Π°Ρ‚ΡŒ Π½Π΅ Ρ‚ΠΎΠ»ΡŒΠΊΠΎ ΠΌΠ°Π³Π½ΠΈΡ‚Π½ΡƒΡŽ ΠΈΠ½Π΄ΡƒΠΊΡ†ΠΈΡŽ, Π½ΠΎ ΠΈ Ρ‚ΠΎΠΊΠΈ смСщСния. Π’ пространствС, Π³Π΄Π΅ Π½Π΅Ρ‚ измСнСния плотности Ρ‚ΠΎΠΊΠ° смСщСния, – Π½Π΅Ρ‚ ΠΌΠ°Π³Π½ΠΈΡ‚Π½ΠΎΠΉ ΠΈΠ½Π΄ΡƒΠΊΡ†ΠΈΠΈ, Π½Π°ΠΏΡ€ΠΈΠΌΠ΅Ρ€, Π²Π½ΡƒΡ‚Ρ€ΠΈ Ρ‚Ρ€ΡƒΠ±ΠΊΠΈ, ΠΏΠΎ ΠΊΠΎΡ‚ΠΎΡ€ΠΎΠΉ Ρ‚Π΅Ρ‡Π΅Ρ‚ Ρ‚ΠΎΠΊ. Π—Π° ΠΏΡ€Π΅Π΄Π΅Π»Π°ΠΌΠΈ ΠΆΠ΅ Ρ‚Ρ€ΡƒΠ±ΠΊΠΈ ΠΏΠ»ΠΎΡ‚Π½ΠΎΡΡ‚ΡŒ Ρ‚ΠΎΠΊΠ° смСщСния измСняСтся, Ρ‡Ρ‚ΠΎ прСдставляСт ΠΌΠ°Π³Π½ΠΈΡ‚Π½ΡƒΡŽ ΠΈΠ½Π΄ΡƒΠΊΡ†ΠΈΡŽ.
Β 
«Если ΠΏΡ€ΠΎΠ²ΠΎΠ΄ ΠΈΠΌΠ΅Π΅Ρ‚ Π²ΠΈΠ΄ Ρ‚Ρ€ΡƒΠ±ΠΊΠΈ, Ρ‚ΠΎ снаруТи индукция B опрСдСляСтся Ρ„ΠΎΡ€ΠΌΡƒΠ»ΠΎΠΉ (6. 18), Π° Π²Π½ΡƒΡ‚Ρ€ΠΈ – ΠΌΠ°Π³Π½ΠΈΡ‚Π½ΠΎΠ΅ ΠΏΠΎΠ»Π΅ отсутствуСт.Β»
Π­Π»Π΅ΠΊΡ‚Ρ€ΠΎΠΌΠ°Π³Π½Π΅Ρ‚ΠΈΠ·ΠΌ. И.Π•.Π˜Ρ€ΠΎΠ΄ΠΎΠ². 2000. Π‘.165.
Β 
Если ΡΠ΄Π΅Π»Π°Ρ‚ΡŒ Π²ΠΈΡ‚ΠΎΠΊ ΠΈΠ· Ρ‚Ρ€ΡƒΠ±ΠΊΠΈ, Π° Π²Π½ΡƒΡ‚Ρ€ΠΈ Ρ‚Ρ€ΡƒΠ±ΠΊΠΈ Ρ€Π°ΡΠΏΠΎΠ»ΠΎΠΆΠΈΡ‚ΡŒ Π²ΠΈΡ‚ΠΎΠΊ ΠΏΡ€ΠΎΠ²ΠΎΠ΄Π°, Ρ‚ΠΎ Π² ΠΏΡ€ΠΎΠ²ΠΎΠ΄Π΅ Π±ΡƒΠ΄Π΅Ρ‚ Π²ΠΎΠ·Π½ΠΈΠΊΠ°Ρ‚ΡŒ ΠΈΠ½Π΄ΡƒΠΊΡ†ΠΈΠΎΠ½Π½Ρ‹ΠΉ Ρ‚ΠΎΠΊ. ΠœΠ°Π³Π½ΠΈΡ‚Π½ΠΎΠ΅ ΠΏΠΎΠ»Π΅ Π²Π½ΡƒΡ‚Ρ€ΠΈ ΠΏΡ€ΠΎΠ²ΠΎΠ΄Π°, ΠΈΠΌΠ΅ΡŽΡ‰Π΅Π³ΠΎ Π²ΠΈΠ΄ Ρ‚Ρ€ΡƒΠ±ΠΊΠΈ, отсутствуСт, Π½ΠΎ ΠΈΠ½Π΄ΡƒΠΊΡ†ΠΈΠΎΠ½Π½Ρ‹ΠΉ Ρ‚ΠΎΠΊ Π²ΠΎΠ·Π½ΠΈΠΊΠ°Π΅Ρ‚, Ρ‚.Π΅. ΠΈΠ·ΠΌΠ΅Π½ΡΡŽΡ‰ΠΈΠΉΡΡ Ρ‚ΠΎΠΊ смСщСния проявляСтся ΠΊΠ°ΠΊ Π²ΠΈΡ…Ρ€Π΅Π²ΠΎΠ΅ элСктричСскоС ΠΏΠΎΠ»Π΅. ΠŸΠ»ΠΎΡ‚Π½ΠΎΡΡ‚ΡŒ ΠΎΠ±Ρ€Π°Ρ‚Π½ΠΎΠ³ΠΎ постоянного Ρ‚ΠΎΠΊΠ° смСщСния Π²Π½ΡƒΡ‚Ρ€ΠΈ прямого ΠΏΡ€ΠΎΠ²ΠΎΠ΄Π° бСсконСчной Π΄Π»ΠΈΠ½Ρ‹, ΠΈΠΌΠ΅ΡŽΡ‰Π΅Π³ΠΎ Π²ΠΈΠ΄ Ρ‚Ρ€ΡƒΠ±ΠΊΠΈ:

jсм =Β -I/2Ο€r2,

Π³Π΄Π΅ r – радиус ΠΏΡ€ΠΎΠ²ΠΎΠ΄Π°, I – постоянный Ρ‚ΠΎΠΊ Π² ΠΏΡ€ΠΎΠ²ΠΎΠ΄Π΅. Если Π½Π΅Ρ‚ измСнСния плотности Ρ‚ΠΎΠΊΠ° смСщСния, Ρ‚ΠΎ Π½Π° двиТущийся заряд Π½Π΅ Π±ΡƒΠ΄Π΅Ρ‚ Π΄Π΅ΠΉΡΡ‚Π²ΠΎΠ²Π°Ρ‚ΡŒ сила Π›ΠΎΡ€Π΅Π½Ρ†Π°. ИзмСнСниС плотности Ρ‚ΠΎΠΊΠ° смСщСния прСдставляСт ΠΌΠ°Π³Π½ΠΈΡ‚Π½ΡƒΡŽ ΠΈΠ½Π΄ΡƒΠΊΡ†ΠΈΡŽ. Π’Π°ΠΊΠΆΠ΅ Π²Π΅Π»ΠΈΡ‡ΠΈΠ½Π° ΠΌΠ°Π³Π½ΠΈΡ‚Π½ΠΎΠΉ ΠΈΠ½Π΄ΡƒΠΊΡ†ΠΈΠΈ зависит ΠΈ ΠΎΡ‚ Π΄Ρ€ΡƒΠ³ΠΈΡ… Ρ„Π°ΠΊΡ‚ΠΎΡ€ΠΎΠ², Π² Ρ‚ΠΎΠΌ числС ΠΈ ΠΎΡ‚ ΠΊΡ€ΠΈΠ²ΠΈΠ·Π½Ρ‹, ΠΏΠΎ ΠΊΠΎΡ‚ΠΎΡ€ΠΎΠΉ Ρ‚Π΅Ρ‡Π΅Ρ‚ Ρ‚ΠΎΠΊ смСщСния.


Β 
Π’ пространствС Π²ΠΎΠΊΡ€ΡƒΠ³ ΠΌΠ°Π³Π½ΠΈΡ‚Π° (Π² ΠΌΠ°Π³Π½ΠΈΡ‚Π½ΠΎΠΌ ΠΏΠΎΠ»Π΅) Π½Π΅ΠΏΡ€Π΅Ρ€Ρ‹Π²Π½ΠΎ Ρ‚Π΅ΠΊΡƒΡ‚ Ρ‚ΠΎΠΊΠΈ элСктричСского смСщСния, ΠΊΠΎΡ‚ΠΎΡ€Ρ‹Π΅ ΠΌΠΎΠΆΠ½ΠΎ ΠΎΠ±Π½Π°Ρ€ΡƒΠΆΠΈΡ‚ΡŒ, Π½Π°ΠΏΡ€ΠΈΠΌΠ΅Ρ€, ΠΊΠ°ΠΊ Π²ΠΈΡ…Ρ€Π΅Π²Ρ‹Π΅ элСктричСскиС поля ΠΏΡ€ΠΈ Π²ΠΊΠ»ΡŽΡ‡Π΅Π½ΠΈΠΈ ΠΈ Π²Ρ‹ΠΊΠ»ΡŽΡ‡Π΅Π½ΠΈΠΈ элСктромагнита.
Β 
Π’ΠΎ ΠΌΠ½ΠΎΠ³ΠΈΡ… случаях ΠΌΠ°Π³Π½ΠΈΡ‚Π½ΠΎΠ΅ ΠΏΠΎΠ»Π΅ ΡƒΠ΄ΠΎΠ±Π½Π΅Π΅ ΠΏΡ€Π΅Π΄ΡΡ‚Π°Π²Π»ΡΡ‚ΡŒ линиями элСктричСского Ρ‚ΠΎΠΊΠ° смСщСния ΠΈΠ»ΠΈ ΠΊΠ°ΠΊ двиТущиСся элСктричСскиС ΠΏΠΎΡ‚ΠΎΠΊΠΈ, Ρ‚Π΅ΠΌ самым ΠΈΠ·-Π·Π° наглядности ΡƒΠΌΠ΅Π½ΡŒΡˆΠ°Π΅Ρ‚ΡΡ Π²Π΅Ρ€ΠΎΡΡ‚Π½ΠΎΡΡ‚ΡŒ тСхничСских ошибок. НапримСр, Π² ΡƒΡ‡Π΅Π±Π½ΠΎΠΉ Π»ΠΈΡ‚Π΅Ρ€Π°Ρ‚ΡƒΡ€Π΅ Π½Π°ΠΏΡ€Π°Π²Π»Π΅Π½ΠΈΠ΅ Π»ΠΈΠ½ΠΈΠΉ ΠΌΠ°Π³Π½ΠΈΡ‚Π½ΠΎΠΉ ΠΈΠ½Π΄ΡƒΠΊΡ†ΠΈΠΈ ΠΌΠ΅ΠΆΠ΄Ρƒ ΠΎΠ±ΠΊΠ»Π°Π΄ΠΊΠ°ΠΌΠΈ кондСнсатора ΠΈΠ·ΠΎΠ±Ρ€Π°ΠΆΠ΅Π½ΠΎ Π½Π΅ΠΏΡ€Π°Π²ΠΈΠ»ΡŒΠ½ΠΎ – Π² ΠΎΠ±Ρ€Π°Ρ‚Π½ΡƒΡŽ сторону, Ρ‚.Π΅. получаСтся Ρ‚Π°ΠΊΠΎΠΉ Ρ€Π°Π·Π²ΠΎΡ€ΠΎΡ‚ Ρ€Π°ΠΌΠΎΠΊ с Ρ‚ΠΎΠΊΠΎΠΌ (стрСлки ΡƒΠΊΠ°Π·Ρ‹Π²Π°ΡŽΡ‚ Π½Π°ΠΏΡ€Π°Π²Π»Π΅Π½ΠΈΠ΅ Ρ‚ΠΎΠΊΠ°).


ΠšΡƒΡ€Ρ Ρ„ΠΈΠ·ΠΈΠΊΠΈ. Π’.И.Π’Ρ€ΠΎΡ„ΠΈΠΌΠΎΠ²Π°. 1998. Π‘.250.

На самом Π΄Π΅Π»Π΅ Ρƒ ΠΌΠ°Π³Π½ΠΈΡ‚Π½ΠΎΠ³ΠΎ поля ΠΌΠ΅ΠΆΠ΄Ρƒ ΠΎΠ±ΠΊΠ»Π°Π΄ΠΊΠ°ΠΌΠΈ кондСнсатора Π»ΠΈΠ½ΠΈΠΈ ΠΌΠ°Π³Π½ΠΈΡ‚Π½ΠΎΠΉ ΠΈΠ½Π΄ΡƒΠΊΡ†ΠΈΠΈ Π½Π°ΠΏΡ€Π°Π²Π»Π΅Π½Ρ‹ Π² ΠΎΠ±Ρ€Π°Ρ‚Π½ΡƒΡŽ сторону. НапримСр, Π²ΠΈΡ…Ρ€Π΅Π²Ρ‹Π΅ Ρ‚ΠΎΠΊΠΈ Π±ΡƒΠ΄ΡƒΡ‚ ΠΈΠΌΠ΅Ρ‚ΡŒ ΠΎΠ±Ρ€Π°Ρ‚Π½ΠΎΠ΅ Π½Π°ΠΏΡ€Π°Π²Π»Π΅Π½ΠΈΠ΅, Ρ‚Π°ΠΊ ΠΊΠ°ΠΊ Ρ‚ΠΎΠΊΠΈ смСщСния ΠΌΠ΅ΠΆΠ΄Ρƒ ΠΎΠ±ΠΊΠ»Π°Π΄ΠΊΠ°ΠΌΠΈ кондСнсатора “прямыС”, Π° Π½Π΅ “ΠΎΠ±Ρ€Π°Ρ‚Π½Ρ‹Π΅”. ВсСм, ΠΊΡ‚ΠΎ пытался ΠΈΠ·ΠΌΠ΅Ρ€ΠΈΡ‚ΡŒ ΠΌΠ°Π³Π½ΠΈΡ‚Π½ΠΎΠ΅ ΠΏΠΎΠ»Π΅, создаваСмоС Ρ‚ΠΎΠΊΠ°ΠΌΠΈ смСщСния (Π½Π΅ Ρ‚ΠΎΠΊΠ°ΠΌΠΈ поляризации) Π² кондСнсаторС, ΡƒΠ΄Π°Π²Π°Π»ΠΎΡΡŒ ΠΎΠ±Π½Π°Ρ€ΡƒΠΆΠΈΡ‚ΡŒ Ρ‚ΠΎΠ»ΡŒΠΊΠΎ ΠΌΠ°Π³Π½ΠΈΡ‚Π½ΠΎΠ΅ ΠΏΠΎΠ»Π΅, ΠΎΠ±Ρ€Π°Π·ΠΎΠ²Π°Π½Π½ΠΎΠ΅ Ρ‚ΠΎΠΊΠ°ΠΌΠΈ проводимости Π² ΠΎΠ±ΠΊΠ»Π°Π΄ΠΊΠ°Ρ… кондСнсатора. Π’ этом случаС Ρ€Π°Π·Π²ΠΎΡ€ΠΎΡ‚ Ρ€Π°ΠΌΠΎΠΊ с Ρ‚ΠΎΠΊΠΎΠΌ Π±ΡƒΠ΄Π΅Ρ‚ Π²Ρ‹Π³Π»ΡΠ΄Π΅Ρ‚ΡŒ Ρ‚Π°ΠΊ.

НаправлСниС ΠΌΠ°Π³Π½ΠΈΡ‚Π½ΠΎΠΉ ΠΈΠ½Π΄ΡƒΠΊΡ†ΠΈΠΈ ΠΌΠ΅ΠΆΠ΄Ρƒ ΠΎΠ±ΠΊΠ»Π°Π΄ΠΊΠ°ΠΌΠΈ кондСнсатора ΠΌΠΎΠΆΠ½ΠΎ просто ΠΎΠΏΡ€Π΅Π΄Π΅Π»ΠΈΡ‚ΡŒ согласно BΒ =Β ΞΌ0[vD], сформулировав ΠΏΡ€Π°Π²ΠΈΠ»ΠΎ возникновСния ΠΌΠ°Π³Π½ΠΈΡ‚Π½ΠΎΠΉ ΠΈΠ½Π΄ΡƒΠΊΡ†ΠΈΠΈ: Ссли ладонь Π»Π΅Π²ΠΎΠΉ Ρ€ΡƒΠΊΠΈ Ρ€Π°ΡΠΏΠΎΠ»ΠΎΠΆΠΈΡ‚ΡŒ Ρ‚Π°ΠΊ, Ρ‡Ρ‚ΠΎΠ±Ρ‹ Ρ‡Π΅Ρ‚Ρ‹Ρ€Π΅ ΠΏΠ°Π»ΡŒΡ†Π° ΡƒΠΊΠ°Π·Ρ‹Π²Π°Π»ΠΈ Π½Π°ΠΏΡ€Π°Π²Π»Π΅Π½ΠΈΠ΅ двиТСния элСктричСского ΠΏΠΎΡ‚ΠΎΠΊΠ°, Π° Π²Π΅ΠΊΡ‚ΠΎΡ€ D Π²Ρ…ΠΎΠ΄ΠΈΠ» Π² ладонь, Ρ‚ΠΎΠ³Π΄Π° отставлСнный большой ΠΏΠ°Π»Π΅Ρ† ΡƒΠΊΠ°ΠΆΠ΅Ρ‚ Π½Π°ΠΏΡ€Π°Π²Π»Π΅Π½ΠΈΠ΅ Π²Π΅ΠΊΡ‚ΠΎΡ€Π° B. Π’.Π΅., Ρ‡Ρ‚ΠΎΠ±Ρ‹ ΠΎΠΏΡ€Π΅Π΄Π΅Π»ΠΈΡ‚ΡŒ Π½Π°ΠΏΡ€Π°Π²Π»Π΅Π½ΠΈΠ΅ Π»ΠΈΠ½ΠΈΠΉ ΠΌΠ°Π³Π½ΠΈΡ‚Π½ΠΎΠΉ ΠΈΠ½Π΄ΡƒΠΊΡ†ΠΈΠΈ, достаточно Ρ€Π°ΡΡΠΌΠΎΡ‚Ρ€Π΅Ρ‚ΡŒ Π΄Π²ΠΈΠΆΠ΅Π½ΠΈΠ΅ элСктричСских ΠΏΠΎΡ‚ΠΎΠΊΠΎΠ², связанных с зарядами, ΠΊΠΎΡ‚ΠΎΡ€Ρ‹Π΅ двиТутся Π² ΠΎΠ±ΠΊΠ»Π°Π΄ΠΊΠ°Ρ… кондСнсатора. Π’Π°ΠΊΠΆΠ΅ ΠΏΠΎΠ½ΡΡ‚ΡŒ, ΠΊΠ°ΠΊ развСрнСтся Ρ€Π°ΠΌΠΊΠ° с Ρ‚ΠΎΠΊΠΎΠΌ ΠΌΠ΅ΠΆΠ΄Ρƒ ΠΎΠ±ΠΊΠ»Π°Π΄ΠΊΠ°ΠΌΠΈ кондСнсатора, ΠΌΠΎΠΆΠ½ΠΎ ΠΏΠΎ ΠΏΡ€Π°Π²ΠΈΠ»Ρƒ: Ссли Ρ‚ΠΎΠΊΠΈ ΠΈΠΌΠ΅ΡŽΡ‚ ΠΎΠ΄ΠΈΠ½Π°ΠΊΠΎΠ²ΠΎΠ΅ Π½Π°ΠΏΡ€Π°Π²Π»Π΅Π½ΠΈΠ΅, Ρ‚ΠΎ Π²ΠΎΠ·Π½ΠΈΠΊΠ°Π΅Ρ‚ сила притяТСния. Π•Ρ‰Π΅ Π½Π°ΠΏΡ€Π°Π²Π»Π΅Π½ΠΈΠ΅ ΠΌΠ°Π³Π½ΠΈΡ‚Π½ΠΎΠ³ΠΎ поля Π²ΠΎΠΊΡ€ΡƒΠ³ ΠΎΠ±ΠΊΠ»Π°Π΄ΠΎΠΊ кондСнсатора ΠΌΠΎΠΆΠ½ΠΎ ΠΎΠΏΡ€Π΅Π΄Π΅Π»ΠΈΡ‚ΡŒ ΠΏΠΎ ΠΏΡ€Π°Π²ΠΈΠ»Ρƒ Π±ΡƒΡ€Π°Π²Ρ‡ΠΈΠΊΠ°, Ссли Ρ€Π°ΡΡΠΌΠΎΡ‚Ρ€Π΅Ρ‚ΡŒ Π΄Π²ΠΈΠΆΠ΅Π½ΠΈΠ΅ зарядов ΠΏΠΎ ΠΎΠ±ΠΊΠ»Π°Π΄ΠΊΠ°ΠΌ.

На рисунС наглядно ΠΏΠΎΠΊΠ°Π·Π°Π½ΠΎ, ΠΊΠ°ΠΊ Ρ€Π΅Π°Π»ΡŒΠ½ΠΎ Π½Π°ΠΏΡ€Π°Π²Π»Π΅Π½ΠΎ ΠΌΠ°Π³Π½ΠΈΡ‚Π½ΠΎΠ΅ ΠΏΠΎΠ»Π΅ ΠΌΠ΅ΠΆΠ΄Ρƒ ΠΎΠ±ΠΊΠ»Π°Π΄ΠΊΠ°ΠΌΠΈ ΠΈ ΠΊΠ°ΠΊ Π½Π΅ΠΏΡ€Π°Π²ΠΈΠ»ΡŒΠ½ΠΎ Ρ€ΠΈΡΡƒΡŽΡ‚ Π² ΡƒΡ‡Π΅Π±Π½ΠΈΠΊΠ°Ρ…. Π’Π°ΠΊΠΆΠ΅ это ΠΌΠΎΠΆΠ½ΠΎ ΡΠΊΡΠΏΠ΅Ρ€ΠΈΠΌΠ΅Π½Ρ‚Π°Π»ΡŒΠ½ΠΎ ΠΏΡ€ΠΎΠ²Π΅Ρ€ΠΈΡ‚ΡŒ: бСрутся Π΄Π²Π΅ ΠΊΠ°Ρ‚ΡƒΡˆΠΊΠΈ с ΠΏΡ€ΠΎΠ²ΠΎΠ΄ΠΎΠΌ, ΠΎΠ΄Π½Π° кладСтся ΠΌΠ΅ΠΆΠ΄Ρƒ ΠΎΠ±ΠΊΠ»Π°Π΄ΠΊΠ°ΠΌΠΈ кондСнсатора, другая рядом с ΠΏΡ€ΠΎΠ²ΠΎΠ΄ΠΎΠΌ, ΠΈΠ΄ΡƒΡ‰ΠΈΠΌ ΠΊ кондСнсатору. ΠŸΠΎΠ΄Π°Π΅Ρ‚ΡΡ ΠΏΠ΅Ρ€Π΅ΠΌΠ΅Π½Π½Ρ‹ΠΉ Ρ‚ΠΎΠΊ Π½Π° кондСнсатор. На Π΄Π²ΡƒΡ…Π»ΡƒΡ‡Π΅Π²ΠΎΠΌ осциллографС Π±ΡƒΠ΄Π΅Ρ‚ Π²ΠΈΠ΄Π½ΠΎ, Ρ‡Ρ‚ΠΎ Ρ‚ΠΎΠΊ, Π²ΠΎΠ·Π½ΠΈΠΊΠ°ΡŽΡ‰ΠΈΠΉ Π² ΠΊΠ°Ρ‚ΡƒΡˆΠΊΠ°Ρ…, сдвинут ΠΏΠΎ Ρ„Π°Π·Π΅ Π½Π° 180 градусов. НаправлСниС Π»ΠΈΠ½ΠΈΠΉ ΠΌΠ°Π³Π½ΠΈΡ‚Π½ΠΎΠΉ ΠΈΠ½Π΄ΡƒΠΊΡ†ΠΈΠΈ ΠΌΠ΅ΠΆΠ΄Ρƒ ΠΎΠ±ΠΊΠ»Π°Π΄ΠΊΠ°ΠΌΠΈ кондСнсатора ΠΈΠΌΠ΅Π΅Ρ‚ ΠΏΡ€ΠΈΠ½Ρ†ΠΈΠΏΠΈΠ°Π»ΡŒΠ½ΠΎΠ΅ Π·Π½Π°Ρ‡Π΅Π½ΠΈΠ΅ для элСктродинамики, Ρ‚Π°ΠΊ ΠΊΠ°ΠΊ это ΡΠΊΡΠΏΠ΅Ρ€ΠΈΠΌΠ΅Π½Ρ‚Π°Π»ΡŒΠ½ΠΎ Π΄ΠΎΠΊΠ°Π·Ρ‹Π²Π°Π΅Ρ‚, Ρ‡Ρ‚ΠΎ сами Ρ‚ΠΎΠΊΠΈ смСщСния (ΠΈΡΠΊΠ»ΡŽΡ‡Π°Ρ Ρ‚ΠΎΠΊΠΈ поляризации) ΠΏΡ€Π΅Π΄ΡΡ‚Π°Π²Π»ΡΡŽΡ‚ ΠΌΠ°Π³Π½ΠΈΡ‚Π½ΠΎΠ΅ ΠΏΠΎΠ»Π΅.
Β 
Β«Π’ΠΎΠ·Π±ΡƒΠΆΠ΄Π΅Π½ΠΈΠ΅ ΠΌΠ°Π³Π½ΠΈΡ‚Π½ΠΎΠ³ΠΎ поля Ρ‚ΠΎΠΊΠ°ΠΌΠΈ поляризации ΠΏΡ€Π°Π²ΠΎΠΌΠ΅Ρ€Π½ΠΎ, Ρ‚Π°ΠΊ ΠΊΠ°ΠΊ Ρ‚ΠΎΠΊΠΈ поляризации ΠΏΠΎ своСй ΠΏΡ€ΠΈΡ€ΠΎΠ΄Π΅ Π½Π΅ ΠΎΡ‚Π»ΠΈΡ‡Π°ΡŽΡ‚ΡΡ ΠΎΡ‚ Ρ‚ΠΎΠΊΠΎΠ² проводимости. Однако Ρ‚ΠΎ, Ρ‡Ρ‚ΠΎ ΠΈ другая Ρ‡Π°ΡΡ‚ΡŒ плотности Ρ‚ΠΎΠΊΠ° смСщСния (Ξ΅0dE/dt), Π½Π΅ связанная с Π΄Π²ΠΈΠΆΠ΅Π½ΠΈΠ΅ΠΌ зарядов, Π° обусловлСнная Ρ‚ΠΎΠ»ΡŒΠΊΠΎ ΠΈΠ·ΠΌΠ΅Π½Π΅Π½ΠΈΠ΅ΠΌ элСктричСского поля Π²ΠΎ Π²Ρ€Π΅ΠΌΠ΅Π½ΠΈ, Ρ‚Π°ΠΊΠΆΠ΅ Π²ΠΎΠ·Π±ΡƒΠΆΠ΄Π°Π΅Ρ‚ ΠΌΠ°Π³Π½ΠΈΡ‚Π½ΠΎΠ΅ ΠΏΠΎΠ»Π΅, являСтся ΠΏΡ€ΠΈΠ½Ρ†ΠΈΠΏΠΈΠ°Π»ΡŒΠ½ΠΎ Π½ΠΎΠ²Ρ‹ΠΌ ΡƒΡ‚Π²Π΅Ρ€ΠΆΠ΄Π΅Π½ΠΈΠ΅ΠΌ МаксвСлла. Π”Π°ΠΆΠ΅ Π² Π²Π°ΠΊΡƒΡƒΠΌΠ΅ всякоС ΠΈΠ·ΠΌΠ΅Π½Π΅Π½ΠΈΠ΅ Π²ΠΎ Π²Ρ€Π΅ΠΌΠ΅Π½ΠΈ элСктричСского поля ΠΏΡ€ΠΈΠ²ΠΎΠ΄ΠΈΡ‚ ΠΊ возникновСнию Π² ΠΎΠΊΡ€ΡƒΠΆΠ°ΡŽΡ‰Π΅ΠΌ пространствС ΠΌΠ°Π³Π½ΠΈΡ‚Π½ΠΎΠ³ΠΎ поля.Β»
ΠšΡƒΡ€Ρ Ρ„ΠΈΠ·ΠΈΠΊΠΈ. Π’.И.Π’Ρ€ΠΎΡ„ΠΈΠΌΠΎΠ²Π°. 1998. Π‘.250.
Β 
Π’ΠΎΠΊ смСщСния (Π½Π΅ поляризации) прСдставляСт ΠΈΠ·ΠΌΠ΅Π½ΡΡŽΡ‰Π΅Π΅ΡΡ элСктричСскоС ΠΏΠΎΠ»Π΅ (dD/dt) ΠΈ Π½Π° Π½Π΅Π³ΠΎ Π½Π΅ распространяСтся ΠΏΡ€Π°Π²ΠΈΠ»ΠΎ Π±ΡƒΡ€Π°Π²Ρ‡ΠΈΠΊΠ°, ΠΊΠΎΡ‚ΠΎΡ€ΠΎΠ΅ дСйствуСт Ρ‚ΠΎΠ»ΡŒΠΊΠΎ для Ρ‚ΠΎΠΊΠ° проводимости, Π³Π΄Π΅ подразумСваСтся Π΄Π²ΠΈΠΆΠ΅Π½ΠΈΠ΅ зарядов, Π° Π½Π΅ ΠΈΠ½Π΄ΡƒΠΊΡ†ΠΈΠΈ D. Для Ρ‚ΠΎΠΊΠ° смСщСния ΠΆΠ΅, ΠΊΠ°ΠΊ ΠΈΠ·ΠΌΠ΅Π½ΡΡŽΡ‰Π΅Π³ΠΎΡΡ элСктричСского поля, ΠΌΠ°Π³Π½ΠΈΡ‚Π½ΡƒΡŽ ΠΈΠ½Π΄ΡƒΠΊΡ†ΠΈΡŽ Π½Π°Π΄ΠΎ ΠΎΠΏΡ€Π΅Π΄Π΅Π»ΡΡ‚ΡŒ согласно BΒ =Β ΞΌ0[vD] (ΠΏΡ€Π°Π²ΠΈΠ»ΠΎ Π»Π΅Π²ΠΎΠΉ Ρ€ΡƒΠΊΠΈ для ΠΌΠ°Π³Π½ΠΈΡ‚Π½ΠΎΠΉ ΠΈΠ½Π΄ΡƒΠΊΡ†ΠΈΠΈ). Когда ΠΌΠ΅ΠΆΠ΄Ρƒ ΠΎΠ±ΠΊΠ»Π°Π΄ΠΊΠ°Ρ… кондСнсатора измСняСтся элСктричСскоС ΠΏΠΎΠ»Π΅, Ρ‚ΠΎ это ΠΎΠ·Π½Π°Ρ‡Π°Π΅Ρ‚, Ρ‡Ρ‚ΠΎ ΠΏΠΎ ΠΎΠ±ΠΊΠ»Π°Π΄ΠΊΠ°ΠΌ Ρ‚Π΅ΠΊΡƒΡ‚ заряды ΠΈ с Π½ΠΈΠΌΠΈ двиТСтся индукция D, ΠΏΡ€Π΅Π΄ΡΡ‚Π°Π²Π»ΡΡŽΡ‰Π°Ρ Ρ‚ΠΎΠΊ смСщСния (dD/dt). ΠŸΠΎΡΡ‚ΠΎΠΌΡƒ, хотя ΠΈΠ·ΠΌΠ΅Π½ΡΡŽΡ‰Π΅Π΅ΡΡ (двиТущССся) элСктричСскоС ΠΏΠΎΠ»Π΅ Π² кондСнсаторС ΠΈ прСдставляСт Ρ‚ΠΎΠΊ смСщСния, Π½ΠΎ ΠΌΠ°Π³Π½ΠΈΡ‚Π½ΠΎΠ΅ ΠΏΠΎΠ»Π΅ ΠΎΠ΄Π½ΠΎΠ²Ρ€Π΅ΠΌΠ΅Π½Π½ΠΎ Ρ‚Π°ΠΊΠΆΠ΅ связано ΠΈ с Ρ‚ΠΎΠΊΠΎΠΌ проводимости, ΠΊΠΎΡ‚ΠΎΡ€Ρ‹ΠΉ Ρ‚Π΅Ρ‡Π΅Ρ‚ ΠΏΠΎ ΠΎΠ±ΠΊΠ»Π°Π΄ΠΊΠ°ΠΌ кондСнсатора. Π’.Π΅. индукция D, ΡΠΎΠ·Π΄Π°ΡŽΡ‰Π°Ρ Ρ‚ΠΎΠΊ смСщСния jсм =Β dD/dt, ΠΏΡ€ΠΈΠ½Π°Π΄Π»Π΅ΠΆΠΈΡ‚ зарядам Π² кондСнсаторС ΠΈ, соотвСтствСнно, ΠΌΠ°Π³Π½ΠΈΡ‚Π½ΠΎΠ΅ ΠΏΠΎΠ»Π΅ Ρ‚Π°ΠΊΠΆΠ΅ связано с Ρ‚ΠΎΠΊΠΎΠΌ Π² ΠΎΠ±ΠΊΠ»Π°Π΄ΠΊΠ°Ρ… кондСнсатора. Волько Π² элСктромагнитной Π²ΠΎΠ»Π½Π΅, Π³Π΄Π΅ Π½Π΅Ρ‚ Ρ‚ΠΎΠΊΠ° проводимости ΠΈ элСктричСская индукция D Π½Π΅ ΠΏΡ€ΠΈΠ½Π°Π΄Π»Π΅ΠΆΠΈΡ‚ зарядам, ΠΌΠ°Π³Π½ΠΈΡ‚Π½ΠΎΠ΅ ΠΏΠΎΠ»Π΅ ΠΈ Π΅Π³ΠΎ энСргия связаны Ρ‚ΠΎΠ»ΡŒΠΊΠΎ с Ρ‚ΠΎΠΊΠΎΠΌ смСщСния. Π‘Π°ΠΌΠ° ΠΆΠ΅ Π²Π΅Π»ΠΈΡ‡ΠΈΠ½Π΅ Ρ‚ΠΎΠΊΠ° смСщСния – это ΡƒΡΠ»ΠΎΠ²Π½ΠΎΡΡ‚ΡŒ (Π½Π΅Ρ‚ двиТСния заряТСнных частиц) ΠΈ ΠΎΠ½Π° Π½ΠΈΠΊΠΎΠ³Π΄Π° Π½Π΅ ΠΈΡΠΏΠΎΠ»ΡŒΠ·ΡƒΠ΅Ρ‚ΡΡ ΠΏΡ€ΠΈ вычислСнии ΠΌΠ°Π³Π½ΠΈΡ‚Π½ΠΎΠΉ ΠΈΠ½Π΄ΡƒΠΊΡ†ΠΈΠΈ, Ρ‚Π°ΠΊ ΠΊΠ°ΠΊ Ρ‚ΠΎΠΊ смСщСния поля – это ΠΈΠ·ΠΌΠ΅Π½ΡΡŽΡ‰Π΅Π΅ΡΡ элСктричСскоС ΠΏΠΎΠ»Π΅, Π° для ΠΏΠΎΠ»Π΅ΠΉ Π² элСктродинамикС: BΒ =Β ΞΌ0[vD]. НапримСр, ΠΊΠΎΠ³Π΄Π° ΠΏΠΎ Π°Π½Ρ‚Π΅Π½Π½Π΅ Ρ‚Π΅Ρ‡Π΅Ρ‚ Ρ‚ΠΎΠΊ проводимости, Ρ‚ΠΎ Ρ‚ΠΎΡ‡Π½ΠΎ Ρ‚Π°ΠΊΠΎΠΉ ΠΆΠ΅ Π²Π΅Π»ΠΈΡ‡ΠΈΠ½Ρ‹, Π½ΠΎ Π² ΠΎΠ±Ρ€Π°Ρ‚Π½ΠΎΠΌ Π½Π°ΠΏΡ€Π°Π²Π»Π΅Π½ΠΈΠΈ Ρ‚Π΅Ρ‡Π΅Ρ‚ Ρ‚ΠΎΠΊ смСщСния (Ρ‚ΠΎΠΊ всСгда Π·Π°ΠΌΠΊΠ½ΡƒΡ‚). ΠŸΡ€ΠΈ этом ΠΌΠ°Π³Π½ΠΈΡ‚Π½ΡƒΡŽ ΠΈΠ½Π΄ΡƒΠΊΡ†ΠΈΡŽ ΠΌΠΎΠΆΠ½ΠΎ Π²Ρ‹Ρ‡ΠΈΡΠ»ΠΈΡ‚ΡŒ Π»ΠΈΠ±ΠΎ ΠΏΠΎ Π²Π΅Π»ΠΈΡ‡ΠΈΠ½Π΅ Ρ‚ΠΎΠΊΠ° проводимости Π² Π°Π½Ρ‚Π΅Π½Π½Π΅, Π»ΠΈΠ±ΠΎ ΠΏΠΎ двиТСнию ΠΏΠΎΡ‚ΠΎΠΊΠΎΠ² элСктричСской ΠΈΠ½Π΄ΡƒΠΊΡ†ΠΈΠΈ BΒ =Β ΞΌ0[vD], связанных с ΠΏΠ΅Ρ€Π΅ΠΌΠ΅Ρ‰Π΅Π½ΠΈΠ΅ΠΌ зарядов Π² Π°Π½Ρ‚Π΅Π½Π½Π΅, Π½ΠΎ Π½Π΅ ΠΏΠΎ Π²Π΅Π»ΠΈΡ‡ΠΈΠ½Π΅ ΠΎΠ±Ρ€Π°Ρ‚Π½ΠΎΠ³ΠΎ Ρ‚ΠΎΠΊΠ° смСщСния поля. Π’ΠΎ ΠΆΠ΅ самоС ΠΈ ΠΌΠ΅ΠΆΠ΄Ρƒ ΠΎΠ±ΠΊΠ»Π°Π΄ΠΊΠ°ΠΌΠΈ кондСнсатора – для опрСдСлСния ΠΌΠ°Π³Π½ΠΈΡ‚Π½ΠΎΠΉ ΠΈΠ½Π΄ΡƒΠΊΡ†ΠΈΠΈ Π½Π°Π΄ΠΎ ΠΈΡΠΏΠΎΠ»ΡŒΠ·ΠΎΠ²Π°Ρ‚ΡŒ BΒ =Β ΞΌ0[vD], примСняя ΠΏΡ€Π°Π²ΠΈΠ»ΠΎ Π»Π΅Π²ΠΎΠΉ Ρ€ΡƒΠΊΠΈ.
Β 
Для наглядности рассмотрим Π½Π°ΠΏΡ€Π°Π²Π»Π΅Π½ΠΈΠ΅ Ρ‚ΠΎΠΊΠ°, Π²ΠΎΠ·Π½ΠΈΠΊΠ°ΡŽΡ‰Π΅Π΅ Π² Π½Π΅Π·Π°ΠΌΠΊΠ½ΡƒΡ‚Ρ‹Ρ… Ρ€Π°ΠΌΠΊΠ°Ρ…, ΠΊΠΎΠ³Π΄Π° ΠΏΠΎ ΠΎΠ±ΠΊΠ»Π°Π΄ΠΊΠ°ΠΌ кондСнсатора Ρ‚Π΅Ρ‡Π΅Ρ‚ Ρ‚ΠΎΠΊ.

Π’ΠΈΠ΄Π½ΠΎ, Ρ‡Ρ‚ΠΎ Ρ‚ΠΎΠΊ Π·Π° ΠΎΠ±ΠΊΠ»Π°Π΄ΠΊΠ°ΠΌΠΈ ΠΈ ΠΌΠ΅ΠΆΠ΄Ρƒ Π½ΠΈΠΌΠΈ ΠΈΠΌΠ΅Π΅Ρ‚ ΠΏΡ€ΠΎΡ‚ΠΈΠ²ΠΎΠΏΠΎΠ»ΠΎΠΆΠ½ΠΎΠ΅ Π½Π°ΠΏΡ€Π°Π²Π»Π΅Π½ΠΈΠ΅, соотвСтствСнно, Ρ‚Π°ΠΊ ΠΆΠ΅ ΠΊΠ°ΠΊ ΠΈ ΠΌΠ°Π³Π½ΠΈΡ‚Π½ΠΎΠ΅ ΠΏΠΎΠ»Π΅.

Β 
ДСмонстрация студСнтам ΠΌΠ°Π³Π½ΠΈΡ‚Π½ΠΎΠ³ΠΎ поля ΠΌΠ΅ΠΆΠ΄Ρƒ ΠΎΠ±ΠΊΠ»Π°Π΄ΠΊΠ°ΠΌΠΈ кондСнсатора.

НСсмотря Π½Π° Ρ‚ΠΎ, Ρ‡Ρ‚ΠΎ сСгодня Π΄Π°ΠΆΠ΅ студСнты ΠΌΠΎΠ³ΡƒΡ‚ Π² ΠΊΠ°Π±ΠΈΠ½Π΅Ρ‚Π΅ Ρ„ΠΈΠ·ΠΈΠΊΠΈ Π½Π° дСмонстрационном ΠΏΡ€ΠΈΠ±ΠΎΡ€Π΅ ΡΠΊΡΠΏΠ΅Ρ€ΠΈΠΌΠ΅Π½Ρ‚Π°Π»ΡŒΠ½ΠΎ ΠΏΡ€ΠΎΠ²Π΅Ρ€ΠΈΡ‚ΡŒ Π½Π°ΠΏΡ€Π°Π²Π»Π΅Π½ΠΈΠ΅ ΠΌΠ°Π³Π½ΠΈΡ‚Π½ΠΎΠ³ΠΎ поля ΠΌΠ΅ΠΆΠ΄Ρƒ ΠΎΠ±ΠΊΠ»Π°Π΄ΠΊΠ°ΠΌΠΈ кондСнсатора, Π² ΡƒΡ‡Π΅Π±Π½ΠΎΠΉ Π»ΠΈΡ‚Π΅Ρ€Π°Ρ‚ΡƒΡ€Π΅ ΠΏΡ€ΠΎΠ΄ΠΎΠ»ΠΆΠ°ΡŽΡ‚ ΠΏΡ€ΠΈΠ²ΠΎΠ΄ΠΈΡ‚ΡŒ рисунки, Π³Π΄Π΅ магнитная индукция ΠΈΠ·ΠΎΠ±Ρ€Π°ΠΆΠ΅Π½Π° Π½Π΅ΠΏΡ€Π°Π²ΠΈΠ»ΡŒΠ½ΠΎ – Π² ΠΎΠ±Ρ€Π°Ρ‚Π½ΡƒΡŽ сторону. Π’.Π΅. ΠΎΡˆΠΈΠ±ΠΎΡ‡Π½ΠΎ ΠΏΡ€ΠΈΠΌΠ΅Π½ΡΡŽΡ‚ для Ρ‚ΠΎΠΊΠ° смСщСния поля ΠΏΡ€Π°Π²ΠΈΠ»ΠΎ Π±ΡƒΡ€Π°Π²Ρ‡ΠΈΠΊΠ°, ΠΊΠ°ΠΊ Π±Ρ‹ забывая, Ρ‡Ρ‚ΠΎ, согласно элСктродинамикС, “Ρ‚ΠΎΠΊ смСщСния ΠΏΠΎ своСй сути – это ΠΈΠ·ΠΌΠ΅Π½ΡΡŽΡ‰Π΅Π΅ΡΡ со Π²Ρ€Π΅ΠΌΠ΅Π½Π΅ΠΌ элСктричСскоС ΠΏΠΎΠ»Π΅”, Π° Π½Π΅ Π΄Π²ΠΈΠΆΠ΅Π½ΠΈΠ΅ зарядов. ΠŸΡ€Π°Π²ΠΈΠ»ΠΎ Π±ΡƒΡ€Π°Π²Ρ‡ΠΈΠΊΠ°, ΠΊΠ°ΠΊ ΠΈ ΠΏΡ€Π°Π²ΠΈΠ»ΠΎ Π»Π΅Π²ΠΎΠΉ Ρ€ΡƒΠΊΠΈ – Ρ‚ΠΎΠ»ΡŒΠΊΠΎ для двиТущихся зарядов, Ρ‚Π°ΠΊ ΠΊΠ°ΠΊ Ρ‚ΠΎΠΊ смСщСния (Π½Π΅ поляризации) Π½Π΅ отклоняСтся Π² ΠΌΠ°Π³Π½ΠΈΡ‚Π½ΠΎΠΌ ΠΏΠΎΠ»Π΅, ΠΊΠ°ΠΊ, Π½Π°ΠΏΡ€ΠΈΠΌΠ΅Ρ€, ΠΊΠ°Ρ‚ΠΎΠ΄Π½Ρ‹Π΅ Π»ΡƒΡ‡ΠΈ, ΠΈ Π½Π° Π½Π΅Π³ΠΎ Π½Π΅ дСйствуСт сила Π›ΠΎΡ€Π΅Π½Ρ†Π° (АмпСра). НСуТСли Π°Π²Ρ‚ΠΎΡ€Ρ‹ рисунков Π½Π΅ Π·Π½Π°ΡŽΡ‚, Ρ‡Ρ‚ΠΎ ΠΌΠ΅ΠΆΠ΄Ρƒ ΠΎΠ±ΠΊΠ»Π°Π΄ΠΊΠ°ΠΌΠΈ кондСнсатора Π½Π΅Ρ‚ Ρ€Π΅Π°Π»ΡŒΠ½ΠΎΠ³ΠΎ Ρ‚ΠΎΠΊΠ°, Π° Π΅ΡΡ‚ΡŒ Ρ‚ΠΎΠ»ΡŒΠΊΠΎ ΠΈΠ·ΠΌΠ΅Π½ΡΡŽΡ‰Π΅Π΅ΡΡ элСктричСскоС ΠΏΠΎΠ»Π΅, ΠΊΠΎΡ‚ΠΎΡ€ΠΎΠ΅ условно Π½Π°Π·Ρ‹Π²Π°ΡŽΡ‚ Ρ‚ΠΎΠΊΠΎΠΌ смСщСния элСктричСского поля. ΠœΠ°Π³Π½ΠΈΡ‚Π½ΠΎΠ΅ ΠΏΠΎΠ»Π΅ Π² этом случаС опрСдСляСтся ΠΏΠΎ двиТСнию элСктричСского ΠΏΠΎΡ‚ΠΎΠΊΠ° BΒ =Β ΞΌ0[vD].
Β 
Рассмотрим Π΅Ρ‰Π΅ ΠΎΠ΄ΠΈΠ½ ΠΏΡ€ΠΈΠΌΠ΅Ρ€. Π’ΠΎΠ·ΡŒΠΌΠ΅ΠΌ Π΄Π²Π° Ρ†ΠΈΠ»ΠΈΠ½Π΄Ρ€Π°, ΠΎΠ΄ΠΈΠ½ ΠΈΠ· ΠΊΠΎΡ‚ΠΎΡ€Ρ‹Ρ… ΠΈΠΌΠ΅Π΅Ρ‚ элСктричСский заряд, Π° Π΄Ρ€ΡƒΠ³ΠΎΠΉ прСдставляСт собой постоянный ΠΌΠ°Π³Π½ΠΈΡ‚ ΠΈΠ· нСпроводящСго ΠΌΠ°Ρ‚Π΅Ρ€ΠΈΠ°Π»Π°. Если Π·Π°ΠΊΡ€Π΅ΠΏΠΈΡ‚ΡŒ ΠΈΡ… Π½Π° ΠΎΠ΄Π½ΠΎΠΉ оси, проходящСй Ρ‡Π΅Ρ€Π΅Π· Ρ†Π΅Π½Ρ‚Ρ€ Ρ†ΠΈΠ»ΠΈΠ½Π΄Ρ€ΠΎΠ², ΠΊΠ°ΠΊ ΠΈΠ·ΠΎΠ±Ρ€Π°ΠΆΠ΅Π½ΠΎ Π½Π° рисункС, ΠΈ Π½Π°Ρ‡Π°Ρ‚ΡŒ Π²Ρ€Π°Ρ‰Π°Ρ‚ΡŒ (синхронно ΠΈ Π² ΠΎΠ΄Π½ΠΎΠΌ Π½Π°ΠΏΡ€Π°Π²Π»Π΅Π½ΠΈΠΈ), Ρ‚ΠΎ Π² зависимости ΠΎΡ‚ направлСния вращСния Ρ†ΠΈΠ»ΠΈΠ½Π΄Ρ€Ρ‹ Π±ΡƒΠ΄ΡƒΡ‚ Π»ΠΈΠ±ΠΎ ΠΏΡ€ΠΈΡ‚ΡΠ³ΠΈΠ²Π°Ρ‚ΡŒΡΡ, Π»ΠΈΠ±ΠΎ ΠΎΡ‚Ρ‚Π°Π»ΠΊΠΈΠ²Π°Ρ‚ΡŒΡΡ, Ρ‚Π°ΠΊ ΠΊΠ°ΠΊ заряТСнный Ρ†ΠΈΠ»ΠΈΠ½Π΄Ρ€ Π±ΡƒΠ΄Π΅Ρ‚ своим Π²Ρ€Π°Ρ‰Π΅Π½ΠΈΠ΅ΠΌ ΡΠΎΠ·Π΄Π°Π²Π°Ρ‚ΡŒ ΠΊΡ€ΡƒΠ³ΠΎΠ²ΠΎΠΉ элСктричСский Ρ‚ΠΎΠΊ ΠΈ, соотвСтствСнно, ΠΌΠ°Π³Π½ΠΈΡ‚Π½ΠΎΠ΅ ΠΏΠΎΠ»Π΅.

ΠΠ°Ρ€ΡƒΡˆΠ΅Π½ΠΈΠ΅ симмСтрии ΠΌΠ΅ΠΆΠ΄Ρƒ ΠΏΡ€Π°Π²Ρ‹ΠΌ ΠΈ Π»Π΅Π²Ρ‹ΠΌ Π²Ρ€Π°Ρ‰Π΅Π½ΠΈΠ΅ΠΌ Π² элСктромагнитном ΠΏΠΎΠ»Π΅ (ΠΏΠΎΠ»Π΅Π²ΠΎΠΌ пространствС) позволяСт ΠΏΠΎΡΡ‚Ρ€ΠΎΠΈΡ‚ΡŒ элСктромагнитный Π΄Π°Ρ‚Ρ‡ΠΈΠΊ, ΠΈΠ·ΠΌΠ΅Ρ€ΡΡŽΡ‰ΠΈΠΉ Π½Π°ΠΏΡ€Π°Π²Π»Π΅Π½ΠΈΠ΅ ΠΈ ΡΠΊΠΎΡ€ΠΎΡΡ‚ΡŒ вращСния. Π’Ρ€Π°Ρ‰Π°Ρ‚Π΅Π»ΡŒΠ½ΠΎΠ΅ Π΄Π²ΠΈΠΆΠ΅Π½ΠΈΠ΅ ΠΌΠ°Π³Π½ΠΈΡ‚Π°, Π² ΠΎΡ‚Π»ΠΈΡ‡ΠΈΠ΅ ΠΎΡ‚ прямолинСйного двиТСния, Π½Π΅ создаСт Π²ΠΈΡ…Ρ€Π΅Π²ΠΎΠ³ΠΎ элСктричСского поля, Ρ‚.Π΅. ΠΌΠ΅ΠΆΠ΄Ρƒ Π²Ρ€Π°Ρ‰Π°ΡŽΡ‰ΠΈΠΌΠΈΡΡ Ρ†ΠΈΠ»ΠΈΠ½Π΄Ρ€Π°ΠΌΠΈ Π²ΠΎΠ·Π½ΠΈΠΊΠ°Π΅Ρ‚ Ρ‚ΠΎΠ»ΡŒΠΊΠΎ сила Π›ΠΎΡ€Π΅Π½Ρ†Π°, ΠΏΠΎ ΠΊΠΎΡ‚ΠΎΡ€ΠΎΠΉ ΠΌΠΎΠΆΠ½ΠΎ ΠΎΠΏΡ€Π΅Π΄Π΅Π»ΠΈΡ‚ΡŒ Π½Π°ΠΏΡ€Π°Π²Π»Π΅Π½ΠΈΠ΅ ΠΈ ΡΠΊΠΎΡ€ΠΎΡΡ‚ΡŒ вращСния. ΠŸΡ€ΠΈ ΠΎΠ΄Π½ΠΎΠ²Ρ€Π΅ΠΌΠ΅Π½Π½ΠΎΠΌ прямолинСйном Π΄Π²ΠΈΠΆΠ΅Π½ΠΈΠΈ Π²ΠΎΠ·Π½ΠΈΠΊΠ°ΡŽΡ‰Π°Ρ сила Π›ΠΎΡ€Π΅Π½Ρ†Π° ΠΌΠ΅ΠΆΠ΄Ρƒ ΠΌΠ°Π³Π½ΠΈΡ‚ΠΎΠΌ ΠΈ зарядом ΡƒΡ€Π°Π²Π½ΠΎΠ²Π΅ΡˆΠΈΠ²Π°Π΅Ρ‚ΡΡ Π²ΠΈΡ…Ρ€Π΅Π²Ρ‹ΠΌ элСктричСским ΠΏΠΎΠ»Π΅ΠΌ, ΠΊΠΎΡ‚ΠΎΡ€ΠΎΠ΅ создаСт двиТущийся ΠΌΠ°Π³Π½ΠΈΡ‚, образуя Π² пространствС ΠΈΠ·ΠΌΠ΅Π½ΡΡŽΡ‰Π΅Π΅ΡΡ ΠΌΠ°Π³Π½ΠΈΡ‚Π½ΠΎΠ΅ ΠΏΠΎΠ»Π΅ (ΠΈΠ·ΠΌΠ΅Π½ΡΡŽΡ‰ΠΈΠΉΡΡ ΠΌΠ°Π³Π½ΠΈΡ‚Π½Ρ‹ΠΉ ΠΏΠΎΡ‚ΠΎΠΊ). ΠŸΡ€ΠΈ Π²Ρ€Π°Ρ‰Π°Ρ‚Π΅Π»ΡŒΠ½ΠΎΠΌ ΠΆΠ΅ Π΄Π²ΠΈΠΆΠ΅Π½ΠΈΠΈ цилиндричСского ΠΌΠ°Π³Π½ΠΈΡ‚Π° с осью вращСния, проходящСй Ρ‡Π΅Ρ€Π΅Π· полюса, Π²ΠΈΡ…Ρ€Π΅Π²ΠΎΠ΅ элСктричСскоС ΠΏΠΎΠ»Π΅ Π½Π΅ Π²ΠΎΠ·Π½ΠΈΠΊΠ°Π΅Ρ‚, Ρ‚Π°ΠΊ ΠΊΠ°ΠΊ ΠΌΠ°Π³Π½ΠΈΡ‚Π½ΠΎΠ΅ ΠΏΠΎΠ»Π΅ Π² пространствС Π½Π΅ измСняСтся. На этом ΠΏΡ€ΠΈΠ½Ρ†ΠΈΠΏΠ΅ ΠΌΠΎΠ³ΡƒΡ‚ Π΄Π΅ΠΉΡΡ‚Π²ΠΎΠ²Π°Ρ‚ΡŒ Ρ€Π°Π·Π»ΠΈΡ‡Π½Ρ‹Π΅ конструкции Π°Π²Ρ‚ΠΎΠ½ΠΎΠΌΠ½Ρ‹Ρ… элСктромагнитных Π΄Π°Ρ‚Ρ‡ΠΈΠΊΠΎΠ² вращСния ΠΎΡ‚Π½ΠΎΡΠΈΡ‚Π΅Π»ΡŒΠ½ΠΎ ΠΏΠΎΠ»Π΅Π²ΠΎΠ³ΠΎ пространства, для ΠΊΠΎΡ‚ΠΎΡ€Ρ‹Ρ… Π½Π΅ Π½ΡƒΠΆΠ½Ρ‹ внСшниС ΠΎΡ€ΠΈΠ΅Π½Ρ‚ΠΈΡ€Ρ‹, Π½Π°ΠΏΡ€ΠΈΠΌΠ΅Ρ€, Ρ‚Π°ΠΊΠΈΠ΅ Π΄Π°Ρ‚Ρ‡ΠΈΠΊΠΈ ΠΌΠΎΠ³ΡƒΡ‚ Π±Ρ‹Ρ‚ΡŒ ΠΈΡΠΏΠΎΠ»ΡŒΠ·ΠΎΠ²Π°Π½Ρ‹ Π² космосС. Если вмСсто ΠΌΠ°Π³Π½ΠΈΡ‚Π° ΡƒΡΡ‚Π°Π½ΠΎΠ²ΠΈΡ‚ΡŒ компас, Π³Π΄Π΅ стрСлку ΡΠ΄Π΅Π»Π°Ρ‚ΡŒ Π½Π΅ острой, Π° ΠΊΡ€ΡƒΠ³Π»ΠΎΠΉ, Ρ‡Ρ‚ΠΎΠ±Ρ‹ ΠΈΡΠΊΠ»ΡŽΡ‡ΠΈΡ‚ΡŒ Ρ†Π΅Π½Ρ‚Ρ€ΠΎΠ±Π΅ΠΆΠ½ΡƒΡŽ силу, Π²ΠΎΠ·Π½ΠΈΠΊΠ°ΡŽΡ‰ΡƒΡŽ ΠΏΡ€ΠΈ Π²Ρ€Π°Ρ‰Π΅Π½ΠΈΠΈ, Ρ‚ΠΎ, Π² зависимости ΠΎΡ‚ направлСния вращСния, магнитная стрСлка Π±ΡƒΠ΄Π΅Ρ‚ ΠΏΠΎΠ²ΠΎΡ€Π°Ρ‡ΠΈΠ²Π°Ρ‚ΡŒΡΡ Π»ΠΈΠ±ΠΎ Π² ΠΎΠ΄Π½Ρƒ, Π»ΠΈΠ±ΠΎ Π² Π΄Ρ€ΡƒΠ³ΡƒΡŽ сторону. На этом ΠΏΡ€ΠΈΠΌΠ΅Ρ€Π΅ Π²ΠΈΠ΄Π½ΠΎ, Ρ‡Ρ‚ΠΎ элСктромагнитный Π΄Π°Ρ‚Ρ‡ΠΈΠΊ вращСния отличаСтся ΠΎΡ‚ Ρ†Π΅Π½Ρ‚Ρ€ΠΎΠ±Π΅ΠΆΠ½ΠΎΠ³ΠΎ Ρ‚Π΅ΠΌ, Ρ‡Ρ‚ΠΎ ΠΌΠΎΠΆΠ½ΠΎ ΠΎΠΏΡ€Π΅Π΄Π΅Π»ΠΈΡ‚ΡŒ Π½Π°ΠΏΡ€Π°Π²Π»Π΅Π½ΠΈΠ΅ вращСния. Π­Ρ‚ΠΎ являСтся Π½Π°Ρ€ΡƒΡˆΠ΅Π½ΠΈΠ΅ΠΌ симмСтрии ΠΌΠ΅ΠΆΠ΄Ρƒ ΠΏΡ€Π°Π²Ρ‹ΠΌ ΠΈ Π»Π΅Π²Ρ‹ΠΌ Π²Ρ€Π°Ρ‰Π΅Π½ΠΈΠ΅ΠΌ ΠΏΡ€ΠΈ элСктромагнитных взаимодСйствиях (Π½Π° ΠΌΠ°ΠΊΡ€ΠΎΡƒΡ€ΠΎΠ²Π½Π΅).
Β 
Β«ΠŸΠΎΡΡ‚ΠΎΠΌΡƒ Π΅ΡΡ‚ΡŒ основаниС ΡΡ‡ΠΈΡ‚Π°Ρ‚ΡŒ, Ρ‡Ρ‚ΠΎ Π»ΠΈΠ±ΠΎ пространство Π½Π΅ ΠΎΠ±Π»Π°Π΄Π°Π΅Ρ‚ симмСтриСй ΠΌΠ΅ΠΆΠ΄Ρƒ ΠΏΡ€Π°Π²Ρ‹ΠΌ ΠΈ Π»Π΅Π²Ρ‹ΠΌ, Π»ΠΈΠ±ΠΎ эта симмСтрия Π½Π°Ρ€ΡƒΡˆΠ°Π΅Ρ‚ΡΡ Π² ΠΎΠΏΡ€Π΅Π΄Π΅Π»Π΅Π½Π½Ρ‹Ρ… Ρ‚ΠΈΠΏΠ°Ρ… взаимодСйствий …Β»
Π§Π΅Ρ‚Π½ΠΎΡΡ‚ΡŒ. http://bse.sci-lib.com/article122181.html
Β 
ΠŸΠ΅Ρ€Π΅ΠΌΠ΅Π½Π½ΠΎΠ΅ ΠΌΠ°Π³Π½ΠΈΡ‚Π½ΠΎΠ΅ ΠΏΠΎΠ»Π΅ всСгда связано с ΠΏΠ΅Ρ€Π΅ΠΌΠ΅Π½Π½Ρ‹ΠΌ Ρ‚ΠΎΠΊΠΎΠΌ смСщСния, ΠΊΠΎΡ‚ΠΎΡ€Ρ‹ΠΉ проявляСтся Π² Π²ΠΈΠ΄Π΅ Π²ΠΈΡ…Ρ€Π΅Π²ΠΎΠ³ΠΎ элСктричСского поля, поэтому:
Β 
Β«… ΠΏΠ΅Ρ€Π΅ΠΌΠ΅Π½Π½Ρ‹Π΅ элСктричСскиС ΠΈ ΠΌΠ°Π³Π½ΠΈΡ‚Π½Ρ‹Π΅ поля Π½Π΅ ΠΌΠΎΠ³ΡƒΡ‚ ΡΡƒΡ‰Π΅ΡΡ‚Π²ΠΎΠ²Π°Ρ‚ΡŒ Π΄Ρ€ΡƒΠ³ Π±Π΅Π· Π΄Ρ€ΡƒΠ³Π°Β …Β»
ЭнциклопСдия элСмСнтарной Ρ„ΠΈΠ·ΠΈΠΊΠΈ. Π­Π›Π•ΠšΠ’Π ΠžΠœΠΠ“ΠΠ˜Π’ΠΠžΠ• ΠŸΠžΠ›Π•.
«ВихрСвая ΡΠΎΡΡ‚Π°Π²Π»ΡΡŽΡ‰Π°Ρ элСктричСского поля Π²ΠΎΠ·Π½ΠΈΠΊΠ°Π΅Ρ‚ ΠΏΡ€ΠΈ ΠΈΠ·ΠΌΠ΅Π½Π΅Π½ΠΈΠΈ Π²ΠΎ Π²Ρ€Π΅ΠΌΠ΅Π½ΠΈ ΠΌΠ°Π³Π½ΠΈΡ‚Π½ΠΎΠ³ΠΎ поля:Β …Β»
ЀизичСская энциклопСдия. Π­Π›Π•ΠšΠ’Π Π˜Π§Π•Π‘ΠšΠžΠ• ΠŸΠžΠ›Π•.
Β 
Π’ΠΈΡ…Ρ€Π΅Π²ΠΎΠ΅ элСктричСскоС ΠΏΠΎΠ»Π΅ – это Π²ΠΈΡ…Ρ€Π΅Π²ΠΎΠΉ ΠΏΠΎΡ‚ΠΎΠΊ элСктричСского смСщСния поля, Ρ‡Ρ‚ΠΎ прСдставляСт ΠΏΠ΅Ρ€Π΅ΠΌΠ΅Π½Π½Ρ‹ΠΉ Ρ‚ΠΎΠΊ смСщСния. ΠŸΠΎΡΡ‚ΠΎΡΠ½Π½ΠΎΠ΅ ΠΌΠ°Π³Π½ΠΈΡ‚Π½ΠΎΠ΅ ΠΏΠΎΠ»Π΅ – это постоянный Ρ‚ΠΎΠΊ смСщСния, Π½Π΅ ΠΏΡ€Π΅Π΄ΡΡ‚Π°Π²Π»ΡΡŽΡ‰ΠΈΠΉ Π²ΠΈΡ…Ρ€Π΅Π²ΠΎΠ΅ элСктричСскоС ΠΏΠΎΠ»Π΅, поэтому ΠΎΠ½ΠΎ Π½Π΅ дСйствуСт Π½Π° покоящиСся элСктричСскиС заряды. ΠŸΠΎΡΡ‚ΠΎΡΠ½Π½Ρ‹ΠΉ Ρ‚ΠΎΠΊ смСщСния, Ρ‚Π°ΠΊ ΠΆΠ΅ ΠΊΠ°ΠΊ ΠΈ свСрхпроводящий Ρ‚ΠΎΠΊ, Π½Π΅ создаСт элСктричСской напряТСнности поля.
Β 
Π₯отя ΠΏΡ€ΠΈΠ²Π΅Π΄Π΅Π½Π½ΠΎΠ΅ описаниС процСссов Π½Π΅ являСтся достаточно ΠΏΠΎΠ»Π½Ρ‹ΠΌ ΠΈ Π±Π΅Π·ΡƒΠΏΡ€Π΅Ρ‡Π½Ρ‹ΠΌ, ΠΎΠ½ΠΎ Π΄Π°Π΅Ρ‚ прСдставлСниС ΠΎ ΠΌΠ΅Ρ…Π°Π½ΠΈΠ·ΠΌΠ΅ элСктромагнитной ΠΈΠ½Π΄ΡƒΠΊΡ†ΠΈΠΈ. Π‘ Π΄Ρ€ΡƒΠ³ΠΎΠΉ стороны, Π±ΠΎΠ»Π΅Π΅ ΠΏΡ€ΠΈΠ²Ρ‹Ρ‡Π½ΠΎ ΠΏΡ€Π΅Π΄ΡΡ‚Π°Π²Π»ΡΡ‚ΡŒ элСктродинамичСскиС взаимодСйствия Ρ‡Π΅Ρ€Π΅Π· Π΄ΠΎΠΏΠΎΠ»Π½ΠΈΡ‚Π΅Π»ΡŒΠ½ΡƒΡŽ характСристику – ΠΈΠ½Π΄ΡƒΠΊΡ†ΠΈΡŽ ΠΌΠ°Π³Π½ΠΈΡ‚Π½ΠΎΠ³ΠΎ поля, ΠΎΡ‚ΡΡŽΠ΄Π° Π½Π°Π·Π²Π°Π½ΠΈΠ΅ – элСктромагнитныС взаимодСйствия, хотя Ρ€Π΅Π°Π»ΡŒΠ½ΠΎ Π² ΠΏΡ€ΠΈΡ€ΠΎΠ΄Π΅ сущСствуСт Ρ‚ΠΎΠ»ΡŒΠΊΠΎ элСктричСскоС ΠΏΠΎΠ»Π΅, Π° ΠΌΠ°Π³Π½ΠΈΡ‚Π½ΠΎΠ΅ ΠΏΠΎΠ»Π΅ ΠΎΠ±Ρ€Π°Π·ΠΎΠ²Π°Π½ΠΎ двиТущимися элСктричСскими ΠΏΠΎΡ‚ΠΎΠΊΠ°ΠΌΠΈ ΠΈ связанными с Π½ΠΈΠΌΠΈ Ρ‚ΠΎΠΊΠ°ΠΌΠΈ смСщСния поля.
Β 
Β«Π’Π°ΠΊΠΈΠΌ ΠΎΠ±Ρ€Π°Π·ΠΎΠΌ, появлСниС ΠΌΠ°Π³Π½ΠΈΡ‚Π½ΠΎΠ³ΠΎ поля Ρ‚ΠΎΠΊΠΎΠ² Π΅ΡΡ‚ΡŒ чисто рСлятивистский эффСкт ΠΈ Π½ΠΈΠΊΠ°ΠΊΠΎΠΉ Π½ΠΎΠ²ΠΎΠΉ физичСской субстанции (Π½Π°ΠΏΡ€ΠΈΠΌΠ΅Ρ€, Π² Π²ΠΈΠ΄Π΅ ΠΌΠ°Π³Π½ΠΈΡ‚Π½Ρ‹Ρ… зарядов) ΠΏΠΎΡΠ²Π»ΡΡ‚ΡŒΡΡ Π½Π΅ Π΄ΠΎΠ»ΠΆΠ½ΠΎ, Ρ‡Ρ‚ΠΎ ΠΈ подтвСрТдаСтся ΡΠΊΡΠΏΠ΅Ρ€ΠΈΠΌΠ΅Π½Ρ‚Π°Π»ΡŒΠ½ΠΎ.Β»
ΠžΡΠ½ΠΎΠ²Ρ‹ Ρ„ΠΈΠ·ΠΈΠΊΠΈ. Π›.А.Π“Ρ€ΠΈΠ±ΠΎΠ², Н.И.ΠŸΡ€ΠΎΠΊΠΎΡ„ΡŒΠ΅Π². 1995. Π‘.299.
Β 
Π’Π°ΠΊ ΠΊΠ°ΠΊ ΠΌΠ°Π³Π½ΠΈΡ‚Π½ΠΎΠ΅ взаимодСйствиС прСдставляСт элСктродинамичСский процСсс, для ΠΌΠ°Π³Π½ΠΈΡ‚Π½ΠΎΠ³ΠΎ поля большС ΠΏΠΎΠ΄Ρ…ΠΎΠ΄ΠΈΡ‚ Ρ‚Π΅Ρ€ΠΌΠΈΠ½ “элСктродинамичСский эффСкт”. Но нСсмотря Π½Π° это, Ρ‡Ρ‚ΠΎΠ±Ρ‹ Π½Π΅ Π²ΠΎΠ·Π½ΠΈΠΊΠ°Π»Π° ΠΏΡƒΡ‚Π°Π½ΠΈΡ†Π°, Π² тСкстС сохранСна привычная тСрминология, Ρ‚.Π΅. ΠΈΡΠΏΠΎΠ»ΡŒΠ·ΡƒΠ΅Ρ‚ΡΡ Ρ‚Π΅Ρ€ΠΌΠΈΠ½ “рСлятивистский” эффСкт, Π° Π½Π΅ “элСктродинамичСский”.
Β 
Надо Π·Π°ΠΌΠ΅Ρ‚ΠΈΡ‚ΡŒ, ΠΈΠ½ΠΎΠ³Π΄Π° Π²ΠΎΠ·Π½ΠΈΠΊΠ½ΠΎΠ²Π΅Π½ΠΈΠ΅ ΠΌΠ°Π³Π½ΠΈΡ‚Π½ΠΎΠ³ΠΎ поля ΠΏΡ‹Ρ‚Π°ΡŽΡ‚ΡΡ ΠΎΠ±ΡŠΡΡΠ½ΠΈΡ‚ΡŒ Ρ‚Π΅ΠΌ, Ρ‡Ρ‚ΠΎ ΠΏΡ€ΠΈ Π΄Π²ΠΈΠΆΠ΅Π½ΠΈΠΈ зарядов Π½Π°ΠΏΡ€ΡΠΆΠ΅Π½Π½ΠΎΡΡ‚ΡŒ элСктричСского поля Π² Π½Π°ΠΏΡ€Π°Π²Π»Π΅Π½ΠΈΠΈ, пСрпСндикулярном двиТСнию, возрастаСт ΠΏΠΎ ΠΎΡ‚Π½ΠΎΡˆΠ΅Π½ΠΈΡŽ ΠΊ покоящимся зарядам.
Β 
Β«… ΠΏΡ€ΠΈ Π΄Π²ΠΈΠΆΠ΅Π½ΠΈΠΈ плоскости создаваСмоС Сю элСктричСскоС ΠΏΠΎΠ»Π΅ Π² Π½Π°ΠΏΡ€Π°Π²Π»Π΅Π½ΠΈΠΈ, пСрпСндикулярном двиТСнию, Π΄ΠΎΠ»ΠΆΠ½ΠΎ возрасти.Β»
ΠžΡΠ½ΠΎΠ²Ρ‹ Ρ„ΠΈΠ·ΠΈΠΊΠΈ. Π›.А.Π“Ρ€ΠΈΠ±ΠΎΠ², Н.И.ΠŸΡ€ΠΎΠΊΠΎΡ„ΡŒΠ΅Π². 1995. Π‘.301.
Β 
ΠŸΡ€ΠΈΠ²ΠΎΠ΄Ρ Ρ‚Π°ΠΊΠΈΠ΅ ΠΈΠ½Ρ‚Π΅Ρ€ΠΏΡ€Π΅Ρ‚Π°Ρ†ΠΈΠΈ, всСгда ΠΊΠ°ΠΊ Π±Ρ‹ Π·Π°Π±Ρ‹Π²Π°ΡŽΡ‚ Ρ€Π°ΡΡΠΌΠΎΡ‚Ρ€Π΅Ρ‚ΡŒ симмСтричноС Π΄Π²ΠΈΠΆΠ΅Π½ΠΈΠ΅ Ρ€Π°Π·Π½ΠΎΠΈΠΌΠ΅Π½Π½Ρ‹Ρ… зарядов. НапримСр, Π΄Π²Π΅ Ρ€Π°Π·Π½ΠΎΠΈΠΌΠ΅Π½Π½ΠΎ заряТСнныС плоскости ΠΎΠ΄Π½ΠΎΠ²Ρ€Π΅ΠΌΠ΅Π½Π½ΠΎ двиТутся Π² ΠΏΡ€ΠΎΡ‚ΠΈΠ²ΠΎΠΏΠΎΠ»ΠΎΠΆΠ½Ρ‹Ρ… направлСниях, ΠΏΡ€ΠΈ этом всС Ρ€Π°Π²Π½ΠΎ Π²ΠΎΠ·Π½ΠΈΠΊΠ°Π΅Ρ‚ ΠΌΠ°Π³Π½ΠΈΡ‚Π½ΠΎΠ΅ ΠΏΠΎΠ»Π΅, Ρ‚.Π΅., Ссли пСрпСндикулярно плоскостям двиТСтся заряд, Ρ‚ΠΎ Π½Π° Π½Π΅Π³ΠΎ Π±ΡƒΠ΄Π΅Ρ‚ Π΄Π΅ΠΉΡΡ‚Π²ΠΎΠ²Π°Ρ‚ΡŒ сила Π›ΠΎΡ€Π΅Π½Ρ†Π°. Π’Π°ΠΊΠΈΠΌ ΠΎΠ±Ρ€Π°Π·ΠΎΠΌ, нСльзя ΠΎΠ±ΡŠΡΡΠ½ΠΈΡ‚ΡŒ Π²ΠΎΠ·Π½ΠΈΠΊΠ½ΠΎΠ²Π΅Π½ΠΈΠ΅ ΠΌΠ°Π³Π½ΠΈΡ‚Π½ΠΎΠ³ΠΎ поля ΠΊΠ°ΠΊ возрастаниС элСктричСской напряТСнности поля двиТущихся зарядов. ΠŸΠΎΡΡ‚ΠΎΠΌΡƒ для ΠΌΠ°Π³Π½ΠΈΡ‚Π½ΠΎΠ³ΠΎ поля ΠΏΡ€Π°Π²ΠΈΠ»ΡŒΠ½Π΅Π΅ Π²Π΅Ρ€Π½ΡƒΡ‚ΡŒΡΡ ΠΊ старой Ρ‚Π΅Ρ€ΠΌΠΈΠ½ΠΎΠ»ΠΎΠ³ΠΈΠΈ - “элСктродинамичСскоС взаимодСйствиС” ΠΈΠ»ΠΈ “элСктродинамичСский эффСкт”.
Β 
Β«Π―Π²Π»Π΅Π½ΠΈΠ΅ взаимодСйствия элСктричСских Ρ‚ΠΎΠΊΠΎΠ² АмпСр Π½Π°Π·Ρ‹Π²Π°Π» элСктродинамичСским взаимодСйствиСм.Β»
Π€ΠΈΠ·ΠΈΠΊΠ°. О.Π€.ΠšΠ°Π±Π°Ρ€Π΄ΠΈΠ½. 1991. Π‘.177.

ΠŸΡ€ΠΎΠ΄ΠΎΠ»ΠΆΠ΅Π½ΠΈΠ΅ Π½Π° сайтС:Β  http://alemanow.narod.ru
Β 
Π—Π°Π³Ρ€ΡƒΠ·ΠΈΡ‚ΡŒ ΠΏΠΎΠ»Π½Ρ‹ΠΉ тСкст ΠΊΠ½ΠΈΠ³ΠΈ Π² ΡƒΠΏΠ°ΠΊΠΎΠ²Π°Π½Π½ΠΎΠΌ Π²ΠΈΠ΄Π΅
http://alemanow.narod.ru/theory.zip


ЛСвитация Π³Ρ€Π°Ρ„ΠΈΡ‚ΠΎΠ²ΠΎΠΉ Ρ„ΠΎΠ»ΡŒΠ³ΠΈ ΠΈ ΠΊΠ°Ρ€Π°Π½Π΄Π°ΡˆΠ½ΠΎΠ³ΠΎ грифСля 0,5 ΠΌΠΌ Π½Π°Π΄ ΠΌΠ°Π³Π½ΠΈΡ‚Π°ΠΌΠΈ!

Π‘Π»ΡƒΡ‡Π°ΠΉΠ½ΠΎ ΠΎΠ±Π½Π°Ρ€ΡƒΠΆΠΈΠ», Ρ‡Ρ‚ΠΎ обычная графитовая Ρ„ΠΎΠ»ΡŒΠ³Π°, ΠΈΠ· ΠΊΠΎΡ‚ΠΎΡ€ΠΎΠΉ Π΄Π΅Π»Π°ΡŽΡ‚ ΠΏΡ€ΠΎΠΊΠ»Π°Π΄ΠΊΠΈ для Π³Π΅Ρ€ΠΌΠ΅Ρ‚ΠΈΠ·Π°Ρ†ΠΈΠΈ, прСкрасно Π»Π΅Π²ΠΈΡ‚ΠΈΡ€ΡƒΠ΅Ρ‚ Π½Π°Π΄ ΠΌΠ°Π³Π½ΠΈΡ‚Π°ΠΌΠΈ (ΠΊΠ°ΠΊ свСрхпроводник). Бвязался с производитСлями Ρ„ΠΎΠ»ΡŒΠ³ΠΈ, ΠΎΠ½ΠΈ Π²ΠΎΠΎΠ±Ρ‰Π΅ Π½Π΅ Π·Π½Π°Π»ΠΈ Π½ΠΈ ΠΏΡ€ΠΎ ΠΊΠ°ΠΊΡƒΡŽ Π»Π΅Π²ΠΈΡ‚Π°Ρ†ΠΈΡŽ. Показал ΠΈΠΌ Π²ΠΈΠ΄Π΅ΠΎ, ΠΎΠ½ΠΈ Π±Ρ‹Π»ΠΈ ΡƒΠ΄ΠΈΠ²Π»Π΅Π½Ρ‹. Π”Π°Π»ΠΈ лист Π³Ρ€Π°Ρ„ΠΈΡ‚ΠΎΠ²ΠΎΠΉ Ρ„ΠΎΠ»ΡŒΠ³ΠΈ для экспСримСнтов ΠΏΠΎ ΠΌΠ°Π³Π½ΠΈΡ‚Π½ΠΎΠΉ Π°Π½Ρ‚ΠΈΠ³Ρ€Π°Π²ΠΈΡ‚Π°Ρ†ΠΈΠΈ.

Π›Π΅Π²ΠΈΡ‚Ρ€ΠΎΠ½ (Π»Π΅Ρ‚Π°ΡŽΡ‰ΠΈΠΉ Π²ΠΎΠ»Ρ‡ΠΎΠΊ). ΠšΠΈΡ‚Π°ΠΉΡΠΊΠ°Ρ ΠΈΠ³Ρ€ΡƒΡˆΠΊΠ°.

Π’ΠΎΠ»Ρ‡ΠΎΠΊ, ΠΊΠΎΡ‚ΠΎΡ€Ρ‹ΠΉ, Π²Ρ€Π°Ρ‰Π°ΡΡΡŒ, висит Π² Π²ΠΎΠ·Π΄ΡƒΡ…Π΅ Π·Π° счСт ΠΌΠ°Π³Π½ΠΈΡ‚Π½Ρ‹Ρ… сил.


ЭлСктромагнитная индукция | ΠœΠ°Π³Π½Π΅Ρ‚ΠΈΠ·ΠΌ ΠΈ элСктромагнСтизм

Π’ Ρ‚ΠΎ врСмя ΠΊΠ°ΠΊ ΡƒΠ΄ΠΈΠ²ΠΈΡ‚Π΅Π»ΡŒΠ½ΠΎΠ΅ ΠΎΡ‚ΠΊΡ€Ρ‹Ρ‚ΠΈΠ΅ элСктромагнСтизма ЭрстСдом ΠΏΡ€ΠΎΠ»ΠΎΠΆΠΈΠ»ΠΎ ΠΏΡƒΡ‚ΡŒ ΠΊ Π±ΠΎΠ»Π΅Π΅ практичСским прилоТСниям элСктричСства, ΠΈΠΌΠ΅Π½Π½ΠΎ Майкл Π€Π°Ρ€Π°Π΄Π΅ΠΉ Π΄Π°Π» Π½Π°ΠΌ ΠΊΠ»ΡŽΡ‡ ΠΊ практичСскому поколСния элСктричСства: элСктромагнитной ΠΈΠ½Π΄ΡƒΠΊΡ†ΠΈΠΈ . Π€Π°Ρ€Π°Π΄Π΅ΠΉ ΠΎΠ±Π½Π°Ρ€ΡƒΠΆΠΈΠ», Ρ‡Ρ‚ΠΎ напряТСниС Π±ΡƒΠ΄Π΅Ρ‚ Π³Π΅Π½Π΅Ρ€ΠΈΡ€ΠΎΠ²Π°Ρ‚ΡŒΡΡ Π½Π° Π΄Π»ΠΈΠ½Π΅ ΠΏΡ€ΠΎΠ²ΠΎΠ΄Π°, Ссли Π½Π° этот ΠΏΡ€ΠΎΠ²ΠΎΠ΄ Π²ΠΎΠ·Π΄Π΅ΠΉΡΡ‚Π²ΠΎΠ²Π°Ρ‚ΡŒ пСрпСндикулярным ΠΏΠΎΡ‚ΠΎΠΊΠΎΠΌ ΠΌΠ°Π³Π½ΠΈΡ‚Π½ΠΎΠ³ΠΎ поля ΠΈΠ·ΠΌΠ΅Π½ΡΡŽΡ‰Π΅ΠΉΡΡ интСнсивности.

ΠŸΡ€ΠΎΡΡ‚ΠΎΠΉ способ ΡΠΎΠ·Π΄Π°Ρ‚ΡŒ ΠΌΠ°Π³Π½ΠΈΡ‚Π½ΠΎΠ΅ ΠΏΠΎΠ»Π΅ ΠΈΠ·ΠΌΠ΅Π½ΡΡŽΡ‰Π΅ΠΉΡΡ интСнсивности – ΠΏΠ΅Ρ€Π΅ΠΌΠ΅ΡΡ‚ΠΈΡ‚ΡŒ постоянный ΠΌΠ°Π³Π½ΠΈΡ‚ рядом с ΠΏΡ€ΠΎΠ²ΠΎΠ»ΠΎΠΊΠΎΠΉ ΠΈΠ»ΠΈ ΠΊΠ°Ρ‚ΡƒΡˆΠΊΠΎΠΉ с ΠΏΡ€ΠΎΠ²ΠΎΠ»ΠΎΠΊΠΎΠΉ.

ΠŸΠΎΠΌΠ½ΠΈΡ‚Π΅: ΠœΠ°Π³Π½ΠΈΡ‚Π½ΠΎΠ΅ ΠΏΠΎΠ»Π΅ Π΄ΠΎΠ»ΠΆΠ½ΠΎ ΡƒΠ²Π΅Π»ΠΈΡ‡ΠΈΠ²Π°Ρ‚ΡŒΡΡ ΠΈΠ»ΠΈ ΡƒΠΌΠ΅Π½ΡŒΡˆΠ°Ρ‚ΡŒΡΡ ΠΏΠΎ напряТСнности пСрпСндикулярно ΠΏΡ€ΠΎΠ²ΠΎΠ΄Ρƒ (Ρ‚Π°ΠΊ, Ρ‡Ρ‚ΠΎΠ±Ρ‹ Π»ΠΈΠ½ΠΈΠΈ ΠΏΠΎΡ‚ΠΎΠΊΠ° «пСрСсСкали» ΠΏΡ€ΠΎΠ²ΠΎΠ΄ ), ΠΈΠ½Π°Ρ‡Π΅ Π½Π΅ Π±ΡƒΠ΄Π΅Ρ‚ ΠΈΠ½Π΄ΡƒΡ†ΠΈΡ€ΠΎΠ²Π°Ρ‚ΡŒΡΡ напряТСниС.

Π€Π°Ρ€Π°Π΄Π΅ΠΉ смог матСматичСски ΡΠ²ΡΠ·Π°Ρ‚ΡŒ ΡΠΊΠΎΡ€ΠΎΡΡ‚ΡŒ измСнСния ΠΏΠΎΡ‚ΠΎΠΊΠ° ΠΌΠ°Π³Π½ΠΈΡ‚Π½ΠΎΠ³ΠΎ поля с Π½Π°Π²Π΅Π΄Π΅Π½Π½Ρ‹ΠΌ напряТСниСм (ΠΎΠ±Ρ€Π°Ρ‚ΠΈΡ‚Π΅ Π²Π½ΠΈΠΌΠ°Π½ΠΈΠ΅ Π½Π° использованиС строчной Π±ΡƒΠΊΠ²Ρ‹ Β«Π΅Β» для обозначСния напряТСния.Π­Ρ‚ΠΎ относится ΠΊ ΠΌΠ³Π½ΠΎΠ²Π΅Π½Π½ΠΎΠΌΡƒ Π½Π°ΠΏΡ€ΡΠΆΠ΅Π½ΠΈΡŽ ΠΈΠ»ΠΈ Π½Π°ΠΏΡ€ΡΠΆΠ΅Π½ΠΈΡŽ Π² ΠΎΠΏΡ€Π΅Π΄Π΅Π»Π΅Π½Π½Ρ‹ΠΉ ΠΌΠΎΠΌΠ΅Π½Ρ‚ Π²Ρ€Π΅ΠΌΠ΅Π½ΠΈ, Π° Π½Π΅ ΠΊ постоянному ΡΡ‚Π°Π±ΠΈΠ»ΡŒΠ½ΠΎΠΌΡƒ Π½Π°ΠΏΡ€ΡΠΆΠ΅Π½ΠΈΡŽ.):

Π’Π΅Ρ€ΠΌΠΈΠ½ Β«dΒ» прСдставляСт собой ΡΡ‚Π°Π½Π΄Π°Ρ€Ρ‚Π½ΡƒΡŽ Π½ΠΎΡ‚Π°Ρ†ΠΈΡŽ расчСтов, ΠΏΡ€Π΅Π΄ΡΡ‚Π°Π²Π»ΡΡŽΡ‰ΡƒΡŽ ΡΠΊΠΎΡ€ΠΎΡΡ‚ΡŒ измСнСния ΠΏΠΎΡ‚ΠΎΠΊΠ° Π²ΠΎ Π²Ρ€Π΅ΠΌΠ΅Π½ΠΈ. Β«NΒ» ΠΎΠ±ΠΎΠ·Π½Π°Ρ‡Π°Π΅Ρ‚ количСство Π²ΠΈΡ‚ΠΊΠΎΠ² ΠΈΠ»ΠΈ Π²ΠΈΡ‚ΠΊΠΎΠ² Π² ΠΊΠ°Ρ‚ΡƒΡˆΠΊΠ΅ с ΠΏΡ€ΠΎΠ²ΠΎΠ»ΠΎΠΊΠΎΠΉ (прСдполагаСтся, Ρ‡Ρ‚ΠΎ ΠΏΡ€ΠΎΠ²ΠΎΠ»ΠΎΠΊΠ° ΠΈΠΌΠ΅Π΅Ρ‚ Ρ„ΠΎΡ€ΠΌΡƒ ΠΊΠ°Ρ‚ΡƒΡˆΠΊΠΈ для максимальной элСктромагнитной эффСктивности).

Π­Ρ‚ΠΎ явлСниС ΠΈΡΠΏΠΎΠ»ΡŒΠ·ΡƒΠ΅Ρ‚ΡΡ Π² конструкции элСктричСских Π³Π΅Π½Π΅Ρ€Π°Ρ‚ΠΎΡ€ΠΎΠ², ΠΊΠΎΡ‚ΠΎΡ€Ρ‹Π΅ ΠΈΡΠΏΠΎΠ»ΡŒΠ·ΡƒΡŽΡ‚ ΠΌΠ΅Ρ…Π°Π½ΠΈΡ‡Π΅ΡΠΊΡƒΡŽ ΡΠ½Π΅Ρ€Π³ΠΈΡŽ для пСрСмСщСния ΠΌΠ°Π³Π½ΠΈΡ‚Π½ΠΎΠ³ΠΎ поля ΠΌΠΈΠΌΠΎ ΠΊΠ°Ρ‚ΡƒΡˆΠ΅ΠΊ с ΠΏΡ€ΠΎΠ²ΠΎΠ»ΠΎΠΊΠΎΠΉ для Π³Π΅Π½Π΅Ρ€Π°Ρ†ΠΈΠΈ напряТСния.Однако это Π΄Π°Π»Π΅ΠΊΠΎ Π½Π΅ СдинствСнноС практичСскоС ΠΏΡ€ΠΈΠΌΠ΅Π½Π΅Π½ΠΈΠ΅ этого ΠΏΡ€ΠΈΠ½Ρ†ΠΈΠΏΠ°.

Если ΠΌΡ‹ вспомним, Ρ‡Ρ‚ΠΎ ΠΌΠ°Π³Π½ΠΈΡ‚Π½ΠΎΠ΅ ΠΏΠΎΠ»Π΅, создаваСмоС Ρ‚ΠΎΠΊΠΎΠ²Π΅Π΄ΡƒΡ‰ΠΈΠΌ ΠΏΡ€ΠΎΠ²ΠΎΠ΄ΠΎΠΌ, всСгда пСрпСндикулярно этому ΠΏΡ€ΠΎΠ²ΠΎΠ΄Ρƒ ΠΈ Ρ‡Ρ‚ΠΎ сила ΠΏΠΎΡ‚ΠΎΠΊΠ° этого ΠΌΠ°Π³Π½ΠΈΡ‚Π½ΠΎΠ³ΠΎ поля измСняСтся Π² зависимости ΠΎΡ‚ Π²Π΅Π»ΠΈΡ‡ΠΈΠ½Ρ‹ Ρ‚ΠΎΠΊΠ°, проходящСго Ρ‡Π΅Ρ€Π΅Π· Π½Π΅Π³ΠΎ, ΠΌΡ‹ ΠΌΠΎΠΆΠ΅ΠΌ Π²ΠΈΠ΄Π΅Ρ‚ΡŒ, Ρ‡Ρ‚ΠΎ ΠΏΡ€ΠΎΠ²ΠΎΠ΄ способСн ΠΈΠ½Π΄ΡƒΡ†ΠΈΡ€ΠΎΠ²Π°Ρ‚ΡŒ напряТСниС Π½Π° Π΅Π³ΠΎ собствСнной Π΄Π»ΠΈΠ½Π΅ просто ΠΈΠ·-Π·Π° измСнСния Ρ‚ΠΎΠΊΠ° Ρ‡Π΅Ρ€Π΅Π· Π½Π΅Π³ΠΎ. Π­Ρ‚ΠΎΡ‚ эффСкт называСтся самоиндукциСй : ΠΈΠ·ΠΌΠ΅Π½ΡΡŽΡ‰Π΅Π΅ΡΡ ΠΌΠ°Π³Π½ΠΈΡ‚Π½ΠΎΠ΅ ΠΏΠΎΠ»Π΅, создаваСмоС измСнСниями Ρ‚ΠΎΠΊΠ° Ρ‡Π΅Ρ€Π΅Π· ΠΏΡ€ΠΎΠ²ΠΎΠ΄, ΠΈΠ½Π΄ΡƒΡ†ΠΈΡ€ΡƒΡŽΡ‰Π΅Π΅ напряТСниС ΠΏΠΎ Π΄Π»ΠΈΠ½Π΅ Ρ‚ΠΎΠ³ΠΎ ΠΆΠ΅ ΠΏΡ€ΠΎΠ²ΠΎΠ΄Π°.Если ΠΏΠΎΡ‚ΠΎΠΊ ΠΌΠ°Π³Π½ΠΈΡ‚Π½ΠΎΠ³ΠΎ поля усиливаСтся ΠΏΡƒΡ‚Π΅ΠΌ сгибания ΠΏΡ€ΠΎΠ²ΠΎΠ»ΠΎΠΊΠΈ Π² Ρ„ΠΎΡ€ΠΌΠ΅ ΠΊΠ°Ρ‚ΡƒΡˆΠΊΠΈ ΠΈ / ΠΈΠ»ΠΈ наматывания этой ΠΊΠ°Ρ‚ΡƒΡˆΠΊΠΈ Π½Π° ΠΌΠ°Ρ‚Π΅Ρ€ΠΈΠ°Π» с высокой ΠΏΡ€ΠΎΠ½ΠΈΡ†Π°Π΅ΠΌΠΎΡΡ‚ΡŒΡŽ, этот эффСкт самоиндуцированного напряТСния Π±ΡƒΠ΄Π΅Ρ‚ Π±ΠΎΠ»Π΅Π΅ интСнсивным. Устройство, созданноС для использования этого эффСкта, называСтся ΠΊΠ°Ρ‚ΡƒΡˆΠΊΠΎΠΉ индуктивности ΠΈ Π±ΡƒΠ΄Π΅Ρ‚ ΠΎΠ±ΡΡƒΠΆΠ΄Π°Ρ‚ΡŒΡΡ Π±ΠΎΠ»Π΅Π΅ ΠΏΠΎΠ΄Ρ€ΠΎΠ±Π½ΠΎ Π² ΡΠ»Π΅Π΄ΡƒΡŽΡ‰Π΅ΠΉ Π³Π»Π°Π²Π΅.

ΠžΠ‘Π—ΠžΠ :

  • ΠœΠ°Π³Π½ΠΈΡ‚Π½ΠΎΠ΅ ΠΏΠΎΠ»Π΅ ΠΈΠ·ΠΌΠ΅Π½ΡΡŽΡ‰Π΅ΠΉΡΡ интСнсивности пСрпСндикулярно ΠΏΡ€ΠΎΠ²ΠΎΠ΄Ρƒ Π±ΡƒΠ΄Π΅Ρ‚ ΠΈΠ½Π΄ΡƒΡ†ΠΈΡ€ΠΎΠ²Π°Ρ‚ΡŒ напряТСниС ΠΏΠΎ всСй Π΄Π»ΠΈΠ½Π΅ этого ΠΏΡ€ΠΎΠ²ΠΎΠ΄Π°.Π’Π΅Π»ΠΈΡ‡ΠΈΠ½Π° ΠΈΠ½Π΄ΡƒΡ†ΠΈΡ€ΠΎΠ²Π°Π½Π½ΠΎΠ³ΠΎ напряТСния зависит ΠΎΡ‚ скорости измСнСния ΠΏΠΎΡ‚ΠΎΠΊΠ° ΠΌΠ°Π³Π½ΠΈΡ‚Π½ΠΎΠ³ΠΎ поля ΠΈ количСства Π²ΠΈΡ‚ΠΊΠΎΠ² ΠΏΡ€ΠΎΠ²ΠΎΠ»ΠΎΠΊΠΈ (Ссли ΠΎΠ½Π° свСрнута), ΠΏΠΎΠ΄Π²Π΅Ρ€ΠΆΠ΅Π½Π½Ρ‹Ρ… измСнСнию ΠΌΠ°Π³Π½ΠΈΡ‚Π½ΠΎΠ³ΠΎ ΠΏΠΎΡ‚ΠΎΠΊΠ°.
  • Π£Ρ€Π°Π²Π½Π΅Π½ΠΈΠ΅ ЀарадСя для ΠΈΠ½Π΄ΡƒΡ†ΠΈΡ€ΠΎΠ²Π°Π½Π½ΠΎΠ³ΠΎ напряТСния: e = N (dΞ¦ / dt)
  • Π’ΠΎΠΊΠΎΠ²Π΅Π΄ΡƒΡ‰ΠΈΠΉ ΠΏΡ€ΠΎΠ²ΠΎΠ΄ Π±ΡƒΠ΄Π΅Ρ‚ ΠΈΡΠΏΡ‹Ρ‚Ρ‹Π²Π°Ρ‚ΡŒ Π½Π°Π²Π΅Π΄Π΅Π½Π½ΠΎΠ΅ напряТСниС ΠΏΠΎ всСй Π΅Π³ΠΎ Π΄Π»ΠΈΠ½Π΅, Ссли Ρ‚ΠΎΠΊ измСнится (Ρ‚Π°ΠΊΠΈΠΌ ΠΎΠ±Ρ€Π°Π·ΠΎΠΌ, измСнится ΠΏΠΎΡ‚ΠΎΠΊ ΠΌΠ°Π³Π½ΠΈΡ‚Π½ΠΎΠ³ΠΎ поля, пСрпСндикулярного ΠΏΡ€ΠΎΠ²ΠΎΠ΄Ρƒ, ΠΈ Π²ΠΎΠ·Π½ΠΈΠΊΠ½Π΅Ρ‚ напряТСниС Π² соотвСтствии с Ρ„ΠΎΡ€ΠΌΡƒΠ»ΠΎΠΉ ЀарадСя). Устройство, созданноС ΡΠΏΠ΅Ρ†ΠΈΠ°Π»ΡŒΠ½ΠΎ для использования этого эффСкта, называСтся ΠΈΠ½Π΄ΡƒΠΊΡ‚ΠΎΡ€ΠΎΠΌ .

БВЯЗАННЫЕ Π ΠΠ‘ΠžΠ§Π˜Π• Π›Π˜Π‘Π’Π«:

Π—Π°ΠΊΠΎΠ½ элСктромагнитной ΠΈΠ½Π΄ΡƒΠΊΡ†ΠΈΠΈ Π›Π΅Π½Ρ†Π°

Π§Ρ‚ΠΎ Ρ‚Π°ΠΊΠΎΠ΅ Π·Π°ΠΊΠΎΠ½ элСктромагнитной ΠΈΠ½Π΄ΡƒΠΊΡ†ΠΈΠΈ Π›Π΅Π½Ρ†Π°

Π§Ρ‚ΠΎ Ρ‚Π°ΠΊΠΎΠ΅ Π·Π°ΠΊΠΎΠ½ Π›Π΅Π½Ρ†Π°?

Богласно Π·Π°ΠΊΠΎΠ½Ρƒ Π›Π΅Π½Ρ†Π° (ΠΊΠΎΡ‚ΠΎΡ€Ρ‹ΠΉ Π±Ρ‹Π» Π²Π²Π΅Π΄Π΅Π½ русским Ρ„ΠΈΠ·ΠΈΠΊΠΎΠΌ ΠΈΠ· балтийских Π½Π΅ΠΌΡ†Π΅Π² Π“Π΅Π½Ρ€ΠΈΡ…ΠΎΠΌ Π€Ρ€ΠΈΠ΄Ρ€ΠΈΡ…ΠΎΠΌ Π­ΠΌΠΈΠ»Π΅ΠΌ Π›Π΅Π½Ρ†Π΅ΠΌ Π² 1834 Π³ΠΎΠ΄Ρƒ) Π½Π°ΠΏΡ€Π°Π²Π»Π΅Π½ΠΈΠ΅ Ρ‚ΠΎΠΊΠ° ΠΌΠΎΠΆΠ½ΠΎ ΠΎΠΏΡ€Π΅Π΄Π΅Π»ΠΈΡ‚ΡŒ. ΠΊΠΎΠ³Π΄Π° Ρ‚ΠΎΠΊ Ρ‡Π΅Ρ€Π΅Π· ΠΊΠ°Ρ‚ΡƒΡˆΠΊΡƒ измСняСт ΠΌΠ°Π³Π½ΠΈΡ‚Π½ΠΎΠ΅ ΠΏΠΎΠ»Π΅, напряТСниС создаСтся Π² Ρ€Π΅Π·ΡƒΠ»ΡŒΡ‚Π°Ρ‚Π΅ измСнСния ΠΌΠ°Π³Π½ΠΈΡ‚Π½ΠΎΠ³ΠΎ поля, Π½Π°ΠΏΡ€Π°Π²Π»Π΅Π½ΠΈΠ΅ ΠΈΠ½Π΄ΡƒΡ†ΠΈΡ€ΠΎΠ²Π°Π½Π½ΠΎΠ³ΠΎ напряТСния Ρ‚Π°ΠΊΠΎΠ²ΠΎ, Ρ‡Ρ‚ΠΎ ΠΎΠ½ΠΎ всСгда противодСйствуСт измСнСнию Ρ‚ΠΎΠΊΠ°.

Π—Π°ΠΊΠΎΠ½ Π›Π΅Π½Ρ†Π° ΡƒΠΊΠ°Π·Ρ‹Π²Π°Π΅Ρ‚, ΠΊΠ°ΠΊ ΠΌΠΎΠΆΠ½ΠΎ ΠΎΠΏΡ€Π΅Π΄Π΅Π»ΠΈΡ‚ΡŒ Π½Π°ΠΏΡ€Π°Π²Π»Π΅Π½ΠΈΠ΅ Π½Π°Π²Π΅Π΄Π΅Π½Π½ΠΎΠΉ Π­Π”Π‘ Π² ΠΊΠ°Ρ‚ΡƒΡˆΠΊΠ΅. Β«Π’Π°ΠΊΠΈΠΌ ΠΎΠ±Ρ€Π°Π·ΠΎΠΌ, ΠΎΠ½ ΡƒΡ‚Π²Π΅Ρ€ΠΆΠ΄Π°Π΅Ρ‚, Ρ‡Ρ‚ΠΎ Π½Π°ΠΏΡ€Π°Π²Π»Π΅Π½ΠΈΠ΅ Π½Π°Π²Π΅Π΄Π΅Π½Π½ΠΎΠΉ Π­Π”Π‘ Ρ‚Π°ΠΊΠΎΠ²ΠΎ, Ρ‡Ρ‚ΠΎ ΠΎΠ½ΠΎ противодСйствуСт Π²Ρ‹Π·Ρ‹Π²Π°ΡŽΡ‰Π΅ΠΌΡƒ Π΅Π΅ измСнСнию.

Π”Ρ€ΡƒΠ³ΠΈΠΌΠΈ словами, Π·Π°ΠΊΠΎΠ½ Π›Π΅Π½Ρ†Π° гласит, Ρ‡Ρ‚ΠΎ ΠΊΠΎΠ³Π΄Π° Π² Ρ†Π΅ΠΏΠΈ индуцируСтся Π­. М. Π€., тСкущая установка всСгда противодСйствуСт двиТСнию ΠΈΠ»ΠΈ измСнСнию Ρ‚ΠΎΠΊΠ°, ΠΊΠΎΡ‚ΠΎΡ€ΠΎΠ΅ Π΅Π³ΠΎ Π²Ρ‹Π·Ρ‹Π²Π°Π΅Ρ‚. Π˜Π›Π˜

Π˜Π½Π΄ΡƒΡ†ΠΈΡ€ΠΎΠ²Π°Π½Π½Π°Ρ Π­Π”Π‘ заставит Ρ‚ΠΎΠΊ Ρ‚Π΅Ρ‡ΡŒ Π² Π·Π°ΠΌΠΊΠ½ΡƒΡ‚ΠΎΠΉ Ρ†Π΅ΠΏΠΈ Π² Ρ‚Π°ΠΊΠΎΠΌ Π½Π°ΠΏΡ€Π°Π²Π»Π΅Π½ΠΈΠΈ, Ρ‡Ρ‚ΠΎ Π΅Π³ΠΎ ΠΌΠ°Π³Π½ΠΈΡ‚Π½Ρ‹ΠΉ эффСкт Π±ΡƒΠ΄Π΅Ρ‚ ΠΏΡ€ΠΎΡ‚ΠΈΠ²ΠΎΠ΄Π΅ΠΉΡΡ‚Π²ΠΎΠ²Π°Ρ‚ΡŒ Π²Ρ‹Π·Π²Π°Π²ΡˆΠ΅ΠΌΡƒ Π΅Π³ΠΎ измСнСнию.

ΠŸΡ€ΠΎΡ‰Π΅ говоря, Π·Π°ΠΊΠΎΠ½ Π›Π΅Π½Ρ†Π° гласит, Ρ‡Ρ‚ΠΎ ΠΈΠ½Π΄ΡƒΡ†ΠΈΡ€ΠΎΠ²Π°Π½Π½Ρ‹ΠΉ эффСкт всСгда Ρ‚Π°ΠΊΠΎΠ², Ρ‡Ρ‚ΠΎ противодСйствуСт ΠΏΡ€ΠΈΡ‡ΠΈΠ½Π΅, Π΅Π³ΠΎ Π²Ρ‹Π·Π²Π°Π²ΡˆΠ΅ΠΉ.

РазъяснСниС Π·Π°ΠΊΠΎΠ½Π° Π›Π΅Π½Ρ†Π°

Π—Π°ΠΊΠΎΠ½ Π›Π΅Π½Ρ†Π° (ΠΊΠΎΡ‚ΠΎΡ€Ρ‹ΠΉ Π½Π΅ΠΌΠ½ΠΎΠ³ΠΎ слоТСн ΠΈ сбиваСт с Ρ‚ΠΎΠ»ΠΊΡƒ Π½ΠΎΠ²ΠΈΡ‡ΠΊΠΎΠ²) ΠΌΠΎΠΆΠ½ΠΎ ΠΏΠΎΠ½ΡΡ‚ΡŒ с ΠΏΠΎΠΌΠΎΡ‰ΡŒΡŽ ΠΏΡ€ΠΈΠ²Π΅Π΄Π΅Π½Π½ΠΎΠΉ Π²Ρ‹ΡˆΠ΅ схСмы, Π³Π΄Π΅ изолированная ΠΊΠ°Ρ‚ΡƒΡˆΠΊΠ° ΠΏΠΎΠ΄ΠΊΠ»ΡŽΡ‡Π΅Π½Π° ΠΊ Ρ‡ΡƒΠ²ΡΡ‚Π²ΠΈΡ‚Π΅Π»ΡŒΠ½ΠΎΠΌΡƒ Π³Π°Π»ΡŒΠ²Π°Π½ΠΎΠΌΠ΅Ρ‚Ρ€Ρƒ ΠΈ статичСскому ΠΈ ΡΠΏΠ»ΠΎΡˆΠ½ΠΎΠΌΡƒ стСрТнСвому ΠΌΠ°Π³Π½ΠΈΡ‚Ρƒ. ΠŸΠΎΡΠΌΠΎΡ‚Ρ€ΠΈΠΌ, ΠΊΠ°ΠΊ это Ρ€Π°Π±ΠΎΡ‚Π°Π΅Ρ‚

  1. Когда ΠΈ стСрТнСвой ΠΌΠ°Π³Π½ΠΈΡ‚, ΠΈ ΠΊΠ°Ρ‚ΡƒΡˆΠΊΠ° находятся Π² статичСском ΠΏΠΎΠ»ΠΎΠΆΠ΅Π½ΠΈΠΈ, отсутствуСт Ρ‚ΠΎΠΊ ΠΈΠ»ΠΈ индуцированная Π­Π”Π‘ (Π΄Π°ΠΆΠ΅ нСбольшая Π²Π΅Π»ΠΈΡ‡ΠΈΠ½Π° ΠΌΠ°Π³Π½ΠΈΡ‚Π½ΠΎΠ³ΠΎ ΠΏΠΎΡ‚ΠΎΠΊΠ° (полюса N статичСского ΠΌΠ°Π³Π½ΠΈΡ‚Π½ΠΎΠ³ΠΎ стСрТня), связанная с Π΄Π²ΠΈΠΆΠ΅Π½ΠΈΠ΅ΠΌ ΠΊΠ°Ρ‚ΡƒΡˆΠΊΠΈ), ΡΠ»Π΅Π΄ΠΎΠ²Π°Ρ‚Π΅Π»ΡŒΠ½ΠΎ, Π³Π°Π»ΡŒΠ²Π°Π½ΠΎΠΌΠ΅Ρ‚Ρ€ Π½Π΅ прогибаСтся.
  2. Когда ΡΡ‚Π΅Ρ€ΠΆΠ΅Π½ΡŒ ΠΌΠ°Π³Π½ΠΈΡ‚Π° быстро двиТСтся ΠΊ ΠΊΠ°Ρ‚ΡƒΡˆΠΊΠ΅, Π² Π³Π°Π»ΡŒΠ²Π°Π½ΠΎΠΌΠ΅Ρ‚Ρ€Π΅, Π½Π°ΠΏΡ€ΠΈΠΌΠ΅Ρ€, происходит быстроС ΠΎΡ‚ΠΊΠ»ΠΎΠ½Π΅Π½ΠΈΠ΅. Π˜ΠΌΠ΅ΠΉΡ‚Π΅ Π² Π²ΠΈΠ΄Ρƒ, Ρ‡Ρ‚ΠΎ ΠΎΡ‚ΠΊΠ»ΠΎΠ½Π΅Π½ΠΈΠ΅ Π±ΡƒΠ΄Π΅Ρ‚ ΠΎΡΡ‚Π°Π²Π°Ρ‚ΡŒΡΡ постоянным Π΄ΠΎ Ρ‚Π΅Ρ… ΠΏΠΎΡ€, ΠΏΠΎΠΊΠ° ΡΡ‚Π΅Ρ€ΠΆΠ΅Π½ΡŒ ΠΌΠ°Π³Π½ΠΈΡ‚Π° Π½Π΅ Π±ΡƒΠ΄Π΅Ρ‚ Π½Π΅ΠΏΡ€Π΅Ρ€Ρ‹Π²Π½ΠΎ ΠΏΠ΅Ρ€Π΅ΠΌΠ΅Ρ‰Π°Ρ‚ΡŒΡΡ ΠΏΠΎ ΠΎΡ‚Π½ΠΎΡˆΠ΅Π½ΠΈΡŽ ΠΊ ΠΊΠ°Ρ‚ΡƒΡˆΠΊΠ΅ (Ρ‚. Π•. ΠžΡ‚Π½ΠΎΡΠΈΡ‚Π΅Π»ΡŒΠ½Ρ‹ΠΉ ΠΌΠΎΠΌΠ΅Π½Ρ‚ ΠΌΠ΅ΠΆΠ΄Ρƒ стСрТнСм ΠΌΠ°Π³Π½ΠΈΡ‚Π° ΠΈ ΠΊΠ°Ρ‚ΡƒΡˆΠΊΠΎΠΉ). Если ΠΈ ΡΡ‚Π΅Ρ€ΠΆΠ΅Π½ΡŒ ΠΌΠ°Π³Π½ΠΈΡ‚Π°, ΠΈ ΠΊΠ°Ρ‚ΡƒΡˆΠΊΠ° Π΄ΠΎΡΡ‚ΠΈΠ³Π°ΡŽΡ‚ статичСского полоТСния, ΠΎΡ‚ΠΊΠ»ΠΎΠ½Π΅Π½ΠΈΠ΅ Π³Π°Π»ΡŒΠ²Π°Π½ΠΎΠΌΠ΅Ρ‚Ρ€Π° Π±ΡƒΠ΄Π΅Ρ‚ Π½ΡƒΠ»Π΅Π²Ρ‹ΠΌ (ΠΊΠ°ΠΊ ΠΏΠΎΠΊΠ°Π·Π°Π½ΠΎ Π½Π° рис. 1 A).
  3. Когда ΡΡ‚Π΅Ρ€ΠΆΠ΅Π½ΡŒ ΠΌΠ°Π³Π½ΠΈΡ‚Π° отодвигаСтся ΠΎΡ‚ ΠΊΠ°Ρ‚ΡƒΡˆΠΊΠΈ, Π³Π°Π»ΡŒΠ²Π°Π½ΠΎΠΌΠ΅Ρ‚Ρ€ снова Π±ΡƒΠ΄Π΅Ρ‚ ΠΎΡ‚ΠΊΠ»ΠΎΠ½ΡΡ‚ΡŒΡΡ Π΄ΠΎ Ρ‚Π΅Ρ… ΠΏΠΎΡ€, ΠΏΠΎΠΊΠ° ΠΎΡ‚Π½ΠΎΡΠΈΡ‚Π΅Π»ΡŒΠ½ΠΎΠ΅ Π΄Π²ΠΈΠΆΠ΅Π½ΠΈΠ΅ ΠΌΠ΅ΠΆΠ΄Ρƒ стСрТнСм ΠΌΠ°Π³Π½ΠΈΡ‚Π° ΠΈ ΠΊΠ°Ρ‚ΡƒΡˆΠΊΠΎΠΉ Π½Π΅ станСт Π½Π΅ΠΏΠΎΠ΄Π²ΠΈΠΆΠ½Ρ‹ΠΌ ΠΈΠ»ΠΈ статичСским.Π˜ΠΌΠ΅ΠΉΡ‚Π΅ Π² Π²ΠΈΠ΄Ρƒ, Ρ‡Ρ‚ΠΎ Π½Π°ΠΏΡ€Π°Π²Π»Π΅Π½ΠΈΠ΅ Π³Π°Π»ΡŒΠ²Π°Π½ΠΎΠΌΠ΅Ρ‚Ρ€Π° ΠΏΡ€ΠΎΡ‚ΠΈΠ²ΠΎΠΏΠΎΠ»ΠΎΠΆΠ½ΠΎ Π½Π°ΠΏΡ€Π°Π²Π»Π΅Π½ΠΈΡŽ Π½Π° рис. 1A (ΠΊΠ°ΠΊ ΠΏΠΎΠΊΠ°Π·Π°Π½ΠΎ Π½Π° рис. 1 B).
  4. Π’ΠΎ ΠΆΠ΅ самоС происходит (этапы 2 ΠΈ 3), Ссли ΡΡ‚Π΅Ρ€ΠΆΠ΅Π½ΡŒ ΠΌΠ°Π³Π½ΠΈΡ‚Π° находится Π² статичСском ΠΏΠΎΠ»ΠΎΠΆΠ΅Π½ΠΈΠΈ, Π² Ρ‚ΠΎ врСмя ΠΊΠ°ΠΊ ΠΊΠ°Ρ‚ΡƒΡˆΠΊΠ° двиТСтся ΠΏΠΎ Π½Π°ΠΏΡ€Π°Π²Π»Π΅Π½ΠΈΡŽ ΠΊ ΡΡ‚Π΅Ρ€ΠΆΠ½ΡŽ статичСского ΠΌΠ°Π³Π½ΠΈΡ‚Π° ΠΈΠ»ΠΈ удаляСтся ΠΎΡ‚ Π½Π΅Π³ΠΎ.

Π­Ρ‚ΠΎ ясно ΠΏΠΎΠΊΠ°Π·Ρ‹Π²Π°Π΅Ρ‚, Ρ‡Ρ‚ΠΎ ΠΊΠΎΠ³Π΄Π° ΠΌΠ°Π³Π½ΠΈΡ‚Π½Ρ‹ΠΉ ΡΡ‚Π΅Ρ€ΠΆΠ΅Π½ΡŒ (Π² Π΄Π²ΠΈΠΆΠ΅Π½ΠΈΠΈ) находится рядом с ΠΊΠ°Ρ‚ΡƒΡˆΠΊΠΎΠΉ, ΠΎΠ½ сокращаСт ΠΈΠ»ΠΈ связываСт Π±ΠΎΠ»ΡŒΡˆΡƒΡŽ Ρ‡Π°ΡΡ‚ΡŒ ΠΌΠ°Π³Π½ΠΈΡ‚Π½ΠΎΠ³ΠΎ ΠΏΠΎΡ‚ΠΎΠΊΠ°, Ρ‚ΠΎΠ³Π΄Π° ΠΊΠ°ΠΊ ΡΠΊΠΎΡ€ΠΎΡΡ‚ΡŒ ΠΌΠ°Π³Π½ΠΈΡ‚Π½ΠΎΠΉ связи мСньшС Π² случаС, ΠΊΠΎΠ³Π΄Π° ΠΌΠ°Π³Π½ΠΈΡ‚Π½Ρ‹ΠΉ ΡΡ‚Π΅Ρ€ΠΆΠ΅Π½ΡŒ пСрСмСщаСтся ΠΎΡ‚ ΠΊΠ°Ρ‚ΡƒΡˆΠΊΠΈ.

ΠžΡ‚ΠΊΠ»ΠΎΠ½Π΅Π½ΠΈΠ΅ Π³Π°Π»ΡŒΠ²Π°Π½ΠΎΠΌΠ΅Ρ‚Ρ€Π° ΠΏΠΎΠΊΠ°Π·Ρ‹Π²Π°Π΅Ρ‚ Π½Π°Π²Π΅Π΄Π΅Π½Π½ΡƒΡŽ Π­Π”Π‘ Π² ΠΊΠ°Ρ‚ΡƒΡˆΠΊΠ΅, которая Π²Ρ‹Π·Π²Π°Π½Π° Π²Π½Π΅Π·Π°ΠΏΠ½Ρ‹ΠΌ ΠΏΠ΅Ρ€Π΅ΠΌΠ΅Ρ‰Π΅Π½ΠΈΠ΅ΠΌ ΠΈΠ»ΠΈ ΠΏΠ΅Ρ€Π΅ΠΌΠ΅Ρ‰Π΅Π½ΠΈΠ΅ΠΌ ΠΌΠ°Π³Π½ΠΈΡ‚Π½ΠΎΠ³ΠΎ стСрТня ΠΊ ΠΊΠ°Ρ‚ΡƒΡˆΠΊΠ΅. Π’ΠΎΡ‡Π½ΠΎΠΉ ΠΏΡ€ΠΈΡ‡ΠΈΠ½ΠΎΠΉ Π½Π°Π²Π΅Π΄Π΅Π½Π½ΠΎΠΉ Π­Π”Π‘ являСтся ΠΈΠ·ΠΌΠ΅Π½Π΅Π½ΠΈΠ΅ ΠΌΠ°Π³Π½ΠΈΡ‚Π½ΠΎΠΉ связи ΠΏΠΎ ΠΎΡ‚Π½ΠΎΡˆΠ΅Π½ΠΈΡŽ ΠΊ ΠΊΠ°Ρ‚ΡƒΡˆΠΊΠ΅, ΠΊΠΎΡ‚ΠΎΡ€ΠΎΠ΅ продолТаСтся Π΄ΠΎ Ρ‚Π΅Ρ… ΠΏΠΎΡ€, ΠΏΠΎΠΊΠ° Π΄Π²ΠΈΠΆΠ΅Π½ΠΈΠ΅ стСрТня ΠΈΠ»ΠΈ ΠΊΠ°Ρ‚ΡƒΡˆΠΊΠΈ ΠΌΠ°Π³Π½ΠΈΡ‚Π° Π½Π΅ Π±ΡƒΠ΄Π΅Ρ‚ остановлСно. Π”Ρ€ΡƒΠ³ΠΈΠΌΠΈ словами, сильноС ΠΌΠ°Π³Π½ΠΈΡ‚Π½ΠΎΠ΅ ΠΏΠΎΠ»Π΅ ΠΈΠ»ΠΈ ΠΌΠ°Π³Π½ΠΈΡ‚Π½Ρ‹ΠΉ ΠΏΠΎΡ‚ΠΎΠΊ Π½Π΅ Π²Ρ‹Π·Ρ‹Π²Π°Π΅Ρ‚ Π­Π”Π‘ Π² ΠΏΡ€ΠΎΠ²ΠΎΠ΄Π½ΠΈΠΊΠ΅ статичСского элСктричСства. Π‘Π»Π΅Π΄ΠΎΠ²Π°Ρ‚Π΅Π»ΡŒΠ½ΠΎ, ΠΈΠ·ΠΌΠ΅Π½Π΅Π½ΠΈΠ΅ ΠΌΠ°Π³Π½ΠΈΡ‚Π½ΠΎΠ³ΠΎ ΠΏΠΎΡ‚ΠΎΠΊΠ° являСтся ΠΎΠ±ΡΠ·Π°Ρ‚Π΅Π»ΡŒΠ½Ρ‹ΠΌ явлСниСм для ΠΈΠ½Π΄ΡƒΠΊΡ†ΠΈΠΈ Π­Π”Π‘ Π² ΠΊΠ°Ρ‚ΡƒΡˆΠΊΠ΅ ΠΈΠ»ΠΈ ΠΏΡ€ΠΎΠ²ΠΎΠ΄Π½ΠΈΠΊΠ°Ρ….

ΠŸΡ€ΠΈΠ²Π΅Π΄Π΅Π½Π½ΠΎΠ΅ Π²Ρ‹ΡˆΠ΅ объяснСниС ΠΏΠΎΠΊΠ°Π·Ρ‹Π²Π°Π΅Ρ‚, Ρ‡Ρ‚ΠΎ, ΠΊΠΎΠ³Π΄Π° полюс Β«NΒ» ΠΌΠ°Π³Π½ΠΈΡ‚Π° пСрСмСщаСтся ΠΊ ΠΊΠ°Ρ‚ΡƒΡˆΠΊΠ΅, Π² ΠΊΠ°Ρ‚ΡƒΡˆΠΊΠ΅ индуцируСтся Π­Π”Π‘ ΠΈ ΠΏΡ€ΠΎΡ‚Π΅ΠΊΠ°Π΅Ρ‚ Ρ‚ΠΎΠΊ Π² Π½Π΅ΠΉ ΠΏΡ€ΠΎΡ‚ΠΈΠ² часовой стрСлки (Ссли ΡΠΌΠΎΡ‚Ρ€Π΅Ρ‚ΡŒ Π½Π° ΠΊΠ°Ρ‚ΡƒΡˆΠΊΡƒ рядом), ΡΠ»Π΅Π΄ΠΎΠ²Π°Ρ‚Π΅Π»ΡŒΠ½ΠΎ, ΠΏΠ΅Ρ€Π΅Π΄Π½ΠΈΠΉ ΠΊΠΎΠ½Π΅Ρ† ΠΊΠ°Ρ‚ΡƒΡˆΠΊΠΈ становится Β«NΒ» полюсом (рис. 1.A).

Π’Π°ΠΊΠΈΠΌ ΠΎΠ±Ρ€Π°Π·ΠΎΠΌ, сСвСрный полюс ΠΌΠ°Π³Π½ΠΈΡ‚Π° ΠΎΡ‚Ρ€Π°ΠΆΠ°Π΅Ρ‚ сСвСрный полюс ΠΊΠ°Ρ‚ΡƒΡˆΠΊΠΈ N. Для управлСния этой силой отталкивания ΠΈΡΠΏΠΎΠ»ΡŒΠ·ΡƒΠ΅Ρ‚ΡΡ мСханичСская энСргия, которая прСобразуСтся Π² ΡΠ»Π΅ΠΊΡ‚Ρ€ΠΈΡ‡Π΅ΡΠΊΡƒΡŽ ΡΠ½Π΅Ρ€Π³ΠΈΡŽ Π² Ρ„ΠΎΡ€ΠΌΠ΅ Π­Π”Π‘ Π² ΠΊΠ°Ρ‚ΡƒΡˆΠΊΠ΅.

Аналогично, ΠΊΠΎΠ³Π΄Π° Π‘Π΅Π²Π΅Ρ€Π½Ρ‹ΠΉ полюс ΠΌΠ°Π³Π½ΠΈΡ‚Π° пСрСмСщаСтся ΠΎΡ‚ ΠΊΠ°Ρ‚ΡƒΡˆΠΊΠΈ, ΠΎΠ±Ρ€Π°Ρ‰Π΅Π½Π½Ρ‹ΠΉ ΠΊΠΎΠ½Π΅Ρ† ΠΊΠ°Ρ‚ΡƒΡˆΠΊΠΈ становится S-полюсом.Π’Π°ΠΊΠΈΠΌ ΠΎΠ±Ρ€Π°Π·ΠΎΠΌ, полюс N ΠΌΠ°Π³Π½ΠΈΡ‚Π½ΠΎΠ³ΠΎ стСрТня притягиваСт полюс S. Π§Ρ‚ΠΎΠ±Ρ‹ ΠΊΠΎΠ½Ρ‚Ρ€ΠΎΠ»ΠΈΡ€ΠΎΠ²Π°Ρ‚ΡŒ эту силу притяТСния ΠΌΠ΅ΠΆΠ΄Ρƒ ΠΊΠ°Ρ‚ΡƒΡˆΠΊΠΎΠΉ ΠΈ стСрТнСм ΠΌΠ°Π³Π½ΠΈΡ‚Π°, снова трСбуСтся мСханичСская энСргия, которая прСобразуСтся Π² ΡΠ»Π΅ΠΊΡ‚Ρ€ΠΈΡ‡Π΅ΡΠΊΡƒΡŽ ΡΠ½Π΅Ρ€Π³ΠΈΡŽ Π² Ρ„ΠΎΡ€ΠΌΠ΅ ΠΈΠ½Π΄ΡƒΡ†ΠΈΡ€ΠΎΠ²Π°Π½Π½ΠΎΠΉ Π­Π”Π‘ Π² ΠΊΠ°Ρ‚ΡƒΡˆΠΊΠ΅.

Π­Ρ‚ΠΎ Π΄ΠΎΠΊΠ°Π·Ρ‹Π²Π°Π΅Ρ‚, Ρ‡Ρ‚ΠΎ ΠΈΠ½Π΄ΡƒΡ†ΠΈΡ€ΠΎΠ²Π°Π½Π½Ρ‹ΠΉ Ρ‚ΠΎΠΊ всСгда Ρ‚Π΅Ρ‡Π΅Ρ‚ Π² Ρ‚Π°ΠΊΠΎΠΌ Π½Π°ΠΏΡ€Π°Π²Π»Π΅Π½ΠΈΠΈ, Ρ‡Ρ‚ΠΎ ΠΎΠ½ противодСйствуСт измСнСнию ΠΌΠ°Π³Π½ΠΈΡ‚Π½ΠΎΠ³ΠΎ поля (ΠΏΠ΅Ρ€Π΅ΠΌΠ΅Ρ‰Π΅Π½ΠΈΠ΅ Π² Π½Π°ΠΏΡ€Π°Π²Π»Π΅Π½ΠΈΠΈ / ΠΎΡ‚ ΠΌΠ°Π³Π½ΠΈΡ‚Π½ΠΎΠ³ΠΎ стСрТня ΠΊ ΠΊΠ°Ρ‚ΡƒΡˆΠΊΠ΅), ΠΊΠΎΡ‚ΠΎΡ€ΠΎΠ΅ Π΅Π³ΠΎ Π²Ρ‹Π·Π²Π°Π»ΠΎ.

ΠŸΠΎΡ…ΠΎΠΆΠΈΠ΅ сообщСния:

Π€ΠΎΡ€ΠΌΡƒΠ»Π° ΠΈ уравнСния для Π·Π°ΠΊΠΎΠ½Π° Π›Π΅Π½Ρ†Π°

ΠœΠ°Ρ‚Π΅ΠΌΠ°Ρ‚ΠΈΡ‡Π΅ΡΠΊΠΈ основная вСрсия Π·Π°ΠΊΠΎΠ½Π° Π›Π΅Π½Ρ†Π° ΠΌΠΎΠΆΠ΅Ρ‚ Π±Ρ‹Ρ‚ΡŒ Π²Ρ‹Ρ€Π°ΠΆΠ΅Π½Π° ΡΠ»Π΅Π΄ΡƒΡŽΡ‰ΠΈΠΌ ΠΎΠ±Ρ€Π°Π·ΠΎΠΌ.

e = – (dΞ¦ B / dt)

ЀактичСски, символ Β«-Β» Π² Π·Π°ΠΊΠΎΠ½Π΅ элСктромагнитной ΠΈΠ½Π΄ΡƒΠΊΡ†ΠΈΠΈ ЀарадСя прСдставляСт Π·Π°ΠΊΠΎΠ½ Π›Π΅Π½Π°. ΠŸΠΎΡΠΊΠΎΠ»ΡŒΠΊΡƒ ΠΎΠ½ΠΎ основано Π½Π° ΠΏΡ€ΠΈΠ½Ρ†ΠΈΠΏΠ΅ сохранСния энСргии, Π° Ρ‚Π°ΠΊΠΆΠ΅ Π½Π° Ρ‚Ρ€Π΅Ρ‚ΡŒΠ΅ΠΌ Π·Π°ΠΊΠΎΠ½Π΅ ΠΡŒΡŽΡ‚ΠΎΠ½Π° (ΠΊΠ°ΠΆΠ΄ΠΎΠ΅ дСйствиС ΠΈΠΌΠ΅Π΅Ρ‚ ΠΎΠ΄ΠΈΠ½Π°ΠΊΠΎΠ²ΡƒΡŽ Ρ€Π΅Π°ΠΊΡ†ΠΈΡŽ, Π½ΠΎ Π²Π΅Π»ΠΈΡ‡ΠΈΠ½Π° ΠΏΡ€ΠΎΡ‚ΠΈΠ²ΠΎΠΏΠΎΠ»ΠΎΠΆΠ½Π°).

Π”Ρ€ΡƒΠ³ΠΈΠ΅ Ρ„ΠΎΡ€ΠΌΡ‹ ΡƒΡ€Π°Π²Π½Π΅Π½ΠΈΠΉ ΠΈ Ρ„ΠΎΡ€ΠΌΡƒΠ» для Π·Π°ΠΊΠΎΠ½Π° Π›Π΅Π½Ρ†Π° ΡΠ»Π΅Π΄ΡƒΡŽΡ‰ΠΈΠ΅.

  • e = – N (dΞ¦ B / dt)
  • e = – N (ΔΦ / Ξ”t)
  • e = – N (δΦ B / Ξ΄t)

Π“Π΄Π΅:

  • e = НавСдСнная Π­Π”Π‘ Π² ΠΊΠ°Ρ‚ΡƒΡˆΠΊΠ΅ / ΠΏΡ€ΠΎΠ²ΠΎΠ΄Π½ΠΈΠΊΠ΅
  • N = количСство Π²ΠΈΡ‚ΠΊΠΎΠ² ΠΈΠ»ΠΈ Π²ΠΈΡ‚ΠΊΠΎΠ² Π² ΠΊΠ°Ρ‚ΡƒΡˆΠΊΠ΅
  • dΞ¦ B , ΔΦ, δΦ B = ИзмСнСниС скорости ΠΌΠ°Π³Π½ΠΈΡ‚Π½ΠΎΠ³ΠΎ ΠΏΠΎΡ‚ΠΎΠΊΠ°
  • dt, Ξ”t, Ξ΄t = ИзмСнСниС скорости Π²Ρ€Π΅ΠΌΠ΅Π½ΠΈ

ΠŸΡ€ΠΈΠΌΠ΅Π½Π΅Π½ΠΈΠ΅ Π·Π°ΠΊΠΎΠ½Π° Π›Π΅Π½Ρ†Π°

БущСствуСт мноТСство Π²Π°Ρ€ΠΈΠ°Π½Ρ‚ΠΎΠ² использования ΠΈ примСнСния, основанных Π½Π° Π·Π°ΠΊΠΎΠ½Π΅ Π›Π΅Π½Π° e. Π³. основной ΠΏΡ€ΠΈΠ½Ρ†ΠΈΠΏ Π·Π°ΠΊΠΎΠ½Π° Π›Π΅Π½Ρ†Π° ΠΈΡΠΏΠΎΠ»ΡŒΠ·ΡƒΠ΅Ρ‚ΡΡ Π²

  • Π­Π»Π΅ΠΊΡ‚Ρ€ΠΎΠΌΠ°Π³Π½ΠΈΡ‚Π½ΠΎΠ΅ Ρ‚ΠΎΡ€ΠΌΠΎΠΆΠ΅Π½ΠΈΠ΅ Π² ΠΏΠΎΠ΅Π·Π΄Π°Ρ….
  • Π˜Π½Π΄ΡƒΠΊΡ†ΠΈΠΎΠ½Π½Ρ‹Π΅ Π³ΠΎΡ€Π΅Π»ΠΊΠΈ, Π²Π°Ρ€ΠΎΡ‡Π½Ρ‹Π΅ ΠΏΠ°Π½Π΅Π»ΠΈ ΠΈ ΠΈΠ½Π΄ΡƒΠΊΡ†ΠΈΠΎΠ½Π½Ρ‹ΠΉ Π½Π°Π³Ρ€Π΅Π².
  • Π­Π»Π΅ΠΊΡ‚Ρ€ΠΎΠ³Π΅Π½Π΅Ρ€Π°Ρ‚ΠΎΡ€Ρ‹, трансформаторы ΠΈ Π΄Π²ΠΈΠ³Π°Ρ‚Π΅Π»ΠΈ (ΠΏΡ€ΠΎΡ‚ΠΈΠ²ΠΎ-Π­Π”Π‘).
  • ΠœΠΈΠΊΡ€ΠΎΡ„ΠΎΠ½Ρ‹, устройства чтСния ΠΊΠ°Ρ€Ρ‚, Π»Π΅Π½Ρ‚Ρ‹ для записи Π°ΡƒΠ΄ΠΈΠΎ / Π²ΠΈΠ΄Π΅ΠΎ, ТСсткий диск ΠΈ Π²Ρ€Π°Ρ‰Π°ΡŽΡ‰ΠΈΠ΅ΡΡ диски, Π΄Π΅Π±Π΅Ρ‚ΠΎΠ²Ρ‹Π΅ / ΠΊΡ€Π΅Π΄ΠΈΡ‚Π½Ρ‹Π΅ ΠΊΠ°Ρ€Ρ‚Ρ‹.
  • Π’ΠΈΡ…Ρ€Π΅Ρ‚ΠΎΠΊΠΎΠ²Ρ‹Π΅ вСсы.
  • Π”ΠΈΠ½Π°ΠΌΠΎΠΌΠ΅Ρ‚Ρ€Ρ‹ ΠΈ мСталлоискатСли.
  • ΠŸΡ€ΠΎΠ°Π½Π°Π»ΠΈΠ·ΠΈΡ€ΠΎΠ²Π°Ρ‚ΡŒ ΠΎΡΠ½ΠΎΠ²Π½ΡƒΡŽ ΠΊΠΎΠ½Ρ†Π΅ΠΏΡ†ΠΈΡŽ Π½Π°ΠΊΠΎΠΏΠ»Π΅Π½Π½ΠΎΠΉ энСргии Π² ΠΈΠ½Π΄ΡƒΠΊΡ‚ΠΎΡ€Π°Ρ….

ΠŸΠΎΡ…ΠΎΠΆΠΈΠ΅ сообщСния

Π—Π°ΠΊΠΎΠ½ ΠΈΠ½Π΄ΡƒΠΊΡ†ΠΈΠΈ ЀарадСя | Ρ„ΠΈΠ·ΠΈΠΊΠ°

Π—Π°ΠΊΠΎΠ½ ΠΈΠ½Π΄ΡƒΠΊΡ†ΠΈΠΈ ЀарадСя , Π² Ρ„ΠΈΠ·ΠΈΠΊΠ΅ количСствСнная связь ΠΌΠ΅ΠΆΠ΄Ρƒ ΠΈΠ·ΠΌΠ΅Π½ΡΡŽΡ‰ΠΈΠΌΡΡ ΠΌΠ°Π³Π½ΠΈΡ‚Π½Ρ‹ΠΌ ΠΏΠΎΠ»Π΅ΠΌ ΠΈ элСктричСским ΠΏΠΎΠ»Π΅ΠΌ, создаваСмым этим ΠΈΠ·ΠΌΠ΅Π½Π΅Π½ΠΈΠ΅ΠΌ, разработанная Π½Π° основС ΡΠΊΡΠΏΠ΅Ρ€ΠΈΠΌΠ΅Π½Ρ‚Π°Π»ΡŒΠ½Ρ‹Ρ… наблюдСний, сдСланных Π² 1831 Π³ΠΎΠ΄Ρƒ английским ΡƒΡ‡Π΅Π½Ρ‹ΠΌ Майклом Π€Π°Ρ€Π°Π΄Π΅Π΅ΠΌ.

Π―Π²Π»Π΅Π½ΠΈΠ΅, Π½Π°Π·Ρ‹Π²Π°Π΅ΠΌΠΎΠ΅ элСктромагнитной ΠΈΠ½Π΄ΡƒΠΊΡ†ΠΈΠ΅ΠΉ, Π±Ρ‹Π»ΠΎ Π²ΠΏΠ΅Ρ€Π²Ρ‹Π΅ Π·Π°ΠΌΠ΅Ρ‡Π΅Π½ΠΎ ΠΈ исслСдовано Π€Π°Ρ€Π°Π΄Π΅Π΅ΠΌ; Π·Π°ΠΊΠΎΠ½ ΠΈΠ½Π΄ΡƒΠΊΡ†ΠΈΠΈ – это Π΅Π³ΠΎ количСствСнноС Π²Ρ‹Ρ€Π°ΠΆΠ΅Π½ΠΈΠ΅. Π€Π°Ρ€Π°Π΄Π΅ΠΉ ΠΎΠ±Π½Π°Ρ€ΡƒΠΆΠΈΠ», Ρ‡Ρ‚ΠΎ всякий Ρ€Π°Π·, ΠΊΠΎΠ³Π΄Π° ΠΌΠ°Π³Π½ΠΈΡ‚Π½ΠΎΠ΅ ΠΏΠΎΠ»Π΅ Π²ΠΎΠΊΡ€ΡƒΠ³ элСктромагнита возрастаСт ΠΈ схлопываСтся Π·Π° счСт замыкания ΠΈ размыкания элСктричСской Ρ†Π΅ΠΏΠΈ, Ρ‡Π°ΡΡ‚ΡŒΡŽ ΠΊΠΎΡ‚ΠΎΡ€ΠΎΠΉ ΠΎΠ½ являСтся, элСктричСский Ρ‚ΠΎΠΊ ΠΌΠΎΠΆΠ΅Ρ‚ Π±Ρ‹Ρ‚ΡŒ ΠΎΠ±Π½Π°Ρ€ΡƒΠΆΠ΅Π½ Π² ΠΎΡ‚Π΄Π΅Π»ΡŒΠ½ΠΎΠΌ ΠΏΡ€ΠΎΠ²ΠΎΠ΄Π½ΠΈΠΊΠ΅ поблизости. ΠŸΠ΅Ρ€Π΅ΠΌΠ΅Ρ‰Π΅Π½ΠΈΠ΅ постоянного ΠΌΠ°Π³Π½ΠΈΡ‚Π° Π² ΠΊΠ°Ρ‚ΡƒΡˆΠΊΡƒ с ΠΏΡ€ΠΎΠ²ΠΎΠ»ΠΎΠΊΠΎΠΉ ΠΈ ΠΈΠ· Π½Π΅Π΅ Ρ‚Π°ΠΊΠΆΠ΅ ΠΈΠ½Π΄ΡƒΡ†ΠΈΡ€ΠΎΠ²Π°Π»ΠΎ Ρ‚ΠΎΠΊ Π² ΠΏΡ€ΠΎΠ²ΠΎΠ»ΠΎΠΊΠ΅, ΠΏΠΎΠΊΠ° ΠΌΠ°Π³Π½ΠΈΡ‚ находился Π² Π΄Π²ΠΈΠΆΠ΅Π½ΠΈΠΈ. ΠŸΡ€ΠΈ ΠΏΠ΅Ρ€Π΅ΠΌΠ΅Ρ‰Π΅Π½ΠΈΠΈ ΠΏΡ€ΠΎΠ²ΠΎΠ΄Π½ΠΈΠΊΠ° рядом с Π½Π΅ΠΏΠΎΠ΄Π²ΠΈΠΆΠ½Ρ‹ΠΌ постоянным ΠΌΠ°Π³Π½ΠΈΡ‚ΠΎΠΌ Π² ΠΏΡ€ΠΎΠ²ΠΎΠ΄Π΅ Ρ‚Π°ΠΊΠΆΠ΅ ΠΏΡ€ΠΎΡ‚Π΅ΠΊΠ°Π» Ρ‚ΠΎΠΊ, ΠΏΠΎΠΊΠ° ΠΎΠ½ двигался.

ΠŸΠΎΠ΄Ρ€ΠΎΠ±Π½Π΅Π΅ ΠΏΠΎ этой Ρ‚Π΅ΠΌΠ΅

Π­Π»Π΅ΠΊΡ‚Ρ€ΠΎΠΌΠ°Π³Π½Π΅Ρ‚ΠΈΠ·ΠΌ: Π·Π°ΠΊΠΎΠ½ ΠΈΠ½Π΄ΡƒΠΊΡ†ΠΈΠΈ ЀарадСя

ΠžΡ‚ΠΊΡ€Ρ‹Ρ‚ΠΈΠ΅ Π€Π°Ρ€Π°Π΄Π΅Π΅ΠΌ Π² 1831 Π³ΠΎΠ΄Ρƒ явлСния ΠΌΠ°Π³Π½ΠΈΡ‚Π½ΠΎΠΉ ΠΈΠ½Π΄ΡƒΠΊΡ†ΠΈΠΈ – ΠΎΠ΄Π½Π° ΠΈΠ· Π²Π°ΠΆΠ½Π΅ΠΉΡˆΠΈΡ… Π²Π΅Ρ… Π½Π° ΠΏΡƒΡ‚ΠΈ ΠΊ пониманию ΠΈ…

Π€Π°Ρ€Π°Π΄Π΅ΠΉ Π²ΠΈΠ·ΡƒΠ°Π»ΠΈΠ·ΠΈΡ€ΠΎΠ²Π°Π» ΠΌΠ°Π³Π½ΠΈΡ‚Π½ΠΎΠ΅ ΠΏΠΎΠ»Π΅ ΠΊΠ°ΠΊ состоящСС ΠΈΠ· мноТСства Π»ΠΈΠ½ΠΈΠΉ ΠΈΠ½Π΄ΡƒΠΊΡ†ΠΈΠΈ, вдоль ΠΊΠΎΡ‚ΠΎΡ€Ρ‹Ρ… Π±ΡƒΠ΄Π΅Ρ‚ ΡƒΠΊΠ°Π·Ρ‹Π²Π°Ρ‚ΡŒ нСбольшой ΠΌΠ°Π³Π½ΠΈΡ‚Π½Ρ‹ΠΉ компас. Π‘ΠΎΠ²ΠΎΠΊΡƒΠΏΠ½ΠΎΡΡ‚ΡŒ Π»ΠΈΠ½ΠΈΠΉ, ΠΏΠ΅Ρ€Π΅ΡΠ΅ΠΊΠ°ΡŽΡ‰ΠΈΡ… Π΄Π°Π½Π½ΡƒΡŽ ΠΎΠ±Π»Π°ΡΡ‚ΡŒ, называСтся ΠΌΠ°Π³Π½ΠΈΡ‚Π½Ρ‹ΠΌ ΠΏΠΎΡ‚ΠΎΠΊΠΎΠΌ. Π’Π°ΠΊΠΈΠΌ ΠΎΠ±Ρ€Π°Π·ΠΎΠΌ, элСктричСскиС эффСкты Π±Ρ‹Π»ΠΈ ΠΎΠ±ΡŠΡΡΠ½Π΅Π½Ρ‹ Π€Π°Ρ€Π°Π΄Π΅Π΅ΠΌ ΠΈΠ·ΠΌΠ΅Π½ΡΡŽΡ‰ΠΈΠΌΡΡ ΠΌΠ°Π³Π½ΠΈΡ‚Π½Ρ‹ΠΌ ΠΏΠΎΡ‚ΠΎΠΊΠΎΠΌ. НСсколькими Π³ΠΎΠ΄Π°ΠΌΠΈ ΠΏΠΎΠ·ΠΆΠ΅ ΡˆΠΎΡ‚Π»Π°Π½Π΄ΡΠΊΠΈΠΉ Ρ„ΠΈΠ·ΠΈΠΊ ДТСймс ΠšΠ»Π΅Ρ€ΠΊ МаксвСлл ΠΏΡ€Π΅Π΄ΠΏΠΎΠ»ΠΎΠΆΠΈΠ», Ρ‡Ρ‚ΠΎ Ρ„ΡƒΠ½Π΄Π°ΠΌΠ΅Π½Ρ‚Π°Π»ΡŒΠ½Ρ‹ΠΌ эффСктом измСнСния ΠΌΠ°Π³Π½ΠΈΡ‚Π½ΠΎΠ³ΠΎ ΠΏΠΎΡ‚ΠΎΠΊΠ° являСтся созданиС элСктричСского поля Π½Π΅ Ρ‚ΠΎΠ»ΡŒΠΊΠΎ Π² ΠΏΡ€ΠΎΠ²ΠΎΠ΄Π½ΠΈΠΊΠ΅ (Π³Π΄Π΅ ΠΎΠ½ ΠΌΠΎΠΆΠ΅Ρ‚ ΠΏΡ€ΠΈΠ²ΠΎΠ΄ΠΈΡ‚ΡŒ Π² Π΄Π²ΠΈΠΆΠ΅Π½ΠΈΠ΅ элСктричСский заряд), Π½ΠΎ ΠΈ Π² космосС Π΄Π°ΠΆΠ΅ ΠΏΡ€ΠΈ отсутствии элСктричСского поля. обвинСния.МаксвСлл сформулировал матСматичСскоС Π²Ρ‹Ρ€Π°ΠΆΠ΅Π½ΠΈΠ΅, ΡΠ²ΡΠ·Ρ‹Π²Π°ΡŽΡ‰Π΅Π΅ ΠΈΠ·ΠΌΠ΅Π½Π΅Π½ΠΈΠ΅ ΠΌΠ°Π³Π½ΠΈΡ‚Π½ΠΎΠ³ΠΎ ΠΏΠΎΡ‚ΠΎΠΊΠ° с Π½Π°Π²Π΅Π΄Π΅Π½Π½ΠΎΠΉ элСктродвиТущСй силой ( E, ΠΈΠ»ΠΈ Π­Π”Π‘ ). Π­Ρ‚ΠΎ ΡΠΎΠΎΡ‚Π½ΠΎΡˆΠ΅Π½ΠΈΠ΅, извСстноС ΠΊΠ°ΠΊ Π·Π°ΠΊΠΎΠ½ ΠΈΠ½Π΄ΡƒΠΊΡ†ΠΈΠΈ ЀарадСя (Ρ‡Ρ‚ΠΎΠ±Ρ‹ ΠΎΡ‚Π»ΠΈΡ‡ΠΈΡ‚ΡŒ Π΅Π³ΠΎ ΠΎΡ‚ Π΅Π³ΠΎ Π·Π°ΠΊΠΎΠ½ΠΎΠ² элСктролиза), гласит, Ρ‡Ρ‚ΠΎ Π²Π΅Π»ΠΈΡ‡ΠΈΠ½Π° Π­Π”Π‘ , индуцированная Π² Ρ†Π΅ΠΏΠΈ, ΠΏΡ€ΠΎΠΏΠΎΡ€Ρ†ΠΈΠΎΠ½Π°Π»ΡŒΠ½Π° скорости измСнСния ΠΌΠ°Π³Π½ΠΈΡ‚Π½ΠΎΠ³ΠΎ ΠΏΠΎΡ‚ΠΎΠΊΠ°, проходящСго Ρ‡Π΅Ρ€Π΅Π· Ρ†Π΅ΠΏΡŒ. Если ΡΠΊΠΎΡ€ΠΎΡΡ‚ΡŒ измСнСния ΠΌΠ°Π³Π½ΠΈΡ‚Π½ΠΎΠ³ΠΎ ΠΏΠΎΡ‚ΠΎΠΊΠ° выраТаСтся Π² Π΅Π΄ΠΈΠ½ΠΈΡ†Π°Ρ… Π’Π΅Π±Π΅Ρ€ΠΎΠ² Π² сСкунду, Ρ‚ΠΎ навСдСнная Π­Π”Π‘ ΠΈΠΌΠ΅Π΅Ρ‚ Π΅Π΄ΠΈΠ½ΠΈΡ†Ρ‹ Π²ΠΎΠ»ΡŒΡ‚.Π—Π°ΠΊΠΎΠ½ ЀарадСя – ΠΎΠ΄Π½ΠΎ ΠΈΠ· Ρ‡Π΅Ρ‚Ρ‹Ρ€Π΅Ρ… ΡƒΡ€Π°Π²Π½Π΅Π½ΠΈΠΉ МаксвСлла, ΠΎΠΏΡ€Π΅Π΄Π΅Π»ΡΡŽΡ‰ΠΈΡ… Ρ‚Π΅ΠΎΡ€ΠΈΡŽ элСктромагнСтизма.

Π­Ρ‚Π° ΡΡ‚Π°Ρ‚ΡŒΡ Π±Ρ‹Π»Π° послСднСй Ρ€Π΅Π΄Π°ΠΊΡ†ΠΈΠ΅ΠΉ ΠΈ ΠΎΠ±Π½ΠΎΠ²Π»Π΅Π½ΠΈΠ΅ΠΌ Уильямом Π›. Π₯ошСм. Π€ΠΎΡ€ΠΌΡƒΠ»Π° ΠΌΠ°Π³Π½ΠΈΡ‚Π½ΠΎΠΉ ΠΈΠ½Π΄ΡƒΠΊΡ†ΠΈΠΈ

– CoolGyan.Org

ΠœΠ°Π³Π½ΠΈΡ‚Π½Π°Ρ индукция Π±Ρ‹Π»Π° ΠΎΡ‚ΠΊΡ€Ρ‹Ρ‚Π° Майклом Π€Π°Ρ€Π°Π΄Π΅Π΅ΠΌ Π² 1831 Π³ΠΎΠ΄Ρƒ. ПозТС МаксвСлл описал Π΅Π΅ матСматичСски, ΠΈ ΠΎΠ½Π° стала извСстна ΠΊΠ°ΠΊ Π·Π°ΠΊΠΎΠ½ ΠΈΠ½Π΄ΡƒΠΊΡ†ΠΈΠΈ ЀарадСя. Π€Π°Ρ€Π°Π΄Π΅ΠΉ ΠΏΡ€ΠΎΠ²Π΅Π» Ρ‚Ρ€ΠΈ экспСримСнта, Ρ‡Ρ‚ΠΎΠ±Ρ‹ ΠΏΠΎΠ½ΡΡ‚ΡŒ ΡΠ»Π΅ΠΊΡ‚Ρ€ΠΎΠΌΠ°Π³Π½ΠΈΡ‚Π½ΡƒΡŽ ΠΈΠ½Π΄ΡƒΠΊΡ†ΠΈΡŽ. Π—Π°Ρ‚Π΅ΠΌ Π·Π°ΠΊΠΎΠ½ ЀарадСя стал Ρ€Π΅ΡˆΠ°ΡŽΡ‰ΠΈΠΌ для понимания ΠΈΠ½Π΄ΡƒΠΊΡ†ΠΈΠΈ, которая Ρ‚Π΅ΠΏΠ΅Ρ€ΡŒ ΠΈΠΌΠ΅Π΅Ρ‚ нСсколько практичСских ΠΏΡ€ΠΈΠΌΠ΅Π½Π΅Π½ΠΈΠΉ, Π½Π°ΠΏΡ€ΠΈΠΌΠ΅Ρ€, Π² Π³Π΅Π½Π΅Ρ€Π°Ρ‚ΠΎΡ€Π°Ρ…, трансформаторах ΠΈ Ρ‚. Π”.

Π—Π°ΠΊΠΎΠ½ ΠΌΠ°Π³Π½ΠΈΡ‚Π½ΠΎΠΉ ΠΈΠ½Π΄ΡƒΠΊΡ†ΠΈΠΈ

ΠœΠ°Π³Π½ΠΈΡ‚Π½Π°Ρ индукция, Ρ‚Π°ΠΊΠΆΠ΅ называСмая элСктромагнитной ΠΈΠ½Π΄ΡƒΠΊΡ†ΠΈΠ΅ΠΉ, относится ΠΊ созданию напряТСния (ΠΈΠ»ΠΈ Π­Π”Π‘) Π½Π° элСктричСском ΠΏΡ€ΠΎΠ²ΠΎΠ΄Π½ΠΈΠΊΠ΅, ΠΏΠΎΠΌΠ΅Ρ‰Π΅Π½Π½ΠΎΠΌ Π² ΠΈΠ·ΠΌΠ΅Π½ΡΡŽΡ‰Π΅Π΅ΡΡ ΠΌΠ°Π³Π½ΠΈΡ‚Π½ΠΎΠ΅ ΠΏΠΎΠ»Π΅. Богласно Π·Π°ΠΊΠΎΠ½Ρƒ ЀарадСя, для Π·Π°ΠΌΠΊΠ½ΡƒΡ‚ΠΎΠΉ Ρ†Π΅ΠΏΠΈ индуцированная элСктродвиТущая сила Ρ€Π°Π²Π½Π° скорости измСнСния ΠΌΠ°Π³Π½ΠΈΡ‚Π½ΠΎΠ³ΠΎ ΠΏΠΎΡ‚ΠΎΠΊΠ°, Π·Π°ΠΊΠ»ΡŽΡ‡Π΅Π½Π½ΠΎΠ³ΠΎ Π² Ρ†Π΅ΠΏΠΈ. Π§Ρ‚ΠΎΠ±Ρ‹ ΡƒΠ·Π½Π°Ρ‚ΡŒ большС ΠΎ ΠΌΠ°Π³Π½ΠΈΡ‚Π½ΠΎΠΉ ΠΈΠ½Π΄ΡƒΠΊΡ†ΠΈΠΈ, посСтитС страницу «ЭлСктромагнитная индукция».

Π€ΠΎΡ€ΠΌΡƒΠ»Π° для ΠΌΠ°Π³Π½ΠΈΡ‚Π½ΠΎΠΉ ΠΈΠ½Π΄ΡƒΠΊΡ†ΠΈΠΈ

Богласно Π·Π°ΠΊΠΎΠ½Ρƒ ЀарадСя, Π­Π”Π‘, индуцированная Π² Π·Π°ΠΌΠΊΠ½ΡƒΡ‚ΠΎΠΉ Ρ†Π΅ΠΏΠΈ, Ρ€Π°Π²Π½Π° –

.

Π—Π΄Π΅ΡΡŒ

– ΠΌΠ°Π³Π½ΠΈΡ‚Π½Ρ‹ΠΉ ΠΏΠΎΡ‚ΠΎΠΊ, t – врСмя, – навСдСнная Π­Π”Π‘.

ΠŸΡ€ΠΈΠΌΠ΅Ρ‡Π°Π½ΠΈΠ΅:

Π“Π΄Π΅ B = ΠΌΠ°Π³Π½ΠΈΡ‚Π½ΠΎΠ΅ ΠΏΠΎΠ»Π΅, Π° ds – ΠΎΡ‡Π΅Π½ΡŒ малСнькая ΠΏΠ»ΠΎΡ‰Π°Π΄ΡŒ.

Π’ ΠΊΠ°Ρ‚ΡƒΡˆΠΊΠ΅ с N Π²ΠΈΡ‚ΠΊΠ°ΠΌΠΈ Π­Π”Π‘ Π±ΡƒΠ΄Π΅Ρ‚ –

.

ПозТС, согласно Π·Π°ΠΊΠΎΠ½Ρƒ Π›Π΅Π½Ρ†Π°, ΡƒΡ€Π°Π²Π½Π΅Π½ΠΈΠ΅ ЀарадСя Π±Ρ‹Π»ΠΎ ΠΈΠ·ΠΌΠ΅Π½Π΅Π½ΠΎ ΡΠΎΠΎΡ‚Π²Π΅Ρ‚ΡΡ‚Π²ΡƒΡŽΡ‰ΠΈΠΌ ΠΎΠ±Ρ€Π°Π·ΠΎΠΌ, ΠΊΠΎΡ‚ΠΎΡ€ΠΎΠ΅ Ρ‚Π΅ΠΏΠ΅Ρ€ΡŒ ΠΈΠΌΠ΅Π΅Ρ‚ Π²ΠΈΠ΄ –

Π’Π΅ΠΏΠ΅Ρ€ΡŒ это ΡƒΡ€Π°Π²Π½Π΅Π½ΠΈΠ΅ опрСдСляСт Π½Π°ΠΏΡ€Π°Π²Π»Π΅Π½ΠΈΠ΅ ΠΈΠ½Π΄ΡƒΡ†ΠΈΡ€ΠΎΠ²Π°Π½Π½ΠΎΠ³ΠΎ Ρ‚ΠΎΠΊΠ° ΠΈ подчиняСтся Π·Π°ΠΊΠΎΠ½Ρƒ сохранСния энСргии.

Для ΠΏΠΎΠ΄Π²ΠΈΠΆΠ½ΠΎΠ³ΠΎ ΠΏΡ€ΠΎΠ²ΠΎΠ΄Π½ΠΈΠΊΠ° Π­Π”Π‘ опрСдСляСтся ΠΏΠΎ Ρ„ΠΎΡ€ΠΌΡƒΠ»Π΅:

Π“Π΄Π΅ l = Π΄Π»ΠΈΠ½Π° ΠΏΡ€ΠΎΠ²ΠΎΠ΄Π½ΠΈΠΊΠ°, v = ΡΠΊΠΎΡ€ΠΎΡΡ‚ΡŒ ΠΏΡ€ΠΎΠ²ΠΎΠ΄Π½ΠΈΠΊΠ° ΠΈ ΞΈ – ΡƒΠ³ΠΎΠ» ΠΌΠ΅ΠΆΠ΄Ρƒ ΠΌΠ°Π³Π½ΠΈΡ‚Π½Ρ‹ΠΌ ΠΏΠΎΠ»Π΅ΠΌ ΠΈ Π½Π°ΠΏΡ€Π°Π²Π»Π΅Π½ΠΈΠ΅ΠΌ двиТСния.

ΠŸΡ€ΠΈΠΌΠ΅Ρ€, связанный с ΠΌΠ°Π³Π½ΠΈΡ‚Π½ΠΎΠΉ ΠΈΠ½Π΄ΡƒΠΊΡ†ΠΈΠ΅ΠΉ, ΠΏΡ€ΠΈΠ²Π΅Π΄Π΅Π½ Π½ΠΈΠΆΠ΅ для Π»ΡƒΡ‡ΡˆΠ΅Π³ΠΎ понимания.

Π Π΅ΡˆΠ΅Π½Π½Ρ‹Π΅ ΠΏΡ€ΠΈΠΌΠ΅Ρ€Ρ‹

ΠŸΡ€ΠΈΠΌΠ΅Ρ€ 1:

РассчитайтС Π½Π°Π²Π΅Π΄Π΅Π½Π½ΡƒΡŽ Π­Π”Π‘, Ссли ΠΌΠ°Π³Π½ΠΈΡ‚Π½Ρ‹ΠΉ ΠΏΠΎΡ‚ΠΎΠΊ, связанный с ΠΊΠ°Ρ‚ΡƒΡˆΠΊΠΎΠΉ, измСняСтся с 12 x 10-3 Π’Ρ‚ Π΄ΠΎ 6 x 10-3 Π’Ρ‚ Π·Π° 0,01 сСкунды.

РСшСниС:

ΠŸΡ€ΠΈΠΌΠ΅Ρ€ 2:

Π”Π»ΠΈΠ½Π½Ρ‹ΠΉ солСноид, ΠΈΠΌΠ΅ΡŽΡ‰ΠΈΠΉ 15 Π²ΠΈΡ‚ΠΊΠΎΠ² Π½Π° см ΠΈ Π½Π΅Π±ΠΎΠ»ΡŒΡˆΡƒΡŽ ΠΏΠ»ΠΎΡ‰Π°Π΄ΡŒ ΠΏΠ΅Ρ‚Π»ΠΈ 2 см2, помСщаСтся Π² солСноид ΠΎΡ‚Π½ΠΎΡΠΈΡ‚Π΅Π»ΡŒΠ½ΠΎ Π΅Π³ΠΎ оси. НайдитС Π½Π°Π²Π΅Π΄Π΅Π½Π½ΡƒΡŽ Π­Π”Π‘ Π² ΠΊΠΎΠ½Ρ‚ΡƒΡ€Π΅, ΠΊΠΎΠ³Π΄Π° Ρ‚ΠΎΠΊ, пСрСносимый солСноидом, постоянно измСняСтся ΠΎΡ‚ 2,0 А Π΄ΠΎ 4,0 А Π·Π° 0,1 с.

Π‘Π»Π΅Π΄ΠΈΡ‚Π΅ Π·Π° обновлСниями CoolGyan’S, Ρ‡Ρ‚ΠΎΠ±Ρ‹ ΡƒΠ²ΠΈΠ΄Π΅Ρ‚ΡŒ большС Ρ‚Π°ΠΊΠΈΡ… интСрСсных статСй. ΠšΡ€ΠΎΠΌΠ΅ Ρ‚ΠΎΠ³ΠΎ, Π·Π°Ρ€Π΅Π³ΠΈΡΡ‚Ρ€ΠΈΡ€ΡƒΠΉΡ‚Π΅ΡΡŒ Π² CoolGyan’S – Learning App, Ρ‡Ρ‚ΠΎΠ±Ρ‹ ΠΏΠΎΠ»ΡƒΡ‡Π°Ρ‚ΡŒ мноТСство ΠΈΠ½Ρ‚Π΅Ρ€Π°ΠΊΡ‚ΠΈΠ²Π½Ρ‹Ρ… ΡƒΠ²Π»Π΅ΠΊΠ°Ρ‚Π΅Π»ΡŒΠ½Ρ‹Ρ… Π²ΠΈΠ΄Π΅ΠΎΡ€ΠΎΠ»ΠΈΠΊΠΎΠ² ΠΏΠΎ Ρ„ΠΈΠ·ΠΈΠΊΠ΅ ΠΈ Π½Π΅ΠΎΠ³Ρ€Π°Π½ΠΈΡ‡Π΅Π½Π½ΡƒΡŽ ΠΏΠΎΠΌΠΎΡ‰ΡŒ Π² ΡƒΡ‡Π΅Π±Π΅.

Какова Ρ„ΠΎΡ€ΠΌΡƒΠ»Π° элСктромагнитной ΠΈΠ½Π΄ΡƒΠΊΡ†ΠΈΠΈ? – MVOrganizing

Какая Ρ„ΠΎΡ€ΠΌΡƒΠ»Π° элСктромагнитной ΠΈΠ½Π΄ΡƒΠΊΡ†ΠΈΠΈ?

Π—Π°ΠΊΠΎΠ½ ΠΈΠ½Π΄ΡƒΠΊΡ†ΠΈΠΈ ЀарадСя – это основной Π·Π°ΠΊΠΎΠ½ элСктромагнСтизма, ΠΊΠΎΡ‚ΠΎΡ€Ρ‹ΠΉ прСдсказываСт, ΠΊΠ°ΠΊ ΠΌΠ°Π³Π½ΠΈΡ‚Π½ΠΎΠ΅ ΠΏΠΎΠ»Π΅ Π±ΡƒΠ΄Π΅Ρ‚ Π²Π·Π°ΠΈΠΌΠΎΠ΄Π΅ΠΉΡΡ‚Π²ΠΎΠ²Π°Ρ‚ΡŒ с элСктричСской Ρ†Π΅ΠΏΡŒΡŽ, создавая ΡΠ»Π΅ΠΊΡ‚Ρ€ΠΎΠ΄Π²ΠΈΠΆΡƒΡ‰ΡƒΡŽ силу (Π­Π”Π‘).Π­Π”Π‘ = βˆ’NΔΦΔt Π­Π”Π‘ = – N Ξ” Ξ¦ Ξ” t. Π­Ρ‚Π° связь извСстна ΠΊΠ°ΠΊ Π·Π°ΠΊΠΎΠ½ ΠΈΠ½Π΄ΡƒΠΊΡ†ΠΈΠΈ ЀарадСя. Π•Π΄ΠΈΠ½ΠΈΡ†Ρ‹ измСрСния Π­Π”Π‘, ΠΊΠ°ΠΊ ΠΎΠ±Ρ‹Ρ‡Π½ΠΎ, – Π²ΠΎΠ»ΡŒΡ‚Ρ‹.

Каков ΠΏΡ€ΠΈΠ½Ρ†ΠΈΠΏ ΠΌΠ°Π³Π½ΠΈΡ‚Π½ΠΎΠ³ΠΎ поля?

ВсС ΠΌΠ°Π³Π½ΠΈΡ‚Ρ‹ ΠΈΠΌΠ΅ΡŽΡ‚ сСвСрный ΠΈ ΡŽΠΆΠ½Ρ‹ΠΉ ΠΏΠΎΠ»ΡŽΡΡ‹. ΠŸΠΎΠ΄ΠΎΠ±Π½Ρ‹Π΅ полюса ΠΎΡ‚Ρ‚Π°Π»ΠΊΠΈΠ²Π°ΡŽΡ‚ΡΡ, Π½ΠΎ ΠΏΡ€ΠΎΡ‚ΠΈΠ²ΠΎΠΏΠΎΠ»ΠΎΠΆΠ½Ρ‹Π΅ полюса ΠΏΡ€ΠΈΡ‚ΡΠ³ΠΈΠ²Π°ΡŽΡ‚ΡΡ Π΄Ρ€ΡƒΠ³ ΠΊ Π΄Ρ€ΡƒΠ³Ρƒ. Π­Π»Π΅ΠΊΡ‚Ρ€ΠΎΠ½Ρ‹ Π² Π°Ρ‚ΠΎΠΌΠ°Ρ… ΠΌΠ°Π³Π½ΠΈΡ‚ΠΎΠ² Π²Ρ€Π°Ρ‰Π°ΡŽΡ‚ΡΡ Π²ΠΎΠΊΡ€ΡƒΠ³ ядра Π² основном Π² ΠΎΠ΄Π½ΠΎΠΌ Π½Π°ΠΏΡ€Π°Π²Π»Π΅Π½ΠΈΠΈ, Ρ‚Π°ΠΊ ΡΠΎΠ·Π΄Π°ΡŽΡ‚ΡΡ Π΄Π²Π° полюса. ΠœΠ°Π³Π½ΠΈΡ‚Π½Π°Ρ сила Ρ‚Π΅Ρ‡Π΅Ρ‚ ΠΎΡ‚ сСвСрного полюса ΠΊ ΡŽΠΆΠ½ΠΎΠΌΡƒ ΠΏΠΎΠ»ΡŽΡΡƒ ΠΌΠ°Π³Π½ΠΈΡ‚Π°.

ΠšΠ°ΠΊΠΎΠ²Ρ‹ 3 Π·Π°ΠΊΠΎΠ½Π° ΠΌΠ°Π³Π½Π΅Ρ‚ΠΈΠ·ΠΌΠ°?

Π Π°Π±ΠΎΡ‚Π° элСктродвигатСлСй рСгулируСтся Ρ€Π°Π·Π»ΠΈΡ‡Π½Ρ‹ΠΌΠΈ Π·Π°ΠΊΠΎΠ½Π°ΠΌΠΈ элСктричСства ΠΈ ΠΌΠ°Π³Π½Π΅Ρ‚ΠΈΠ·ΠΌΠ°, Π²ΠΊΠ»ΡŽΡ‡Π°Ρ Π·Π°ΠΊΠΎΠ½ ΠΈΠ½Π΄ΡƒΠΊΡ†ΠΈΠΈ ЀарадСя, Π·Π°ΠΊΠΎΠ½ АмпСра, Π·Π°ΠΊΠΎΠ½ Π›Π΅Π½Ρ†Π° ΠΈ силу Π›ΠΎΡ€Π΅Π½Ρ†Π°. ΠŸΠ΅Ρ€Π²Ρ‹Π΅ Π΄Π²Π° – Π·Π°ΠΊΠΎΠ½ ЀарадСя ΠΈ Π·Π°ΠΊΠΎΠ½ АмпСра – Π²ΠΊΠ»ΡŽΡ‡Π΅Π½Ρ‹ Π² уравнСния МаксвСлла.

КакоС Π·Π½Π°Ρ‡Π΅Π½ΠΈΠ΅ ΠΈΠΌΠ΅Π΅Ρ‚ ΠΌΠ°Π³Π½ΠΈΡ‚Π½ΠΎΠ΅ ΠΏΠΎΠ»Π΅?

Π’Ρ€Π°Ρ‰Π΅Π½ΠΈΠ΅ Π—Π΅ΠΌΠ»ΠΈ Π²ΠΎΠΊΡ€ΡƒΠ³ своСй оси заставляСт эти элСктричСскиС Ρ‚ΠΎΠΊΠΈ ΠΎΠ±Ρ€Π°Π·ΠΎΠ²Ρ‹Π²Π°Ρ‚ΡŒ ΠΌΠ°Π³Π½ΠΈΡ‚Π½ΠΎΠ΅ ΠΏΠΎΠ»Π΅, ΠΊΠΎΡ‚ΠΎΡ€ΠΎΠ΅ распространяСтся Π²ΠΎΠΊΡ€ΡƒΠ³ ΠΏΠ»Π°Π½Π΅Ρ‚Ρ‹.ΠœΠ°Π³Π½ΠΈΡ‚Π½ΠΎΠ΅ ΠΏΠΎΠ»Π΅ Ρ‡Ρ€Π΅Π·Π²Ρ‹Ρ‡Π°ΠΉΠ½ΠΎ Π²Π°ΠΆΠ½ΠΎ для поддСрТания ΠΆΠΈΠ·Π½ΠΈ Π½Π° Π—Π΅ΠΌΠ»Π΅. Π‘Π΅Π· этого ΠΌΡ‹ Π±Ρ‹Π»ΠΈ Π±Ρ‹ ΠΏΠΎΠ΄Π²Π΅Ρ€ΠΆΠ΅Π½Ρ‹ Π²ΠΎΠ·Π΄Π΅ΠΉΡΡ‚Π²ΠΈΡŽ большого количСства солнСчной Ρ€Π°Π΄ΠΈΠ°Ρ†ΠΈΠΈ, ΠΈ наша атмосфСра ΠΌΠΎΠ³Π»Π° Π±Ρ‹ свободно ΠΏΡ€ΠΎΡΠ°Ρ‡ΠΈΠ²Π°Ρ‚ΡŒΡΡ Π² космос.

Каков ΠΏΡ€ΠΈΠ½Ρ†ΠΈΠΏ ΠΌΠ°Π³Π½Π΅Ρ‚ΠΈΠ·ΠΌΠ° ΠΈ элСктричСства?

ЭлСктричСство ΠΈ ΠΌΠ°Π³Π½Π΅Ρ‚ΠΈΠ·ΠΌ – Π΄Π²Π° взаимосвязанных явлСния, создаваСмых элСктромагнитной силой. ВмСстС ΠΎΠ½ΠΈ ΠΎΠ±Ρ€Π°Π·ΡƒΡŽΡ‚ элСктромагнСтизм. ДвиТущийся элСктричСский заряд создаСт ΠΌΠ°Π³Π½ΠΈΡ‚Π½ΠΎΠ΅ ΠΏΠΎΠ»Π΅. ΠœΠ°Π³Π½ΠΈΡ‚Π½ΠΎΠ΅ ΠΏΠΎΠ»Π΅ Π²Ρ‹Π·Ρ‹Π²Π°Π΅Ρ‚ Π΄Π²ΠΈΠΆΠ΅Π½ΠΈΠ΅ элСктричСского заряда, Π² Ρ€Π΅Π·ΡƒΠ»ΡŒΡ‚Π°Ρ‚Π΅ Ρ‡Π΅Π³ΠΎ Π²ΠΎΠ·Π½ΠΈΠΊΠ°Π΅Ρ‚ элСктричСский Ρ‚ΠΎΠΊ.

ЯвляСтся Π»ΠΈ элСктричСство Ρ„ΠΎΡ€ΠΌΠΎΠΉ ΠΌΠ°Π³Π½Π΅Ρ‚ΠΈΠ·ΠΌΠ°?

3) ЭлСктричСство ΠΈ ΠΌΠ°Π³Π½Π΅Ρ‚ΠΈΠ·ΠΌ – это, ΠΏΠΎ сути, Π΄Π²Π° аспСкта ΠΎΠ΄Π½ΠΎΠ³ΠΎ ΠΈ Ρ‚ΠΎΠ³ΠΎ ΠΆΠ΅, ΠΏΠΎΡ‚ΠΎΠΌΡƒ Ρ‡Ρ‚ΠΎ ΠΈΠ·ΠΌΠ΅Π½ΡΡŽΡ‰Π΅Π΅ΡΡ элСктричСскоС ΠΏΠΎΠ»Π΅ создаСт ΠΌΠ°Π³Π½ΠΈΡ‚Π½ΠΎΠ΅ ΠΏΠΎΠ»Π΅, Π° ΠΈΠ·ΠΌΠ΅Π½ΡΡŽΡ‰Π΅Π΅ΡΡ ΠΌΠ°Π³Π½ΠΈΡ‚Π½ΠΎΠ΅ ΠΏΠΎΠ»Π΅ создаСт элСктричСскоС ΠΏΠΎΠ»Π΅. (Π’ΠΎΡ‚ ΠΏΠΎΡ‡Π΅ΠΌΡƒ Ρ„ΠΈΠ·ΠΈΠΊΠΈ ΠΎΠ±Ρ‹Ρ‡Π½ΠΎ Π½Π°Π·Ρ‹Π²Π°ΡŽΡ‚ «элСктромагнСтизм» ΠΈΠ»ΠΈ «элСктромагнитныС» силы вмСстС, Π° Π½Π΅ ΠΏΠΎ ΠΎΡ‚Π΄Π΅Π»ΡŒΠ½ΠΎΡΡ‚ΠΈ.)

Π’ Ρ‡Π΅ΠΌ основныС сходства ΠΈ различия ΠΌΠ΅ΠΆΠ΄Ρƒ элСктричСским ΠΏΠΎΠ»Π΅ΠΌ ΠΈ ΠΌΠ°Π³Π½ΠΈΡ‚Π½Ρ‹ΠΌ ΠΏΠΎΠ»Π΅ΠΌ?

Бходства ΠΌΠ΅ΠΆΠ΄Ρƒ ΠΌΠ°Π³Π½ΠΈΡ‚Π½Ρ‹ΠΌΠΈ полями ΠΈ элСктричСскими полями: элСктричСскиС поля производятся двумя Π²ΠΈΠ΄Π°ΠΌΠΈ зарядов: ΠΏΠΎΠ»ΠΎΠΆΠΈΡ‚Π΅Π»ΡŒΠ½Ρ‹ΠΌ ΠΈ ΠΎΡ‚Ρ€ΠΈΡ†Π°Ρ‚Π΅Π»ΡŒΠ½Ρ‹ΠΌ.ΠœΠ°Π³Π½ΠΈΡ‚Π½Ρ‹Π΅ поля связаны с двумя ΠΌΠ°Π³Π½ΠΈΡ‚Π½Ρ‹ΠΌΠΈ полюсами, сСвСрным ΠΈ ΡŽΠΆΠ½Ρ‹ΠΌ, хотя ΠΎΠ½ΠΈ Ρ‚Π°ΠΊΠΆΠ΅ ΡΠΎΠ·Π΄Π°ΡŽΡ‚ΡΡ зарядами (Π½ΠΎ двиТущимися зарядами). Как столбы ΠΎΡ‚Ρ‚Π°Π»ΠΊΠΈΠ²Π°ΡŽΡ‚ΡΡ; Π² ΠΎΡ‚Π»ΠΈΡ‡ΠΈΠ΅ ΠΎΡ‚ полюсов ΠΏΡ€ΠΈΡ‚ΡΠ³ΠΈΠ²Π°ΡŽΡ‚.

Π§Ρ‚ΠΎ сильнСС ΠΈΠ· Π΄Π²ΡƒΡ…: элСктричСского поля ΠΈΠ»ΠΈ ΠΌΠ°Π³Π½ΠΈΡ‚Π½ΠΎΠ³ΠΎ поля?

Бпособ взаимодСйствия зарядов ΠΈ Ρ‚ΠΎΠΊΠΎΠ² с элСктромагнитным ΠΏΠΎΠ»Π΅ΠΌ описываСтся уравнСниями МаксвСлла ΠΈ Π·Π°ΠΊΠΎΠ½ΠΎΠΌ силы Π›ΠΎΡ€Π΅Π½Ρ†Π°. Π‘ΠΈΠ»Π°, создаваСмая элСктричСским ΠΏΠΎΠ»Π΅ΠΌ, Π½Π°ΠΌΠ½ΠΎΠ³ΠΎ сильнСС, Ρ‡Π΅ΠΌ сила, создаваСмая ΠΌΠ°Π³Π½ΠΈΡ‚Π½Ρ‹ΠΌ ΠΏΠΎΠ»Π΅ΠΌ.

Π§Ρ‚ΠΎ ΠΎΠ±ΠΎΠ·Π½Π°Ρ‡Π°ΡŽΡ‚ Π»ΠΈΠ½ΠΈΠΈ Π½Π° Π΄ΠΈΠ°Π³Ρ€Π°ΠΌΠΌΠ΅ элСктричСского поля?

Π›ΠΈΠ½ΠΈΠΈ элСктричСского поля Ρ€Π°ΡΠΊΡ€Ρ‹Π²Π°ΡŽΡ‚ ΠΈΠ½Ρ„ΠΎΡ€ΠΌΠ°Ρ†ΠΈΡŽ ΠΎ Π½Π°ΠΏΡ€Π°Π²Π»Π΅Π½ΠΈΠΈ (ΠΈ силС) элСктричСского поля Π² ΠΎΠΏΡ€Π΅Π΄Π΅Π»Π΅Π½Π½ΠΎΠΉ области пространства. Если Π»ΠΈΠ½ΠΈΠΈ ΠΏΠ΅Ρ€Π΅ΡΠ΅ΠΊΠ°ΡŽΡ‚ΡΡ Π΄Ρ€ΡƒΠ³ с Π΄Ρ€ΡƒΠ³ΠΎΠΌ Π² Π΄Π°Π½Π½ΠΎΠΌ мСстС, Ρ‚ΠΎΠ³Π΄Π° Π΄ΠΎΠ»ΠΆΠ½ΠΎ Π±Ρ‹Ρ‚ΡŒ Π΄Π²Π° Ρ‡Π΅Ρ‚ΠΊΠΎ Ρ€Π°Π·Π½Ρ‹Ρ… значСния элСктричСского поля с ΠΈΡ… собствСнным ΠΈΠ½Π΄ΠΈΠ²ΠΈΠ΄ΡƒΠ°Π»ΡŒΠ½Ρ‹ΠΌ Π½Π°ΠΏΡ€Π°Π²Π»Π΅Π½ΠΈΠ΅ΠΌ Π² этом Π·Π°Π΄Π°Π½Π½ΠΎΠΌ мСстС.

ΠšΠ°ΠΊΠΎΠ²Ρ‹ 4 основных ΠΏΡ€Π°Π²ΠΈΠ»Π° построСния Π΄ΠΈΠ°Π³Ρ€Π°ΠΌΠΌ силовых Π»ΠΈΠ½ΠΈΠΉ элСктричСского поля?

1 ΠΎΡ‚Π²Π΅Ρ‚

  • 1) Π›ΠΈΠ½ΠΈΠΈ элСктричСского поля всСгда ΠΏΡ€ΠΎΡ‚ΡΠ³ΠΈΠ²Π°ΡŽΡ‚ΡΡ ΠΎΡ‚ высокого ΠΏΠΎΡ‚Π΅Π½Ρ†ΠΈΠ°Π»Π° ΠΊ.
  • 2) Π”Π²Π΅ силовыС Π»ΠΈΠ½ΠΈΠΈ элСктричСского поля Π½ΠΈΠΊΠΎΠ³Π΄Π° Π½Π΅ ΠΌΠΎΠ³ΡƒΡ‚ ΠΏΠ΅Ρ€Π΅ΡΠ΅ΠΊΠ°Ρ‚ΡŒΡΡ Π΄Ρ€ΡƒΠ³ с Π΄Ρ€ΡƒΠ³ΠΎΠΌ.
  • 3) ЧистоС элСктричСскоС ΠΏΠΎΠ»Π΅ Π²Π½ΡƒΡ‚Ρ€ΠΈ ΠΏΡ€ΠΎΠ²ΠΎΠ΄Π½ΠΈΠΊΠ° Ρ€Π°Π²Π½ΠΎ Π½ΡƒΠ»ΡŽ.
  • 4) Линия элСктричСского поля ΠΎΡ‚ ΠΏΠΎΠ»ΠΎΠΆΠΈΡ‚Π΅Π»ΡŒΠ½ΠΎΠ³ΠΎ заряда тянСтся Ρ€Π°Π΄ΠΈΠ°Π»ΡŒΠ½ΠΎ Π½Π°Ρ€ΡƒΠΆΡƒ, Π° ΠΎΡ‚ ΠΎΡ‚Ρ€ΠΈΡ†Π°Ρ‚Π΅Π»ΡŒΠ½ΠΎΠ³ΠΎ заряда – Ρ€Π°Π΄ΠΈΠ°Π»ΡŒΠ½ΠΎ Π²Π½ΡƒΡ‚Ρ€ΡŒ.

Как ΠΎΠΏΡ€Π΅Π΄Π΅Π»ΠΈΡ‚ΡŒ элСктричСскоС ΠΏΠΎΠ»Π΅?

ЭлСктричСскоС ΠΏΠΎΠ»Π΅, элСктричСскоС свойство, связанноС с ΠΊΠ°ΠΆΠ΄ΠΎΠΉ Ρ‚ΠΎΡ‡ΠΊΠΎΠΉ Π² ​​пространствС, ΠΊΠΎΠ³Π΄Π° заряд присутствуСт Π² любой Ρ„ΠΎΡ€ΠΌΠ΅. Π’Π΅Π»ΠΈΡ‡ΠΈΠ½Π° ΠΈ Π½Π°ΠΏΡ€Π°Π²Π»Π΅Π½ΠΈΠ΅ элСктричСского поля Π²Ρ‹Ρ€Π°ΠΆΠ°ΡŽΡ‚ΡΡ Π²Π΅Π»ΠΈΡ‡ΠΈΠ½ΠΎΠΉ E, Π½Π°Π·Ρ‹Π²Π°Π΅ΠΌΠΎΠΉ Π½Π°ΠΏΡ€ΡΠΆΠ΅Π½Π½ΠΎΡΡ‚ΡŒΡŽ элСктричСского поля ΠΈΠ»ΠΈ Π½Π°ΠΏΡ€ΡΠΆΠ΅Π½Π½ΠΎΡΡ‚ΡŒΡŽ элСктричСского поля ΠΈΠ»ΠΈ просто элСктричСским ΠΏΠΎΠ»Π΅ΠΌ.

Как ΠΎΠΏΡ€Π΅Π΄Π΅Π»ΠΈΡ‚ΡŒ Π½Π°ΠΏΡ€Π°Π²Π»Π΅Π½ΠΈΠ΅ элСктричСского поля?

ЭлСктричСскоС ΠΏΠΎΠ»Π΅ опрСдСляСтся ΠΊΠ°ΠΊ элСктричСская сила Π½Π° Π΅Π΄ΠΈΠ½ΠΈΡ†Ρƒ заряда. НаправлСниС поля принимаСтся Π·Π° Π½Π°ΠΏΡ€Π°Π²Π»Π΅Π½ΠΈΠ΅ силы, ΠΊΠΎΡ‚ΠΎΡ€ΠΎΠ΅ ΠΎΠ½ΠΎ Π±ΡƒΠ΄Π΅Ρ‚ ΠΎΠΊΠ°Π·Ρ‹Π²Π°Ρ‚ΡŒ Π½Π° ΠΏΠΎΠ»ΠΎΠΆΠΈΡ‚Π΅Π»ΡŒΠ½Ρ‹ΠΉ ΠΈΡΠΏΡ‹Ρ‚Π°Ρ‚Π΅Π»ΡŒΠ½Ρ‹ΠΉ заряд. ЭлСктричСскоС ΠΏΠΎΠ»Π΅ Π½Π°ΠΏΡ€Π°Π²Π»Π΅Π½ΠΎ Ρ€Π°Π΄ΠΈΠ°Π»ΡŒΠ½ΠΎ Π½Π°Ρ€ΡƒΠΆΡƒ ΠΎΡ‚ ΠΏΠΎΠ»ΠΎΠΆΠΈΡ‚Π΅Π»ΡŒΠ½ΠΎΠ³ΠΎ заряда ΠΈ Ρ€Π°Π΄ΠΈΠ°Π»ΡŒΠ½ΠΎ Π²Π½ΡƒΡ‚Ρ€ΡŒ ΠΊ ΠΎΡ‚Ρ€ΠΈΡ†Π°Ρ‚Π΅Π»ΡŒΠ½ΠΎΠΌΡƒ Ρ‚ΠΎΡ‡Π΅Ρ‡Π½ΠΎΠΌΡƒ заряду.

Как ΠΎΡ‚Ρ€ΠΈΡ†Π°Ρ‚Π΅Π»ΡŒΠ½Ρ‹ΠΉ тСстовый заряд Π±ΡƒΠ΄Π΅Ρ‚ вСсти сСбя Π² элСктричСском ΠΏΠΎΠ»Π΅?

ΠžΡ‚Ρ€ΠΈΡ†Π°Ρ‚Π΅Π»ΡŒΠ½Ρ‹Π΅ заряды Π² ΠΏΡ€ΠΎΠ²ΠΎΠ΄Π½ΠΈΠΊΠ΅ Π²Ρ‹Ρ€Π°Π²Π½ΠΈΠ²Π°ΡŽΡ‚ΡΡ ΠΏΠΎ Π½Π°ΠΏΡ€Π°Π²Π»Π΅Π½ΠΈΡŽ ΠΊ ΠΏΠΎΠ»ΠΎΠΆΠΈΡ‚Π΅Π»ΡŒΠ½ΠΎΠΌΡƒ ΠΊΠΎΠ½Ρ†Ρƒ элСктричСского поля, оставляя ΠΏΠΎΠ»ΠΎΠΆΠΈΡ‚Π΅Π»ΡŒΠ½Ρ‹Π΅ заряды Π½Π° ΠΎΡ‚Ρ€ΠΈΡ†Π°Ρ‚Π΅Π»ΡŒΠ½ΠΎΠΌ ΠΊΠΎΠ½Ρ†Π΅ поля.

ЭлСктричСская сила ΠΏΠΎΠ»ΠΎΠΆΠΈΡ‚Π΅Π»ΡŒΠ½Π° ΠΈΠ»ΠΈ ΠΎΡ‚Ρ€ΠΈΡ†Π°Ρ‚Π΅Π»ΡŒΠ½Π°?

Богласно ΠšΡƒΠ»ΠΎΠ½Ρƒ, элСктричСская сила для покоящихся зарядов ΠΈΠΌΠ΅Π΅Ρ‚ ΡΠ»Π΅Π΄ΡƒΡŽΡ‰ΠΈΠ΅ свойства: ΠΎΠ΄ΠΈΠ½Π°ΠΊΠΎΠ²Ρ‹Π΅ заряды ΠΎΡ‚Ρ‚Π°Π»ΠΊΠΈΠ²Π°ΡŽΡ‚ΡΡ Π΄Ρ€ΡƒΠ³ ΠΎΡ‚ Π΄Ρ€ΡƒΠ³Π°; Π² ΠΎΡ‚Π»ΠΈΡ‡ΠΈΠ΅ ΠΎΡ‚ ΠΎΠ±Π²ΠΈΠ½Π΅Π½ΠΈΠΉ ΠΏΡ€ΠΈΠ²Π»Π΅ΠΊΠ°ΡŽΡ‚. Π’Π°ΠΊΠΈΠΌ ΠΎΠ±Ρ€Π°Π·ΠΎΠΌ, Π΄Π²Π° ΠΎΡ‚Ρ€ΠΈΡ†Π°Ρ‚Π΅Π»ΡŒΠ½Ρ‹Ρ… заряда ΠΎΡ‚Ρ‚Π°Π»ΠΊΠΈΠ²Π°ΡŽΡ‚ΡΡ Π΄Ρ€ΡƒΠ³ ΠΎΡ‚ Π΄Ρ€ΡƒΠ³Π°, Π° ΠΏΠΎΠ»ΠΎΠΆΠΈΡ‚Π΅Π»ΡŒΠ½Ρ‹ΠΉ заряд притягиваСт ΠΎΡ‚Ρ€ΠΈΡ†Π°Ρ‚Π΅Π»ΡŒΠ½Ρ‹ΠΉ. ΠŸΡ€ΠΈΡ‚ΡΠΆΠ΅Π½ΠΈΠ΅ ΠΈΠ»ΠΈ ΠΎΡ‚Ρ‚Π°Π»ΠΊΠΈΠ²Π°Π½ΠΈΠ΅ дСйствуСт вдоль Π»ΠΈΠ½ΠΈΠΈ ΠΌΠ΅ΠΆΠ΄Ρƒ двумя зарядами.

Как Ρ€Π΅ΡˆΠΈΡ‚ΡŒ ΠΏΡ€ΠΎΠ±Π»Π΅ΠΌΡƒ элСктричСского заряда?

Если Π²Π°ΠΌ извСстСн элСктричСский Ρ‚ΠΎΠΊ, ΠΏΠΎΡ‚ΠΎΠΊ элСктричСского заряда Ρ‡Π΅Ρ€Π΅Π· ΠΎΠ±ΡŠΠ΅ΠΊΡ‚, проходящий ΠΏΠΎ Ρ†Π΅ΠΏΠΈ ΠΈ ΠΏΡ€ΠΎΠ΄ΠΎΠ»ΠΆΠΈΡ‚Π΅Π»ΡŒΠ½ΠΎΡΡ‚ΡŒ дСйствия Ρ‚ΠΎΠΊΠ°, Π²Ρ‹ ΠΌΠΎΠΆΠ΅Ρ‚Π΅ Ρ€Π°ΡΡΡ‡ΠΈΡ‚Π°Ρ‚ΡŒ элСктричСский заряд, ΠΈΡΠΏΠΎΠ»ΡŒΠ·ΡƒΡ ΡƒΡ€Π°Π²Π½Π΅Π½ΠΈΠ΅ для Ρ‚ΠΎΠΊΠ° Q = It, Π² ΠΊΠΎΡ‚ΠΎΡ€ΠΎΠΌ Q – ΠΈΠ·ΠΌΠ΅Ρ€Π΅Π½Π½Ρ‹ΠΉ ΠΎΠ±Ρ‰ΠΈΠΉ заряд. Π² ΠΊΡƒΠ»ΠΎΠ½Π°Ρ…, I – Ρ‚ΠΎΠΊ Π² Π°ΠΌΠΏΠ΅Ρ€Π°Ρ…, Π° t – врСмя, когда…

15 – ЭлСктромагнитная индукция – ΠŸΡ€ΠΈΠΌΠ΅Ρ‡Π°Π½ΠΈΡ ΠΊ Π½Π°Ρ‡Π°Π»ΡŒΠ½Ρ‹ΠΌ испытаниям


  • ΠžΡ€Π΅ΡΡ‚Π΅Π΄ ΠΎΠ±Π½Π°Ρ€ΡƒΠΆΠΈΠ» ΠΌΠ°Π³Π½ΠΈΡ‚Π½ΠΎΠ΅ ΠΏΠΎΠ»Π΅ ΠΈΠ·-Π·Π° ΠΊΠ°Ρ‚ΡƒΡˆΠΊΠΈ с Ρ‚ΠΎΠΊΠΎΠΌ.
  • Π€Π°Ρ€Π°Π΄Π΅ΠΉ ΠΎΡ‚ΠΊΡ€Ρ‹Π» ΡΠ»Π΅ΠΊΡ‚Ρ€ΠΎΠΌΠ°Π³Π½ΠΈΡ‚Π½ΡƒΡŽ ΠΈΠ½Π΄ΡƒΠΊΡ†ΠΈΡŽ (производство Π½Π°Π²Π΅Π΄Π΅Π½Π½ΠΎΠΉ Π­Π”Π‘ ΠΈ ΠΈΠ½Π΄ΡƒΡ†ΠΈΡ€ΠΎΠ²Π°Π½Π½ΠΎΠ³ΠΎ Ρ‚ΠΎΠΊΠ° Π² зависимости ΠΎΡ‚ скорости измСнСния ΠΌΠ°Π³Π½ΠΈΡ‚Π½ΠΎΠ³ΠΎ ΠΏΠΎΡ‚ΠΎΠΊΠ° Π²ΠΎ Π²Ρ€Π΅ΠΌΠ΅Π½ΠΈ).
  • НавСдСнный Ρ‚ΠΎΠΊ зависит ΠΎΡ‚ скорости, с ΠΊΠΎΡ‚ΠΎΡ€ΠΎΠΉ двиТСтся ΠΏΡ€ΠΎΠ²ΠΎΠ΄Π½ΠΈΠΊ, ΠΈ ΠΎΡ‚ сопротивлСния ΠΊΠΎΠ½Ρ‚ΡƒΡ€Π°.
  • НавСдСнный Ρ‚ΠΎΠΊ ΠΌΠΎΠΆΠ½ΠΎ ΡƒΠ²Π΅Π»ΠΈΡ‡ΠΈΡ‚ΡŒ:
    • с ΠΏΠΎΠΌΠΎΡ‰ΡŒΡŽ Π±ΠΎΠ»Π΅Π΅ сильного ΠΌΠ°Π³Π½ΠΈΡ‚Π½ΠΎΠ³ΠΎ поля.
    • ΠΏΠ΅Ρ€Π΅ΠΌΠ΅Ρ‰Π΅Π½ΠΈΠ΅ ΠΏΠ΅Ρ‚Π»ΠΈ быстрСС
    • Π—Π°ΠΌΠ΅Π½Π° ΠΏΠ΅Ρ‚Π»ΠΈ Π½Π° ΠΌΠ½ΠΎΠ³ΠΎΠΎΠ±ΠΎΡ€ΠΎΡ‚Π½ΡƒΡŽ ΠΊΠ°Ρ‚ΡƒΡˆΠΊΡƒ

Π˜Π½Ρ„ΠΎΡ€ΠΌΠ°Ρ†ΠΈΡ

ΠŸΠΎΡΠΊΠΎΠ»ΡŒΠΊΡƒ ΠΌΠ°Π³Π½ΠΈΡ‚Π½Ρ‹ΠΉ ΠΏΠΎΡ‚ΠΎΠΊ Ρ€Π°Π²Π΅Π½ $$ \ phi = BA \ cos \ theta $$ Π’Π°ΠΊΠΈΠΌ ΠΎΠ±Ρ€Π°Π·ΠΎΠΌ, ΠΈΠ·ΠΌΠ΅Π½Π΅Π½ΠΈΠ΅ ΠΌΠ°Π³Π½ΠΈΡ‚Π½ΠΎΠ³ΠΎ ΠΏΠΎΡ‚ΠΎΠΊΠ° ΠΌΠΎΠΆΠ΅Ρ‚ Π±Ρ‹Ρ‚ΡŒ Π²Ρ‹Π·Π²Π°Π½ΠΎ ΡΠ»Π΅Π΄ΡƒΡŽΡ‰ΠΈΠΌ ΠΎΠ±Ρ€Π°Π·ΠΎΠΌ:

  • ИзмСнСниС напряТСнности ΠΌΠ°Π³Π½ΠΈΡ‚Π½ΠΎΠ³ΠΎ поля
  • ИзмСнСниС ΠΏΠ»ΠΎΡ‰Π°Π΄ΠΈ ΠΏΡ€ΠΎΠ΅ΠΊΡ†ΠΈΠΈ.
  • ИзмСнСниС ΠΎΡ€ΠΈΠ΅Π½Ρ‚Π°Ρ†ΠΈΠΈ ΠΊΠ°Ρ‚ΡƒΡˆΠΊΠΈ Π² ΠΏΠΎΠ»Π΅.

Π—Π°ΠΊΠΎΠ½ ЀарадСя

ΠŸΠ΅Ρ€Π²Ρ‹ΠΉ Π·Π°ΠΊΠΎΠ½

Π˜Π½Π΄ΡƒΡ†ΠΈΡ€ΠΎΠ²Π°Π½Π½Π°Ρ Π­Π”Π‘ длится Π΄ΠΎ Ρ‚Π΅Ρ… ΠΏΠΎΡ€, ΠΏΠΎΠΊΠ° Π³Ρ€ΠΈΠΏΠΏ Π½Π΅ измСнится.

Π’Ρ‚ΠΎΡ€ΠΎΠΉ Π·Π°ΠΊΠΎΠ½

Π˜Π½Π΄ΡƒΡ†ΠΈΡ€ΠΎΠ²Π°Π½Π½Π°Ρ Π­Π”Π‘ прямо ΠΏΡ€ΠΎΠΏΠΎΡ€Ρ†ΠΈΠΎΠ½Π°Π»ΡŒΠ½Π° скорости измСнСния ΠΌΠ°Π³Π½ΠΈΡ‚Π½ΠΎΠ³ΠΎ ΠΏΠΎΡ‚ΠΎΠΊΠ° Π²ΠΎ Π²Ρ€Π΅ΠΌΠ΅Π½ΠΈ, Ρ‚.Π΅. $$ \ epsilon = -N \ frac {\ Delta \ phi_B} {\ Delta t} $$

  • НаправлСниС ΠΈΠ½Π΄ΡƒΡ†ΠΈΡ€ΠΎΠ²Π°Π½Π½ΠΎΠ³ΠΎ f опрСдСляСтся Π·Π°ΠΊΠΎΠ½ΠΎΠΌ Π›Π΅Π½Ρ†Π° , ΠΊΠΎΡ‚ΠΎΡ€Ρ‹ΠΉ устанавливаСт

Π—Π°ΠΊΠΎΠ½ Π›Π΅Π½Ρ†Π°

Π­Π”Π‘ индуцируСтся Ρ‚Π°ΠΊΠΈΠΌ ΠΎΠ±Ρ€Π°Π·ΠΎΠΌ, Ρ‡Ρ‚ΠΎ противодСйствуСт своСй собствСнной ΠΏΡ€ΠΈΡ‡ΠΈΠ½Π΅ Π³Π΅Π½Π΅Ρ€Π°Ρ†ΠΈΠΈ.

  • ΠœΠ΅Ρ…Π°Π½ΠΈΠ·ΠΌ Π³Π΅Π½Π΅Ρ€Π°Ρ†ΠΈΠΈ Π½Π°Π²Π΅Π΄Π΅Π½Π½ΠΎΠΉ Π­Π”Π‘ Ρ‚Π°ΠΊΠΎΠ², Ρ‡Ρ‚ΠΎ энСргия, подаваСмая для измСнСния ΠΏΠΎΡ‚ΠΎΠΊΠ°, прСвращаСтся Π² ΠΌΠ°Π³Π½ΠΈΡ‚Π½ΡƒΡŽ силу, которая ΠΏΠ΅Ρ€Π΅ΠΌΠ΅Ρ‰Π°Π΅Ρ‚ свободныС элСктроны, вызывая Ρ‚ΠΎΠΊ ΠΈ, ΡΠ»Π΅Π΄ΠΎΠ²Π°Ρ‚Π΅Π»ΡŒΠ½ΠΎ, ΠΈΠ½Π΄ΡƒΡ†ΠΈΡ€ΠΎΠ²Π°Π½Π½ΡƒΡŽ Π­Π”Π‘.
  • Π‘Π»Π΅Π΄ΡƒΡŽΡ‰ΠΈΠ΅ способы ΠΌΠΎΠ³ΡƒΡ‚ ΠΈΠ·ΠΌΠ΅Π½ΠΈΡ‚ΡŒ ΠΌΠ°Π³Π½ΠΈΡ‚Π½Ρ‹ΠΉ ΠΏΠΎΡ‚ΠΎΠΊ / ΠΌΠΎΠ³ΡƒΡ‚ Π²Ρ‹Π·Π²Π°Ρ‚ΡŒ Π½Π°Π²Π΅Π΄Π΅Π½Π½ΡƒΡŽ Π­Π”Π‘.
    • Π”Π²ΠΈΠΆΠ΅Π½ΠΈΠ΅ ΠΌΠ°Π³Π½ΠΈΡ‚Π°
    • Π”Π²ΠΈΠΆΠ΅Π½ΠΈΠ΅ ΠΊΠ°Ρ‚ΡƒΡˆΠΊΠΈ
    • ΠžΡ‚Π½ΠΎΡΠΈΡ‚Π΅Π»ΡŒΠ½ΠΎΠ΅ Π΄Π²ΠΈΠΆΠ΅Π½ΠΈΠ΅ ΠΊΠ°Ρ‚ΡƒΡˆΠΊΠΈ ΠΈ ΠΌΠ°Π³Π½ΠΈΡ‚Π°
    • ИзмСнСниС Ρ‚ΠΎΠΊΠ°
  • Если Π½Π΅ сущСствуСт Π·Π°ΠΊΠΎΠ½Π° Π›Π΅Π½Ρ†Π°, Ρ‚ΠΎ Π²ΠΎΠ·ΠΌΠΎΠΆΠ΅Π½ Π²Π΅Ρ‡Π½Ρ‹ΠΉ Π΄Π²ΠΈΠ³Π°Ρ‚Π΅Π»ΡŒ Ρ‚ΠΈΠΏΠ° 1 st .Π’Π°ΠΊΠΈΠΌ ΠΎΠ±Ρ€Π°Π·ΠΎΠΌ, процСсс Π±ΡƒΠ΄Π΅Ρ‚ бСсконСчным.

Π”Π²ΠΈΠΆΠ΅Π½ΠΈΠ΅ Π­Π”Π‘

  • Π­Π”Π‘, вызванная Π΄Π²ΠΈΠΆΠ΅Π½ΠΈΠ΅ΠΌ ΠΏΡ€ΠΎΠ²ΠΎΠ΄Π½ΠΈΠΊΠ° ΠΏΠΎΠΏΠ΅Ρ€Π΅ΠΊ ΠΌΠ°Π³Π½ΠΈΡ‚Π½ΠΎΠ³ΠΎ поля, называСтся Π­Π”Π‘ двиТСния. $$ \ epsilon _ {(motional)} = – vBL $$ Π³Π΄Π΅ \ (v \) – ΡΠΊΠΎΡ€ΠΎΡΡ‚ΡŒ ΠΏΡ€ΠΎΠ²ΠΎΠ΄Π½ΠΈΠΊΠ°, \ (L \) – Π΄Π»ΠΈΠ½Π° ΠΏΡ€ΠΎΠ²ΠΎΠ΄Π½ΠΈΠΊΠ°

  • Если Π΅ΡΡ‚ΡŒ нСкоторая ориСнтация ΠΊΠ°Ρ‚ΡƒΡˆΠΊΠΈ ΠΎΡ‚Π½ΠΎΡΠΈΡ‚Π΅Π»ΡŒΠ½ΠΎ ΠΌΠ°Π³Π½ΠΈΡ‚Π½ΠΎΠ³ΠΎ поля, Ρ‚ΠΎ $$ \ epsilon _ {(motional)} = – vBL \ sin \ theta $$, Π³Π΄Π΅ \ (\ theta \) – ΡƒΠ³ΠΎΠ» ΠΌΠ΅ΠΆΠ΄Ρƒ \ (v \) & \ ( Π’ \).

  • Π•Π΄ΠΈΠ½ΠΈΡ†Π° Π΄Π²ΠΈΠ³Π°Ρ‚Π΅Π»ΡŒΠ½ΠΎΠΉ Π­Π”Π‘ – \ (Π²ΠΎΠ»ΡŒΡ‚ \).
  • ΠŸΡ€ΠΈ Π²Ρ‹Π²ΠΎΠ΄Π΅ Π²Ρ‹ΡˆΠ΅ΡƒΠΊΠ°Π·Π°Π½Π½ΠΎΠ³ΠΎ выраТСния Π½Π°ΠΏΡ€ΡΠΆΠ΅Π½Π½ΠΎΡΡ‚ΡŒ ΠΌΠ°Π³Π½ΠΈΡ‚Π½ΠΎΠ³ΠΎ поля считаСтся ΠΎΠ΄Π½ΠΎΡ€ΠΎΠ΄Π½ΠΎΠΉ.

Взаимная индукция ΠΈ самоиндукция

  • Взаимная индукция – это явлСниС, ΠΏΡ€ΠΈ ΠΊΠΎΡ‚ΠΎΡ€ΠΎΠΌ ΠΈΠ·ΠΌΠ΅Π½ΡΡŽΡ‰ΠΈΠΉΡΡ Ρ‚ΠΎΠΊ Π² ΠΎΠ΄Π½ΠΎΠΉ ΠΊΠ°Ρ‚ΡƒΡˆΠΊΠ΅ ΠΈΠ½Π΄ΡƒΡ†ΠΈΡ€ΡƒΠ΅Ρ‚ Π­Π”Π‘ Π² Π΄Ρ€ΡƒΠ³ΠΎΠΉ ΠΊΠ°Ρ‚ΡƒΡˆΠΊΠ΅. $$ \ epsilon_ {sec} = – M \ frac {\ Delta I_p} {\ Delta t} $$ Π“Π΄Π΅ \ (M = N_s \ phi_s / I_p \) называСтся Π²Π·Π°ΠΈΠΌΠ½ΠΎΠΉ ΠΈΠ½Π΄ΡƒΠΊΡ‚ΠΈΠ²Π½ΠΎΡΡ‚ΡŒΡŽ.

Взаимная ΠΈΠ½Π΄ΡƒΠΊΡ‚ΠΈΠ²Π½ΠΎΡΡ‚ΡŒ

ΠžΡ‚Π½ΠΎΡˆΠ΅Π½ΠΈΠ΅ ΠΎΠ±Ρ€Π°Ρ‚Π½ΠΎΠΉ Π­Π”Π‘ Π²ΠΎ Π²Ρ‚ΠΎΡ€ΠΈΡ‡Π½ΠΎΠΉ ΠΎΠ±ΠΌΠΎΡ‚ΠΊΠ΅ ΠΊ Ρ€Π΅Π°ΠΊΡ†ΠΈΠΈ измСнСния Ρ‚ΠΎΠΊΠ° Π² ΠΏΠ΅Ρ€Π²ΠΈΡ‡Π½ΠΎΠΉ ΠΎΠ±ΠΌΠΎΡ‚ΠΊΠ΅.$$ M = – \ frac {\ epsilon} {\ Delta I_p / \ Delta t} $$

  • Бамоиндукция – это явлСниС Π³Π΅Π½Π΅Ρ€Π°Ρ†ΠΈΠΈ Π½Π°Π²Π΅Π΄Π΅Π½Π½ΠΎΠΉ Π­Π”Π‘ Π² ΠΊΠ°Ρ‚ΡƒΡˆΠΊΠ΅ ΠΈΠ·-Π·Π° измСнСния самого Ρ‚ΠΎΠΊΠ°. $$ \ epsilon_ {self} = – L \ frac {\ Delta I} {\ Delta t} $$, Π³Π΄Π΅ \ (L \) называСтся самоиндукциСй. Π’Π°ΠΊΠΆΠ΅ \ (N \ phi = LI \)

БобствСнная ΠΈΠ½Π΄ΡƒΠΊΡ‚ΠΈΠ²Π½ΠΎΡΡ‚ΡŒ

ΠžΡ‚Π½ΠΎΡˆΠ΅Π½ΠΈΠ΅ ΠΎΠ±Ρ€Π°Ρ‚Π½ΠΎΠΉ Π­Π”Π‘ ΠΊ скорости измСнСния Ρ‚ΠΎΠΊΠ° Π² Ρ‚ΠΎΠΉ ΠΆΠ΅ ΠΊΠ°Ρ‚ΡƒΡˆΠΊΠ΅ $$ L = \ frac {- \ epsilon _ {\ text {self}}} {\ Delta I / \ Delta t} $$

ЭнСргия, запасСнная Π² ΠΈΠ½Π΄ΡƒΠΊΡ‚ΠΎΡ€Π΅

Π˜Π½Π΄ΡƒΠΊΡ‚ΠΎΡ€ – это просто изолированная ΠΊΠ°Ρ‚ΡƒΡˆΠΊΠ°, которая обСспСчиваСт ΠΎΡ‡Π΅Π½ΡŒ ΠΌΠ°Π»ΠΎΠ΅ сопротивлСниС постоянному Ρ‚ΠΎΠΊΡƒ, Π½ΠΎ высокоС сопротивлСниС ΠΏΠ΅Ρ€Π΅ΠΌΠ΅Π½Π½ΠΎΠΌΡƒ Ρ‚ΠΎΠΊΡƒ. 2 $$

  • ΠšΠ°Ρ‚ΡƒΡˆΠΊΠ° индуктивности Π² Ρ†Π΅ΠΏΠΈ ΠΏΠ΅Ρ€Π΅ΠΌΠ΅Π½Π½ΠΎΠ³ΠΎ Ρ‚ΠΎΠΊΠ° всСгда сопротивляСтся Ρ€Π΅Π·ΠΊΠΎΠΌΡƒ измСнСнию Ρ‚ΠΎΠΊΠ°.

  • Π˜Π½Ρ„ΠΎΡ€ΠΌΠ°Ρ†ΠΈΡ

    ΠŸΠ»ΠΎΡ‚Π½ΠΎΡΡ‚ΡŒ накоплСния энСргии Π² ΠΈΠ½Π΄ΡƒΠΊΡ‚ΠΎΡ€Π΅ прямо ΠΏΡ€ΠΎΠΏΠΎΡ€Ρ†ΠΈΠΎΠ½Π°Π»ΡŒΠ½Π° ΠΊΠ²Π°Π΄Ρ€Π°Ρ‚Ρƒ напряТСнности ΠΌΠ°Π³Π½ΠΈΡ‚Π½ΠΎΠ³ΠΎ поля ΠΏΡ€ΠΈ условии, Ρ‡Ρ‚ΠΎ всС ΠΎΡΡ‚Π°Π»ΡŒΠ½Ρ‹Π΅ Ρ„Π°ΠΊΡ‚ΠΎΡ€Ρ‹ ΠΎΡΡ‚Π°ΡŽΡ‚ΡΡ постоянными.

    ΠŸΠΎΡΠ»Π΅Π΄ΠΎΠ²Π°Ρ‚Π΅Π»ΡŒΠ½Π°Ρ ΠΈ ΠΏΠ°Ρ€Π°Π»Π»Π΅Π»ΡŒΠ½Π°Ρ ΠΈΠ½Π΄ΡƒΠΊΡ‚ΠΈΠ²Π½ΠΎΡΡ‚ΡŒ

    сСрии

    Если Π΄Π²Π΅ ΠΊΠ°Ρ‚ΡƒΡˆΠΊΠΈ индуктивности \ (L_1 \) ΠΈ \ (L_2 \) соСдинСны ΠΏΠΎΡΠ»Π΅Π΄ΠΎΠ²Π°Ρ‚Π΅Π»ΡŒΠ½ΠΎ, Ρ‚ΠΎ: $$ \ epsilon = \ epsilon_1 + \ epsilon_2 \; \; \ ΠΏΠΎΠ΄Ρ€Π°Π·ΡƒΠΌΠ΅Π²Π°Π΅Ρ‚ \; \; L_s \ frac {\ Delta I} {\ Delta t} = L_1 \ frac {\ Delta I_1} {\ Delta t} + L_2 \ frac {\ Delta I_2} {\ Delta t} $$ Но ΠΏΠΎΡΠ»Π΅Π΄ΠΎΠ²Π°Ρ‚Π΅Π»ΡŒΠ½ΠΎ Ρ‚ΠΎΠΊ остаСтся Ρ‚Π°ΠΊΠΈΠΌ ΠΆΠ΅ \ (I = I_1 = I_2 \) Π‘Π»Π΅Π΄ΠΎΠ²Π°Ρ‚Π΅Π»ΡŒΠ½ΠΎ, $$ L_s = L_1 + L_2 $$

    ΠŸΠ°Ρ€Π°Π»Π»Π΅Π»ΡŒΠ½Ρ‹ΠΉ

    Π’ ΠΏΠ°Ρ€Π°Π»Π»Π΅Π»ΡŒΠ½ΠΎΠΉ ΠΊΠΎΠΌΠ±ΠΈΠ½Π°Ρ†ΠΈΠΈ Ρ‚ΠΎΠΊ дСлится Ρ‚Π°ΠΊ, $$ I = I_1 + I_2 \; \; \ ΠΏΠΎΠ΄Ρ€Π°Π·ΡƒΠΌΠ΅Π²Π°Π΅Ρ‚ \; \; \ frac {\ Delta I} {\ Delta t} = \ frac {\ Delta I_1} {\ Delta t} + \ frac {\ Delta I_2} {\ Delta t} $$ As, \ (\ frac {\ Delta I } {\ Delta t} = – \ frac {\ epsilon} {L} \) Ρ‚Π°ΠΊ Ρ‡Ρ‚ΠΎ $$ \ frac {\ epsilon} {L_p} = \ frac {\ epsilon_1} {L_1} + \ frac {\ epsilon_2} { L_2} $$ Π’Π°ΠΊΠΈΠΌ ΠΎΠ±Ρ€Π°Π·ΠΎΠΌ, \ (\ epsilon = \ epsilon_1 = \ epsilon_2 \) Π’Π°ΠΊΠΈΠΌ ΠΎΠ±Ρ€Π°Π·ΠΎΠΌ, $$ \ frac {1} {L_p} = \ frac {1} {L_1} + \ frac {1} {L_2} \; \; \ ΠΏΠΎΠ΄Ρ€Π°Π·ΡƒΠΌΠ΅Π²Π°Π΅Ρ‚ \; \; L_p = \ frac {L_1L_2} {L_1 + L_2} $$

    Π“Π΅Π½Π΅Ρ€Π°Ρ‚ΠΎΡ€ постоянного ΠΈ ΠΏΠ΅Ρ€Π΅ΠΌΠ΅Π½Π½ΠΎΠ³ΠΎ Ρ‚ΠΎΠΊΠ°

    Π“Π΅Π½Π΅Ρ€Π°Ρ‚ΠΎΡ€ Ρ‚ΠΎΠΊΠ° – это устройство, ΠΏΡ€Π΅ΠΎΠ±Ρ€Π°Π·ΡƒΡŽΡ‰Π΅Π΅ ΠΌΠ΅Ρ…Π°Π½ΠΈΡ‡Π΅ΡΠΊΡƒΡŽ ΡΠ½Π΅Ρ€Π³ΠΈΡŽ Π² ΡΠ»Π΅ΠΊΡ‚Ρ€ΠΈΡ‡Π΅ΡΠΊΡƒΡŽ Π² присутствии ΠΌΠ°Π³Π½ΠΈΡ‚Π½ΠΎΠ³ΠΎ поля.

    ΠŸΡ€ΠΈΠ½Ρ†ΠΈΠΏ

    Π­Π”Π‘ индуцируСтся ΠΏΠΎΠΏΠ΅Ρ€Π΅ΠΊ ΠΊΠ°Ρ‚ΡƒΡˆΠΊΠΈ, Π²Ρ€Π°Ρ‰Π°ΡŽΡ‰Π΅ΠΉΡΡ Π² ΠΌΠ°Π³Π½ΠΈΡ‚Π½ΠΎΠΌ ΠΏΠΎΠ»Π΅, ΠΈΠ·-Π·Π° измСнСния ΠΌΠ°Π³Π½ΠΈΡ‚Π½ΠΎΠ³ΠΎ ΠΏΠΎΡ‚ΠΎΠΊΠ°.

    ΠžΡΠ½ΠΎΠ²Π½Ρ‹Π΅ части

    • Полюса (ΠΌΠ°Π³Π½ΠΈΡ‚ U-ΠΎΠ±Ρ€Π°Π·Π½ΠΎΠΉ Ρ„ΠΎΡ€ΠΌΡ‹) с Π²ΠΎΠ³Π½ΡƒΡ‚Ρ‹ΠΌΠΈ полюсами.
    • Π―ΠΊΠΎΡ€ΡŒ (сборка ΠΊΠ°Ρ‚ΡƒΡˆΠΊΠΈ Π½Π° ΠΆΠ΅Π»Π΅Π·Π½ΠΎΠΌ Ρ†ΠΈΠ»ΠΈΠ½Π΄Ρ€Π΅)
    • ΠšΠΎΠ½Ρ‚Π°ΠΊΡ‚Π½Ρ‹Π΅ ΠΊΠΎΠ»ΡŒΡ†Π° (Π² качСствС соСдинитСля)
    • Угольная Ρ‰Π΅Ρ‚ΠΊΠ° (внСшняя поставка)

    Π Π°Π±ΠΎΡ‡ΠΈΠΉ

    • Π­Π”Π‘ индуцируСтся стороной ΠΏΠ΅Ρ‚Π»ΠΈ, ΠΏΠ΅Ρ€Π΅ΡΠ΅ΠΊΠ°ΡŽΡ‰Π΅ΠΉ ΠΌΠ°Π³Π½ΠΈΡ‚Π½ΠΎΠ΅ ΠΏΠΎΠ»Π΅.
    • Буммарная Π­Π”Π‘ для N ΠΏΠ΅Ρ‚Π΅Π»ΡŒ опрСдСляСтся ΠΊΠ°ΠΊ: $$ \ epsilon = N (2vB \ sin \ theta) \; \; \; \ тСкст {Ρ‚Π°ΠΊΠΆΠ΅} \; \; \; \ epsilon = N \ omega AB sin \ omega t $$ $$ \ epsilon_ {max} = N \ omega AB \; \; \; \ тСкст {ΠΊΠΎΠ³Π΄Π°} \; \; \; \ theta = 90 \ градус $$ Π˜Ρ‚Π°ΠΊ, $$ \ epsilon = \ epsilon_ {max} \ sin \ omega t $$ if \ (\ omega = 2 \ pi f \), Ρ‚ΠΎ $$ \ epsilon = \ epsilon_ {max} \ sin 2 \ pi ft $$ Π’ Ρ‚Π΅Ρ€ΠΌΠΈΠ½Π°Ρ… разности ΠΏΠΎΡ‚Π΅Π½Ρ†ΠΈΠ°Π»ΠΎΠ² \ (V = V_o \ sin 2 \ pi ft \)
      Π’ Ρ‚Π΅Ρ€ΠΌΠΈΠ½Π°Ρ… Ρ‚ΠΎΠΊΠ° \ (I = I_o \ sin 2 \ pi ft \)
      Π³Π΄Π΅ f – частота, с ΠΊΠΎΡ‚ΠΎΡ€ΠΎΠΉ измСняСтся Π½Π°ΠΏΡ€Π°Π²Π»Π΅Π½ΠΈΠ΅ Ρ‚ΠΎΠΊΠ°. мСняСтся.

    • Π’ ΠŸΠ°ΠΊΠΈΡΡ‚Π°Π½Π΅ \ (f = 50 Π“Ρ† \) это ΠΎΠ·Π½Π°Ρ‡Π°Π΅Ρ‚, Ρ‡Ρ‚ΠΎ 50 Ρ€Π°Π· измСняСтся Π²Ρ‚ΠΎΡ€ΠΎΠ΅ Π½Π°ΠΏΡ€Π°Π²Π»Π΅Π½ΠΈΠ΅.

    • Π’ Π³Π΅Π½Π΅Ρ€Π°Ρ‚ΠΎΡ€Π°Ρ… постоянного Ρ‚ΠΎΠΊΠ° вмСсто ΠΊΠΎΠ½Ρ‚Π°ΠΊΡ‚Π½Ρ‹Ρ… ΠΊΠΎΠ»Π΅Ρ† ΠΈΡΠΏΠΎΠ»ΡŒΠ·ΡƒΡŽΡ‚ΡΡ Ρ€Π°Π·ΡŠΠ΅ΠΌΠ½Ρ‹Π΅ ΠΊΠΎΠ»ΡŒΡ†Π° ΠΈΠ»ΠΈ ΠΊΠΎΠΌΠΌΡƒΡ‚Π°Ρ‚ΠΎΡ€Ρ‹, ΠΈΠ½Π²Π΅Ρ€Ρ‚ΠΈΡ€ΠΎΠ²Π°Π½Π½Ρ‹Π΅ Уильямом Π‘Ρ‚Π΅Ρ€Π΄ΠΆΠ΅Π½ΠΎΠΌ.
    • Π“Π΅Π½Π΅Ρ€Π°Ρ‚ΠΎΡ€ постоянного Ρ‚ΠΎΠΊΠ°
    • ΠΏΡ€ΠΎΠΈΠ·Π²ΠΎΠ΄ΠΈΡ‚ ΠΎΠ΄Π½ΠΎΠ½Π°ΠΏΡ€Π°Π²Π»Π΅Π½Π½Ρ‹ΠΉ ΠΏΡƒΠ»ΡŒΡΠΈΡ€ΡƒΡŽΡ‰ΠΈΠΉ постоянный Ρ‚ΠΎΠΊ, Π° Π½Π΅ чистый постоянный Ρ‚ΠΎΠΊ.
    • Для чистого постоянного Ρ‚ΠΎΠΊΠ° ΠΌΠ½ΠΎΠ³ΠΈΠ΅ ΠΊΠ°Ρ‚ΡƒΡˆΠΊΠΈ Π½Π°ΠΌΠΎΡ‚Π°Π½Ρ‹ Π½Π° цилиндричСский сСрдСчник.

    ΠžΠ±Ρ€Π°Ρ‚Π½Ρ‹ΠΉ ΠΌΠΎΡ‚ΠΎΡ€Π½Ρ‹ΠΉ эффСкт Π² Π³Π΅Π½Π΅Ρ€Π°Ρ‚ΠΎΡ€Π΅

    • Устройство Π² элСктричСской Ρ†Π΅ΠΏΠΈ, ΠΏΠΎΡ‚Ρ€Π΅Π±Π»ΡΡŽΡ‰Π΅Π΅ ΡΠ»Π΅ΠΊΡ‚Ρ€ΠΎΡΠ½Π΅Ρ€Π³ΠΈΡŽ, извСстно ΠΊΠ°ΠΊ Π½Π°Π³Ρ€ΡƒΠ·ΠΊΠ° .
    • Π§Π΅ΠΌ большС Π½Π°Π³Ρ€ΡƒΠ·ΠΊΠ°, Ρ‚Π΅ΠΌ больший Ρ‚ΠΎΠΊ Π²Ρ‹Π΄Π°Π΅Ρ‚ Π³Π΅Π½Π΅Ρ€Π°Ρ‚ΠΎΡ€.
    • Когда Ρ†Π΅ΠΏΡŒ Π·Π°ΠΌΠΊΠ½ΡƒΡ‚Π°, Ρ‡Π΅Ρ€Π΅Π· ΠΊΠ°Ρ‚ΡƒΡˆΠΊΡƒ ΠΏΡ€ΠΎΡ‚Π΅ΠΊΠ°Π΅Ρ‚ Ρ‚ΠΎΠΊ. ΠœΠ°Π³Π½ΠΈΡ‚Π½ΠΎΠ΅ ΠΏΠΎΠ»Π΅ дСйствуСт Π½Π° ΠΊΠ°Ρ‚ΡƒΡˆΠΊΡƒ с Ρ‚ΠΎΠΊΠΎΠΌ. Π­Ρ‚Π° сила создаСт ΠΏΡ€ΠΎΡ‚ΠΈΠ²ΠΎΠ΄Π΅ΠΉΡΡ‚Π²ΡƒΡŽΡ‰ΠΈΠΉ крутящий ΠΌΠΎΠΌΠ΅Π½Ρ‚, ΠΊΠΎΡ‚ΠΎΡ€Ρ‹ΠΉ противодСйствуСт Π²Ρ€Π°Ρ‰Π°Ρ‚Π΅Π»ΡŒΠ½ΠΎΠΌΡƒ двиТСнию ΠΊΠ°Ρ‚ΡƒΡˆΠΊΠΈ. Π­Ρ‚ΠΎΡ‚ эффСкт называСтся ΠΎΠ±Ρ€Π°Ρ‚Π½Ρ‹ΠΌ ΠΌΠΎΡ‚ΠΎΡ€Π½Ρ‹ΠΌ эффСктом Π³Π΅Π½Π΅Ρ€Π°Ρ‚ΠΎΡ€Π°.
    • Богласно Π·Π°ΠΊΠΎΠ½Ρƒ сохранСния энСргии, энСргия, потрСбляСмая Π½Π°Π³Ρ€ΡƒΠ·ΠΊΠΎΠΉ, Π΄ΠΎΠ»ΠΆΠ½Π° ΠΏΠΎΡΡ‚ΡƒΠΏΠ°Ρ‚ΡŒ ΠΈΠ· источника энСргии, ΠΈΡΠΏΠΎΠ»ΡŒΠ·ΡƒΠ΅ΠΌΠΎΠ³ΠΎ для ΠΏΡ€ΠΈΠ²ΠΎΠ΄Π° Ρ‚ΡƒΡ€Π±ΠΈΠ½.
    • Π§Π΅ΠΌ большС Ρ‚ΠΎΠΊ, потрСбляСмый Π½Π°Π³Ρ€ΡƒΠ·ΠΊΠΎΠΉ, Ρ‚Π΅ΠΌ большС создаСтся ΠΏΡ€ΠΎΡ‚ΠΈΠ²ΠΎΠ΄Π΅ΠΉΡΡ‚Π²ΡƒΡŽΡ‰ΠΈΠΉ крутящий ΠΌΠΎΠΌΠ΅Π½Ρ‚.

    Π”Π²ΠΈΠ³Π°Ρ‚Π΅Π»ΡŒ постоянного Ρ‚ΠΎΠΊΠ°

    Π”Π²ΠΈΠ³Π°Ρ‚Π΅Π»ΡŒ – это устройство, ΠΏΡ€Π΅ΠΎΠ±Ρ€Π°Π·ΡƒΡŽΡ‰Π΅Π΅ ΡΠ»Π΅ΠΊΡ‚Ρ€ΠΈΡ‡Π΅ΡΠΊΡƒΡŽ ΡΠ½Π΅Ρ€Π³ΠΈΡŽ Π² ΠΌΠ΅Ρ…Π°Π½ΠΈΡ‡Π΅ΡΠΊΡƒΡŽ. * Π“Π΅Π½Π΅Ρ€Π°Ρ‚ΠΎΡ€, Ρ€Π°Π±ΠΎΡ‚Π°ΡŽΡ‰ΠΈΠΉ Π² ΠΎΠ±Ρ€Π°Ρ‚Π½ΠΎΠΌ Π½Π°ΠΏΡ€Π°Π²Π»Π΅Π½ΠΈΠΈ, называСтся Π΄Π²ΠΈΠ³Π°Ρ‚Π΅Π»Π΅ΠΌ. * ΠŸΡ€ΠΈΠ½Ρ†ΠΈΠΏ двигатСля Π·Π°ΠΊΠ»ΡŽΡ‡Π°Π΅Ρ‚ΡΡ Π² Ρ‚ΠΎΠΌ, Ρ‡Ρ‚ΠΎ всякий Ρ€Π°Π·, ΠΊΠΎΠ³Π΄Π° ΠΊΠ°Ρ‚ΡƒΡˆΠΊΠ° с Ρ‚ΠΎΠΊΠΎΠΌ помСщаСтся Π² ΠΌΠ°Π³Π½ΠΈΡ‚Π½ΠΎΠ΅ ΠΏΠΎΠ»Π΅, ΠΎΠ½Π° испытываСт ΠΏΠ°Ρ€Ρƒ сил, ΠΊΠΎΡ‚ΠΎΡ€Ρ‹Π΅ Π²Ρ‹Π·Ρ‹Π²Π°ΡŽΡ‚ крутящий ΠΌΠΎΠΌΠ΅Π½Ρ‚.

    Π­Ρ„Ρ„Π΅ΠΊΡ‚ ΠΎΠ±Ρ€Π°Ρ‚Π½ΠΎΠΉ Π­Π”Π‘ Π² Π΄Π²ΠΈΠ³Π°Ρ‚Π΅Π»Π΅

    • Бамоиндуцированная Π­Π”Π‘ (\ (\ epsilon \)) Π² Π΄Π²ΠΈΠ³Π°Ρ‚Π΅Π»Π΅ противодСйствуСт Π½Π°ΠΏΡ€ΡΠΆΠ΅Π½ΠΈΡŽ \ (v \) Π½Π° Π΄Π²ΠΈΠ³Π°Ρ‚Π΅Π»Π΅.Π’Π°ΠΊΠΈΠΌ ΠΎΠ±Ρ€Π°Π·ΠΎΠΌ, Π­Π”Π‘ ΠΈΠ½Π΄ΡƒΠΊΡ†ΠΈΠΈ, называСтся ΠΎΠ±Ρ€Π°Ρ‚Π½ΠΎΠΉ Π­Π”Π‘ двигатСля.
    • Π’Π΅Π»ΠΈΡ‡ΠΈΠ½Π° ΠΎΠ±Ρ€Π°Ρ‚Π½ΠΎΠΉ Π­Π”Π‘ увСличиваСтся со ΡΠΊΠΎΡ€ΠΎΡΡ‚ΡŒΡŽ двигатСля.
    • \ (V \) ΠΈ \ (\ эпсилон \) находятся Π² ΠΏΡ€ΠΎΡ‚ΠΈΠ²ΠΎΠΏΠΎΠ»ΠΎΠΆΠ½ΠΎΠΉ полярности.
    • Π’ΠΎΠΊΠ°, ΠΏΡ€ΠΎΡ‚Π΅ΠΊΠ°ΡŽΡ‰ΠΈΠ΅ Π² Π΄Π²ΠΈΠ³Π°Ρ‚Π΅Π»Π΅: $$ I = \ Π³ΠΈΠ΄Ρ€ΠΎΡ€Π°Π·Ρ€Ρ‹Π²Π° {(V- \ эпсилон)} {R} \; \; \ ΠΏΠΎΠ΄Ρ€Π°Π·ΡƒΠΌΠ΅Π²Π°Π΅Ρ‚ \; \; V = \ эпсилон + IR $$

    • Как Π΄Π²ΠΈΠ³Π°Ρ‚Π΅Π»ΡŒ ускоряСт, Ρ‚ΠΎ эдс возрастаСт ΠΈ Ρ‚ΠΎΠΊ становится всС мСньшС ΠΈ мСньшС. Π’Π°ΠΊΠΈΠΌ ΠΎΠ±Ρ€Π°Π·ΠΎΠΌ, Ρ‚ΠΎΠΊ Π΄ΠΎΠ»ΠΆΠ΅Π½ Π±Ρ‹Ρ‚ΡŒ достаточным, Ρ‡Ρ‚ΠΎΠ±Ρ‹ ΠΎΠ±Π΅ΡΠΏΠ΅Ρ‡ΠΈΡ‚ΡŒ крутящий ΠΌΠΎΠΌΠ΅Π½Ρ‚.

    • Если Π΄Π²ΠΈΠ³Π°Ρ‚Π΅Π»ΡŒ ΠΏΠ΅Ρ€Π΅Π³Ρ€ΡƒΠΆΠ΅Π½, Ρ‚ΠΎ эдс ΡƒΠΌΠ΅Π½ΡŒΡˆΠ°Π΅Ρ‚ΡΡ ΠΈ ΠΏΠΎΠ·Π²ΠΎΠ»ΠΈΡ‚ΡŒ Π΄Π²ΠΈΠ³Π°Ρ‚Π΅Π»ΡŽ ΠΏΡ€ΠΈΠ²Π»Π΅Ρ‡ΡŒ большС Ρ‚ΠΎΠΊΠ°.
    • Если Π΄Π²ΠΈΠ³Π°Ρ‚Π΅Π»ΡŒ ΠΏΠ΅Ρ€Π΅Π³Ρ€ΡƒΠΆΠ΅Π½ свСрх допустимых ΠΏΡ€Π΅Π΄Π΅Π»ΠΎΠ², Ρ‚ΠΎΠΊ ΠΌΠΎΠΆΠ΅Ρ‚ Π±Ρ‹Ρ‚ΡŒ Π½Π°ΡΡ‚ΠΎΠ»ΡŒΠΊΠΎ высоким, Ρ‡Ρ‚ΠΎ ΠΌΠΎΠΆΠ΅Ρ‚ ΡΠΆΠ΅Ρ‡ΡŒ ΠΎΠ±ΠΌΠΎΡ‚ΠΊΡƒ двигатСля.

    Врансформатор

    Врансформатор – это элСктромагнитноС устройство, ΠΈΡΠΏΠΎΠ»ΡŒΠ·ΡƒΠ΅ΠΌΠΎΠ΅ для ΠΏΠΎΠ²Ρ‹ΡˆΠ΅Π½ΠΈΡ ΠΈΠ»ΠΈ пониТСния ΠΏΠ΅Ρ€Π΅ΠΌΠ΅Π½Π½ΠΎΠ³ΠΎ напряТСния ΠΈΠ»ΠΈ ΠΏΠ΅Ρ€Π΅ΠΌΠ΅Π½Π½ΠΎΠ³ΠΎ Ρ‚ΠΎΠΊΠ°, Π° Π½Π΅ постоянного.

    • ΠŸΡ€ΠΈΠ½Ρ†ΠΈΠΏ трансформатора Π²Π·Π°ΠΈΠΌΠ½ΠΎΠΉ ΠΈΠ½Π΄ΡƒΠΊΡ†ΠΈΠΈ .
    • ΠšΠ°Ρ‚ΡƒΡˆΠΊΠ° трансформатора, Π½Π° ΠΊΠΎΡ‚ΠΎΡ€ΡƒΡŽ подаСтся Π²Ρ…ΠΎΠ΄, называСтся ΠΏΠ΅Ρ€Π²ΠΈΡ‡Π½ΠΎΠΉ ΠΎΠ±ΠΌΠΎΡ‚ΠΊΠΎΠΉ, Π° другая, Π³Π΄Π΅ получаСтся Π²Ρ‹Ρ…ΠΎΠ΄Π½ΠΎΠΉ сигнал, называСтся Π²Ρ‚ΠΎΡ€ΠΈΡ‡Π½ΠΎΠΉ ΠΎΠ±ΠΌΠΎΡ‚ΠΊΠΎΠΉ.
    • Π‘Π΅Ρ€Π΄Π΅Ρ‡Π½ΠΈΠΊ состоит ΠΈΠ· Π»Π°ΠΌΠΈΠ½ΠΈΡ€ΠΎΠ²Π°Π½Π½Ρ‹Ρ… ΠΏΠ΅Ρ€Ρ„ΠΎΡ€ΠΈΡ€ΠΎΠ²Π°Π½Π½Ρ‹Ρ… листов ΠΆΠ΅Π»Π΅Π·Π°, Π½Π° ΠΊΠΎΡ‚ΠΎΡ€Ρ‹Π΅ Π½Π°ΠΌΠΎΡ‚Π°Π½Π° ΠΊΠ°Ρ‚ΡƒΡˆΠΊΠ°.
    • Π›Π°ΠΌΠΈΠ½ΠΈΡ€ΠΎΠ²Π°Π½ΠΈΠ΅ сСрдСчника выполняСтся для ΡƒΠΌΠ΅Π½ΡŒΡˆΠ΅Π½ΠΈΡ ΠΏΠΎΡ‚Π΅Ρ€ΡŒ Π½Π° Π²ΠΈΡ…Ρ€Π΅Π²Ρ‹Π΅ Ρ‚ΠΎΠΊΠΈ (Π²ΠΈΡ…Ρ€Π΅Π²Ρ‹Π΅ Ρ‚ΠΎΠΊΠΈ ΠΈ ΠΈΠ½Π΄ΡƒΡ†ΠΈΡ€ΠΎΠ²Π°Π½Π½Ρ‹ΠΉ Ρ‚ΠΎΠΊ, Π²ΠΎΠ·Π½ΠΈΠΊΠ°ΡŽΡ‰ΠΈΠΉ Π² сСрдСчникС).
    • Врансформатор Π½Π΅ ΠΌΠΎΠΆΠ΅Ρ‚ ΠΏΠΎΠ²Ρ‹ΡˆΠ°Ρ‚ΡŒ ΠΈΠ»ΠΈ ΠΏΠΎΠ½ΠΈΠΆΠ°Ρ‚ΡŒ ΡΠ½Π΅Ρ€Π³ΠΈΡŽ ΠΈΠ»ΠΈ ΠΌΠΎΡ‰Π½ΠΎΡΡ‚ΡŒ.
    • Врансформатор
    • потрСбляСт ΠΎΡ‡Π΅Π½ΡŒ Π½Π΅Π±ΠΎΠ»ΡŒΡˆΡƒΡŽ Ρ‡Π°ΡΡ‚ΡŒ Π²Ρ…ΠΎΠ΄Π½ΠΎΠΉ мощности, поэтому ΠΎΠ½ ΠΌΠΎΠΆΠ΅Ρ‚ Ρ€Π°Π±ΠΎΡ‚Π°Ρ‚ΡŒ нСсколько часов Π±Π΅Π· Π·Π½Π°Ρ‡ΠΈΡ‚Π΅Π»ΡŒΠ½Ρ‹Ρ… ΠΏΠΎΡ‚Π΅Ρ€ΡŒ мощности. $$ \ frac {V_s} {V_p} = \ frac {N_s} {N_p} = \ frac {I_p} {I_s} $$

    Π˜Π½Ρ„ΠΎΡ€ΠΌΠ°Ρ†ΠΈΡ

    Врансформатор – ΠΎΠ΄Π½ΠΎ ΠΈΠ· статичСских устройств Π² элСктричСских ΠΏΡ€ΠΈΠ±ΠΎΡ€Π°Ρ….Π’Π°ΠΊΠΈΠΌ ΠΎΠ±Ρ€Π°Π·ΠΎΠΌ, ΠΏΡ€ΠΈ Ρ‚Ρ€Π΅Π½ΠΈΠΈ Π½Π΅Ρ‚ ΠΏΠΎΡ‚Π΅Ρ€ΡŒ энСргии. Π‘Π»Π΅Π΄ΠΎΠ²Π°Ρ‚Π΅Π»ΡŒΠ½ΠΎ, ΠΎΠ½ΠΈ ΠΌΠΎΠ³ΡƒΡ‚ ΠΈΠΌΠ΅Ρ‚ΡŒ Π±ΠΎΠ»Π΅Π΅ высокий ΠšΠŸΠ”. ΠœΡ‹ ΠΌΠΎΠΆΠ΅ΠΌ Π΄ΠΎΡΡ‚ΠΈΡ‡ΡŒ ΠšΠŸΠ” трансформатора Π΄ΠΎ 95%.

    ΠŸΠΎΡ‚Π΅Ρ€ΠΈ энСргии

    • ΠŸΠΎΡ‚Π΅Ρ€ΠΈ Π² ΠΌΠ΅Π΄ΠΈ Π² ΠΎΠ±ΠΌΠΎΡ‚ΠΊΠ΅: Из-Π·Π° сопротивлСния ΠΎΠ±ΠΌΠΎΡ‚ΠΊΠΈ ΠΏΠ΅Ρ€Π²ΠΈΡ‡Π½ΠΎΠΉ ΠΈ Π²Ρ‚ΠΎΡ€ΠΈΡ‡Π½ΠΎΠΉ ΠΎΠ±ΠΌΠΎΡ‚ΠΎΠΊ нСкоторая Ρ‡Π°ΡΡ‚ΡŒ элСктричСской энСргии всСгда тСряСтся Π² Π²ΠΈΠ΄Π΅ Ρ‚Π΅ΠΏΠ»ΠΎΠ²ΠΎΠΉ энСргии.
    • ΠŸΠΎΡ‚Π΅Ρ€ΠΈ ΠΏΠΎΡ‚ΠΎΠΊΠ°: Бвязь ΠΏΠ΅Ρ€Π²ΠΈΡ‡Π½ΠΎΠΉ ΠΈ Π²Ρ‚ΠΎΡ€ΠΈΡ‡Π½ΠΎΠΉ ΠΎΠ±ΠΌΠΎΡ‚ΠΎΠΊ Π½ΠΈΠΊΠΎΠ³Π΄Π° Π½Π΅ Π±Ρ‹Π²Π°Π΅Ρ‚ идСальной, ΠΈ вСсь ΠΌΠ°Π³Π½ΠΈΡ‚Π½Ρ‹ΠΉ ΠΏΠΎΡ‚ΠΎΠΊ, создаваСмый Π² ΠΏΠ΅Ρ€Π²ΠΈΡ‡Π½ΠΎΠΉ ΠΎΠ±ΠΌΠΎΡ‚ΠΊΠ΅, Π½Π΅ связываСт Π²Ρ‚ΠΎΡ€ΠΈΡ‡Π½ΡƒΡŽ ΠΎΠ±ΠΌΠΎΡ‚ΠΊΡƒ.Π­Ρ‚ΠΎ ΠΏΡ€ΠΈΠ²ΠΎΠ΄ΠΈΡ‚ ΠΊ ΠΏΠΎΡ‚Π΅Ρ€Π΅ энСргии.
    • ΠŸΠΎΡ‚Π΅Ρ€ΠΈ Π² ΠΆΠ΅Π»Π΅Π·Π΅ Π² сСрдСчникС: ΠŸΠΎΡ‚Π΅Ρ€ΠΈ Π² ΠΆΠ΅Π»Π΅Π·Π΅ Π±Ρ‹Π²Π°ΡŽΡ‚ Π΄Π²ΡƒΡ… Ρ‚ΠΈΠΏΠΎΠ²:
      1. ΠŸΠΎΡ‚Π΅Ρ€ΠΈ Π½Π° Π²ΠΈΡ…Ρ€Π΅Π²Ρ‹Π΅ Ρ‚ΠΎΠΊΠΈ: Из-Π·Π° пСриодичСски ΠΈΠ·ΠΌΠ΅Π½ΡΡŽΡ‰Π΅ΠΉΡΡ ΠΏΡ€ΠΈΡ€ΠΎΠ΄Ρ‹ ΠΏΠ΅Ρ€Π΅ΠΌΠ΅Π½Π½ΠΎΠ³ΠΎ Ρ‚ΠΎΠΊΠ°, ΠΏΠΎΠ΄Π°Π²Π°Π΅ΠΌΠΎΠ³ΠΎ Π² ΠΏΠ΅Ρ€Π²ΠΈΡ‡Π½ΡƒΡŽ ΠΎΠ±ΠΌΠΎΡ‚ΠΊΡƒ, измСняСтся ΠΏΠΎΡ‚ΠΎΠΊ, связанный с сСрдСчником, ΠΈ, ΡΠ»Π΅Π΄ΠΎΠ²Π°Ρ‚Π΅Π»ΡŒΠ½ΠΎ, Π² Π½Π΅ΠΌ ΠΈΠ½Π΄ΡƒΡ†ΠΈΡ€ΡƒΡŽΡ‚ΡΡ Π²ΠΈΡ…Ρ€Π΅Π²Ρ‹Π΅ Ρ‚ΠΎΠΊΠΈ. . Π’ΠΈΡ…Ρ€Π΅Π²Ρ‹Π΅ Ρ‚ΠΎΠΊΠΈ, ΠΈΠ½Π΄ΡƒΡ†ΠΈΡ€ΡƒΠ΅ΠΌΡ‹Π΅ Π² сСрдСчникС, Π½Π΅ΠΆΠ΅Π»Π°Ρ‚Π΅Π»ΡŒΠ½Ρ‹, ΠΏΠΎΡΠΊΠΎΠ»ΡŒΠΊΡƒ ΠΎΠ½ΠΈ Π½Π°Π³Ρ€Π΅Π²Π°ΡŽΡ‚ сСрдСчник ΠΈ приводят ΠΊ потСрям энСргии. Для ΠΌΠΈΠ½ΠΈΠΌΠΈΠ·Π°Ρ†ΠΈΠΈ ΠΏΠΎΡ‚Π΅Ρ€ΡŒ Π½Π° Π²ΠΈΡ…Ρ€Π΅Π²Ρ‹Π΅ Ρ‚ΠΎΠΊΠΈ сСрдСчник Π»Π°ΠΌΠΈΠ½ΠΈΡ€ΠΎΠ²Π°Π½.
      2. Hysterisis Loss: ΠŸΠ΅Ρ€Π΅ΠΌΠ΅Π½Π½Ρ‹ΠΉ Ρ‚ΠΎΠΊ, ΠΏΡ€ΠΎΡ‚Π΅ΠΊΠ°ΡŽΡ‰ΠΈΠΉ Ρ‡Π΅Ρ€Π΅Π· ΠΊΠ°Ρ‚ΡƒΡˆΠΊΠΈ, Π½Π°ΠΌΠ°Π³Π½ΠΈΡ‡ΠΈΠ²Π°Π΅Ρ‚ ΠΈ Ρ€Π°Π·ΠΌΠ°Π³Π½ΠΈΡ‡ΠΈΠ²Π°Π΅Ρ‚ ΠΆΠ΅Π»Π΅Π·Π½Ρ‹ΠΉ сСрдСчник снова ΠΈ снова.ΡΠ»Π΅Π΄ΠΎΠ²Π°Ρ‚Π΅Π»ΡŒΠ½ΠΎ, Π²ΠΎ врСмя ΠΊΠ°ΠΆΠ΄ΠΎΠ³ΠΎ Ρ†ΠΈΠΊΠ»Π° намагничивания ΠΎΠ΄Π½Π° ΠΈ Ρ‚Π° ΠΆΠ΅ энСргия тСряСтся ΠΈΠ·-Π·Π° гистСриза. Π§Ρ‚ΠΎΠ±Ρ‹ свСсти ΠΊ ΠΌΠΈΠ½ΠΈΠΌΡƒΠΌΡƒ эти ΠΏΠΎΡ‚Π΅Ρ€ΠΈ, ΠΌΡ‹ Π²Ρ‹Π±ΠΈΡ€Π°Π΅ΠΌ ΠΌΠ°Ρ‚Π΅Ρ€ΠΈΠ°Π» сСрдСчника с мСньшими гистСрСзисными потСрями, ΠΊΠ°ΠΊ ΠΏΡ€Π°Π²ΠΈΠ»ΠΎ, мягкоС ΠΆΠ΅Π»Π΅Π·ΠΎ.

    ΠšΠŸΠ” трансформатора

    Π˜Π΄Π΅Π°Π»ΡŒΠ½Ρ‹ΠΉ ΠšΠŸΠ” трансформатора составляСт 100% ΠΈΠ»ΠΈ 1, Π½ΠΎ фактичСская выходная ΠΌΠΎΡ‰Π½ΠΎΡΡ‚ΡŒ трансформатора всСгда мСньшС, Ρ‡Π΅ΠΌ внутрСнняя ΠΌΠΎΡ‰Π½ΠΎΡΡ‚ΡŒ, поэтому ΠšΠŸΠ” Ρ‚Π°ΠΊΠΆΠ΅ всСгда мСньшС 100%. Π’ Ρ†Π΅Π»ΠΎΠΌ ΠšΠŸΠ” трансформатора ΠΎΡ‡Π΅Π½ΡŒ высок ΠΈ составляСт порядка 90%.

    • Π­Ρ„Ρ„Π΅ΠΊΡ‚ΠΈΠ²Π½ΠΎΡΡ‚ΡŒ опрСдСляСтся ΠΏΠΎ Ρ„ΠΎΡ€ΠΌΡƒΠ»Π΅: $$ \ eta = \ frac {\ text {Output Power}} {\ text {Input Power}} \ times 100 $$
    • По Π²Ρ‚ΠΎΡ€ΠΈΡ‡Π½ΠΎΠΌΡƒ ΠΈ ΠΏΠ΅Ρ€Π²ΠΈΡ‡Π½ΠΎΠΌΡƒ Π½Π°ΠΏΡ€ΡΠΆΠ΅Π½ΠΈΡŽ ΠΈ Ρ‚ΠΎΠΊΡƒ $$ \ eta = \ frac {V_sI_s} {V_pI_p} $$
    • As, \ (\ text {Imput} = \ text {Output} + \ text {Losses} \) Ρ‚Π°ΠΊ, $$ \ eta = \ frac {\ text {Output Power}} {\ text {Output Power} + \ text {Losses}} $$

    ΠŸΡ€ΠΈΠ»ΠΎΠΆΠ΅Π½ΠΈΡ

    • ΠŸΡ€ΠΈΠΌΠ΅Π½ΡΡŽΡ‚ΡΡ ΠΏΡ€ΠΈ распрСдСлСнии элСктроэнСргии Π² сСйфС ΠΎΡ‚ элСктростанций ΠΊ ΠΊΠΎΠΌΠΌΡƒΠ½Π°Π»ΡŒΠ½Ρ‹ΠΌ станциям.
    • ΠŸΡ€ΠΈΠ½Ρ†ΠΈΠΏ ΠΏΠ΅Ρ€Π΅Π΄Π°Ρ‡ΠΈ Π·Π°ΠΊΠ»ΡŽΡ‡Π°Π΅Ρ‚ΡΡ Π² Ρ‚ΠΎΠΌ, Ρ‡Ρ‚ΠΎ ΠΌΡ‹ ΡƒΠΌΠ΅Π½ΡŒΡˆΠ°Π΅ΠΌ Π·Π½Π°Ρ‡Π΅Π½ΠΈΠ΅ Ρ‚ΠΎΠΊΠ° Π·Π° счСт увСличСния напряТСния ΠΏΠ΅Ρ€Π΅Π΄Π°Ρ‡ΠΈ, Ρ‡Ρ‚ΠΎ позволяСт ΡΡΠΊΠΎΠ½ΠΎΠΌΠΈΡ‚ΡŒ Π½Π° стоимости ΠΌΠ΅Π΄ΠΈ, Ρ‚Π°ΠΊ ΠΊΠ°ΠΊ мСньшСС количСство ΠΏΡ€ΠΎΠ²ΠΎΠ΄ΠΎΠ² ΠΌΠΎΠΆΠ΅Ρ‚ ΠΏΠΎΠ»Π½ΠΎΡΡ‚ΡŒΡŽ Π·Π°ΠΏΠΎΠ»Π½ΠΈΡ‚ΡŒ Ρ†Π΅Π»ΡŒ ΠΈΠ·-Π·Π° ΠΌΠ°Π»Ρ‹Ρ… Ρ‚ΠΎΠΊΠΎΠ².
    • Врансформатор
    • Ρ‚Π°ΠΊΠΆΠ΅ обСспСчиваСт ΠΊΠΎΠ½Ρ‚Ρ€ΠΎΠ»ΡŒ распрСдСлСния элСктроэнСргии Π² Π³ΠΎΡ€ΠΎΠ΄Π΅.

    ЭлСктромагнитная индукция 1, Поля ΠΈ эффСкты

    ОписаниС явлСния

    Быстро ΠΌΠ΅Π½ΡΡŽΡ‰Π΅Π΅ΡΡ ΠΌΠ°Π³Π½ΠΈΡ‚Π½ΠΎΠ΅ ΠΏΠΎΠ»Π΅ ΠΈΠ½Π΄ΡƒΡ†ΠΈΡ€ΡƒΠ΅Ρ‚ элСктричСскиС Ρ‚ΠΎΠΊΠΈ, ΠΏΡ€ΠΎΡ‚Π΅ΠΊΠ°ΡŽΡ‰ΠΈΠ΅ ΠΏΠΎ Π·Π°ΠΌΠΊΠ½ΡƒΡ‚ΠΎΠΉ Ρ†Π΅ΠΏΠΈ.

    На ΠΏΡ€ΠΈΠ²Π΅Π΄Π΅Π½Π½ΠΎΠΉ Π²Ρ‹ΡˆΠ΅ Π΄ΠΈΠ°Π³Ρ€Π°ΠΌΠΌΠ΅ стСрТнСвой ΠΌΠ°Π³Π½ΠΈΡ‚ опускаСтся Π²Π΅Ρ€Ρ‚ΠΈΠΊΠ°Π»ΡŒΠ½ΠΎ Ρ‡Π΅Ρ€Π΅Π· ΠΊΠ°Ρ‚ΡƒΡˆΠΊΡƒ, ΡΠΎΠ΅Π΄ΠΈΠ½Π΅Π½Π½ΡƒΡŽ с Π³Π°Π»ΡŒΠ²Π°Π½ΠΎΠΌΠ΅Ρ‚Ρ€ΠΎΠΌ с Π½ΡƒΠ»Π΅Π²Ρ‹ΠΌ Ρ†Π΅Π½Ρ‚Ρ€ΠΎΠΌ.

    Π“Ρ€Π°Ρ„ΠΈΠΊ зависимости Π­Π”Π‘ ΠΊΠ°Ρ‚ΡƒΡˆΠΊΠΈ ΠΎΡ‚ Π²Ρ€Π΅ΠΌΠ΅Π½ΠΈ ΠΏΠΎΠΊΠ°Π·Ρ‹Π²Π°Π΅Ρ‚, Ρ‡Ρ‚ΠΎ:

    Когда ΠΏΠ΅Ρ€Π²Ρ‹ΠΉ полюс (S) ΠΏΠ°Π΄Π°Π΅Ρ‚ Ρ‡Π΅Ρ€Π΅Π· ΠΊΠ°Ρ‚ΡƒΡˆΠΊΡƒ, Π­Π”Π‘ увСличиваСтся Π΄ΠΎ уровня, Π° Π·Π°Ρ‚Π΅ΠΌ ΡƒΠΌΠ΅Π½ΡŒΡˆΠ°Π΅Ρ‚ΡΡ.

    Когда сСрСдина ΠΌΠ°Π³Π½ΠΈΡ‚Π° ΠΏΡ€ΠΎΡ…ΠΎΠ΄ΠΈΡ‚ Ρ‡Π΅Ρ€Π΅Π· ΠΊΠ°Ρ‚ΡƒΡˆΠΊΡƒ, Π­Π”Π‘ минимальна.ΠšΠ°Ρ‚ΡƒΡˆΠΊΠ° Π½Π΅ ΠΏΠ΅Ρ€Π΅Ρ€Π΅Π·Π°Π΅Ρ‚ силовыС Π»ΠΈΠ½ΠΈΠΈ.

    Максимальная Π­Π”Π‘ достигаСтся, ΠΊΠΎΠ³Π΄Π° Π²Ρ‚ΠΎΡ€ΠΎΠΉ полюс (N) проваливаСтся Ρ‡Π΅Ρ€Π΅Π· ΠΊΠ°Ρ‚ΡƒΡˆΠΊΡƒ. Π­Ρ‚ΠΎ ΠΊΠΎΠ³Π΄Π° ΡΠΊΠΎΡ€ΠΎΡΡ‚ΡŒ Ρ€Π΅Π·ΠΊΠΈ силовых Π»ΠΈΠ½ΠΈΠΉ самая высокая, ΠΏΠΎΡ‚ΠΎΠΌΡƒ Ρ‡Ρ‚ΠΎ ΠΌΠ°Π³Π½ΠΈΡ‚ ΠΏΠ°Π΄Π°Π΅Ρ‚ быстрСС. Π’ Ρ€Π΅Π·ΡƒΠ»ΡŒΡ‚Π°Ρ‚Π΅ увСличСния скорости ΠΏΠ΅Ρ€ΠΈΠΎΠ΄ высокой Π­Π”Π‘ ΠΊΠΎΡ€ΠΎΡ‡Π΅.

    NB Π’ Ρ€Π΅Π·ΡƒΠ»ΡŒΡ‚Π°Ρ‚Π΅ Ρ‚ΠΎΠ³ΠΎ, Ρ‡Ρ‚ΠΎ Π½Π°ΠΏΡ€Π°Π²Π»Π΅Π½ΠΈΠ΅ поля мСняСтся Π½Π° ΠΏΡ€ΠΎΡ‚ΠΈΠ²ΠΎΠΏΠΎΠ»ΠΎΠΆΠ½ΠΎΠ΅, ΠΊΠΎΠ³Π΄Π° полюса ΠΎΠΏΡƒΡΠΊΠ°ΡŽΡ‚ΡΡ Ρ‡Π΅Ρ€Π΅Π· ΠΊΠ°Ρ‚ΡƒΡˆΠΊΡƒ, Π½Π°ΠΏΡ€Π°Π²Π»Π΅Π½ΠΈΠ΅ ΠΈΠ½Π΄ΡƒΡ†ΠΈΡ€ΠΎΠ²Π°Π½Π½ΠΎΠ³ΠΎ Ρ‚ΠΎΠΊΠ° Ρ‚Π°ΠΊΠΆΠ΅ мСняСтся Π½Π° ΠΏΡ€ΠΎΡ‚ΠΈΠ²ΠΎΠΏΠΎΠ»ΠΎΠΆΠ½ΠΎΠ΅.

    Π—Π½Π°Ρ‡ΠΈΡ‚, Π­Π”Π‘ Ρ‚ΠΎΠΆΠ΅ обратная (Π­Π”Π‘ прямо ΠΏΡ€ΠΎΠΏΠΎΡ€Ρ†ΠΈΠΎΠ½Π°Π»ΡŒΠ½Π° Ρ‚ΠΎΠΊΡƒ).

    ΠΊ Π½Π°Ρ‡Π°Π»Ρƒ

    Π—Π°ΠΊΠΎΠ½ ЀарадСя

    Рассмотрим ΠΊΠ°Ρ‚ΡƒΡˆΠΊΠΈ Ρ€Π°Π·Π½ΠΎΠ³ΠΎ Ρ€Π°Π·ΠΌΠ΅Ρ€Π°, ΠΊΠΎΠ³Π΄Π° ΠΎΠ΄ΠΈΠ½ ΠΈ Ρ‚ΠΎΡ‚ ΠΆΠ΅ ΠΌΠ°Π³Π½ΠΈΡ‚ вводится Π² корпус ΠΊΠ°ΠΆΠ΄ΠΎΠΉ ΠΊΠ°Ρ‚ΡƒΡˆΠΊΠΈ с ΠΎΠ΄ΠΈΠ½Π°ΠΊΠΎΠ²ΠΎΠΉ ΡΠΊΠΎΡ€ΠΎΡΡ‚ΡŒΡŽ.

    УстановлСно, Ρ‡Ρ‚ΠΎ,

    Π’Π°ΠΊ навСдСнная Π­Π”Π‘ E прямо ΠΏΡ€ΠΎΠΏΠΎΡ€Ρ†ΠΈΠΎΠ½Π°Π»ΡŒΠ½Π° количСству Π²ΠΈΡ‚ΠΊΠΎΠ² N ,

    Π’Π΅ΠΏΠ΅Ρ€ΡŒ рассмотрим Ρ‚ΠΎΠ»ΡŒΠΊΠΎ ΠΎΠ΄Π½Ρƒ ΠΊΠ°Ρ‚ΡƒΡˆΠΊΡƒ ΠΈ ΠΏΠΎ ΠΎΡ‡Π΅Ρ€Π΅Π΄ΠΈ прСдставим Ρ‚Ρ€ΠΈ ΠΌΠ°Π³Π½ΠΈΡ‚Π°. ΠœΠ°Π³Π½ΠΈΡ‚Ρ‹ ΠΈΠΌΠ΅ΡŽΡ‚ Ρ€Π°Π·Π½ΡƒΡŽ силу ΠΈ вводятся Π² ΠΊΠ°Ρ‚ΡƒΡˆΠΊΡƒ с ΠΎΠ΄ΠΈΠ½Π°ΠΊΠΎΠ²ΠΎΠΉ ΡΠΊΠΎΡ€ΠΎΡΡ‚ΡŒΡŽ.

    ΠŸΡƒΡ‚Π΅ΠΌ измСрСния максимальной Π­Π”Π‘ ΠΈ ΠΌΠ°Π³Π½ΠΈΡ‚Π½ΠΎΠ³ΠΎ ΠΏΠΎΡ‚ΠΎΠΊΠ° для ΠΊΠ°ΠΆΠ΄ΠΎΠ³ΠΎ ΠΌΠ°Π³Π½ΠΈΡ‚Π° Π±Ρ‹Π»ΠΎ ΠΎΠ±Π½Π°Ρ€ΡƒΠΆΠ΅Π½ΠΎ, Ρ‡Ρ‚ΠΎ

    Π’Π°ΠΊ навСдСнная Π­Π”Π‘ E прямо ΠΏΡ€ΠΎΠΏΠΎΡ€Ρ†ΠΈΠΎΠ½Π°Π»ΡŒΠ½Π° ΠΏΠΎΡ‚ΠΎΠΊΡƒ Ο† ,

    Π—Π°ΠΊΠΎΠ½ ЀарадСя просто гласит:

    Π˜Π½Π΄ΡƒΡ†ΠΈΡ€ΠΎΠ²Π°Π½Π½Π°Ρ Π­Π”Π‘ Π² Π·Π°ΠΌΠΊΠ½ΡƒΡ‚ΠΎΠΉ Ρ†Π΅ΠΏΠΈ прямо ΠΏΡ€ΠΎΠΏΠΎΡ€Ρ†ΠΈΠΎΠ½Π°Π»ΡŒΠ½Π° ΠΏΠΎΡ‚ΠΎΠΊΠΎΡΡ†Π΅ΠΏΠ»Π΅Π½ΠΈΡŽ.

    ΠŸΠΎΡ‚ΠΎΠΊΠΎΠ²Π°Ρ связь NΟ† – ΠΏΡ€ΠΎΠΈΠ·Π²Π΅Π΄Π΅Π½ΠΈΠ΅ ΠΌΠ°Π³Π½ΠΈΡ‚Π½ΠΎΠ³ΠΎ ΠΏΠΎΡ‚ΠΎΠΊΠ° Ο† ΠΈ количСства Π²ΠΈΡ‚ΠΊΠΎΠ² N Π½Π° ΠΊΠ°Ρ‚ΡƒΡˆΠΊΠ΅.

    ΠœΡ‹ Π²ΠΈΠ΄Π΅Π»ΠΈ это,

    Π‘Π»Π΅Π΄ΠΎΠ²Π°Ρ‚Π΅Π»ΡŒΠ½ΠΎ,

    ΠΊ Π½Π°Ρ‡Π°Π»Ρƒ

    Π—Π°ΠΊΠΎΠ½ Π›Π΅Π½Ρ†Π°

    НаправлСниС Π½Π°Π²Π΅Π΄Π΅Π½Π½ΠΎΠΉ Π­Π”Π‘ Ρ‚Π°ΠΊΠΎΠ²ΠΎ, Ρ‡Ρ‚ΠΎ Π½Π°Π²Π΅Π΄Π΅Π½Π½Ρ‹ΠΉ Ρ‚ΠΎΠΊ противодСйствуСт измСнСнию

    Π΅Π³ΠΎ производят.

    Π˜Ρ‚Π°ΠΊ, ΠΊΠΎΠ³Π΄Π° ΡŽΠΆΠ½Ρ‹ΠΉ ΠΌΠ°Π³Π½ΠΈΡ‚Π½Ρ‹ΠΉ полюс пСрСмСщаСтся ΠΊ ΠΊΠ°Ρ‚ΡƒΡˆΠΊΠ΅ Π² Ρ†Π΅ΠΏΠΈ, ΠΏΠΎΠ²Π΅Ρ€Ρ…Π½ΠΎΡΡ‚ΡŒ ΠΊΠ°Ρ‚ΡƒΡˆΠΊΠΈ прСдставляСт собой ΡŽΠΆΠ½Ρ‹ΠΉ полюс.

    Π˜Π½Π΄ΡƒΡ†ΠΈΡ€ΠΎΠ²Π°Π½Π½Ρ‹ΠΉ Ρ‚ΠΎΠΊ противодСйствуСт Π²Ρ‹Π·Π²Π°Π²ΡˆΠ΅ΠΌΡƒ Π΅Π³ΠΎ измСнСнию, ΠΏΡ‹Ρ‚Π°ΡΡΡŒ ΠΏΡ€Π΅Π΄ΠΎΡ‚Π²Ρ€Π°Ρ‚ΠΈΡ‚ΡŒ ΠΏΠΎΠΏΠ°Π΄Π°Π½ΠΈΠ΅ юТного полюса Π² ΠΊΠ°Ρ‚ΡƒΡˆΠΊΡƒ (отталкивая Π΅Π³ΠΎ).

    Π’ΠΎΡ‡Π½ΠΎ Ρ‚Π°ΠΊ ΠΆΠ΅, ΠΊΠΎΠ³Π΄Π° ΡŽΠΆΠ½Ρ‹ΠΉ полюс вытаскиваСтся ΠΈΠ· ΠΊΠ°Ρ‚ΡƒΡˆΠΊΠΈ Π² Ρ†Π΅ΠΏΠΈ, лицСвая сторона ΠΊΠ°Ρ‚ΡƒΡˆΠΊΠΈ прСдставляСт собой сСвСрный полюс.

    Π˜Π½Π΄ΡƒΡ†ΠΈΡ€ΠΎΠ²Π°Π½Π½Ρ‹ΠΉ Ρ‚ΠΎΠΊ противодСйствуСт Π²Ρ‹Π·Π²Π°Π²ΡˆΠ΅ΠΌΡƒ Π΅Π³ΠΎ измСнСнию, ΠΏΡ‹Ρ‚Π°ΡΡΡŒ ΠΏΡ€Π΅Π΄ΠΎΡ‚Π²Ρ€Π°Ρ‚ΠΈΡ‚ΡŒ Π²Ρ‹Ρ…ΠΎΠ΄ юТного полюса ΠΈΠ· ΠΊΠ°Ρ‚ΡƒΡˆΠΊΠΈ (притягивая Π΅Π³ΠΎ).

    NB

    1.) ΠΊΠ°ΠΊ Π½Π°ΠΏΡ€Π°Π²Π»Π΅Π½ΠΈΠ΅ Ρ‚ΠΎΠΊΠ° измСняСтся Π½Π°ΠΏΡ€Π°Π²Π»Π΅Π½ΠΈΠ΅ΠΌ ΠΌΠ°Π³Π½ΠΈΡ‚Π°.

    2.) Π½Π° ΠΊΠ°ΠΆΠ΄ΠΎΠΉ повСрхности ΠΊΠ°Ρ‚ΡƒΡˆΠΊΠΈ, ΠΊΠ°ΠΊ линия, провСдСнная ΠΌΠ΅ΠΆΠ΄Ρƒ ΠΊΠΎΠ½Ρ†Π°ΠΌΠΈ стрСлок (сСрого Ρ†Π²Π΅Ρ‚Π°), ΠΎΠ±Ρ€Π°Π·ΡƒΠ΅Ρ‚

    Β« N Β» ΠΈ Β« S Β», Π·Π°Π΄Π°ΡŽΡ‰ΠΈΠ΅ ΠΏΠΎΠ»ΡΡ€Π½ΠΎΡΡ‚ΡŒ ΠΊΠ°Ρ‚ΡƒΡˆΠ΅ΠΊ.

    ΠΊ Π½Π°Ρ‡Π°Π»Ρƒ

    Π£Ρ€Π°Π²Π½Π΅Π½ΠΈΠ΅ НСймана

    Π­Ρ‚ΠΎ ΠΎΠ±ΡŠΠ΅Π΄ΠΈΠ½ΡΠ΅Ρ‚ ΠΏΡ€ΠΎΠΏΠΎΡ€Ρ†ΠΈΠΎΠ½Π°Π»ΡŒΠ½ΠΎΡΡ‚ΡŒ Π·Π°ΠΊΠΎΠ½Π° ЀарадСя с Π½Π°ΠΏΡ€Π°Π²Π»Π΅Π½ΠΈΠ΅ΠΌ ΠΈΠ½Π΄ΡƒΡ†ΠΈΡ€ΠΎΠ²Π°Π½Π½ΠΎΠ³ΠΎ Ρ‚ΠΎΠΊΠ° ΠΈΠ· Π·Π°ΠΊΠΎΠ½Π° Π›Π΅Π½Ρ†Π°.

    Π’ Ρ€Π΅Π·ΡƒΠ»ΡŒΡ‚Π°Ρ‚Π΅ Сдинообразия ΠΈΡΠΏΠΎΠ»ΡŒΠ·ΡƒΠ΅ΠΌΡ‹Ρ… Π΅Π΄ΠΈΠ½ΠΈΡ† (БИ) Π½Π΅Ρ‚ нСобходимости Π² константС ΠΏΡ€ΠΎΠΏΠΎΡ€Ρ†ΠΈΠΎΠ½Π°Π»ΡŒΠ½ΠΎΡΡ‚ΠΈ.

    Π—Π½Π°ΠΊ минус взят ΠΈΠ· Π·Π°ΠΊΠΎΠ½Π° Π›Π΅Π½Ρ†Π°, Ρ‡Ρ‚ΠΎ ΡƒΠΊΠ°Π·Ρ‹Π²Π°Π΅Ρ‚ Π½Π° ΠΏΡ€ΠΎΡ‚ΠΈΠ²ΠΎΠΏΠΎΠ»ΠΎΠΆΠ½ΡƒΡŽ ΠΏΡ€ΠΈΡ€ΠΎΠ΄Ρƒ Π½Π°Π²Π΅Π΄Π΅Π½Π½ΠΎΠΉ Π­Π”Π‘ ΠΈ скорости срСзания ΠΌΠ°Π³Π½ΠΈΡ‚Π½ΠΎΠΉ связи.

    Π’ ΡƒΡ€Π°Π²Π½Π΅Π½ΠΈΠ΅ ΠΌΠΎΠΆΠ½ΠΎ внСсти ΠΏΠΎΠΏΡ€Π°Π²ΠΊΠΈ, Ρ‡Ρ‚ΠΎΠ±Ρ‹ Π²ΠΊΠ»ΡŽΡ‡ΠΈΡ‚ΡŒ ΡΠΊΠΎΡ€ΠΎΡΡ‚ΡŒ рСзания ΠΏΠΎΡ‚ΠΎΠΊΠ° dΟ† / dt , Ссли Π²Ρ‹Ρ‡Π΅ΡΡ‚ΡŒ количСство Π²ΠΈΡ‚ΠΊΠΎΠ² N ΠΈΠ· Π΄ΠΈΡ„Ρ„Π΅Ρ€Π΅Π½Ρ†ΠΈΠ°Π»Π°.

    ΠΊ Π½Π°Ρ‡Π°Π»Ρƒ

    ΠŸΡ€Π°Π²ΠΈΠ»ΠΎ ΠΏΡ€Π°Π²ΠΎΠΉ Ρ€ΡƒΠΊΠΈ Π€Π»Π΅ΠΌΠΌΠΈΠ½Π³Π°

    ΠŸΡ€Π°Π²ΠΈΠ»ΠΎ описываСт Ρ€Π΅Π·ΡƒΠ»ΡŒΡ‚ΠΈΡ€ΡƒΡŽΡ‰Π΅Π΅ Π½Π°ΠΏΡ€Π°Π²Π»Π΅Π½Π½ΠΎΠ΅ Π΄Π²ΠΈΠΆΠ΅Π½ΠΈΠ΅ ΠΈΠ½Π΄ΡƒΡ†ΠΈΡ€ΠΎΠ²Π°Π½Π½ΠΎΠ³ΠΎ Ρ‚ΠΎΠΊΠ° для ΠΏΡ€ΠΎΠ²ΠΎΠ΄Π½ΠΈΠΊΠ°, двиТущСгося ΠΏΠΎΠ΄ прямым ΡƒΠ³Π»ΠΎΠΌ ΠΊ β€‹β€‹Π½Π°ΠΏΡ€Π°Π²Π»Π΅Π½ΠΈΡŽ поля.

    Π’Ρ€ΠΈ Π²Π΅Π»ΠΈΡ‡ΠΈΠ½Ρ‹ FIELD , CURRENT И MOTION Π²Π·Π°ΠΈΠΌΠ½ΠΎ пСрпСндикулярны Π΄Ρ€ΡƒΠ³ Π΄Ρ€ΡƒΠ³Ρƒ.

    Π˜ΡΠΏΠΎΠ»ΡŒΠ·ΡƒΡ ΠΏΡ€Π°Π²ΡƒΡŽ Ρ€ΡƒΠΊΡƒ, располоТитС ΠΏΠ΅Ρ€Π²Ρ‹ΠΉ, Π²Ρ‚ΠΎΡ€ΠΎΠΉ ΠΈ большой ΠΏΠ°Π»ΡŒΡ†Ρ‹ Ρ‚Π°ΠΊ, Ρ‡Ρ‚ΠΎΠ±Ρ‹ ΠΎΠ±Ρ€Π°Π·ΠΎΠ²Π°Ρ‚ΡŒ оси x, y, z.

    Π’Ρ‹Π΄Π΅Π»Π΅Π½Π½Ρ‹Π΅ Π±ΡƒΠΊΠ²Ρ‹ Π² словах ΠΏΠΎΠΌΠΎΠ³Π°ΡŽΡ‚ Π·Π°ΠΏΠΎΠΌΠ½ΠΈΡ‚ΡŒ Ρ‚Ρ€ΠΈ Π²Π΅Π»ΠΈΡ‡ΠΈΠ½Ρ‹.

    F ΠŸΠ΅Ρ€Π²Ρ‹ΠΉ ΠΏΠ°Π»Π΅Ρ† – F НаправлСниС поля

    se C прямой ΠΏΠ°Π»Π΅Ρ† – C Ρ‚Π΅ΠΊΡƒΡ‰Π΅Π΅ Π½Π°ΠΏΡ€Π°Π²Π»Π΅Π½ΠΈΠ΅

    Ρ‡Ρ‚ M b – M ΠΏΡ€ΠΎΠΈΠ·Π²Π΅Π΄Π΅Π½ΠΎ

    ΠΊ Π½Π°Ρ‡Π°Π»Ρƒ

    Π­Π”Π‘, индуцированная Π² мСталличСском стСрТнС

    Для создания Π½Π°Π²Π΅Π΄Π΅Π½Π½ΠΎΠΉ Π­Π”Π‘ E ΠΏΠΎ Π΄Π»ΠΈΠ½Π΅ L мСталличСского стСрТня Π½Π΅ΠΎΠ±Ρ…ΠΎΠ΄ΠΈΠΌΠΎ, Ρ‡Ρ‚ΠΎΠ±Ρ‹ ΠΌΠ°Π³Π½ΠΈΡ‚Π½ΠΎΠ΅ ΠΏΠΎΠ»Π΅ B , ΡΠΊΠΎΡ€ΠΎΡΡ‚ΡŒ v ΠΈ большая ось стСрТня. всС ΠΎΠ½ΠΈ Π΄ΠΎΠ»ΠΆΠ½Ρ‹ Π½Π°Ρ…ΠΎΠ΄ΠΈΡ‚ΡŒΡΡ ΠΏΠΎΠ΄ прямым ΡƒΠ³Π»ΠΎΠΌ Π΄Ρ€ΡƒΠ³ ΠΊ Π΄Ρ€ΡƒΠ³Ρƒ.

    ΠŸΠΎΠ»ΡƒΡ‡Π΅Π½ΠΈΠ΅ E = BLv :

    Рассмотрим мСталличСский ΡΡ‚Π΅Ρ€ΠΆΠ΅Π½ΡŒ Π΄Π»ΠΈΠ½ΠΎΠΉ L .

    Если ΡΡ‚Π΅Ρ€ΠΆΠ΅Π½ΡŒ двиТСтся со ΡΠΊΠΎΡ€ΠΎΡΡ‚ΡŒΡŽ v пСрпСндикулярно Π΅Π³ΠΎ Π΄Π»ΠΈΠ½Π΅, Ρ‚ΠΎΠ³Π΄Π° ΠΏΠ»ΠΎΡ‰Π°Π΄ΡŒ A , уносимая Π·Π° сСкунду, опрСдСляСтся ΠΊΠ°ΠΊ:

    ΠžΠ±Ρ‰ΠΈΠΉ ΠΏΠΎΡ‚ΠΎΠΊ Ο† , проходящий Ρ‡Π΅Ρ€Π΅Π· эту ΠΎΠ±Π»Π°ΡΡ‚ΡŒ Π² сСкунду, являСтся ΠΏΡ€ΠΎΠΈΠ·Π²Π΅Π΄Π΅Π½ΠΈΠ΅ΠΌ ΠΏΠ»ΠΎΡ‰Π°Π΄ΠΈ A ΠΈ плотности ΠΏΠΎΡ‚ΠΎΠΊΠ° B .

    Π—Π°ΠΌΠ΅Π½Π° ΠΏΠ»ΠΎΡ‰Π°Π΄ΠΈ A ,

    Π’ этом случаС, ΠΏΠΎΡΠΊΠΎΠ»ΡŒΠΊΡƒ ΠΎΠ±Ρ‰ΠΈΠΉ ΠΏΠΎΡ‚ΠΎΠΊ Ο† относится ΠΊ 1 сСкундС, ΠΌΡ‹ ΠΌΠΎΠΆΠ΅ΠΌ Π½Π°ΠΏΠΈΡΠ°Ρ‚ΡŒ:

    , Π³Π΄Π΅ dΟ† / dt – ΡΠΊΠΎΡ€ΠΎΡΡ‚ΡŒ Ρ€Π΅Π·ΠΊΠΈ Ρ„Π»ΡŽΡΠ°.

    ΠžΡΡ‚Π°Π²ΠΈΡ‚ΡŒ ΠΊΠΎΠΌΠΌΠ΅Π½Ρ‚Π°Ρ€ΠΈΠΉ